Mksa: Lnternal Medicine
Mksa: Lnternal Medicine
ACP *MKSA
lVle# i*eI Hr-#=lrl**g* Self-A,=s*s*rrrffi nt Frmilr#ffi re
General lnternal
Medicine 2
MKSAP 19 strives to provide the clinical knowledge its learners need to navigate their longitudinal learning paths. MKSAP l9's
core content contains essential, newly researched information in 11 subspecialty areas of internal medicine created by dozens
of expert generalists and subspecialists. Development of MKSAP 19's syllabus and its 1200 all-new peer reviewed. psychomet
rically validated multiple choice questions (MCQs) has been informed by ABIM Certification and Maintenance of Certification
(MOC) requirements, emerging internal medicine knowledge, and our learners' feedback. MKSAP 19 continues to include
High Value Core (HVC) recommendations and MCQs, based on the concept of balancing clinical benefit with costs and harms.
Hospital-based internists can continue to trust that MKSAP's comprehensive hospitalist content, integrated throughout the syl
labus, and hospitalist focused MCQs, specially designated with the blue hospitalist icon (E), continue to align with the ABIM's
Focused Practice in Hospital Medicine MOC exam blueprint and enhance learning for hospital based practitioners.
More than ever before, MKSAP 19 Digital focuses on individualized learning and convenience. In addition to custom quizzes
and interlinked questions and syllabus sections, MKSAP 19 Digital's new learning dashboard enables users to create a self-
directed learning plan, with topic-specific links to resources within MKSAP and ACP Online. Multimedia formats, including
whiteboard animations and clinical videos, will benefit our audiovisual learners, while MKSAP's Earn-as-You-Go CME/MOC
feature now allows subscribers to earn CME/MOC as they answer individual questions. In addition to Extension Questions and
New Info Updates, MKSAP 19 Complete and Complete Green continue to offer Virtual Dx and Flashcards and non,offer brand
new enhancements: MKSAP Quick Qs, a set of concise questions mapped to high frequency/high-importance areas of the
ABIM blueprint mirroring boards-style MCQs, and an embedded digital version of Board Basics for easy-access exam prep.
Language can be imprecise and imperfect, but MKSAP 19's Editors and contributors commit to using language and images
that support ACP's commitment to being an anti racist organization that supports diversi$, equity, and inclusion through
out health care and health education. ACP also continues to ensure diversity among MKSAP's physician-contributors. When
appropriate, the MKSAP Editors also rely on MKSAP 19 Digital's expanded use of multimedia enhancements, including video
and audio, to explore and more fully explain issues surrounding the presentation of MKSAP 19 clinical content as it relates to
race and ethnicity. MKSAP 19 users are encouraged to contact the Editors at mksap_editors6racponline.org to help us identiff
opportunities for improvement in this area.
On behalf of the many internists and editorial staff who have helped us create our new edition, we are honored that y,ou have
chosen to use MKSAP 19 to meet your lifelong learning needs.
Sincerely,
ll
General lnternal Medicine 2
llr
Opting in for MOC found with the list of contributors' names and those of ACP
principal staff listed in the beginning of this book.
MKSAP 19 users can opt in for simultaneous submission of
CME and MOC points as they answer self assessment ques-
tions. To opt in, users will be required to complete a form Language Reflecting Diversity, Equily,
requesting their name, date of birth, and ABIM number. The and lnclusion Within MKSAP 19
MOC Opt-in Form will be presented during a user's first MKSAP 19's Editors and contributors commit to using
CME submission and needs to be completed only once. language and images that support ACP's commitmellt to
being an anti racist organization that supports diversify,
ABIM Maintenance of Certification equity, and inclusion throughout health care and health
Successful completion of the CME activiry which includes education. ACP also continues to ensure diversity among
participation in the evaluation component, enables the par MKSAP's physician contributors. When appropriate,
ticipant to earn up to 300 medical knowledge MOC points in the MKSAP Editors will also rely on MKSAP 19 Digital's
the ABIM's MOC program. It is the CME activity provider's expanded use of multimedia enhancements. including video
responsibility to submit participant completion information and audio, to explore and more fully explain issues sur-
to ACCME for the purpose of granting MOC credit. rounding the presentation of MKSAP 19 clinical content as it
relates to race and ethnicity. MKSAP 19 users are encouraged
to contact the Editors at mksap-editors6lacponline.org to
Disclosure Policy help us identi$z opporhrnities for improvement in this area.
It is the policy of the American College of Physicians (ACP)
to ensure balance, independence, objectivity, and scientific
rigor in all of its educational activities. To this end, and con
Hospital-Based Medicine
sistent with the policies of the ACP and the Accreditation For the convenience of subscribers who provide care itr
Council for Continuing Medical Education (ACCME), con hospital settings, comprehensive hospital focused content
tributors to all ACP continuing medical education activities aligned with the ABIM Focused Practice in tJospital Mt:dicine
are required to disclose all relevant financial relationships blueprint is integrated throughout the syllabus, and self
with any entity producing, marketing, re selling, or dis assessment questions that are specific to the hospital setting
tributing health care goods or services consumed by, or are specially designated with the blue hospitalist icon tlEl).
vl
I
by the ACP. The ACP does not warrant the safety, reliability, not set up MKSAP CME/MOC accounts for purchasers of
accuracy, completeness, or usefulness of and disclaims any MKSAP sold by unauthorized sellers (e.g., Amazon, eBay),
and all liability for damages and claims that may result from with whom ACP has no relationship. We do not honor
the use of information, publications, technologies, products, third party sales. CME credits and MOC points cannot be
and/or services discussed in this program. awarded to those purchasers who have purchased the pro
gram from non-authorized sellers.
Permission for Use of Figures Shown in
MKSAP 19 General lnternal Medicine 2 Unauthorized Use of This Book ls
Multiple-Choice Ouestions Against the Law
Figure shown in the critique of Self-Assessment Test Unauthorized reproduction of this publication is unlaw-
Item 37 is reprinted with permission from Walker HK, ful. ACP prohibits reproduction of this publication or any
Hall WD, Hurst JW, Editors. Clinical Methods: The of its parts in any form either for individual use or for
\
History, Physical, and Laboratory Examinations, third distribution.
:
t
L
Disclaimer Regarding Direct Purchases Errata and Revisions
I from Online Retailers Errata and Revisions for MKSAP 19 will be available
\ CME and/or MOC for MKSAP 19 is available only to custom- through MKSAP 19 Digital at mksaplg.acponline.org as
t
I ers who purchase the program directly from ACP. ACP will new information becomes known to the editors.
,
t
i
t
t
t
t ]
t
t
\
I
t
\
I
t
I
t
t
\
,
I
t
t
t
t
i
tI vll
t
L
t
:
Table of Contents
lx
1
..
x
Stevens-Johnson Syndrome and Nail Disorders . 115
Toxic Epidermal Necrolysis Infection. . tr6
Acute Generalized Exanthematous Pustulosis, Inflammatory Nail Disorders. . . ..tt7
Pruritus IngrownToenail .... . .lt7
Acneiform Eruptions . . Melanonychia . 118
Acne Vulgaris Squamous Cell Carcinoma. . . . . . 118
Rosacea. Benign Nodules and Tumors . 119
Hidradenitis Suppurativa Seborrheic Keratosis . tLg
Pigment Disorders Warts, Corns, and Skin Tags. . . . . 119
Vitiligo Dermatofibroma ... . t20
Nevi... . L20
Autoimmune Bullous Diseases DysplasticNevi ....... ....... . r2l
Pemphigus Vulgaris Premalignant and Malignant Tumors . t2t
Bullous Pemphigoid Actinic Keratosis . 127
Photo- or LightJnduced Dermatoses. . . . . Squamous Cell Carcinoma and
Porphyria Cutanea Tarda . Keratoacanthoma. . t22
Phototoxic and Photoallergic Conditions. . . . . Basal Cell Carcinoma t23
Polymorphous Light Eruption. Melanoma L24
Superficial Skin Infections. . . . . . Inflammatory Dermatoses 126
Bacterial Skin Infections. . . . . . Psoriasis 126
Dermatophytes .... Lichen Planus 126
Yeast .. Erythema Multiforme 127
Viral Skin Infections and Exanthems . . . . . . . . Erythema Nodosum t27
Ectoparasites Erythroderma ..... t28
Lice... SweetSyndrome... t28
Scabies
Bed Bugs. Bibliognphy t29
Burns .
Alopecia
lelf.AssesmentTest. 135
Nonscarring Localized and
Generalized Alopecia
Scarring Localized and Generalized Alopecia lndex 237
xt
I
1
I
1
I
1
I
1
I
I
General lnternal Medicine 2
High Value Care Recommendations
The American College of Physicians, in collaboration with r Preoperative ECG should not be performed in patients
multiple other organizations, is engaged in a worldwide undergoing low-risk procedures.
initiative to promote the practice of High Value Care o Patients with coronary artery disease should not undergo
(HVC). The goals of the HVC initiative are to improve routine coronary angiography or revascularization before
health care outcomes by providing care ofproven benefit surgery exclusively to reduce perioperative events.
and reducing costs by avoiding unnecessary and even r Chest radiography, arterial blood gas analysis, or spirom
harmful interventions. The initiative comprises several etry is not routinely indicated in the preoperative evalua
programs that integrate the important concept of health tion of patients with chronic lung disease (see Item 23).
care value (balancing clinical benefit with costs and o There is no benefit of incentive spirometry with or with
harms) for a given intervention into a broad range of out deep breathing exercises, in preventing postoperative
educational materials to address the needs of trainees. pulmonary complications (see Item 111).
practicing physicians, and patients. . Patients with untreated, asymptomatic mild hypothy
roidism may proceed to surgery without further testing
HVC content has been integrated into MKSAP 19 in sev
or treatment.
eral important ways. MKSAP 19 includes HVC-identified
r Pregnant patients should undergo the same preoperative
key points in the text, HVC-focused multiple-choice ques-
medical evaluation as nonpregnant patients; additional
tions, and, in MKSAP Digital, an HVC custom quiz. From
diagnostic testing is unnecessary.
the text and questions, we have generated the following
list of HVC recommendations that meet the definition
. There is little evidence that over-the-counter weight loss
supplements are effective.
below ofhigh value care and bring us closer to our goal
of improving patient outcomes while conserving finite
. Aspirin should be used infrequently in primary prevention
of atherosclerotic cardiovascular disease (see Item 77).
resources.
r Routine screening for abdominal aortic aneurysm (AAA)
High Value Care Recommendation: A recommendation to in women who have never smoked and who have no
choose diagnostic and management strategies for patients family history of AAA is not recommended (see Item 97).
in specific clinical situations that balance clinical benefit r Do not screen for coronary artery disease with either
with cost and harms with the goal of improving patient resting or exercise ECG in asymptomatic patients at low
outcomes. risk (see Item 22).
xltl
. with acute pharyngitis who present with fewer
Patients o Topical antihistamines for urticaria are not recom-
than three Centor criteria do not need to be tested or mended because they are ineffective and may lead to
treated for bacterial pharyngitis. allergic contact dermatitis.
o There is a generic equfualent in each class of topical r Pruritus may persist for weeks after eradication of scabies
glucocorticoids, allowing for a cost-effective approach. and does not require retreatment.
r Crmbined topical glucocorticoids and antifungal agents o Treatrnent of onychomycosis is unnecessary in most cases.
should be avoided because they can worsen some tinea o Warts are usually self-limited and treatnent is unnecessary
infections and may cause striae. o Perioperaflve glucocorticoid stress dosing may not be
. Inexpensive treatment of seborrheic dermatitis includes required in many patients receiving low-dose and short-
over-the-counter medications, such as selenium sulfide course glucocorticoid therapy (see Item 4).
or zinc pyrithione shampoos. o The choice of a topical broad-qpectrum antibiotic for bacte-
o Diagnostic evaluation for urticaria is not recommended rial conjunctivitis is usually empiric; the most c\cn/enient
unless the history suggests a specific cause. or least expenshrc option can be selected (see Item 17).
xtY
I
1
Routine Care of the Healthy Patient
2
Routine Care of the Healthy patient
and false negative results. population based screening and otherwise would not have become clinically apparent or
case-finding are often confused. Case-finding is a strategr of caused harm in the patient's lifetime, is an extreme example
targeted testing that relies on clinical judgment ofappropriate_
of length time bias. Overdiagnosis is an increasingly recog-
ness and potential benefit for a given patient that meets
nized harm ofbreast and prostate cancer screening and may
evidence-based criteria for inclusion in a high-risk group. The
also occur with incidental detection of thyroid and kidney
evidence that case finding improves morbidity or mortality is
cancers on imaging studies. Selection bias, also referred to as
often weak or absent.
volunteer bias, referral bias, or compliance bias, occurs
The effectiveness of screening tests in reducing mor-
when patients who undergo screening tests are healthier
bidity and mortality is evaluated through clinical trials;
and more interested in their health than nonadherent
however, studies of screening tests are problematic and
patients or the general population. Intention-to-treat analy
subject to three types of bias. Lead-time bias occurs when
ses, in which patients are analyzed according to their origi
early detection artificially results in an increase in meas-
nal group assignment in randomized clinical trials regardless
ured survival. The time between early detection and clinical
of intervention received. reduce selection bias.
diagnosis is mistakenly counted as survival time; however,
only the measured time with diagnosed disease, not sur-
vival time, has increased (Figure l). Using disease-specific
Screening Recommendations for Adults
mortality rates rather than survival time as the primary The USPSTF and many specialfy societies routinely aggregate
outcome in studies of screening tests can help minimize and review available evidence to inform clinical practice
lead time bias. Length-time bias occurs when screening guidelines for screening, counseling, and use of preventive
detects more cases of disease with a prolonged asympto medications. The USPSTF recommendations are available at
matic phase than cases of disease with a short asympto- www.uspreventiveservicestaskforce.org. The Emergency
matic phase. Slowly progressive disease is more likely than Care Research Institute (ECRI) is designated an Evidence
aggressive disease to be detected with screening, leading to based Practice Center by the Agency for Healthcare Research
an overestimation of survival benefit in those with screen- and Quality (AHRQ) and maintains a clinical practice guide-
detected disease. An example of length-time bias is the line repository at https://guidelines.ecri.org. The American
detection of indolent and slow growing prostate cancer College of Physicians (ACP) has developed several types of
rather than more aggressive tumors with a short asympto- clinical recommendations, including clinical practice guide-
matic phase, with a resultant overestimation of survival lines, clinical guidance statements, best practice advice, and
benefit. Overdiagnosis, or finding and treating illness that recommendations regarding high value care, all of which are
A. Natural history
Primary prevention Secondary prevention Tertiary prevention
Prevent disease Detect and treat Reduce
from developing asymptomatic disease complications
I
Death
I
Death
No disease Asymptomatic
Early detection
Disease course t
Death
longer. (Q Screening effectively detects disease during the asymptomatic phase, and survival time is lengthened.
3
Routine Care of the Healthy Patient
available at wnvw.acponline.org/clinical information/guide- TABLE 2. U.S. Preventive Services Task Force Grading
lines. Although there is much agreement among screening and Suggestions for Practice
recommendations, guidelines often disagree when (1) suffi Grade Definition Suggestions for
cient evidence is lacking and expert opinion plays a larger Practice
role or (2) potential benefits and harms both exist and the A The USPSTF recommends Offer or provide
the service. There is high this service.
balance depends on a person's risk, preferences, and values.
certainty that the net
An additional resource to help clinicians identifiz appropriate benefit is substantial.
screening tests and preventive services is Prevention TaskForce, an
B The USPSTF recommends Offer or provide
electronic tool that is available in Web and mobile application- the service. There is high this service.
based formats at w\iwvuspreventiveservicestaskforce.org/apps/. certainty that the net
With this tool, users can select USPSTF recommended practices benefit is moderate or
there is moderate
on the basis of patient age, sex, and other characteristics (such as certainty that the net
tobacco use or pregnancy). benefit is moderate to
Screening recommendations frequently change as sup substantial.
portive evidence emerges. It is important to be aware of C The USPSTF recommends Offer or provide
inrecommendations, reflect on the rationale and selectively offering or this service for
changes
providing this service to selected patients
implications of the changes, and incorporate these changes individual patients based depending on
appropriately into practice. on professional judgment individual
and patient preferences. circumstances.
There is at least moderate
Specific Screening Tests certainty that the net
benefit is small.
The following section describes screening recommendations
from the USPSTF and other organizations. The grading system D The USPSTF recommends Discourage the
against the service. There use of this service
of the USPSTF (A, B, C, or D, or I statement) is explained in is moderate or high
Table 2. certainty that the service
has no net benefit or that
the harms outweigh the
Screening for Chronic Diseases benefits.
Abdominal Aortic Aneurysm I statement The USPSTF concludes Read the Clinical
The USPSTF recommends one time abdominal ultrasonogra that the current evidence Considerations
is insufficient to assess the seaion of USPSTF
phy to screen for abdominal aortic aneurysm (AAA) in all men
balance of benefits and Recommendation
aged 65 to 75 years who have ever smoked (grade B). Ever harms of the service. Statement. lf the
smokers are commonly defined as persons who have smoked Evidence is lacking, of service is offered,
more than 100 cigarettes (five packs) in their lifetime. Surgical poor quality, or patients should
confliaing, and the understand the
repair is usually considered once the diameter exceeds 5.5 cm. balance of benefits and uncertainty about
The number needed to screen to prevent one death from AAA harms cannot be the balance of
in this population is 714. In men aged 65 to 75 years who have determined. benefits and
harms.
never smoked, selective screening is recommended (grade C),
USPSTF = U.S. Preventive Seruices Task Force.
especially in those with a first-degree relative with a history of
treated or ruptured AAA. The USPSTF assessed the benefits of Reproduced from U.S. Preventive Services Task Force. Procedure Manual.
www.uspreventiveseruicestaskforce-org/Page/Name/procedure manual. Accessed
screening for AAA in women aged 65 to 75 years who have ever June 17 .2021.
smoked as uncertain and makes no recommendation regard-
ing screening in this population (l statement). This likely
reflects the low prevalence of AAA in women (1'1,) versus men The USPSTF does not recommend screening for coronary
(4"/,, 7"/,,) as well as limited evidence to recommend against artery disease with either resting or exercise ECG in asympto
screening. The USPSTF specifically recommends against rou- matic patients at low risk, defined by the USPSTF as a l0 year
tine screening in women who have never smoked and who cardiovascular event risk of less than l0'/. using the Pooled
have no family history of AAA (grade D). Women, howevet Cohort Equations (grade D). In patients at intermediate or high
may have higher risk for rupture with AAAs of smaller sizes. risk for such events, evidence was inadequate to assess the rela-
tive benefits and harms of screening (l statement). Similarly,
Cardiouascular and Cerebrouascular Disease the ACP recommends against screening low-risk and asympto
Cardiovascular risk assessment is performed in asympto matic adults with resting ECG or stress testing. No specialty
matic adults to evaluate a patient's risk for future cardiac organization recommends screening these populations with
events; it does not identify preexisting disease and is there coronary calcium scoring or coronary angiography.
fore considered separate from screening (see MKSAP 19 The USPSTF also does not recommend screening for
Cardiovascular Medicine). carotid artery stenosis in the general adult population (grade D).
4
Routine Care of the Healthy Patient
!
5
Routine Care of the Healthy Patient
settings. Before treatment is initiated, when possible and in Risk assessment and screening fbr osteoporosis are fur
the absence ofsevere hyperlension or end organ damage, the ther discussed in MKSAP 19 Endocrinolory and Metabolism.
diagnosis should be confirmed with blood pressure measure
ments outside of the clinical setting, such as ambulatory or Thyroid Dysfunction
home blood pressure monitoring. The USPSTF concludes that evidence is insufficient to recom
The ACC/AHA recommend annual evaluation of patients mend for or against screening fbr thyroid dysfunction (l state
with normal blood pressure (<120/<80 mm Hg). Adults with ment). The American Thyroid Association and the American a
elevated blood pressure or stage t hyperlension (130 to 139/80 Association of Clinical Endocrinologists, hou,ever. recommend
to 89 mm Hg) who have an estimated 10 year ASCVD risk less measuring thyroid stimulating hormone in individuals at risk
than 10'7, should be managed with nonpharmacologic therapy for hypothyroidism (for example, personal history of autoim
and have a repeat blood pressure evaluation within 3 to mune disease. neck radiation, or thyroid surgery)r they addi
6 months. tionally recommend considering screening in adults aged
60 years or older.
Obesity
f,EY POIlIIS
The USPSTF no longer recommends screening for obesity but
does recommend referring or initiating weight loss interven
. The U.S. Preventive Services Task Force supports routine
screening for depression, hypertension. tobacco use,
tions for adults with a BMI of 30 or higher (grade B).The ACC,
and alcohol misuse in asymptomatic, a'verage risk
the AHA, and The Obesity Socieg recommend annual screen-
adults.
ing with BMI and waist circumference measurements.
o Adults aged 35 to 70 years who are overweight or obese
6
Routine Care of the Healthy Patient
TABLE 4. U.S. Preventive Services Task Force Recommendations on Screening for lnfectious Diseases
lnfectious 5creening Screening Test Populations at Risk
Disease Recommendation
Chlamydia and Screen all sexually active Nucleic acid amplification Persons with a history of STls, persons with new or
gonorrhea women aged <24 y and test multiple sexual partners, those who use condoms
women aged >24 y who are inconsistently, persons who exchange sex for money
at increased risk for infection or drugs
HIV Perform one-time Combination HIV antibody Men who have sex with men, active users of injection
screening for all adults immunoassay/p24 antigen drugs, persons who engage in risky behaviors
aged 1 5-65 y; repeat test (unprotected vaginal or anal intercourse; sexual
screening for adults at partners who are HIV infected, bisexual, or users of
increased risk injection drugs; transactional sex), persons with other
STls, persons who live and receive care in a high-
prevalence setting (HlV seroprevalence oI >1%)
HBV Screen all adults at high Hepatitis B surface antigen Persons born in countries with >2% prevalence of HBV
risk test; obtain antibodies infection, persons with HIV infection, users of injection
(anti-HBs, anti-HBc) to drugs, men who have sex with men, household
differentiate between contacts or sexual partners of persons with HBV
immunity and in{ection infection
HCV Perform one time Anti-HCV anti body test, Users of injection/intranasal drugs, persons who
screening in all patients followed by PCR viral load received a blood transfusion before 1992, persons
aged 1 B-79 y test if result is positive receiving long-term hemodialysis, prisoners, and
persons who received unregulated tattoos
Syphilis Screen all adults at VDRL or RPR test Persons with HIV infection, prisoners, men who have sex
increased risk with men, persons who exchange sex for money or drugs
Latent TB Screen populations at Tuberculin skin test or Persons born in or former residents of countries with
increased risk interferon-'y release assay high TB prevalence, close contacts o{ persons with
known or suspected TB, persons who live or work in
high-risk settings
reagrn; STI = se^ually transmited infection; TB = tuberculosis; VDRL = venereal disease research laboratory.
TABLE 5. Screening for lnfectious Diseases in High-Risk 13 to 64 years. Pregnancy is an indication fbr repeat screening
Populations fbr HIV infection. The USPS'l'F suggests repeat screening of'
Specific Populations Recommended Screenino persons at increased risk for HIV (such as men who have sex
with men and active injection drug users) but does not specily
Pregnant women Chlamydia and gonorrhea,
hepatitis B, HlV, syphilis, the interval for repeat screening. The CDC recommends at
asymptomatic bacteriuria least annual screening lbr r.nen who have sex with men but
Persons engaging in high-risk Hepatitis B and C, HIV notes that more frequent screening may be appropriate in
sexual behavior syphilis other populations. Combination IllV antibody immunoassay/
Men who have sex with men Chlamydia and gonorrhea, p24 antigen testing is recommended lbr screening. Diagnosis
hepatitis B, HlV, syphilis of HIV is discussed in MKSAP 19 lnlbctious Disease.
lnjection drug users Hepatitis B and C, HIV
I(EY POITT
Prisoners Hepatitis C, hepatitis B",
syphilis, tuberculosis
o The U.S. Preventive Services]ask Force recommends
that all persons aged 15 to 65 years receive one time
Persons receiving Hepatitis Bu and C,
hemodialysis tubercu losis HIV screening and persons 18 to 79 years receive one
Persons born in or living in Hepatitis B and tuberculosis
time hepatitis C screening regardless of risk.
countries with22"/" hepatitis B depending on prevalence of
virus prevalenceb disease
Health care workers Tuberculosis (baseline testing Screening for Substance Use Disorders
only); pre'employment 'l'he USPSTF recommends tl.rat cliniciar.rs ask all adults about
verification of immunity to
hepatitis B virus, measles, tobacco use, advise them to stop using tobacco, and provide
mumps, rubella, and varicella behavioral intervention and pharmacotherapy for tobacco cessa
7
:
drinking with brief behavioral counseling interventions to TABLE 6. Recommendations for Breast Cancer Screening
reduce alcohol misuse (grade B). Screening instruments to in Women at Average Risk
identify harmful drinking include the Alcohol Use Disorders Expert Group Recommendation
Identification Test (AUDIT), the AUDIT-Consumption American Cancer Age 40-44y: Provide women with the
(AUDIT-C), and single item screening. The AUDIT (https,// Society (2015) opportunity to begin annual screening
pubs.niaaa.nih.gov/publications/Audit.pdfl is a validated mammography
10 item screening test that takes approximately 2 to 3 minutes Age 45-54 y: Perform annual screening
to administerr the AUDIT C is a briefer (three item) version of mammography
the AUDIT. With single item screening, the clinician asks. Age >55 y: Perform biennial screening
"How many times in the past year have you had five [four for mammography with the opportunity to
continue annual screening
women] or more drinks in 1 day?" A positive test result,
Do not perform CBE for breast cancer
defined as any answer other than "zero," has a sensitivity and
screening
specificity of approximately 80'2, for unhealthy alcohol use.
American Age 25-39 y: May offer CBE every 1 -3 y
The USPSTF recommends asking adults aged 18 years or
College of
Age 40-49 y: Offer screening mammography
older about the use of illegal or nonprescribed psychoactive Obstetricians
and engage women in a shared decision-
drugs if the physician is also able to offer treatment or refer the and
making process; may offer annual CBE
Gynecologists
patient to treatment. Screening tools include the Drug Abuse (2017) Age 50-75 y: Perform annual or biennial
Screening Test (DAST 10) (wr,rrw.integration.samhsa.gov/ screening mammography based on a
clinical-practice/DAST 10.pdf) and the CAGE questionnaire shared decision-making process
expanded to include drugs (CAGE AID) (www.integration. Age >75 y: Engage women in a shared
samhsa.gov/images/res/CAGEAI D. pdf). decision-making process about
discontinuing screening
8
Routine Care of the Healthy Patient
9
Routine Care of the Healthy Patient
less frequent screening intervals (>Z years), which may reduce this age group because patients who have not yet undergone
overdiagnosis and the number of false-positive results while screening are the most likely to benefit (grade C). The USPSTF
preserving most ofthe screening benefit. The AUA also recom suggests that screening may be discontinued in patients older
mends that the interval for rescreening may be based on the than 85 years. In contrast, the ACP suggests discontinuing
baseline PSA level. Screening is not recommended for men screening in average risk patients older than 75 years or when
with life expectancy ofless than 10 to 15 years. the expected life expectancy is less than l0 years.
There is little head to head comparative evidence that
Colorectal Cancer any one recommended screening modality provides a greater
There is significant variation in colorectal cancer screening benefit than the others. In addition, despite unequivocal evi
guidelines. The ACP recommends screening fbr colorectal cancer dence that colon cancer screening reduces mortaliry an esti-
in asymptomatic adults aged 50 to 75 years (gfade A). In contrast, mated one in three U.S. adults who are eligible for colon
the USPSTF and the American Cancer Society (ACS) make a cancer screening has not been screened. Therefore. the
qualified recommendation to initiate screening for colorectal USPSTF supports using the test that is most likely to result in
cancer at age 45 years. The USPSTF also concludes with moder completion of screening. Understanding a patient's values and
ate certainty that screening for colorectal cancer in adults aged preferences and selecting a test to which the patient is most
45 to 49 years has moderate net benefit (Grade B). According to likely to adhere may improve screening rates. The ACP sug
the USPSTE, screening decisions in patients aged76 to 85 years gests a discussion with the patient and consideration of simi
should be individualized according to life expectancy and ability lar factors in selecting a screening stratery. Clinicians should
to tolerate treatment ofcolorectal cancer ifdiagnosed. Screening be familiar with the characteristics of each screening stratery
history should also be considered before screening patients in to facilitate effective discussion with patients (Table 9).
ACG = American College of Castroenterology; FIT = fecal immunochemical test; gFOBT = g!aiac fecal occult blood test; MSTF - U.S. Multi-society Task Force on Colorectal
Cancer
hal{ of the preparation on the day o{ colonoscopy, starting 4 to 5 hours before the procedure start and finishing 3 hours beforJthe procedure start.
Adapted from Davidson KW, Barry MJ, Mangione CM, et al; US Preventive Services Task Force- Screening for colorectal cancer: US preventive Services Task Force recommendation
statement. JAMA. 2O21;325:1965 1 977. IPM lD: 340032 1 8] doi:1 0.1 001 /)ama.2O21 .6238
10
Routine Care of the Healthy Patient
The ACP suggests colonoscopy every 10 years, flexible of prior screening cannot be confirmed should undergo
sigmoidoscopy every 10 years plus fecal immunochemical screening with cytologr every 3 years, high-risk HPV testing
testing (FIT) every 2 years, or FIT or high sensitivity guaiac every 5 years, or combined high risk HPV testing and cytologz
based fecal occult blood testing every 2 years as acceptable every 5 years until the criteria for adequate screening have
screening regimens. In contrast, the U.S. Multi Society Task been fulfilled.
Force on Colorectal Cancer (MSTF), an initiative of U.S. gastro Screening should not be performed in women who have
enterologi societies, has ranked colorectal cancer screening had hysterectomy with removal of the cervix unless a high-
tests in tiers based on the available evidence. cost effective grade precancerous lesion (cervical intraepithelial neoplasia 2
ness, test availability, and several other factors. The MSTF rec- or 3) was present. in which case screening should be contin
ommends colonoscopy every 10 years or annual FIT as first-tier ued for at least 20 years after hysterectomy.
tests; CT colonography every 5 years, FIT DNA testing every
3 years, or flexible sigmoidoscopy every 5 to 10 years as sec Lung Cancer
ond tier tests; and capsule colonography every 5 years as a Although lung cancer screening with chest radiography is not
third tier test. The serum circulating methylated SEPTg DNA recommended, annual screening with low-dose CT in patients
test is an FDA-approved screening strateS/. However, this test's who are heavy smokers results in a 2O% to 24o/,, reduction in
sensitivity for detecting colorectal cancer is only 4B%, and the lung cancer mortality. The USPSTF recommends lung cancer
MSTF does not recommend its use. screening with annual low dose CT for persons aged 50 years
to B0 years with a 20 pack year smoking history; screening
Ceruicql Cancer should be discontinued in former smokers who have not
The USPSTF recommends screening women aged 21 to smoked for 15 years (see MKSAP 19 Pulmonary and Critical
with cy.tologr (Pap test). In women aged
65 years every 3 years Care Medicine).
30 to 65 years who want to lengthen the screening interval,
high risk human papillomavirus (HPV) testing (preferred) or Additiona.l Cancer Screening Tests
cy.tologr combined with high risk HPV testing can be per- The USPSTF recommends using a brief familial risk assess-
formed every 5 years. The USPSTF recommends against ment tool, such as the Ontario Family History Assessment
screening women younger than 27 years regardless of sexual Tool, to screen lbr BRCAL/2 gene mutations risk in patients
history because screening has not been shown to reduce cervi- with a personal or family history of breast, ovarian, tubal, or
cal cancer incidence or mortalit5z compared with starting peritoneal cancer or who have an ancestry associated with the
screening at age 21 years. mutations. Genetic counseling and possibly genetic testing
In 2O2O, the ACS substantially shifted its recommenda should be provided to those identified to be at higher risk.
tions fbr cervical cancer screening to focus on HPV testing. The The USPSTF. the Society of Gynecologic Oncologr, and the
ACS recommends initiating screening at age 25 years with American College of Obstetricians and Gynecologists all rec-
HPV testing alone (primary HPV testing) every 5 years until ommend against screening for ovarian cancer with serum
age 65 years. When primary HPV testing is unavailable, CA 125 testing or ultrasonography in women at average risk.
patients in this age range should preferably be screened with Women with a family history indicating a possible hereditary
HPV cotesting (HPV testing with cervical c1,tology) every cancer syndrome should be referred to a genetic counselor for
5 years, or alternatively with cytologz alone every 3 years. consideration of genetic testing (see MKSAP 19 Oncolo5r).
t Currently, there are two FDA-approved primary HPV tests and According to the USPSTE, evidence is insufficient to deter
five FDA approved HPV tests for cotesting. mine the balance of benefits and harms of screening for skin
Screening can be discontinued at age 65 years in non cancer with a visual skin examination (l statement). However,
high-risk women with adequate prior screening, commonly the USPSTF recommends that persons younger than 24 years
defined as three consecutive negative cytologz results or two who have fair skin receive counseling to minimize exposure to
consecutive negative cytologi plus HPV test results within the ultraviolet radiation to reduce risk for skin cancer (grade B)
past 10 years, with the most recent test occurring within and recommends offering selective counseling to adults older
5 years. In women older than 65 years with life expectancy of than age 24 years with fair skin types (grade C).
at least 10 years and risk factors for cervical cancer (history of The USPSTF recommends against screening for pancre
abnormal Pap smears, history of a high-grade precancerous atic cancer in asymptomatic adults (grade D). Because up to
Iesion, in utero exposure to diethylstilbestrol, immunocom- 15'/n of pancreatic ductal adenocarcinomas are attributable to
promise, previous HPV infection), continued screening should genetic factors, patients with a family history suggestive of a
be considered. genetic syndrome associated with pancreatic cancer (BRCA1/2
Women who have never been screened have the highest mutations, Peutz Jeghers syndrome, Lynch syndrome) should
incidence of and mortality from cervical cancer. The mortality be referred for genetic counseling and possible genetic
reduction from screening in women who have not been previ testing.
ously screened may be as high as 74%. Women older than Evidence is insufflcient to recommend routine anal can
65 years who have never been screened or in whom adequacy cer screening in average risk populations, but such screening
11
Routine Care of the Healthy Patient
may be considered in high risk populations. The Infectious TABLE 10. Live Vaccines and Contraindications to
Diseases Society of America suggests screening patients with Administration
genital warts, men who have sex with men, and women who Live Vaccines
have a history of abnormal cervical cytologr or participate in Live attenuated influenza vaccine (LAIV)
receptive anal intercourse.
Measles, mumps, rubella vaccine (MMR)
The USPSTF recommends against screening for testicular
Zoster vaccine, live (ZVL)
cancer and thyroid cancer in asymptomatic adults (grade D)'
Routine screening for bladder cancer is not recommended by Varicella vaccine (VAR)
any expert group. Contraindications to Live Vaccines
. All women aged 50 to 74 years should undergo screening HIV with CD4 cell count <200 cells/gL or CD4 cells <15% of
total lymphocytesu
mammography; the recommended screening interval
lmmunosuppressant therapy, including high-dose
varies by expert group.
glucocorticoids (>20 mg/d of prednisone or equivalent {or >2 wk)
. The ACP recommends screening for colorectal cancer in Leukemia, lymphoma, or other bone marrow and lymphatic
asymptomatic adults aged 50 to 75 years (grade A); in system malignancies
contrast, the USPSTF and the American Cancer Society I Cellular immunodeficiency
(ACS) make a qualified recommendation to initiate
Solid-organ transplant recipient
screening for colorectal cancer at age 45 years.
Recent hematopoietic stem cell transplantation
o The choice of modality for colorectal cancer screening
"Live attenuated influenza vaccine is contraindicated in patients with HIV infection
should be based on discussion with patients, including
consideration of likelihood of adherence and patient [':"1':: "'i"1:"""'": ._
12
Routine Care of the Healthy Patient
Unvaccinated adults should receive a three-dose series decision making in patients with HIV infection and a CD,l cell
consisting of Td or Tdap, with at least one Tdap dose (pref'era count of 200 cells/trrL or greater.
bly as the first dose). Adults who have received fewer than All adults aged 50 years or older, including those with a
three doses of the primary series should complete the series previous episode of zoster, should receive the recombinant
with the Td or Tdap vaccine, while ensuring that at least one (inactivated) herpes zoster vaccine to reduce the incidence o1'
dose ofthe Tdap vaccine is received. zoster and postherpetic neuralgia. The recombinant vaccine is
Pregnant women should receive at least one dose of the administered intramuscularly in two doses, with an interval of
Tdap vaccine between 27 and 36 weeks' gestation with every 2 to 6 months between doses. Vaccination is recommended
pregnancy. regardless of previous zoster inf'ection or vaccination with the
live vaccine. The safety of the vaccines in pregnant women has
Vaccinations Recommended for Some Adults not been determined. and the ACIP recommends delaying
Varicella and Herpes Zoster immunization in pregnant women. Use of the lir,e attenuated
All immunocompetent adults without evidence of varicella vaccine in individuals with HIV infection and a CD,l count less
immunity should receive two varicella vaccine doses. Evidence than 200 pr L is under revicw.
of varicella immunity includes laboratory confirmed disease
or immunity, diagnosis or verificaticln of varicella or zoster by Pneumococcal Disease
a provider, or documentation of age appropriate varicella vac Pneumococcal vaccination is recommended in all adults aged
cination. U.S. birth before 1980 is also considered to be evi with certain
65 years or older and in adults aged 19 to 64 years
dence of immuniry except in pregnant women and immuno high risk conditions (Table 1l). Two pneumococcal vaccines
compromised persons (who are at risk fbr severe disease) and are available: the 13 valent conjugate vaccine (PCV13) and the
health care workers (who are at risk for repeated varicella 23 valent polysaccharide vaccine (PPSV23).
exposure and spreading the disease to those at high risk fbr All adults aged 65 years or older who have not previously
severe disease). These patient groups must meet the other been vaccinated should receive PPSV23. In 2019. ACIP recom
criteria for varicella immunity. Although varicella vaccination mended shared decision making regarding the use ol PCVlll
is contraindicated in patients with severe immunosuppres lbr healthy adults aged 65 years or older. lf PCV13 is given, it
sion, it can be considered on the basis of shared clinical should be administered at age 65 years, fbllowed one year later
PCVI 3 PPSV23
Risk Group UnderlyingMedicalCondition Recommended Recommended Revaccinationat
5 Years After First Dose
lmmunocompetent persons Chronic heart disease" X
Diabetes mellitus X
CSF leaks X X
Cochlear implants X X
Alcoholism X
Cigarette smoking X
lmmunocompromised persons' X X X
CSF = cereb,rospinal fluid; PCVl 3 = 1 3 valent pneumococcal conjugate vaccine; PPSV23 23 va ent pneumococcal polysaccharide vacc ne
and phagocytii disorders Iexcluding chronic granulomatous disease]); HIV nfectron; chron c kidney failure; nephrotic syndrome; leukemia; lymphoma; Hodgkin lymphoma;
13
Routine Care of the Healthy Patient
by PPSV23. This recommendation does not change the recom immunity should be vaccinated. Pregnant women who lack
nrendations regarding use of PPSV23 and PCV13 in high-risk immunity should be vaccinated at the time of delivery before
patients aged 19 to 64 years. Among high risk patients who leaving the hospital or at the tinle of pregnancy termination.
require vaccination with both PCVI3 and PPSV23 but who The MMR vaccine is a live virus vaccine and should not be
have not yet received either vaccine, a single dose <lf PCV13 administered to immunocompromised individuals or preg-
should be given first, followed by a single dose of PPSV23 given n:lnt women.
at least B weeks later. Adults aged 19 to 64 years with high risk
cc.rnditions who require vaccinatiol.t with both PCV13 and Meningococcal Disease
PPSV23 and who have already received PPSV23 should be Meningococcal vaccines used in the adult population
adn.rinistered a single dose of PCV13 no sooner than 1 year af ter include the quadrivalent meningococcal conjugate vaccine
receiving the most recent PPSV23. A second dose of PPSV23 (MenACWY). which protects against serogroups A' C' W135.
shriuld also be administered 5 years after the first PPSV23 dose and Y. and the meningococcal group B (MenB) vaccine'
in adults aged 19 to 64 years with certain immunocomprolllis which protects against serogroup B disease' MenACWY is
ing conditions (see Table 11). recommended in adolescents aged 11 to 18 years' If the
patient was vaccinated at age 1l to 12 years (preferred), a
Human Papillomavirus booster dose is administered at age 16 lears; if the initial
H PV vaccination prevents persistent FIPV infection, which can dose was administered at age 13 to 15 years, a booster dose
lead to cervical, anogenital, and nasopharyngeal cancers. A is administered at 16 to 18 years. A booster dose may be
nine valent HPV vaccine is available and approved for use in administered to persons aged 19 to 2l years who did not
both f'emales and males. Patients should be administered the receive a dose after age 16 years. ll the first dose is adminis
vaccine series at age 11 or 12 years or between the ages of13 and tered after age 16 years, a booster dose is not required. First
26 years ifnot given previously. In unvaccinated patients aged year college students living ir.r residence halls should receive
27 lo 45 years, vaccination can be cor.tsidered on the basis of at least one dose of MenACWY within 5 years before college
risk, using a shared decision making process. If administered entry. MenACWY vaccines are interchangeable. Routine
befbre the age of 15 years, a two dose series is recommended. two dose series of MenB can be administered at age 16 to
where:rs a three-dose series is recommended in older indi. 23 years on basis of shared clinical decision making. A
viduals. Vaccination is not recommended during pregnancy, booster dose is not recommended unless the patient devel
although no harmful eff'ects have been noted when inadver ops increased risk for meningococcal disease. MenB vac'
tently given to pregnant women and pregnancy testing is not cines are not interchangeable and dosing schedules differ.
lrecessilry befbre vaccination. Indications for meningococcal vaccination in adults at
increased risk are summarized in Table 12.
Measles, Mumps, and Rubella
All U.S. adults born before 1957 are considered immune tcr Hepatitis A
measles and mumps. Adults born in 1957 or later without Vaccination against hepatitis A is recommended for all persons :
documented evidence of receiving one or more doses of the who desire vaccination and fbr persons who are at increased
measles, mumps, and rubella (MMR) vaccine or laboratory risk lbr infection or complications of infection, including :
confirmed immunity against all tl-rree diseases should receive pregnant women. Persons at increased risk include those who
at least one MMR dose. A second MMR dose should be admir.r work in or travel to endemic areas. men who have sex with
istered to postsecondary students. household or close personal men. individuals with chronic liver disease, individuals with
contacts of immunocompromised persons with no evidence of l'llV infection, users of injection or noninjection illicit drugs,
imnlunity. and international travelers. For persons who have homeless persons. persons who conduct hepatitis A related
been previously vaccinated with two doses of a mumps virus research, household or close contacts of children adopted
containing vaccine but are :rt increased risk because of an fiom endemic areas, and those who work in settings of possi I
outbreak, the ACIP recommends administering a third dose of ble exposure (group homes, nonresidential day care facilities
munlps virus containing vaccine to improve protection. fbr developmentally disabled persons, and health care pro- :
[]ealth care workers should be considered separately. For grams serving users of injection or noninjection drugs).
those who',tere born in or after 1957 and who lack evidence of Immunization can be provided as a two dose series of single
immunity to measles, mumps, or rubella, a two dose MMR antigen hepatitis A vaccine or a three dose series of combina-
i
:
series, with at least 4 weeks between doses, should be admir.r tion hepdtitis A and B vuccine.
istered fbr measles or mumps intmunity, whereas at least a
sir.rgle dose should be administered fbr rubella immunity. The Hepatitis B
srrme irdministration schedule should be considered in health Hepatitis B vaccination is recomnrended for any nonimmune
care workers born before 1957. adult who desires vaccination or who is considered to be at high :
Ir.r women of childbearing age, it is necessary to deter risk for infection (Table 13). The typical hepatitis B vaccination :
mir.re rubella immunity. Nonpregnant women who lack series is a three dose series, with doses administered at 0, l, and
14
Routine Care of the Healthy Patient
TABLE 12 lndications for Meningococcal Vaccination in Adults with lncreased Risk for Meningococcal Disease
Population MenACWY MenB
Persistent complement component Primary vaccination followed by booster Primary vaccination followed by booster
deficiencies (C5-C9, faaor H, factor D, every 5 y 1 y later and every 2-3 y thereafter
properdin) or persons taking complement
inhibitors (e.9, eculizumab or ravulizumab)
Functional or anatomic asplenia (including Primary vaccination followed by booster Primary vaccination followed by booster
sickle cell disease) every 5 y 1 y later and every 2-3 y thereafter
Exposure to vaccine-preventable Single dose if >5 y since vaccination Primary vaccination; booster if >1 y since
serogroup meningococcal disease primary series
outbreak
Travel or residence in country with Primary vaccination followed by booster Same as general population
hyperendemic or epidemic every 5 y
meningococcal disease
College freshmen living in residence halls Primary vaccination Same as general population
Military recruits Primary vaccination followed by booster Same as general population
every 5 y (depending on military
assignment)
AdaptedfromMbaeyiSA,BozioCH,DuffyJ,etal Meningococcalvaccination:recommendationso{theAdvisoryCommitteeonlmmunizationPractices,UnitedStates,2020.
MMWR Recomm Rep. 2020;69:1 4 1 . IPMID: 3341 7592] doi:
1 0.1 5585/mmwr.rr6909a 1
L relationship (any person with more than one sexual partner Checking serum antibodies is not typically recommended
within the past 6 mo) after routine vaccination but is indicated in persons in whom
t Sexual partners of persons who are HBsAg positive subsequent clinical management is dependent on knowledge
I
L Men who have sex with men of serologic response (e.g., patients on chronic hemodialysis,
persons with HIV health care and public saf'ety workers, and
I Household contacts of persons who are HBsAg positive
needle sharing partners of persons who are positive for hepa
Residents and staff members of institutions for persons who
I
titis B surface antigen).
t are developmentally disabled
I
t
Persons who are current or recent users of injection drugs
Health care and public safety workers with anticipated risk for Vaccinations Recommended for
I
I exposure Specific Populations
Persons with end-stage kidney disease, including those Health care workers are at increased risk fbr acquiring and
L receiving hemodialysis and peritoneal dialysis transmitting hepatitis B, influenza, measles, mumps,
I
L lnternational travelers to regions with intermediate or high rubella, pertussis, and varicella. All health care workers,
I levels of endemic hepatitis B infection regardless ol patient contact, should receive the influenza
t Persons with chronic liver disease vaccine annually. Health care workers without immunity
i
L Persons with HIV infection should be vaccinated against hepatitis B; measles, mumps,
I
Pregnant patients with risk for infeaion or serious outcome and rubella; and varicella. In addition, all health care work
L from infection. ers who have not previously received the Tdap vaccine
I
I Persons with diabetes mellitus who are aged <60 y should receive one dose, irrespective of when they last
)60 the basis received the Td vaccine.
Persons with diabetes mellitus who are aged y, on
t
I
of shared clinical decision making Patients with anatomic or functional asplenia are at
!
I
lncarcerated persons increased risk for infection from encapsulated organisms.
t such as Haemophilus inJ'luenzae type B, meningococcus, and
i HBsAg = hepatitis B surface antrgen.
L pneumococcus, and should be appropriately vaccinated.
I
i 15
Routine Care of the Healthy Patient
Pirtients who have undergone hematopoietic stem cell prevention of ASCVD because of lack of net benefit; sPecifi
transplantation should receive lhe Haemophilus influenzae cally. low dose aspirin may be considered for primary preven
type B vaccine. ti<-rn of ASCVD in adults aged 40 to 70 years who are at higher
Vaccination recommendations tbr international travelers ASCVD risk but not at increased bleeding risk. Similarly, the
vary depending on the destination. 1'rip specific recommen ADA reaffirmed its recommendations in 2019 that low-dose
dations from the CDC can be accessed at wr,vwnc.cdc.gov/ aspirin may be considered in patients with diabetes who are at
travel. For more information on vaccination in travelers, see i ncreased cardiovascular risk.
MKSAP 19 Infectious Disease. The decision to initiate low dose aspirin should be
infbrmed by the totality of ASCVD risk and bleeding risk as
l(EY P0lilTS
well as patient values and pref'erences. Factors that increase
. Annual influenza vaccination is recommended for all bleeding risk include concurrent anticoagulant or NSAID use,
individuals aged 6 months or older. history of gastrointestinal ulcer, upper gastrointestinal pain,
. Pregnant women should receive at least one dose of the uncontrolled hypertension, male sex, and increasing age. After
tetanus toxoid, reduced diphtheria toxoid, and acellular age 70 years, the benefit of aspirin decreases and bleeding
pertussis vaccine between 27 and 36 weeks' gestation risks are higher; thus, neither the ACC/AHA nor the ADA rec
with every pregnancy. ommends aspirin for primary prevention in this age group.
. All adults aged 50 years or older should receive the There is some evidence for the use of aspirin in cancer
recombinant (inactivated) herpes zoster vaccine, prevention. lvith one trial showing a 34'X, to 407, reduction in
regardless of previous immunization or clinical infec crilorectal cancer mortality with at least 5 to 10 years of aspirin
tion, to reduce the incidence ofzoster and postherpetic therapy. The benefit is not apparent until 10 to 20 years aFter
neuralgia. initiation of aspirinr no mortality benefit was observed in the
o first 10 years offollow up.
Pneumococcal vaccination is recommended in all adults
Prophylactic aspirin use is :rlso recommended for preg
aged os years or older and adults aged 19 to 64 years with
nant women at high risk for preeclampsia and should be
certain high risk conditions.
considered in those at moderate risk (see Women's Health).
r Patients with anatomic or functional asplenia should be
vaccinated against Haemophilus influenzae type B,
meningococcal, and pneumococcal diseases. Healthy Lifestyle Counseling
tlealthy lif'estyle counseling is directed at the leading prevent
able causes of death. Tobacco and obesity contribute signiti
Aspirin for Primary Prevention cantly to the top two leading causes ofdeath. heart disease and
Because of its low cost and eflicacy ir.r preventing cardiovas cancer, which account for almost 50'){, ol all deaths in the
cullr and cerebrovascular events, aspirin can be considered United States. Other preventable causes of death include alco
fbr primary prevention of vascular disease. In 2016, the hol and drug use, infectious diseases, toxins, accidents, and
USPSTF recommended low dose aspirin tbr the primary pre firearms.
vention of ASCVD and colorectal cancer in willing adults aged Tobacco cessation is a high priority intervention fiom a
50 to 59 years with a 10 year ASCVD risk of 10')(, or higher health and cost effectiveness standpoint. Smokers should be
(using the Pooled Cohort Equation risk calculator). a life ofl'ered pharmacologic therapy and behavioral interv'entions.
expectancy of at least 10 years. ancl no increased risk for For more information. see MKSAP 19 General Internal
bleeding (grade B). The USPSTF suggests individualizing the Medicine 1.
decision in adults aged 60 to 69 years with a 10 year ASCVD All patients who are overweight or obese should undergo
risk o1 10'1, or higher (grade C). counseling on the benefits of a healthy weight, regular exer
In 2018, however, three large trials, ASCEND, ARRIVE, cise, and a healthy diet (see Behavioral Counseling).
ancl ASPREE, showed that primary prevention with aspirin led lnjury prevention, including seat belt use, use of safety
to no reduction in nonfatal myocardial infarction or mortality. helmets lbr motorcycles and bicycles, and home safety meas
In 2019, a meta analysis of the use of aspirin for primary pre ures, should be emphasized. Patients should be counseled to
ventiolr was updated; it included nrore than 164,000 patients use smoke alarms in the home and to set water heaters to
and showed no benefit in any mortality end point. Reductions lower than 49 "C (l2o "F) in households with infants or young
ir.r nonfatal myocardial inlarction (relative risk [RR]. 0.82; children (see Table 1).
O.72 O.94) and ischemic stroke (RR. O.B7: O.79 0.95) were Even small changes at the population level, such as
ofliet by a significant increase in bleeding events. including improving social determinants of health, can create large pre
bleeding (RR. 1.5; 1.33 1.69). intracranial bleeding (RR,
r.r'rajor ventive benefits. Healthy People 2020 is a CDC initiative with
l|\2:1.12 1.55), and major gastrointestinal bleeding (RR, 1.52; the goal of creating social and physical environments that
1.34 1.73). Subsequently, the ACC/AHA recommended that promote good health, and Healthy People 2030 is currently in
aspirir-r sl-rould be used inftequently in the routine primary development. Using strategies ftorr-r bel-ravioral econor-r-r ics,
16
Routine Care of the Healthy Patient
17
Routine Care of the Healthy Patient
maintain a healthy weight and adopt healtiry practices, includ- the risk for ASCVD and type 2 diabetes. However, artificially
ing eliminating sugar-containing beverages, implementing a sweetened beverages may be a useful replacement for sugar
reduced-calorie diet, avoiding processed foods, and practicing containing beverages as a transition to drinking water (plain.
mindful eating, can be beneficial (see Nutrition and Physical carbonated. or with unsweetened flavor).
Activity). Providers may also help patients by modeling Adults of legal drinking age should be advised to limit
healthy behaviors; evidence shows that doctors who improve consumption of alcohol to no more than one drink per day for
their health habits may be better able to counsel their patients women and two drinks per day for men.
regarding preventive and healthful behaviors.
Physical Activity
Nutrition lncreased physical activity decreases risk for most chronic
Plant-based and Mediterranean diets as well as consumption diseases and cancer. The U.S. Department of Health and
of fruits, vegetables, legumes, and vegetable- and lean-animal Human Services 2018 Physical Activity Guidelines for
protein (mostly fish) are consistently associated with lower Americans recommend that adults perlorm at least 150 to
all cause mortality than other types of diets and should be the 300 minutes of moderate intensity or 75 minutes of vigorous
primary components of dietary intake. Dietary guidelines intensity aerobic activity per week. ideally spread throughout
from both the U.S. Department of Health and Human Services the week. Muscle strengthening activities should be per
and the ACC/AHA recommend following a healthy eating pat formed at least 2 days per week. Physical activity does not have
tern that consists of a variety of vegetables, whole fruits, leg to take the form of formal exercise or be performed for lengthy
umes, nuts, whole grains, and fish while minimizing intake of intervals. Any physical activity is better than none. and adults
added sugars, saturated and trans fats, sodium, and refined should move more and sit less. Patients should be encouraged
carbohydrates. Common nutrient deficiencies and associated to discover ways to naturally and habitually incorporate physi
conditions are listed in Table 15. cal activity into their daily life.
In the Adventist Health Study 2 cohort, consuming meat
as a primary source of protein increased mortality by 61'7, with
a corresponding 40% reduction in mortality in those who
Supplements and Herbal Therapies
replaced meat with nuts and seeds as their protein source. Dietary supplements, including vitamins, minerals. botani
Similarly, foods that include trans and saturated fats have con cals, herbals, metabolites, and amino acids, are categorized as
sistently been shown to be harmful and increase mortality, foods by the FDA. Therefore, manufacturers are not required
and highly processed foods increase the risk for ASCVD and to demonstrate efficacy or safety of their products unless the
type 2 diabetes. supplement includes ingredients that were introduced alter
Water should be recommended as the drink of choice. 1994. Manufacturers are not allowed to make specific medical
Sugar-containing and artifi cially sweetened beverages increase claims: however, the product's intended elfect on body struc
ture or function may be described.
TABLE 15. Common Nutrient Deficiencies Patients take dietary supplements lor various reasons,
such as to prevent and treat illness, manage symptoms, and
Nutrient Associated Disease/Disorder and
Deficiency Manifestations enhance health. In the United States, approximately 50')(, ol
adults report using vitamins or dietary supplements. The
Vitamin D Osteoporosis, fractures, weakness
USPSTF recommends against the use of p-carotene or vitamin
Calcium Osteoporosis, fractures, weakness
E supplements for the prevention ofcardiovascular disease or
Vitamin C Scurvy: perifollicular hemorrhage, poor cancer and has concluded that the current evidence is insuf-
wound healing, gingivitis
ficient to assess the use of multivitamins for the prevention of
lron Anemia cardiovascular disease or cancer.
Folate Anemia In general, there is little good quality evidence showing
Vitamin B,2 Anemia, mental status and psychiatric the efficacy of dietary supplementation. and use carries the
changes, subacute combined degeneration of potential for harm. There are several populations. however. lor
the dorsal and lateral spinal cord column
which vitamin or supplement use is recommended. Women of
$rridoxine Skin problems, neuropathy childbearing age are advised to take 0.4 to 0.8 mg (400 800 pg)
(vitamin Bu)
of folic acid daily to prevent fetal neural tube defects. Vegans
Thiamine Beriberi, Wernicke-Korsakoff syndrome:
(vitamin B,)
and older adults may consider vitamin B1 , supplementation to
peripheral neuropathy, heart failure,
nystagmus, ataxia, ophthalmoplegia, address dietary deficiency. Calcium and/or vitamin D may also
encephalopathy be considered for older adults or individuals with suboptimal
Vitamin K Coagulopathy, elevated INR bone health. Calcium supplementation may increase the risk
r
Vitamin A Dry eyes, dry skin, night blindness for cardiovascular disease and kidney stones, thus eating a
calcium rich diet may be a preferable means of assuring ade
Dietary fiber lntestinal problems
quate intake. Persons with age related macular degeneration
18
Patient Safety and Ouality lmprovement
19
Patient Safety and Ouality lmprovement
Ginkgo biloba Treatment and prevention Not effective Headache, Gl upset, Mayincrease riskfor
of cognitive decline allergic skin reactions, bleeding when used whh :
20 t
l
\
Patient Safety and Ouality lmprovement
CYP2C9 = cytochrome P450 2C9; CYP3A4 = cytochrome P450 3A4; Gl = gastrointestinal; MAOI = monoamine oxidase inhibitor.
21
Patient Safety and Ouality lmprovement
patient taking fewer pills. However. it may require clinician For more information. see Hospital Medicine Principles'
time for making changes and combination pills may be more f,EY POIXTS
expensive. Consultation rn'ith tr clir.rical pharmacist is a team
based approach that facilitates disease co management, allow
. To prevent medication errors, physicians must review a
patient's complete medical record, including medical
ing physicians to utilize their time perfbrming other tasks.
history; medication list, including prescription and
This method may be costly and may not be available at all
over the counter medications, herbal remedies, and
institutions. Utilizing cost incentives (such as selecting medi
supplements; allergies; and laboratory data that may
cations with lowest copays or available discounts) can be time
affect medication dosing.
intensive and may require clinicians to maintain knowledge of
shifting copays among a large number ol health insurance o The most widely used method for handoff communica-
companies. tion during transition of care is the SBAR (Situation.
Patient education (online resources, handouts) is low Background, Assessment, and Recommendation)
cost and widely available and usually requires the patient to technique.
make a time investment to become educated. Cognitive behav
ioral therapy in the form of selfr management strategies and
motivational interviewing to promote adherence can be usedr Patient Safety and Ouality lssues
when performed by a psychologist. clinicians can share
responsibility of patient management. However, cognitive
at the Systems Level
behavioral therapy may not be available for all patients, Ouality lmprovement Models
requires a patient's time investment. and has associated costs There are multiple QI models that provide structure for imple
fbr use of a specialist. Patient reminders, such as text or voice menting QI interventions on a systems level. The American
alerts, cirn be programmed into smart phones or other techno- College of Physicians (ACP) Advance curriculum provides a
logical devices, but not allpatier.rts may have access to technol useful tool set and additional resources for implementing
ogr and personal and interactive reminders may be more QI nterventions
i (r,r,r,rrw.acpon Iine. org r practice - resources
ef fective. Pill boxes may also be used. acp quality improvement/acp advanceiquality improvement
curriculum). The Plan Do Study Act (PDSA) cycle is a fre
Transitions of Care quently used method in each of the models described in this
When patients transition care between providers or care envi section and provides structure fbr continuous changes.
ronments, they are at increased risk for harm. Handoffs
between clinicians and teams are integral to this process and Model for Improvement
are delined as the transf'er and acceptirnce of responsibility of The Model for Improvement is the most commonly used QI
patient care between clinicians through effective communica model in health care. lt provides a simple framework for QI
tion. Factors that contribute to suboptimal transitions include interventions that relies on rapid PDSA cycles. A key underly
poor communication between health care team members, ing theme is that although not every change leads to improve
incomplete transler of information, and inadequate patient ment, improvement cannot occur without change. This model
education. Patient factors that increase likelihood ofsubopti works best when frontline health care staff,members are actile
mal transition of care include older age, multiple medical participants. Users of this model begin by developing a goal by
comorbidities, poor health literacy. language barriers, and asking, "What are we trying to accomplish?" Next. to deter
cognitive impairment. mine how to track success. users must anst ler, "Hort'u'ill rte
Excellent communication among health care team mem nleasure if a change yields an impro'"ement?" The final step in
bers and between providers and patients is the key to success this model is to determine rthat change is going to be made b1'
ful transition of care. The n.rost widely used method fbr asking the question, "What change can we make that rvill
handoff communication is the SBAR (Situation, Background, result in improvement?" Because this QI model does not help
Assessment, and Recommendation) technique (see Table 18). to identify causes of health care problems. it works best for
Transitions are also enhanced by physician involvement in a problems with easily identifiable causes and evidence based
multidisciplinary team utilizing a continuous, case based solutions. For example, a medical center may set the specific
multidisciplinary team management approach. During transi goal of increasing adherence to stroke treatment guidelines
tions, physicians should strive to provide clear, concise, and and use PDSA cycles to rapidly implement and assess the
accurate patient records to subsequent providers. Accurate inlpact of changes. such as using a computerized admission
medication reconciliation. although tinte consuming. is para order set.
mount because errors frequently occur: several studies have
shown medication reconciliation is best performed by a phar Lean
macist. Upon hospital discharge, physicians must focus on The Lean model focuses on improving health care processes
follow up plans, including educating patients on the expected through two principles: waste elimination and respect for
course and red Ilag symptoms that necessitate reevaluation. individuals. ln this model, activities are classified as being
22
Patient Safety and Ouality lmprovement
23
Patient Safety and Ouality lmprovement
for clinicians and medical educators, clinical guidelines, best CDS is the use of information technolory to facilitate
practice advice, and patient resources on a wide variety of clinical decision making. When integrated into a CPOE system,
related topics (r,tr,tr,v.acponline.orgiclinical information/high CDS can highlight potential contraindications to diagnostic
value care). Components of the High Value Care initiative that tests, specilz dose recommendations, identiff potential drug
are evident in MKSAP 19 include the identification of High interactions, and suggest modifications to drug dosages in
Value Care key points in the text and a list ofhigh value care patients with kidney or liver dysfunction. CDS within the EHR
recommendations assembled for each MKSAP section. can also promote protocols to improve care and provide ready
access to clinical guidelines.
Choosing Wisely Limitations of HIT include the expense associated with
The Choosing Wisely initiative was developed by the American system implementation and maintenance as well as concerns
Board of Internal Medicine Foundation in collaboration with related to protection of patient privacy. Although useful for
Consumer Reports to encourage discussions between clini preventing many Spes of errors, HIT does not provide a lail
cians and patients on selecting tests, treatments, and proce- safe against errors and may facilitate errors itself, such as those
dures that are evidence-based and truly necessary thereby resulting from charting templates and use of the copy-and-
avoiding unnecessary care. More than 80 specialist orgaliza paste function in composing notes. Alert fatigue, wherein cli-
tions have participated to create lists of overused tests and nicians are subjected to so many alerts they begin to ignore all
treatments in their specialties (wwwchoosingwisely.org/ of them. is a substantial drawback to CDS and can decrease the
clinician-lists), and Consumer Reporfs has generated patient effectiveness of alert systems.
education materials based on these lists to engage and Useful resources for incorporation of HIT in practice are
empower patients to participate in care discussions. available from ACP (www.acponline.org/practice resources/
business resources/health information-technolory) and
National Patient Safety Goals AHRQ (www.ahrq.gov/professionals/prevention chronic care/
The Joint Commission establishes annual National Patient Safety improve/health-it).
Goals to address important issues in health care safety.
Recommended by a panel of patient safef experts and applied
to a variety of patient care settings, the National Patient Safety
Health Literacy
Goals center on interventions that have the highest impact on As defined by AHRQ, health literacy is the degree to which
quality and safety. In publishing the goals, the Joint Commission individuals have the capacity to obtain, process, and under
also provides specific metrics for each goal to facilitate implemen- stand the information required to make informed health deci
tation. Goals are customized to the care setting and separate goals sions. Low health Iiteracy is more common among older
are provided for hospitals, ambulatory health centers, and nurs adults, minority populations, persons with lower socioeco
ing care centers, among others. The goals are updated annually nomic status, and medically underserved groups. Low health
and are available at wr,vw.jointcommission.org/standards/ literacy may hinder patients' ability to describe their health
national-patient safegr goals and www.jointcommission.org/ concerns, complete health forms accurately, understand
standards /national-patient- safety goals /hospital-national medical information, and manage their health conditions.
patient- safety- goals. Furthermore, evidence shows that low health literacy is asso-
ciated with poorer health outcomes and decreased utilization
of care.
Clinicians need to be aware of the health literacy of their
Health I nformation Tech nology patients and identiflz those who may need assistance. Tools to
and Patient Safety assess health literacy in specific populations are available from
Health information technolory (HIT) is the use of an electronic AHRQ (www.ahrq.gov/health-literacy/quality resources/
environment to share patient health information and facilitate tools/literacy/index.html) and include the Short Assessment
patient management. The electronic health record (EHR), of Health Literacy and the Rapid Estimate of Adult Literacy in
computerized physician order entry (CPOE), and computer Medicine. Another instrument available for use is the Newest
ized clinical decision support (CDS) are some common exam- Vital Sign, which can help predict a patient's ability to com
ples of HIT. prehend common instructions related to frequent primary
The EHR is a compilation of all health data for a specific care scenarios. This tool and multiple other health literacy
patient, including medical notes and test results, in a digital measures are available through the Health Literacy Tool Shed
format. The EHR enables the timely sharing of patient infor (http: //healthliteracy.bu.edu/all).
mation by multiple users, resulting in improved communica Clinicians can improve patient understanding by using
tion and care efficiency. simple sentences, repeating information, providing an oppor
CPOE is a system by which clinicians electronically enter tunity for the patient to ask questions, and supplying the
medication, radiologr, and laboratory orders, thereby elimi patient with educational materials written in plain language.
nating errors and delays related to illegible handwriting. The teach back method, wherein the clinician asks the patient
24
i
I
L
Hospital Medicine Principles
t
I
I
! prioritize, and implement recommendations by specialists and
to describe his or her understanding of the situation in his or
I her own words, can further assess a patient's command of the other consulting services in a timely fashion, if within the
I
situation and provide valuable insight into patient knowledge goals of care. Hospitalists are instrumental in synthesizing
I
I information and clearly communicating care plans to other
and health literacy.
i providers, clinicians, patients, and families. Meticulous coor-
I
dination of information among services and specialties
t improves consistency of messaging to patients and reduces
;
t
Hospital Medicine fragmentation of care that can result in omissions, duplica
Principles tions, or discrepancies of care.
The frequent hand offs between clinical teams inherent
I The specialty of hospital medicine has established itself as an to hospital medicine require specific aftention on the part of
important leader in key domains critical to hospital function- the hospitalist to prevent errors. Two thirds ofsentinel events
\
ing. ln addition to expertise in the management of inpatient in the hospital occur in the setting of inadequate handoffs.
I
medical issues, a hospitalist's core competencies, as defined by Handoffs are defined as the transfer and acceptance ofrespon-
the Society of Hospital Medicine, include expertise in interdis-
sibility of patient care between clinicians through effective
ciplinary communication and team-based care, perioperative communication. Handoffs are high frequency events in the
r
and consultative medicine, prevention of health care associ- hospital and can be difficult to do well. Strategies aimed at
I
25
Hospital Medicine Principles
Call-outs Communicating important or Emergent settings (e.9., code Code leader: 'Airway status?"
critical information verbally, blue, rapid response team)
Resident:'Airway clear."
which informs allteam
members simultaneously Code leader: "Breath sounds?"
Resident: "Breath sounds decreased on right."
I-PASS lllness severity: stability level of Used during handoffs betvveen l: "Mr. is the sickest patient on our team
patient clinicians in hospital and will- need watching."
Patient summary: events Designed to enhance P: "He is a 57-year-old man with DM and
leading to admission, hospital information exchange during cellulitis of the right leg. He was admitted
course, assessment, and plan transitions of care with hypotension, fever, and concerns for
fasciitis. He has stabilized after 2 liters of NS
Action list: to-do list for patient, and vancomycin and piperacillin/tazobactam,
timeline, and ownership but continues to have fevers and BPs in the
Situation awareness and 90s/50s. Leg CT is pendin9."
contingency planning: plan for A: "Vancomycin level needs to be followed
what might happen up at 8 PM and vital signs and volume status
Synthesis: recipient of patient evaluated every 2 hours."
information su mmarizes what S: "lf SBP <80 mm Hg and there! no volume
was heard, asks questions, and overload, give 500 mL NS lV bolus. lf
restates action items vancomycin level is <1 5, discuss with
pharmary dose changes. Follow up on leg CT."
BP = blood pressure; DM = diabetes mellitus; lV = intravenous; NS = normal saline; SBP = systolic blood pressure.
Adapted {rom TeamSTEPPSo curriculum materials. Agency for Healthcare Research and Ouality, Rockville, MD. Content last reviewed October 201 9. https://www.ahrq.gov/
teamstepps/curriculum-materials.html. Accessed June I 7, 2021 .
26
:
i
I
i
i
Hospital Medicine Principles
L
I
L
I
A model of shared care known as co management has TABLE 19.
Padua Prediction Score for Risk for
L evolved between hospitalists and many surgical and other ser Venous Thromboembolism
I vices. There are several styles of co management, depending on Risk Factor Points
t the institution, resources, and specific agreements between
I Active cancer 3
L
services. For example, a hospitalist may serve as the primary
Previous VTE 3
clinician with surgeons serving as consultants for a primary
t
I
ties among services. Although literature has been mixed on Acute infection or rheumatologic 1
\
I
Hospita l-Based Prevention using a validated risk assessment model, such as the Padua
L
;
Strategies Prediction Score (Table 19). Ifbleeding risk is not elevated and
t Hospitalized patients are at risk fbr many medical complica VTE risk is high (Padua Score >4), use of pharmacologic prophy
tions, several of which can be mitigated through strategies laxis with daily low-molecular-weight heparin is recommended
t
aimed at prevention. Hospitalists are uniquely positioned to throughout the patient's hospital stay or until mobility is fully
\
l
reduce inpatient complications. Risk reduction strategies that recovered. In patients with an unacceptably high bleeding risk,
are commonly initiated include venous thromboembolism mechanical prophylaxis with intermittent pneumatic compres-
\ (VTE) prophylaxis, delirium prevention and screening, anti- sion may be used, although data supporting this approach in
5 biotic stewardship, discontinuation of catheters to prevent non-surgical patients is limited. In certain high-risk settings,
catheter associated infections, prevention of aspiration, such as orthopedic and oncologic surgery extended duration
interventions to reduce deconditioning and falls, bedsore VTE prophylaxis is recommended postdischarge. For more infbr-
prevention initiatives, and medication reconciliation. mation, see Perioperative Medicine and MKSAP 19 Hematologr.
VTE risk is elevated in hospitalized, immobile, and acutely Delirium is particularly common among hospitalized
ill patients. More than 50'1, of VTE events occur during or after patients. For more information, see MKSAP 19 Pulmonary and
hospitalization for acute illness or recent surgery. VTE risk is Critical Care Medicine and MKSAP 19 Neurologr. Patients at
particularly high in ICU patients and patients with cancer or highest risk include those with advanced age, polypharmacy,
stroke. Early ambulation, mechanical prophylaxis (such as inter- prior cognitive impairment, recent surgery and acute illness.
mittent pneumatic compression devices applied to the lower Predisposing and provoking risk factors for delirium are listed
exlremities), and prophylactic anticoagulants are used either in Table 20. Up to 40'7, ol cases of delirium in hospitalized
alone or in combination to prevent VTE in these patients. patients can be prevented. Preventive strategies include the
Guidance on VTE prevention strategies is provided by the use of assistive visual and hearing devices, optimization
American Society of Hematologr and American College of Chest of pain control, minimization of psychoactive medications,
Physicians. A patient's thrombotic risk can be determined by frequent reorientation, early mobilization, and allowance of
27
Perioperative Medicine
TABLE 20. Risk Factors for Delirium During Hospitalization recent stroke or neuromuscular disease), bowel obstruction or
ileus, dementia, delirium, or progressive somnolence. Aspiration
Predisposing Factors
prevention strategies include elevation of the head of the bed,
Advanced age (>70 y)
swallow evaluation by a speech therapist, assisted feeding, and
Preexisti ng cognitive impairment avoidance of nasogastric tubes.
Poor nutritional status Several fall prevention strategies can be used in the hospi
Poor functional status tal setting. These strategies include physical and occupational
Multiple medical comorbidities therapy, bed alarms, limitation of medications that cause pos-
tural hypotension or weakness, and hospital sitter services for
Renal disease
patients with cognitive impairment. Although no single inter
Alcohol use disorder
vention has been shown to decrease falls in hospitalized
Depression patients, various bundles of multifactorial intervention pro
Polypharmacy grams have been successful in reducing falls in the hospital
Deliriogenic medications (e.g., benzodiazepines, anticholinergics, setting (see MKSAP 19 General Internal Medicine 1).
opioids)" Prevention of pressure related injury is a prioriry espe-
History of delirium cially in patients at higher risk. For more interventions. see
Vision or hearing impairment MKSAP 19 General Internal Medicine 1.
Medication errors are one of the most common elTors
Provoking Factors
during hospitalization and can be reduced by vigilance with
Poorly controlled pain
medication review and reconciliation upon admission and
Deliriogenic medications (e.g., benzodiazepines, discharge as well as upon transfer between services (see
anticholinergics, opioids)"
Patient Safety and Quality Improvement).
Disruption of sleep-wake cycle
l(EY POll{Tt
Sleep deprivation
r Thrombotic risk in hospitalized patients can be deter-
Constipation
mined by using a validated risk assessment model (e.9.,
Dehydration
Padua Prediction Score) to inform the use ofpharmaco-
Urinary retention logic prophylaxis.
Severe illness o Strategies to prevent delirium in hospitalized patients
lnfection include the use of assistive visual and hearing devices,
Hypoxia optimization of pain control, minimization of psychoac-
Surgical stress tive medications, frequent reorientation, early mobiliza-
tion, and allowance of unintemrpted sleep to promote
Use of urinary catheter and other tethers
a normal sleep-wake cycle.
Sensory deprivation (e.g., no hearing aids or glasses)
'See the American Geriatrics Society Beers criteria for potentially inappropriate
medication use in older adults.
Perioperative Med ici ne
uninterrupted sleep to promote a normal sleep-wake cycle.
General Responsi bi lities
Early detection of delirium is of equal importance for preven Perioperative medicine encompasses risk assessment and
tion of additional complications. The Confusion Assessment optimization of comorbidities before surgery intraoperative
Method is an evidence-based screening tool that should be management. and postoperative care. The internist's role is to
used regularly in high risk patients. determine operative risk; communicate this information to
Hospitalists play a key role in preventing hospital-acquired patients, surgeons, and anesthesiologists; and recommend
infections by championing early removal of urinary and central strategies to mitigate this risk. Decisions regarding the modal
venous catheters and judicious use of antibiotics. Antibiotic ity ofsurgical approach and anesthesia are best deferred to the
stewardship activity in collaboration with infectious disease anesthesiologist and surgeon. Commonly, hospitalists are also
specialists and pharmacists targets appropriate and timely involved in the postoperative management of surgical patients.
antibiotic use to prevent antibiotic resistance and decrease the Internists can also assist with setting patient and family
likelihood of complications of antibiotic therapy, including expectations in the perioperative period and ensuring that
Clostridioides difficile infection (see MKSAP 19 Infectious expected outcomes align with their goals.
Disease). Aspiration and resultant respiratory distress can be a
devastating pulmonary complication in hospitalized patients, Preoperative Laboratory Testin g
leading to pneumonitis and/or pneumonia. Patients at higher AII preoperative laboratory testing should be tailored on the
risk for aspiration include those with dysphagia (for example, basis of the nature of surgery known comorbidities, and
28
i
L
t
t
Perioperative Medicine
t
I
29
I
'l
I
Perioperative Medicine
't
TASLE 22. Perioperative Medication Management.
:
Medication Perioperative Special Considerations
:
Recommendation
Cardiovascular Agents
I
o1-Blockers Continue Noti{y surgeon of risk for intraoperative floppy iris syndrome in cataract surgery; temporary
cessation may not decrease risk
a2-Blockers Continue Do not initiate clonidine for preoperative cardiovascular risk reduction (increases risk for
perioperative hypotension) I
I
p-Blockers Continue Consider init:ating preoperatively in patients with three or more RCRI riskfactors and those with
intermediate- or high-risk myocardial ischemia on preoperative stresstesting
Begin p-blocker with enough time to assess tolerability; P-blocker should not be started on
the day of surgery (>2 wk before surgery is preferred)
Calcium channel blockers Continue
ACE inhibitors and ARBs lndividualize lncreases risk for hypotension and AKI and may increase risk for adverse cardiovascular events;
typically withhold if prescribed for hypertension (unless blood pressure is poorly controlled);
check institutional anesthesiology guidelines; restart as soon as possible postoperatively
Diuretics Withhold Monitor volume status closely if heart failure is present and restart as soon as possible
postoperatively
Nitrates Continue
Vasodilators Continue
Statins Continue Thought to have beneficial pleiotropic effects in addition to lipidJowering properties
Starting preoperatively recommended in patients undergoing vascular surgery and those
undergoing elevated-risk noncardiac surgery with long-term indication for statin
Withhold all other li medications
Analgesic Agents
NSAIDs Withhold Withhold for 3-7 days before surgery depending on half-life of NSAID
Opioids lndividualize lf surgery is elective, may consider preoperative pain management consultation and opioid
taper
Continue in most patients receiving long-term opioid therapy
Risk for poorly controlled postoperative pain and respiratory depression
Acetami Continue
Gastro:ntestinal Agents
Antacid medications Continue
(including H2-blockers and
proton pump inhibitors)
ne withhold Risk for side effects
Rheumatologic Agents
Hydroxychloroquine Continue
Methotrexate lndividualize Limited high-quality evidence
Probably safe to continue in most situations; withhold if significant concern for infection or
history of septic complications; dose adjust in cases of kidney injury
Biologics withhold Balance risks for infection and postoperative disease flare
Reasonable to schedule surgery at end of dosing cycle, withholding dose at time of surgery
and not resuming for 22 wk after surgery
Tofacitinib withhold Hold for >7 d before elective total hip or total knee replacement and do not resume for
22 wk after su
Psychiatric Agents
Selective serotonin Usually continue May increase risk for bleeding, especially in conjunction with antiplatelet agents; risk for
reuptake inhibitors withdrawal symptoms with abrupt cessation
Benzodiazepines Continue Risk for withdrawal with abrupt cessation; monitor for respiratory depression
Antipsvchotics Continue Potential for QT orolonqation
Supplements
Herbal preparations Withhold Withhold 1-2 wk preoperatively
Vitamins and supplements Withhold
AKI = acute kidney injury; ARB = angiotensin receptor blocker; RCRI = Revised Cardiac Risk lndex.
'Perioperative management of antiplatelet agents, anticoagulants, antiepileptic drugs, glucoconicoids, and diabetes medications is discussed later in this chapter.
lnformation from Fleisher LA, Fleischmann KE, Auerbach AD, et al; American College of Cardiology. 201 4 ACC/AHA guideline on perioperative cardiovascular evaluation and
management oI patients undergoing noncardiac surgery: a report of the American College of Cardiology/American Heart Association Task Force on practice guidelines. J Am
undergoing noncardiac surgery. N Engl J Med. 201 4;370:1 504 13.IPMID: 24679061 ] doi:1 0.1 056/NEJMoa 1 40 1 1 06 and Goodman SM, Springer B, Guyatt G, et al. 201 7
American College of Rheumatology/American Association of Hip and Knee Surgeons guideline for the perioperative management of antirheumatic medication in patients with
rheumatic diseases undergoing elective total hip or total knee arthroplasty. Arthritis Rheumatol. 201 7;69:1 538 5 1 . IPMID: 28620948] doi:1 0.1 O02l an.401 49
30
Perioperative Medicine
No
No
Risk factors for CAD?! Proceed to surgery
Yes
No
'Risk fact06 for CAD: not specifically delined in ACC/AHA guidelines; examples include Normal
known CID cerebrovascular disease(i.e., stroke ortransient ischemitattack), chroni(
Pharmacologic stress test
kidney disease, diabetes mellalus, and hean failure.
Recommendations from fleisher [A, Fleischmann Kt,Auerbach AD, etal; American Col'
Coronary revascularization
lege of Cardiology.2014 ACC/AHA guideline on perioperative cardiovascular evaluation
and management of palients undergoing nontardia( surgery: a report oftheAmelican
according to existing
College of Grdiology/AmeiGn Heanlssociation Task Force on pradice guid€lines. J Am clinical practice guideline
Coll Cardiol. 201 4;64:e77t 37. PMID: 25091 544 doi:1 0.1 01 6/jjact.201 4.07.944
31
Perioperative Medicine
TABLE 23, Revised Cardiac Risk lndex and Prediaed Rate Patients with low calculated cardiovascular risk (<l'7, risk
of Major Cardiac Complications Perioperatively for perioperative MACE) may proceed to surgery whereas
Risk Factor (1 point for each) patients with elevated risk (>1'7, risk for perioperative MACE)
High-risk surgery (intrathoracic, intraperitoneal, suprainguinal
should have functional capacity assessed. Metabolic equiva
vascular) lents (METs) are used to represent a patient's functional capac
lschemic heart disease ity based on the intensity of activity the patient is able to
perform. According to the ACC/AHA guideline, if functional
Heart failure (compensated)
capacity exceeds 4 METs, the patient may proceed to surgery
Diabetes mellitus (requiring insulin)
without further testing. The Duke Activity Starus Index (DASI)
Cerebrovascular disease score (Table 25) is a standardized and accurate means of esti-
Chronic kidney disease (serum creatinine >2.0 mg/dL mating exercise capacity; data lrom the METS trial showed
[176.8 gmol/L])" that clinician estimates of exercise capacity without use of a
Number of Points Risk for Major Cardiac Complicationsb standardized tool do not correlate with actual patient perfor-
0 0.47"-0.5o/" mance on cardiopulmonary exercise testing nor with postop
1 1.0%-2.6% erative cardiac outcomes. Preoperative cardiac stress testing
should be considered in patients at elevated risk for MACE
2 2.4"/"-7.2"/"
with functional capaci[z objectively assessed to not be low risk,
.J 5.4%-14.4"/"
but only if the results will change perioperative management.
aEstimated glomerular filtration rate <30 mUmin/1 .73 m2 is also shown to predict An alternative approach from the Canadian Cardiovascular
cardiovascular risk.
Society does not use preoperative cardiac stress testing and
bDefined as cardiac death, nonfatal myocardial infarction, and nonfatal cardiac
arrest (original validation also included pulmonary edema and complete heart
instead suggests assessing perioperative risk using patient age,
block). Percentages represent incidence measured during inpatient timeframe. presence of known cardiovascular disease, the RCRI, and
Validations using 30 day time frames have demonstrated significantly higher rates, l
even among patients with a score of 0. B-type natriuretic peptide.
Data from Lee TH, Marcantonio ER, Mangione CM, et al. Derivation and prospenrve Preoperative ECG is reasonable in patients with known
validation of a simple index for prediction of cardiac risk of major noncardiac
surgery. Circulation. 1 999;100:1043-9. IPMID: 10477528] doi:10.1 1 61l01. cardiovascular disease undergoing moderate to high risk sur-
L
TABLE 24. Comparison of Tools for Estimating Major Adverse Cardiovascular Event Risk
ASC NSOIP Calculators
Characteristic RCRI MtCA Risk Surgical Risk
External validation Multiple studies Minimal Minimal
Surgical population Inpatient (hospitalization for All, except for some low-risk All, except for some low-risk
2 d) for most validations procedures procedures
Measured cardiovascular Ml, cardiac death/ventricular MICA Ml and cardiac death
outcomes fibrillation, pulmonary edema,
complete heart block
Outcome measurement time lnpatient only {or most 30 d postoperatively 30 d postoperatively
frame validations
Limitations Tendency to overestimate risk Surgical types combined into Tendency to overestimate risk
broad categories that could
lead to inaccurate assessment
of risk
I acs NSOIP = American Cancer Society National Surgical Ouality lmprovement Program; Ml = myocardial infarction; N4ICA = myocardial infarction and cardiac arrest;
] acnr = Revised Cardiac Risk lndex
l
32
Perioperative Medicine
TABLE 25. Duke Activity Status lndex (DAS|) Score considered 90 days after drug-eluting placement. patients
Questionnaire with an acute coronary syndrome not managed with coro
Ouestionnaire ltem Points (each nary intervention are still at elevated risk for postoperative
"Yes" answer)" MACE, and elective surgery should be delayed for at least
1. Can you take care of yourself (e.g., 2.75 60 days after the event.
eat, dress, bathe, or use the toilet)? Patients taking B blockers, statins, and most antihyper
2. Can you walk indoors (e.g., 1.75 tensive medications (with the possible exceptions of diuretics
throughout the rooms of your and ACE inhibitors) should continue these medications peri
house)?
operatively unless prohibited by hypotension. In some circum
3. Can you walk one to two blocks on 2.75 stances, p-blocker or statin therapy is initiated preoperatively
level ground?
(see Table 22). Postoperative p blocker administration should
4. Can you climb a flight of stairs or walk 5.50 be guided by clinical circumstances, but dose reduction is pre,
up a hill?
ferred to discontinuation if hypotension develops.
5. Can you run a shortdistance? 8.00
The ACC/AHA perioperative evaluation and management
6. Can you do /ight work around the 2.70 guideline does not recommend routine measurement of post-
house (e.9., dusting, washing dishes)?
operative troponin or ECG, but these studies should be obtained
7. Can you do moderate work around 3.50 if signs or symptoms of myocardial ischemia develop. However,
the house (e.g., vacuuming, sweeping
floors, or carrying groceries)? in many cases, postoperative myocardial ischemia is asympto
matic or presents a!/pically. The Canadian Cardiovascular
:
33
Perioperative Medicine
HVC . Patients with coronary artery disease should not ASA = American Society of Anesthesiologists.
undergo routine coronary angiography or revasculariza- 'ASA classes are as follows: class 1, normal healthy patient; class 2, patient with
mild system c disease; class 3, patient with severe systemic disease; class 4, patient
tion before surgery exclusively to reduce perioperative with systemlc disease that is a constant threat to li{e; and class 5, moribund patient
events. who s not expected to survive for 24 hours with or wrthout operation.
34
Perioperative Medicine
TABLE 27.STOP-BANG Obstructive Sleep Apnea muscle training) and postoperative continuous positive pres-
Screening Tool sure ventilation. Surgery should be postponed and appropriate
Survey ltems (1 point for each) treatment initiated if there is concern for exacerbation of
Snoring underlying disease or respiratory illness. Smoking cessation
reduces pulmonary risk and should be encouraged as far in
Tiredness or sleepiness during the day
advance of surgery as possible. Prescribed inhaled medica
Observed apnea during sleep
tions should be continued throughout the perioperative
Pressure, high blood period. Perioperative management should also include goal
BMt>35 directed fluid management and consideration of lung protec
Age >50 y tive ventilation strategies.
In patients diagnosed with OSA, continuous positive air
Neck circumference >40 cm (15.7 in)
way pressure should be initiated preoperatively when possible.
Gender = male
For patients with suspected OSA undergoing nonelective sur-
STOP-BANG Score Risk Correlation gery, management includes postoperative monitoring with
0-2 Low risk for OSA continuous pulse oximetry or capnography, nonsupine posi
>3 lncreased risk for OSA tioning, and limiting sedating medications. Patients with
>5 lncreased risk for moderate to severe OSA
known OSA should bring their continuous positive airway
pressure device to the hospital for use in the perioperative
OSA = obstructive sleep apnea.
period.
Adapted with permission from Chung F, Yegneswaran B, Liao P, et al. STOP
questionnaire: a tool to screen patients for obstructive sleep apnea.
Anesthesiology. 2008;108:812-21.{PMlDr 18431 1 161 doi:10.1097/
t(tY PottTs
. Preoperative chest radiography is indicated only in HVC
patients with signs or symptoms of pulmonary illness
or in those with underlying cardiac or pulmonary dis
ease and new or unstable symptoms.
and other available risk calculators do not incorporate OSA but o Spirometry should not be routinely performed preoper HVC
are useful in estimating general pulmonary risk. atively except in patients undergoing lung resection.
A11 patients should be screened for OSA, which is associ
o All patients undergoing surgery should be screened for
ated with cardiac events, pulmonary complications, and ICU
obstructive sleep apnea, which is associated with adverse
admissions. A commonly used screening tool for OSA is the
perioperative outcomes.
STOP BANG score (Ihble 27). Although there is a paucity of
data supporting delay of surgery in patients at high risk for
OSA, the Society of Anesthesia and Sleep Medicine recom
mends preoperative sleep medicine evaluation in patients
Hematologic Perioperative
screened as high risk for OSA who also have evidence of Management
hypoventilation (CO2 retention), severe or uncontrolled Venous Thromboembolism Prophylaxis
comorbidities, or resting hypoxemia not attributable to other l'he American College of Chest Physicians (ACCP) and the
cardiopul monary causes. American Society for Hematologr (ASI'i) provide guidelines for
Chest radiography is indicated only in patients with signs WE prophyla-ris in both orthopedic and nonorthopedic surgery
or symptoms of pulmonary illness or underlying cardiac or populations (Table 28). In patients undergoing general surgery
pulmonary disease with new or unstable symptoms. or abdominal pelvic surgery the Caprini score can be used to
Spirometry is not useful fbr predicting risk and should not be esti mate risk for postoperative thronr bosi s (w.vwv. mdca c. com /
I
routinely ordered for preoperative evaluation. including in caprini-score venous thromboembolism 2005).
patients with COPD. Furthermore, evidence does not support Hip fracture surgery total knee arthroplasty, and total hip
a spirometric threshold below which the risk of surgery is arthroplasty pose a very high risk for VTE, and concomitant
unacceptable. However, spirometry is warranted in patients mechanical and pharmacologic VTE prophylaxis are recom
undergoing lung resection to help predict postoperative lung mended. Mechanical prophylaxis is provided with an inter
function. mittent pneumatic compression device lor the duration of the
hospital stay. For pharmacologic prophylaxis, direct oral anti
Perioperative Risk-Reduction Strategies coagulants (DOACs), aspirin, low molecular-weight heparin,
Risk for perioperative pulmonary complications can be miti- warfarin, or low dose unfractionated heparin may be used in
gated with selected interventions, including oral hygiene, total joint arthroplasty, and low molecular weight heparin,
, early mobilization, use of regional or neuraxial anesthesia low dose unfractionated heparin, or fondaparinux may be
and analgesia, prophylactic respiratory physiotherapy (sputum used in hip fracture repair. The minimum recommended
clearance techniques, deep breathing exercises, and inspiratory duration of pharmacologic VTE prophylaxis atter these
35
Perioperative Medicine
Orthopedic Hip or knee arthroplasty IPC during hospital stay and aspirin
or DOAC for 21-35 days; LMWH over
warfarin or LDUH as secondary
options; if high bleeding risk, IPC
alone during hospital stay
High VTE risk (e.9., malignancy, IPC + LMWH (when bleeding risk
a nterior-posterior approach) sufficiently low)
DOAC = direct oral anticoagulant; IPC = intermittent pneumatic compression; LDUH = low-dose unfractionated heparin; LMWH = low-molecular-weight heparin; NA = not
applicable; UFH = unfractionated heparin; VKA = vitamin K antagonist; VTE = venous thromboembolism.
uDuration is for postoperative hospitalization unless noted otheMise.
bSee the Ca pri ni Score for Venous Throm boem bol ism: www. mdca lc.com/caprin i-score-venous-thromboem bolism-2005.
'Caprini and colleagues' 201 7 meta analysis of 1 4,700 patients in 1 I studies showed that Caprini scores >7 benefited from VTE chemoprophylaxis-but it was less clear for
intermediate risk {Caprini 3-6). The American Society o{ Hematology does not specifically suggest using the Caprini risk tool for determining high risk for WE but does
recommend LMWH or LDUH (tlPC)for patients with acceptable bleeding risk.
Recommendations from Gould MK, Garcia DA, Wren SM, et al. Prevention of WE in nonorthopedic surgical patients: antithrombotic therapy and prevention o{ thrombosis, 9th ed
Francis CW, Johanson NA, et al. Prevention of VTE in orthopedic surgery patients: antithrombotic therapy and prevention of thrombosis, fth ed: American College of Chest
et al. American Society of Hematology 201 9 guidelines for management of venous thromboembolism: prevention of venous thromboembolism in surgical hospitalized patients.
Blood Adv. 201 9;3:3898-944. IPMID: 3 1 794602] doi:1 0.1 1 82lbloodadvances.20l 9000975
Key NS, Khorana AA, Kuderer NM, et al. Venous thromboembolism prophylaxis and treatment in patients with cancer: ASCO Clinical Practice Guideline Update Ipublished online
ahead of print, 20 1 9 Aug 51. J Clin Oncol. 201 9;JCO1 901 461 . [PMID: 31 381 464] doi:1 0.1 200/JCO.1 9.01 46
Pannucci CJ, Swistun L, MacDonald JK, et al. lndividualized venous thromboembolism risk stratification using the 2005 caprini score to identi{y the benefits and harms of
chemoprophylaxis in surgical patients: a meta analysis. Ann 5urg. 201 7;265:1 094-1 03. IPMID: 281 06607] doi:1 0.1 097/S1A.0000000000002126
36
Perioperative Medicine
procedures is l0 to 14 days; however, provided that the bleed with the surgical team. If bridging is required, therapeutic
ing risk is low extended duration postoperative prophylaxis anticoagulation is typically delayed until 24 hours after proce
(up to 35 days from the day of surgery) is preferred. dures with low bleeding risk and 48 to72 hours after surgery
If bleeding risk is especially high, mechanical prophylaxis with higher bleeding risk. Owing to their delayed effect, the
is recommended over no prophylaxis. first dose of a vitamin K antagonist is typically administered 12
Both ACCP and ASH recommend against preoperative to 24 hours after surgery. Postoperative timing of DOAC rein-
placement of inferior vena cava filters for VTE prophylaxis and stitution depends on postoperative kidney function, bleeding
routine surveillance for VTE with venous compression risk, and hemostasis. DOACs reach therapeutic levels in 1 to
ultrasonography. 3 hours and can typically be resumed 48 to 72 hours after
major surgery.
Perioperative Management of Because of the risk for spinal epidural hematoma, antico-
Anticoagulant Therapy agulant use with concomitant neuraxial (spinal and epidural)
Anticoagulant therapy increases the risk for perioperative hem anesthesia should be avoided.
orrhage and should be withheld in most patients before major
surgery. For minor surgery such as cataracts, simple skin sur Atrial Fibrillation
gery or simple dental extractions, anticoagulation may be con According to AHA/ACC/Heart Rhythm Society and ACCP
tinued. When necessary to discontinue, vitamin K antagonists guidelines, preoperative bridging is not recommended in
should be withheld at least 5 days before surgery; most proce patients with atrial fibrillation taking warfarin unless they
dures can be safely perfbrmed with an INR less than 1.5. The have a mechanical valve. Decisions on bridging therapy should
duration for which DOACs are withheld before surgery depends balance the risks ofstroke and bleeding.
on the bleeding risk of the procedure, kidney function, and
medication half life; in general, DOACs can be stopped 2 to Prosthetic Heart Valves
3 days preoperatively because oftheir shorter half lives. In patients receiving warfarin anticoagulant therapy for a
Bridging anticoagulation is the administration of thera- mechanical prosthetic heart valve, continuation of anticoagu-
peutic doses of short-acting parenteral therapy, usually a hep- lation is recommended when the surgical procedure is minor.
arin, when oral anticoagulant therapy is being withheld In patients undergoing surgery with a higher risk for bleeding,
during the perioperative period. Newer guidelines recom the 2017 ACC/AHA guideline on valvular heart disease suggests
mend against bridging in most scenarios due to Iimited benefit that bridging should be considered on an individualized basis
and elevated bleeding risk. Bridging with a parenteral agent in patients with a mechanical mitral valve; a mechanical aortic
should only be considered in patients with a high thrombotic valve with thromboembolic risk factors (such as atrial fibrilla-
risk (atrial fibrillation with a high CHATDS, VASc score, recent tion, previous stroke or transient ischemic attack, hyperten
thromboembolic event [e.g., such as stroke or presence of a sion, diabetes mellitus, heart failure, or age >75 years); or an
high thrombotic risk mechanical valvel). Bridging is not older generation mechanical aortic valve. Bridging is not nec
indicated with DOACs owing to the rapid onset and short half- essary in patients with a bileaflet mechanical aortic valve and
lif'e of these drugs. no other risk factors for thrombosis. The ACCP provides simi
Postprocedural management of anticoagulation should lar recommendations for bridging anticoagulation in patients
be guided by thrombotic and bleeding risk and collaboration with prosthetic heart valves (Table 29).
TABLE 29. Recommendations for Perioperative Bridging in Patients With a Prosthetic Heart Valve
RiskforThromboembolism PatientHistory Bridging Anticoagulation
Recommendation
High (annual risk >10%) Any mitral valve prosthesis Bridging
Any caged-ball or tilting disc aortic valve prosthesis
I Recent (within 6 mo) stroke or TIA
Moderate (annual risk of Bileaflet aortic valve prosthesis and one or more Bridging unless procedure is associated
5%-10y"\ of the following risk factors: atrialfibrillation, with a high bleeding risk
previous stroke or TlA, hypertension, diabetes
mellitus, heart failure, age >7 5 y
Low (annual risk <5%) Bileaflet aortic valve prosthesis without atrial No bridging
fibrillation and no other risk factors for stroke
TIA = transient ischemic attack.
Recommendations from Douketis JD, Spyropoulos AC, Spencer FA, et al. Perioperative management of antithrombotic therapy: antithrombotic therapy and prevention of
2298 and Nishimura RA, Otto CM, Bonow RO, et al. 201 7 AHA,/ACC focused update o{ the 201 4 AHA/ACC guideline lor the management of patients with valvular heart disease:
a repon of the American College of Cardiology/American Heart Association Task Force on clinical practice guidelines. J Am Coll Cardiol. 2017:70:252-29. IPMID:283157321
doi:1 O.1 01 6/j.jacc.20 1 7.03.0 1 1
37
Perioperative Medicine
TABLE 3O. Recommendations for Perioperative Bridging in Patients With Venous Thromboembolism Requiring lnterruption
of Vitamin K Antagonist Therapy
Risk for Patient History Bridging Anticoagulation
Thromboembolism Recommendation
High (annual risk >10%) Recent (within 3 mo)WE Bridging
Severe thrombophilia (e.9., deficienry of protein C,
protein S, or antithrombin; antiphospholipid antibodies;
multiple thrombophilic abnormalities)
Moderate (annual risk of 5%-10ol.) WE within the past 3,12 mo No bridging
Nonsevere thrombophilia (e.g., heterozygous factor V
Leiden or prothrombin gene mutation)
Recurrent VTE
Active cancer (treated within 6 mo or palliative)
Low (annual risk <5%) VTE >12 mo ago and no other risk factors No bridging
VTE = venous thromboembolism
Recommendations from Witt DlV, Nieuwlaat R, Clark NP, et al. American Society oI Hematology 2018 guidelines for management of venous rhromboembolism: optimal
management of anticoagulation therapy. Blood Adv. 20 1 8;2:3257 91 . IPMID:30482 7651 doi:1 0.1 1 82lbloodadvances.2o 1 8024893 and Gould M K, Garcia DA, Wren SM, et al.
practice guidel nes. Chest. 20 1 2;1 4 1:e2215 e2775. IPMID: 22315263) dot:10.1318/cl-,est.1 1 2297
Venous Thromboembolic Disease events. However, a subgroup analysis of this trial showed
Recommendations for bridging anticoagulation in patients greater benefit than harm to continuing perioperative aspirin
with a history of VTE, including those with thrombophilias, in patients with prior percutaneous coronary intervention
are listed in Table 30. during any time frame.
38
Perioperative Medicine
eluting stent placement. should be deferred until thyroid disease can be controlled.
I HVC . In orthopedic and cardiac surgery patients and those Consultation with an endocrinologist is advised if emer
with a history of coronary artery disease, the American gent surgery is required in patients with severe thyroid disease.
Association of Blood Banks recommends a restrictive
transfusion threshold (hemoglobin level of 8 g/dl [80 g/L1);
otherwise, a transfusion threshold of 7 gldL (ZO gll-) is
Adrenal tnsufficiency
recommended. High-quality evidence to guide the use of perioperative sup
plemental glucocorticoid dosing, known as stress dosing, is
Iacking. Increasingly, stress dosing is only administered in the
perioperative setting in the case of major surgery when there
Perioperative Management is known primary or secondary adrenal insufficiency or if a
of Endocrine Diseases patient is at high risk for hypothalamic-pituitary adrenal axis
Diabetes Mellitus (HPAA) suppression. Despite the lack of evidence, the anesthe-
Evidence demonstrates that patients with perioperative hyper- siologz literature recommends an aggressive approach.
glycemia are at increased risk fbr postoperative complications, Guidance for perioperative management of patients at risk fbr
including inf'ections, cardiovascular events, and mortality. adrenal insufficiency is provided in Table 31.
Patients at risk for diabetes should be evaluated fbr diabetes The exact time course fbr recovery from HPAA suppres
before elective surgery. In patients with known diabetes, it is sion ftom previous glucocorticoid use varies between individu
reasonable to obtain hemoglobin A," within 3 months of sur als. When perioperative stress dosing is indicated owing to
gery. Efforts should be made to optimize glycemic control suspected HPAA suppression, glucocorticoid dosing recom-
before major elective surgery; however, it is undecided mendations are tailored to the stress ofthe planned procedure.
whether delaying surgery to do so improves outcomes. For minor surgical procedures using only local anesthesia,
Oral and injectable noninsulin medications should be patients with primary adrenal insufficiency should take twice
withheld the morning of major surgery replaced with insulin their usual dose ofglucocorticoids on the day ofsurgery and fbr
during hospitalization, and resumed at hospital discharge 1 to 2 days after; other patients at risk for adrenal insufficiency
when oral intake and kidney function have normalized. can take their usual daily glucocorticoids dose on the morning
Sodium glucose cotransporter 2 inhibitors should be with- of the procedure. Recommendations lbr non minor surgery
held 3 to 4 days owing to the risk fbr euglycemic ketoacidosis. vary. For non-minor surgical procedures, the Association of
For patients on insulin regimens, basal insulin should be con Anaesthetists recommends giving 100 mg of intravenous (lV)
tinued perioperatively. While lasting for surgery, patients hydrocortisone with induction of anesthesia followed by con
should receive 50'X, of their usual intermediate acting insulin tinuous infusion of 200 mg/d of IV hydrocortisone until the
dose, or 60'7, to B0% of their long acting analogue dose or patient can resume oral glucocorticoids. Once oral glucocorti
pump basal insulin rate. coids are resumed, twice the usual daily dose should be admin
Postoperatively, if the patient is eating, the ideal insulin istered fbr 48 hours after surgery. Counter to these guidelines,
regimen is a basal bolus regimen, with nutritional coverage many experts suggest replacing the continuous infusion with
and correction boluses for pre meal hyperglycemia. Fbr a dis- intermittent dosing of 5O mg of hydrocortisone every 6 hours
cussion of the management of hyperglycemia in the hospital until oral intake is resumed. In addition, using lower doses
setting, see MKSAP 19 Endocrinologr and Metabolism. with induction for minor or intermediate-stress surgeries and
39
Perioperative Medicine
TABLE 31. Stress Dosing Strategies in Patients at Risk for Perioperative Management
Adrenal lnsufficiency
Patient Risk Patient Characteristics Management
of Kidney Disease
i
Patients with chronic kidney disease (CKD) are at increased
High risk Established primary or Stress dosing
perioperative risk for acute kidney injury (AKI). fluid and
secondary adrenal
insufficiency electrolyte imbalance, cardiac events, metabolic acidosis,
Cushingoid features anemia, and bleeding. In all patients with CKD undergoing
surgery it is important to ensure the preoperative stability of
Current equivalent of
>20 mg/d prednisone kidney function, volume status, and electrolytes. Iodinated
for >3 wk contrast dye and other nephrotoxic agents should be avoided
Moderate or Past use equivalent of Stress dosing in the perioperative period, and hypotension should be mini-
possible risk >20 mg/d prednisone for or mized to maintain renal perfusion pressure. For patients on
>3 wk during past year preoperative
hemodialysis, a nephrologist should be consulted for input
adrenal axis
Current or past equivalent on perioperative dialysate prescription, hemodialysis timing
testing may be
of >5-20 mg/d prednisone
indicated and heparin management, and adjustment of fluid removal
for >3 wk during past year
perioperatively.
Chronic high-dose inhaled or
topical glucocorticoid therapy Perioperative AKI portends an increased risk for long
Recent intra-articular
term decline in kidney function (including progression to
glucocorticoid therapy end stage kidney disease). The two most important means
(>3 injectable glucocorticoid of mitigating risk for AKI are maintenance of renal blood
treatments within 3 mo
before surgery)
flow and avoidance of further insults to the kidneys.
Renal blood flow is maintained by avoiding renal hypoper
Low risk Equivalent of <5 mg/d of No stress
prednisone for any duration dosing fusion ; effectively managing diuresis and antihypertensive
medications; and treating anemia, which may impair
Any dose steroid for <3 wk
peripheral vasodilation. Careful medication review is also
Low-dose inhaled or topical
glucocorticoid therapy
warranted to ensure appropriate dosing based on renal
clearance.
Adapted from Liu MM, Reidy AB, Saatee S, et al. Perioperative steroid
management: approaches based on current evidence. Anesthesiology. XEY POII'I
2011;127:1 66-72.IPMlD: 284528061 doi:1 0.1 097/A1N.0000000000001 659 and
Woodcock T, Barker P, Daniel S, et al. Guidelines for the management of o The two most important means of mitigating the risk
glucocorticoids during the peri-operative period for patients with adrenal
insufliciency: Guidelines from the Association o{ Anaesthetists, the Royal College for acute kidney injury in the perioperative period are
of Physicians and the Society for Endocrinology UK. Anaesthesia.2020;75:654 63.
IPMID: 3201 701 2] doi:1 0.1 1 1 1 /anae.1 4963 maintenance ofrenal blood flow and avoidance offur
ther insults to the kidneys.
40
Perioperative Medicine
41
Obesity
loss, and illness severity (among other factors), is critical to f,EY POIXT
mitigate complications. such as inf'ection and poor wound . Pregnant patients who require surgery should HVC
healing. Loss of 107, to l5'X, of total body weight during the
undergo the same preoperative medical evaluation as
preceding 6 months, a BMI less than 18.5. and a serum albu
nonpregnant patients; additional diagnostic testing is
min level less than 3.0 g/dl (30 g/L) indicate severe malnu-
unnecessary.
trition and are key predictive factors of poor surgical
outcomes. Patients with these features warrant preoperative
nutritional interventions with high-protein, high calorie
oral supplementation.
Finally, before any major surgery geriatric patients and
Obesity
their families should be educated on delirium risk and preven, Definition and Epidemiology
tion, potential fbr functional and cognitive decline, and pos Obesitll defined as a BMI of 30 orgreater. is one olthe big
sible need fbr skilled nursing at discharge. gest health problems in the United States and increases risk
For more information, see Hospital Medicine Principles. for mortality, type 2 diabetes mellitus, cardiovascular dis-
ease. obstructive sleep apnea, mental health disorders. liver
In women of childbearing ently associated with increased risk tbr dementia. The
age undergoing surgery a men.
United States is the most obese developed country,. rvith
strual history should be obtained and pregnancy testing per
rates of obesity at 39.8'1, and overweight (BMI of 25.0 29.9)
formed if pregnancy is possible.
at 71.8"/,.
Although high quality evidence is lacking and the
Waist circumference is another marker of health related
current body of evidence suggests that surgery does not
to obesity. Measurements at the top of the iliac crests of 102 cm
negatively affect obstetric or maternal outcomes. elective
(40 in) or greater in men and 88 cm (35 in) or greater in
surgery should be delayed until after pregnancy. If surgery
women is correlated with visceral adiposity and increased risk
cannot be delayed until after delivery, pertbrming surgery
for diabetes mellitus, cardiovascular disease, and all cause
during the second trimester is pref'erred, if possible.
mortality (Table 32).
Pregnant patients who require surgery should undergo a
standard preoperative medical evaluation; additional diag xtl PorxTs
nostic testing is unnecessary unless directed by the obste o Obesity increases risk for mortality, type 2 diabetes
trician. Modifications to surgical and anesthetic techniques mellitus, cardiovascular disease, obstructive sleep
may be required because of the anatomic and physiologic apnea, mental health disorders, liver disease, and
changes of pregnancy. Close collaboration among the many cancers.
obstetrician, surgeon, anesthesiologist, and internist is . Waist circumference of 102 cm (40 in) or greater in men
essential. Notably, pregnancy is considered a hypercoagu
and 88 cm (35 in) or greater in women is associated
lable state, and the ACCP recommends perioperative
with increased risk fbr diabetes mellitus, cardiovascular
mechanical or pharmacotogic VTE prophylaxis for preg
disease, and all-cause mortality.
nant patients.
Normal 18.5-24.9
Overweight 25.0-29.9 lncreased High
Obesity 30.0-34.9 I High Very high
3s.0-39.9 il Very high Very high
Extreme obesity >40 lil Extremely high Extremely high
normal weight
42
Obesity
43
Obesity
Evidence shows that any diet that achieves a calorie defi- also shown success. Continued visits for at least 1 year after
cit will produce weight loss. Those who choose low-fat options initial weight loss increase the chances of maintaining weight
should be vigilant about the sugar and sodium contents of loss.
these products. Although general teaching is that calories are
equivalent, calories listed on nutrition labels are measured in Pharmacologic Therapy
a calorimeter and may be processed differently physiologi Pharmacotherapy is an option for patients with BMI of 30 or
cally; calories from sugar-sweetened sodas may not be pro- greater, or with BMI of 27 or greater and at least one obesity
cessed in the same way as calories from leafy green vegetables. associated comorbid condition who have not achieved weight
Patients should be advised to gradually consume more loss goals with a trial of at least 3 to 6 months of lifestyle modi
guideline-advised healthy foods, including fruits, vegetables, fication. Weight is typically regained once these medications
legumes, nuts, whole grains, fish, and lean meats (see Routine are stopped and thus should be used alongside lifestyle modi-
Care of the Healthy Patient). Patients should be advised to fication. Pharmacotherapy is effective, but potential weight
choose a diet that they will be most likely able to maintain and loss benefit should be balanced against risk for adverse events
encouraged to find healthy foods they enjoy. and cost.
Very low calorie diets (<800 kcal/d) produce accelerated Table 34 describes the mechanism of action, expected
weight loss but require medical supervision. They should be weight loss, and side effects of commonly used agents; all have
reserved for situations requiring rapid weight loss, such as demonstrated higher rates of achieving 5% or greater weight
preparation for surgery. loss compared with placebo. Orlistat has the best long term
safeg data and Iowest rate of discontinuation due to side
Exercise effects. Liraglutide and phentermine topiramate produced the
Lifestyle interventions should include moderate- to vigorous- greatest weight loss but had higher rates of discontinuation
intensity physical activity for at least 150 minutes per week due to side effects.
and resistance training at least tvvice per week. Exercise does As with all other comprehensive weight loss interven
not contribute as much to initial weight loss as does reduced tions, patients should be monitored regularly. Patients who do
calorie intake, but long term commitment to regular physical not show weight reduction after 12 weeks of therapy should
activity is important for maintaining weight loss and improv- discontinue pharmacotherapy.
ing cardiovascular health. Because patients are often initially Patients are inundated with advertisements for over-the
deconditioned, gradual progression may be required, and it counter weight loss supplements and devices, which may
should be emphasized that any physical activity is better claim greater safety and effectiveness than prescription medi-
than none. cations. Forweight loss supplements, systematic reviews show
little evidence of effectiveness. Moreover, some supplements
Behavioral Therapy may be associated with significant adverse effects. Ma huang/
The Centers for Medicare & Medicaid Services allow payment ephedra (ephedrine), for example, has been associated with
for "intensive behavioral weight loss counseling" by primary myocardial infarction and stroke. Some weight loss supple
care providers. The content ofthis counseling in practice var- ments may also have additional undisclosed ingredients, most
ies widely, as does its effectiveness. Using motivational inter commonly sibutramine. Physicians should discuss the lack of
viewing techniques is encouraged (see Routine Care of the effectiveness and potential for side effects of supplements dur
Healthy Patient). Specific components associated with ing weight loss counseling and advise discontinuation.
increased effectiveness include a calorie deficit of at least Commonly used herbal weight loss supplements and potential
500 kcal/d, at least 150 minutes of moderate to vigorous physi, side effects can be found at https://ods.od.nih.gov/factsheets/
cal activity per week, and the use of trained interventionists Weightloss HealthProfessional/ and https://medlineplus.gov/
(nutritionists, behavioral therapists, or exercise therapists). druginfo /herb _ Al1. html.
Interventions should incorporate regular self-monitoring of
weight using a scale and calorie intake as well as education on Bariatric Surgery
controlling or altering the environment to avoid excess calorie Guidelines recommend surgery for patients with a BMI of 40
intake. Other interventions include removing calorie-dense or greater or for those with a BMI of 35 or greater who have
snacks and beverages from the home and workplace or replac- obesity associated comorbid conditions. Bariatric surgery reli
ing them with lower-calorie options, and engaging in alternate ably results in weight loss and may also produce improve
behaviors (walking, chewing gum) in situations in which the ments in diabetes control, blood pressure, and lipid profiles.
patient might be tempted to eat. Cardiovascular and overall mortality may also be improved in
High-intensity programs (>14 sessions over >6 months) patients with severe obesity.
delivered by trained interventionists are associated with suc The risks associated with bariatric surgery exceed those
cessful weight loss. Face to-face interventions most reliably associated with nonsurgical treatments. Therefore, candidates
result in weight loss and have the largest effects on weight, but for bariatric surgery should be selected carefully on the basis
interventions delivered electronically or by telephone have of risk-benefit analysis. Patients should have acceptable
44
Obesity
Orlistat Lipase inhibitor; decreases 2.6 kg (s.B lb) Oi ly stools, increased defecation,
triglyceride absorption fecal urgency/incontinence ( 1 .84)
Contraindications: pregnancy,
malabsorption syndrome,
cholestasis
Phentermine- Noradrenergic/GABA receptor B.B ks (19.4 lb) Paresthesias, dizziness, taste
topiramate activator and AMPA glutamate alterations, insomnia, constipation,
receptor inhibitor; suppresses dry mouth, tachycardia, cognitive
appetite changes (2.32)
Contraindications: g laucoma,
hyperthyroidism
Data from Khera R, Murad MH, Chandar AK, et al. Association of pharmacological treatments for obesity with weight loss and adverse events: a systematic review and meta-
analysis. JAMA. 201 6;315:2424 34. IPMID: 27299618] doi:1O.1O01/jama.2016.7 602 I
surgical risk, understand the necessity oflifelong dietary and Gastric banding involves placement of a silicone fluid
lifestyle measures for sustained weight loss, and be willing to filled band around the proximal stomach, creating a small
adhere to lifelong follow up. Candidates should not have psy stomach pouch with subsequent reduction in calorie intake by
chological or psychiatric conditions that impede adherence to increasing satiety. Gastric banding has greatly fallen in popu-
these requirements. larity owing to modest efficacy and high rates of revision and
has been largely replaced by sleeve gastrectomy.
Techniques All bariatric procedures result in loss of excess weight in
The most commonly performed bariatric surgical procedures the short term and up to 70'l. of excess weight with Roux-en-Y
are sleeve gastrectomy and Roux-en-Y gastric bypass. Sleeve and 60% with sleeve gastrectomy at 2 years. Long-term
gastrectomy has become the most common bariatric proce (5 year) data are less robust but suggest sustained weight loss.
dure in the United States. Sleeve gastrectomy is accomplished More recently, other surgical and nonsurgical procedures
by excising the part ofthe stomach along the greater curva have been developed. These include restrictive procedures (endo-
ture, creating an approximately 857, reduction in size. It scopic suturing or stapling in a manner that replicates sleeve gas
results in restriction of caloric intake via a smaller stomach trectomy), pills that expand in the stomach, devices intended to
and hormonal (glucagon-like peptide-l [GLP 1] and related decrease calorie absorption (duodenal jejunal liners), and intra
hormones) appetite suppression. The smaller gastric surface gastric balloons. Intragastric balloons are tlpically placed endo-
area also results in less production of ghrelin, an appetite scopically and are FDA indicated in patients with BMI of 30 to 40
stimulant. with one or more obesi$ related comorbidities. These and other
Roux-en-Y gastric bypass involves detaching the proximal related techniques are less invasive and may carry less risk than
stomach and creating a small pouch, which is reattached to a surgical procedures, but more data on long term outcomes are
limb of the small intestine. Weight loss results from decrease needed.
in calorie intake because of the small stomach pouch, malab
sorption due to bypassing much of the stomach and proximal Postoperative Care
small intestines, and appetite suppression due to changes in Rates of 30 day postoperative complications range from 1.3'1,
GLP 1 and related hormones. to 8.7"/,,. Complications include hemorrhage or leakage at the
45
\
Men's Health
anastomosis (bypass procedures) or staple line (sleeve gastrec- TAB LE 3 5 . Nutrient Deficiencies and Replacement After
tomy), venous thromboembolism, and bowel obstruction. Bariatric Surgeryo
Anastomotic leaks usually occur within the first week and Nutrient Routine Replacement
may present with nonspecific findings, including low-grade Deficiency Replacement Therapy if Deficient
fever, tachycardia, and respiratory symptoms. Barium swallow
lron Multivitamin with iron, Ferrous sulfate
or contrast-enhanced CT is recommended diagnostic testing or elemental iron 325 mg/d orally, or
when a leak is suspected; if suspicion is high and imaging is 40-80 mg/d orally; lV iron if oral is
take with vitamin C ineffective or not
negative, surgical exploration should be considered.
500 mg/d tolerated
Weight should be monitored closely in the early postop
Vitamin 812 Vitamin 812 500 pgld Vitamin B12 1000 pgld
erative period. In patients with
diabetes, sulfbnylureas orally, or 1000 gg lM orally, or 1000 pg lM
should be discontinued and insulin should be adjusted. As monthly monthly
patients lose weight, frequent reassessment of medications Folic acid Multivitamin with Folate 1 mg/d until
is required. folate replete, then
400-800 pg/d orally
Long term postsurgical care focuses on preventing and For women of
identifying nutritional deficiencies, managing adherence to childbearing age,
folate 1 mg/d orally
lifestyle modifications, and monitoring for behaviors that
lead to weight regain. Vitamin D deficiency is universal in Calcium Calcium citrate 1200-
1500 mg/d orally
patients after bariatric surgery leading to reduced intestinal
Vitamin D Vitamin D 400-1000 vitamin D 50,000 u
absorption of calcium and phosphorus with resultant sec
U/d orally weekly orally for 3 mo,
ondary hypoparathyroidism. Persistent secondary hyper then reassess
parathyroidism can last for years alter bariatric surgery and Thiamine Thiamine 50-100 mg/d 100 mg three times
annual monitoring of calcium, albumin, parathyroid hor orally daily until symptoms
mone, and 2S-hydroxyvitamin D levels is recommended. resolve
Annual monitoring of other nutrients, including vitamin B,r, Vitamin A Multivitamin daily Vitamin A 10,000 U/d
folate, iron, and ferritin is often recommended (see Routine orally with ongoing
monitoring
Care of the Healthy Patient). Other considerations for moni-
toring include thiamine, vitamin A, zinc, copper, and 24-hour Vitamin E Multivitamin daily Vitamin E 400 U/d
orally
urinary calcium. Table 35 lists the anticipated nutritional
deficiencies and recommended replacement strategies.
Vitamin K Multivitamin daily Mtamin K 10 mg/d
orally
Table 36 describes post bariatric surgery syndromes and
Copper Multivitamin with Copper 2-4 mg/d
their management. minerals daily orally
Tinc Multivitamin with Zinc22O mg/d orally
I(EY POIlIIS minerals daily,
HVC . Lifestyle modifications that are eflective lor the treatment
15 mg/d
of obesity include a calorie deficit of at least 500 kcal/d, Selenium Multivitamin with 2pg/kg/d if related
m ine ra ls cardiomyopathy
at least 150 minutes of moderate to vigorous physical
activity per week, and the use of trained intervention lM = intramuscularly.
ists (nutritionists, behavioral therapists, or exercise "See Table 1 5. All medications and supplements should be in liquid, crushed, or
chewableformforthel 2monthsaftersleevegastrectomyandforthefirst3months
therapists). after Roux en-Y gastric bypass.
o Pharmacologic therapy may be used as an adjunct to Data from Marcotte E, Chand B. Management and prevention of surgical and
nutritional complications after bariatric surgery. Surg Clin North Am. 201 6;96:843 56.
lifestyle modifications in patients with a BMI of 30 or IPMID: 274738051 doi: 1 0.1 01 6/j.suc.201 6.03.006
greater or in patients with a BMI of 27 or greater who
have overweight- or obesity-associated comorbid
conditions.
HVC . Systematic reviews show little evidence that over-the Men's Health
counter weight loss supplements are effective.
o Bariatric surgery should be reserved for patients with a
Male Sexual Dysfunction
BMI of 40 or greater or for those with a BMI of 35 or Erectile Dysfunction
greater who have obesity associated comorbid conditions. Erectile dysfunction (ED) refers to the inability to achieve or
o Long term postsurgical care focuses maintain an erection necessary for satisfactory sexual per
on preventing
formance. The most common causes are vascular disease:
nutritional deficiencies, managing adherence to lifestyle
neurologic disease, including stroke and postoperative
modifications, and monitoring for behaviors that lead
to weight regain.
nerve injury; medications; androgen deficiency; and
psychological issues.
46
Men's Health
Depression May be present preoperatively or develop Monitor for signs of depression and treat with
postoperatively multimodal approach
SSRIs may be less effective post bariatric
surgery due to reduced bioavailability
Regain of weight lost Disordered eating, excess intake of high-calorie Re-educate patient on diet modifications
liquid/semisolid foods or supplements
Refer to psychiatric care or counseling if
indicated
Gl = gastrointestinal; PPI = proton pump inhibitor; RCT = randomized controlled trial; SSRI = selective serotonin reuptake inhibitor.
Assessment of ED begins with obtaining a comprehen with ED include coronary atherosclerosis, diabetes mellitus,
sive history including medical, surgical, sexual, and psycho hyperlipidemia, hypertension, androgen deficiency, obesity,
'
social histories. Clinicians should clarify whether the problem spinal cord injury and prostate cancer therapy.
!
is attaining or maintaining erections, whether it occurs with Psychogenic ED, which is ED due to psychological or
t masturbation or with partners, and whether nocturnal and interpersonal factors, is common and may coexist with other
I
early morning erections are present. The International Index etiologies. ED that occurs with preserved ability to achieve
of Erectile Function (www.croesoffi ce.orglPortals / 0 /Short nocturnal and early morning erections should raise suspicion
I
5 IIEF.pdfl is a tool to rapidly assess ED symptoms and subse for psychogenic ED. Psychogenic ED is more common in
l quently monitor treatment effect. An accurate medication list younger patients and in those with a history ofsexual abuse.
t should be elicited because many medications are associated Physical examination should include assessment of
i
with ED (Table 37). Medical conditions commonly associated heart rate, blood pressure, and body mass index as well as
t
I
! 47
i
t
Men's Health
TABLE 37. Drugs CommonlyAssociated With Erectile although its use is associated with chronic fibrotic changes
Dysfunction (4.97, incidence) and injection site pain. Priapism is a side effect
; Antidepressants (monoamine oxidase inhibitors, selective of both formulations (approximately 1.9% incidence), and
. serotonin reuptake inhibitors, tricyclic antidepressants) proper patient education is required.
Benzodiazepines When pharmacotherapy is not an option, patients can be
offered a vacuum erection device. which has been shown to be
I Opioids, nicotine, alcohol, amphetamines, barbiturates,
, cocaine, marijuana both safe and effective. Implantation of a penile prosthesis is
Anticonvu lsants (phenytoin, phenobarbital) reserved for patients in whom other management options fail.
48
Men's Health
TABLE 38. Third Princeton Consensus Conference Guidelines for Treatment of Erectile Dysfunction in Patients with
Cardiovascular Disease or Cardiac Risk Factors
Risk Level Treatment Recommendation
Low Risk
Patients who are able to do moderate-intensity exercise without Can initiate or resume sexual activity ortreatfor ED with a PDE-5
symptoms inhibitor (if not using nitrates)
Successfully revascularized patients (e.g., coronary artery bypass
grafting, coronary stenting, or angioplasty)
Asymptomatic controlled hypertension
Mild valvular disease
Mild left ventricular dysfunction (NYHA functional class I or ll) who
can achieve 5 METs without ischemia as determined by recent
exercise testing
lntermediate/lndeterminate Risk
Mild to moderate stable angina Further cardiac evaluation and restratification before resumption
of sexual activity or treatment for ED
Recent Ml (2-8 wk ago)without intervention and awaiting exercise
ECG lf the patientcan complete 4 minutes of the standard Bruce
treadmill protocol without symptoms, arrhythmias, or a decrease
Heart failure (NYHA functional class lll)
in blood pressure, treatment for ED can be safely initiated
Noncardiac atherosclerotic disease (clinically evident PAD, history
of stroke/TlA)
High Risk
Unstable or refractory angina Defer sexual activity or ED treatment until cardiac condition is
stabilized and reassessed
Uncontrolled hypertension
Moderate to severe heart failure (NYHA functional class lV)
Recent Ml(<2 wk ago) without intervention
High-risk arrhythmia (exercise-induced ventricu lar tachycardia,
ICD with frequent shocks, poorly controlled atrial fibrillation)
ED=erectiledysfunction;lCD=implantablecardioverter-defibrillator;METs=metabolicequivalents;Ml=myocardialinfarction;NYHA=NewYorkHeartAssociation;
p{! = peripheral artery disease; PDE = phosphodiesterase; TIA = transient ischemic attack.
Recommendations from Nehra A, Jackson G, Miner M, et al. The Princeton lll Consensus recommendations for the management of erectile dysfunction and cardiovascular disease.
Mayo Clin Proc.2012;87:7 66-78.IPMID:22862865] doi:1 0.1 0 1 6/j.mayocp.20 1 2.06.0 1 5
considered only when sexual dysfunction is accompanied by vagina before ejaculation), and vasectomy. The efficacy of
documented androgen deficiency and after a discussion of these contraceptive options are described in Table 39.
risks and benefits of treatment with the patient. Vasectomy, or male sterilization, is an office surgical pro-
cedure performed under local anesthesia in which the vas
I(EY POITTS
deferens is either occluded or severed to prevent transport of
o Erectile dysfunction is a significant risk factor for coro
spermatozoa during ejaculation. Vasectomy, considered both
nary arteriosclerosis, and its presence without clear eti- highly safe and effective, appears to be underutilized com-
ologr should prompt evaluation for cardiovascular risk pared with female sterilization (tubal ligation).
factors. Men interested in undergoing vasectomy should be coun-
. Oral phosphodiesterase-s inhibitors are first line medi- seled that the procedure is intended to be permanent and that
cal therapy for erectile dysfunction; however, they are it does not result in immediate sterility. Patients must use
contraindicated in patients taking nitrates. another contraceptive until azoospermia is conftrmed by post-
vasectomy semen analysis. Complications are generally minor
and include local bleeding and infection. Asymptomatic sper-
Reproductive Counseling and matic granuloma formation is common but symptomatic
Male Sterilization granulomas are infrequent (approximately 5%); granulomas
Three forms of male contraception are available: male con typically resolve within 3 to 4 weeks with conservative treat-
doms, the withdrawal method (withdrawing penis from ment, such as NSAIDs. Vasectomy does not increase the risk
49
Men's Health
TABLE 39. Pregnancy Rates With Use of Male TABLE 40. Symptoms and Signs Suggestive of Androgen
Contraception Deficiency in Men
Women Experiencing Unintended More Specific Symptoms and Signs
Pregnancy Within First Year
lncomplete or delayed sexual development, eunuchoidism
of Use (%)
Reduced sexual desire (libido) and activity
Method Perfect Usea Typical Useb
(Efficacy) (Effectiveness) Decreased spontaneous erections
Withdrawal method 4% 20"/" Loss of body (axillary and pubic) hair, reduced shaving
50
Men's Health
51
Men's Health
are effective only in patients with an enlarged prostate. TABLE 42. Empiric AntimicrobialTherapy for Epididymitis
So-Reductase inhibitors function by preventing the conver, and Epididymo-orchitis
sion of testosterone to dihydroxytestosterone, inhibiting pros- Patient Group Therapy
tate growth. These agents must be taken for up to 6 months
Age <35 y lntramuscular ceftriaxone and
before symptomatic improvement may be seen. In patients oral doxycycline
with LUTS who have an enlarged prostate, combining a Age >35 y Oral levofloxacin
5o reductase inhibitor with an cr blocker appears to be more
Age >35 y and at risk for lntramuscular ceft riaxone and
effective than either agent alone. Side effects of 5a reductase chlamydia or gonococcal oral doxycycline
inhibitors include decreased libido, erectile dysfunction, infection
breast tenderness, and S/necomastia. Although several trials All men who practice lntramuscu lar ceftriaxone and
have shown reduced incidence of low-grade prostate cancer insertive anal intercourse oral levofloxacin
with So-reductase inhibitors, there appears to be a slight
increase in high-grade prostate cancer.
undergo imaging studies and instead be directly referred for
Antimuscarinic agents, such as trospium, oxybutynin, immediate exploratory surgery because surgical delay reduces
and tolterodine, can be used to treat bladder storage symptoms
the probability of testicular viability.
by relaxing bladder detrusor muscles. These are generally not
Epididymitis is usually secondary to infection: when
used as monotherapy but instead in combination with other
infection spreads to the adjacent testis, it is termed epididymo
agents.
orchitis. Both conditions typically cause unilateral scrotal pain
Saw palmetto is frequently used to treat LUTS secondary
ofgradual onset that worsens over a period ofdays. In younger
to BPH but lacks data supporting its efficacy. patients (age <35 years) , Chlamydia trochomotis and Neisserio
Surgical treatment for LUTS secondary to BPH, most gonorrhoeae are the most commonly implicated pathogens. In
commonly transurethral resection of the prostate, is usually men older than 35 years and those at low risk lor sexually
reserved for individuals who do not respond to pharmaco- transmitted infections, colonic bacteria, such as Escherichia
therapy or develop BPH complications, such as urinary reten coli, are the most common pathogens. Examination reveals
tion or persistent gross hematuria. tenderness and swelling of involved structures. Pain may be
XEY POITTS relieved with scrotal elevation. Urinalysis and culture should
. All patients with lower urinary tract symptoms should be obtained in all patients, as well as testing for sexually trans
52
Men's Health
53
Men's Health
TABLE 44. National lnstitute of Health Consensus normal, tender. and/or boggr. In patients $'ith an initially
Definition and Classification of Prostatitis negative urine culture, a repeat urine culture obtained after
Category Description prostate massage can increase the likelihood of identifuing the
causati\€ organism. Antimicrobial therapy is similar to that
Acute bacterial prostatitis
used for acute bacterial prostatitis and should be guided by
Chronic bacteria I prostatitis
urine culture results and continued fbr 4 to 6 weeks.
ill Chronic prostatitis/chronic pelvic pain More than 9O"/,, of patients with prostatitis have chronic
syndrome (A, inflammatory; B, noninflammatory)
prostatitis pelvic pain syndrome u,ithout evidence of infection.
Asymptomatic infla m matory prostatitis Symptoms vary widely and include chronic urinary symptoms.
Data from Krieger JN, Nyberg L Jr, Nickel JC. NIH consensus definition and pelvic pain, and pain with ejaculation. Although there is no
classi{ication of prostatitis ILetter]. JAM A 1999;282:236 7 .IPMID: 1042299A)
doi:1 0.1 001/jama.282.3.236 clearly established first-line treatment. cr blockers. NSAIDs.
and extended courses olantimicrobials with good prostatic tis
sue penetration are frequently used. Pharmacotherapl' should
are four categories of prostatitis (Table a ). Assessment of not be repeated if no improvement is noted after an initial
patients with suspected prostatitis begins with a thorough his-
course of appropriate duration. Approximately 30'li, of patients
tory including assessment of urinary and systemic symptoms u,ill improve with or without therapy
and urologic and sexual history. Vital sign assessment and Asymptomatic inflammatory prostatitis is usually diag
abdominal, genital, and digital rectal examinations should be nosed when urologic evaluation is performed for other indica
perflormed. Urinalysis and urine culture should be obtained in tions. such as prostate biopsy for an elerated PSA ler,el. The
all patients. significance of this form of prostatitis is unclear and. rt'hen
Patients with acute bacterial prostatitis typically present present without symptoms. does not require specific therapy.
with acute onset of local symptoms (dysuria, urinary fre-
quency and urgency, suprapubic andror rectal pain) and sys TEY POIilIS
temic symptoms (fevers, chills, nausea,,vomiting, malaise). . Oral ciprofloxacin, levofloxacin, and trimethoprim
Digital rectal examination reveals an enlarged, tender, and sulfamethoxazole are options for treating acute bacte-
often boggl prostate. Prostate massage may lead to bacteremia rial prostatitis; severely ill patients and those with
and should be avoided. Empiric antimicrobial therapy should suspected sepsis should be hospitalized and initially
be initiated at time of diagnosis and modified as appropriate treated with parenteral antibiotics.
on the basis of urine culture results. Most patients can be man . Although there is no clearly established first line treat-
aged with oral agents as outpatients, but severely ill patients ment for chronic pelvic pain syndrome, cr blockers,
and those with suspected sepsis should be hospitalized and NSAIDs, and extended courses of antimicrobials with
initially treated with parenteral antibiotics (Table 45). good prostatic tissue penetration are frequently used.
Antimicrobial agents are typically continued for 2 to 4 weeks;
some experts recommend up to 6 weeks.
Chronic bacterial prostatitis involves persistent prostate Hernias
infection lasting longer than 3 months. It commonly manifests A hemia is a protrusion of tissue through a defect in the sur
as recurrent lower urinary tract infections with the same rounding fascia. Inguinal hernias, which account for nearly
organism. On digital rectal examination, the prostate may be 75'X, of hernias in men, may be asymptomatic but may present
as groin pain and a scrotal or inguinal bulge. lndirect inguinal
TABLE 45. Empiric Antimicrobial Therapy for Acute hernias. the most common type, originate at the internal
Bacterial Prostatitis inguinal ring, pass through the inguinal canal and external
Oral Agents internal ring, and may descend into the scrotum. Direct ingui
Trimethoprim-sulfamethoxazole nal hernias originate near the external inguinal ring and usu
ally do not descend into the scrotum. Examination may reveal
Ciprofloxacin
a visible or palpable bulge in the groin or scrotum that is
Levofloxacin
accentuated by the Valsalva maneuver. Inguinal hernia diagno
lntravenous Agents" sis is usually clinical. and imaging studies are rarely needed.
Ciprofloxacin or levofloxacin (alone or combined with Small, asymptomatic or minimally symptomatic inguinal
gentamycin or tobramycin) hernias may be observed or undergo surgical repair; patients
Cefotaxime or ceftazidime (alone or combined with in whom repair is deferred should be advised to seek emergent
gentamycin or tobramycin) care if they develop increasing pain or symptoms of bort'el
Piperacillin/tazobactam (alone or combined with gentamycin obstruction. Large or bothersome inguinal hernias are usually
or tobramycin)
repaired surgically. Laparoscopic and open approaches have
'For patients unable to tolerate oral medication or who require hospltalization similar outcomes, but recovery time is shorter for the former.
ow ng to severlty of llness
Surgical complications of both methods include hematomas,
54
Women's Health
seromas, and wound infbction. Incarcerated or nonreducible TABTE 46.Breast lmaging Reporting and Data System
hernias can be reduced by applying pressure to the area with (Bl-RADS) Assessment Categories
the patient in the Trendelenburg position. If the hernia Category 0 Mammography: lncomplete-Need additional
remains nonreducible, then surgical evaluation should occur. imaging evaluation and/or prior mammograms
for comparison
Strangulation of a nonreducible hernia is a surgical emergency
because it represents vascular compromise of the entrapped Ultrasound and MRI: lncomplete-Need
additional imaging evaluation
hernia. Strangulation should be suspected when there is sig
nificant tenderness, overlying erythema, and accompanying Category 1 Negative
nausea and vomiting. Category 2 Benign
Athletic pubalgia (sports hernia) is caused by tearing of Category 3 Probably benign
groin muscle fibers and should be suspected in athletes who Category 4 Suspicious
participate in high intensity activities and are experiencing
Mammography and ultrasound:
hernia like symptoms but who have no palpable bulge on
Category 4A; Low suspicion for malignancy
examination. lnitial treatment consists of rest, physical ther
apy, and NSAIDs. Surgery is reserved for those in whom con Category 48: Moderate suspicion for
malignancy
servative measures fail.
Category 4C: High suspicion for malignancy
f,EY POITTS
Category 5 Highly suggestive of malignancy
HVC o Small, asymptomatic or minimally symptomatic ingui-
Category 6 Known biopsy-proven malignancy
nal hernias may be observed or undergo surgical repair.
Reproduced with permission of the American College of Radiology (ACR) {rom
o Strangulation of a nonreducible hernia is a surgical D'Orsi CJ, Sickles EA, Mende son EB, et al. ACR Bl RADS' Atlas, Ereast lmagrng
Reporting and Data System. Reston, VA, American College of Radiology; 201 3. No
emergency and should be suspected when there is sig other representation of this material is authorized without expressed, wriften
nificant tenderness, overlying erythema, and accompa- permissionfromtheACR.RefertotheACRwebsiteatwww.acrorg/Clinica Resources/
Reporting and-Data Systems/Bi-Rads for the most current and complete version of
nying nausea and vomiting. the Bl RADS' Atlas.
55
Women's Health
of a breast mass in women younger than 30 years. olrrlation. There is no increased risk for WE with progestin-
o only contraception.
A suspicious breast mass should be evaluated with a
The progestin based subdermal implant is placed in the
biopsy even if imaging results are negative.
arm and is approved for 3 years of use. It is the most effective
HVC . Cyclic breast pain often occurs in the premenstrual contraceptive option, with a failure rate belo$'0.05'X, at 1 year.
phase, tends to be bilateral, and resolves with onset of It is well tolerated but may cause irregular vaginal bleeding;
menstruation and conservative management. there is a small risk for infection or hematoma with both
(Continued)
insertion and removal.
56
Women's Health
57
Women's Health
Long- Acting Reu ersible Contraception after unprotected intercourse. Levonorgestrel is available over
Long acting reversible contraceptives are highly effective and the counter and is effective in the 3 days after intercourse. It is
include intrauterine devices (lUDs) and subdermal implants less effective in overweight women, and women with a BMI
(see Progesterone-Only Contraception). An IUD is a small, greater than 26 should use either the copper IUD or ulipristal
T-shaped device placed inside the uterus. There are five acetate.
FDA-approved IUDs, including one copper device and four
levonorgestrel releasing (LNG-IUD) devices. Both types work Preconception Care
by preventing fertilization and implantation. The copper IUD Preconception counseling refers to education provided before
releases copper ions, which are toxic to sperm; it is FDA pregnancy, aimed at ensuring a healthy pregnancy and reduc-
approved for up to 10 years of use. The LNG-IUD inhibits ovu- ing the risk for preterm birth and congenital anomalies. This
lation and thickens cervical mucus, obstructing sperrn pene differs from prenatal counseling, which occurs after preg
tration; it is FDA approved for 3 to 5 years ofuse, depending on nancy is diagnosed.
the brand. It is also highly effective at treating menorrhagia. Any primary care visit for a woman of reproductive age is
Changes in menstrual bleeding patterns are the most an oppoftunity to routinely ask whether she would like to
common adverse effect. Complications include expulsion (3%, become pregnant in the next year. Women who are not inter
6% in the first year) and uterine perforation (approximately ested in pregnancy should be offered contraception. For
0.1'1, of insertions). Risk for pelvic infection with IUD place- women considering pregnancy, a comprehensive risk assess
ment is low and prophylactic antibiotics are not recom ment should be completed (Table 48). An obstetric history of
mended during the procedure. If a sexually transmitted gestational hypertension, preeclampsia, or gestational diabetes
infection (STI) occurs with an IUD in place, the IUD does not is particularly predictive of future risk. Women at increased
need to be removed. Contraindications to IUD placement risk for preeclampsia should be counseled about the use of
include pregnancy, anatomic uterine abnormalities with dis- low dose aspirin in pregnancy forprevention.
tortion of the uterine cavity, and acute untreated pelvic
infection. TABLE 48. Preconception Risk Assessment
Risk Category Specific ltems to Assess
Barrier Contraception
Family planning Desire for pregnancy; number and timing
Barrier contraceptive methods, such as diaphragms and con
and pregnancy of desired pregnancies; age-related
doms, are among the least effective of all the modes of contra spacing changes in fertility, sexuality, contraception
ception. Their efficacy is improved when combined with a Exposure to radiation, lead, and mercury
Environmental
spermicidal agent. Use of either male or female condoms hazards and toxins
reduces the risk for STIs. However, the use of spermicide may Nutrition and Healthy diet; folic acid supplementation;
increase risk for HIV transmission by disrupting the vaginal folic acid recommended daily intake of iron;
epithelial wall. consumption restricting consumption of fish with high
mercury levels (e.g., shark, swordfish, king
mackerel, and tilefish)
Permanent Contraception
Genetics Family history of genetic disorders
There are several methods of female sterilization. Options
include salpingectomy and clipping or electrosurgical desic- Substance use Use of tobacco, alcohol, marijuana, and
illicit drugs
cation of the fallopian tubes; salpingectomy may be associ
Medical Diabetes mellitus, hypertension, thyroid
ated with a decreased risk flor ovarian cancer. Women who
conditions disease, HIV infection, bariatric surgery,
are considering sterilization should be counseled regarding mood disorders, thrombophilia, asthma,
procedural risk, the permanency of the method, and the seizure disorder
availability of alternative long-acting reversible contraceptive Obstetric history Gestational hypertension, preeclampsia,
methods. gestational diabetes
Medications Over-the-counter and prescription
Emergency Contraception medications, potential teratogens
Emergency contraception is contraception using a device or lnfectious Vaccinations up to date; immunity to
medication to prevent pregnancy after inadequately protected diseases and varicella and rubella; risk for HIV and
vaccinations hepatitis B infection; need for STI
intercourse. The most effective form of emergency contracep- screening; potential exposure to Zika virus
tion is the placement of a copper IUD within 5 days of inter-
Psychosocial Depression, interpersonal/family
course, which can reduce the risk for pregnancy by 99'1,,. concerns relationships, exposure to violence, risk
Recent data indicate that the 52-mg LNG IUD is noninferior to for abuse (physical, sexual, emotional)
the copper IUD for emergency contraception; however, it is STI = sexually transmitted infection.
not currently approved for this indication. Two oral contracep
lnformation from American College of Obstetricians and Gynecologists. ACOG
tive options are ulipristal acetate and levonorgestrel. Ulipristal Committee Opinion No. 762 Summary: Prepregnancy counseling. Obstet Gynecol.
2019;133:228 230. IPMID: 30575672] doi:1 0.1 097IAOG.000000000000301 4
acetate requires a prescription and is elfective for up to 5 days
58
Women's Health
TABLE 49. Commonly Used Medications With Potential information regarding Zika virus and pregnancy (wwr,rr.cdc.
for Teratogenic Effects gov I zika I pregnancy/index. html).
59
Women's Health
60
Women's Health
history. Physical examination should assess for evidence of nausea, vomiting, and diarrhea. Dysmenorrhea is classified as
acute bleeding. Pelvic examination can determine the source primary or secondary. Primary dysmenorrhea involves cramp
of the bleeding, assess for trauma to the genital track, and look ing lower abdominal and pelvic pain that occurs during
for uterine enlargement or irregularity. Initial laboratory test menstrual cycles without an identifiable cause. Secondary
ing includes a pregnancy test and complete blood count. dysmenorrhea may result from pelvic conditions, of which
Additional testing should be based on clinical indications and endometriosis is the most common.
may include coagulation tests, TSH level, iron studies. and A detailed history is necessary to assess the timing of pain
hormone levels. and relationship to the menstrual cycle. Sexual history is
Transvaginal ultrasonography is often perfbrmed to assess important to assess for infection risk. Primary dysmenorrhea
for structural abnormalities and endometrial thickness. can be treated with NSAIDs or contraception, including
Endometrial thickness greater than 4 mm in postmenopausal estrogen progesterone and progesterone only methods.
women may indicate endometrial hyperplasia or malignancy Secondary dysmenorrhea requires treatment of the underly-
i
and necessitates biopsy. Proceeding directly to biopsy without ing condition.
ultrasound is also a reasonable alternative in postmenopausal
I(EY POIilTs
women with uterine bleeding.
In premenopausal women, endometrial thickness is not a o Evaluation of all patients with abnormal uterine bleed,
reliable indicator because thickness varies according to the phase ing includes a detailed history physical and pelvic
of the menstrual cycle. Regardless of endometrial thickness, examinations, and pregnancy testing and complete
endometrial biopsy should be done in premenopausal women blood count; additional testing should be based on
aged 45 years or older or those younger than 45 years who are at clinical indications.
increased risk for endometrial cancer secondary to a history of . Risk factors for endometrial cancer include age older
unopposed estrogen (polycystic ovary syndrome, obesity); than 45 years; anovulatory bleeding; tamoxifen use;
tamoxifen use; Lynch or Cowden syndrome; or reproductive fac- Lynch or Cowden syndrome; obesity; and reproductive
tors, such as early menarche, nulliparity, and late menopause. factors, such as early menarche, nulliparity, and late
menopause.
Treatment o Primary dysmenorrhea can be treated with NSAIDs or
Management of abnormal uterine bleeding is aimed at the estrogen-progesterone or progesterone-only methods of
underlying cause. Structural abnormalities, such as endome- contraception.
trial polyps or submucosal fibroids, may be surgically resected.
Treatment of underlying endocrine disorders (thyroid disease,
polycystic ovary syndrome) may result in improvement.
For women with anovulatory cycles, treatment is aimed
Menopause
at providing adequate progestin to maintain endometrial sta- Diagnosis
bility. The type of therapy depends on the patient's plans for Menopause is the permanent cessation of menses. It is diag
contraception. I,br women who wish to preserve fertility, nosed retrospectively after a woman has not experienced a
medroxyprogesterone acetate used fbr the second half of the menstrual period for 12 months. Menopause may be natural;
menstrual cycle will restore cyclic withdrawal bleeding. For surgical, after bilateral oophorectomy; or medical, as a result
women interested in contraception, oral contraceptive pills of such treatments as chemotherapy. Menopause occurring
containing estrogen and progesterone, the LNG-IUD, or depot before age 40 years is considered premature menopause or
medroxyprogesterone may be used. NSAIDs may also be used premature ovarian insufficiency. Perimenopause refers to the
because these drugs reduce synthesis of prostaglandins in the years preceding menopause when ovarian function declines
endometrium, leading to vasoconstriction and reduced bleed and is often associated with anovulatory cycles and hot flashes.
ing. Tranexamic acid, an antithrombolytic agent, stabilizes Symptoms of menopause include vasomotor symptoms
clots and may be used for treatment of heavy vaginal bleeding (hot flashes, night sweats). Hot flashes generally start in the
and in women who cannot tolerate hormonal treatment. perimenopausal period and can last lor a few years to lif'elong.
For women experiencing acute and severe bleeding, such Women may experience other symptoms, such as depression,
agents as gonadotropin-releasing hormone or intravenous anxiety, and irritability, but it is unclear whether these symp
administration of high dose estrogens may be used. For toms can be attributed to menopause. Genitourinary syn
women who do not respond to medical therapy, operative drome of menopause results from estrogen deficiency and is
procedures, such as endometrial ablation, uterine artery characterized by vaginal symptoms, such as burning or irrita
embolization, or hysterectomy, may be performed. tion; sexual symptoms, such as dyspareunia or sexual dys
function; or urinary symptoms, such as dysuria or recurrent
Dysmenorrhea urinary infections. Pelvic examination findings include a pale,
Dysmenorrhea is characterized by pain during menstruation; dry vaginal lining with reduction in rugae. Increases in LDL
itmay also be associated with low back pain, headache, cholesterol and bone loss may also occur (Figure 4).
61
Women's Health
Step 3: Assess the patient's baseline risk for CVD, WE, and !
menopausal symptoms. i
!
Step 5: Add systemic progesterone therapy to estrogen therapy l
1
Routine laboratory testing for the diagnosis of menopause Step 7: Reassess symptoms and risk factors {or CVD, WE, and l
is not recommended. Patients with possible early menopause breast cancer annually.
should have pregnancy excluded and undergo measurement Step 8: Discontinue systemic hormone therapy if the risks of
of lollicle stimulating hormone, TSH, and prolactin. Younger treatment outweigh the benefits. 1
62
Women's Health
Hormonal preparations include estradiol or conjugated Other laboratory testing may include STI testing, complete
estrogen in tablet, cream. or ring fbrnrs. Low dose vaginal blood cell count, and type and screen depending on the clini
estrogen is recommended for the management of vaginal atro cal scenario. Transvaginal ultrasonography can help diagnosis
phy because it builds the vaginal epithelium and restores the ectopic pregnancy, ovarian cysts, ovarian torsion, and tubo
acidic pti and microenvironment. Moreover, improving the ovarian abscess as a complication of PID.
lining of the lower urethra reduces dysuria and recurrent uri The management of acute pelvic pain depends on the
nary tract infection. For women with breast cancer (current or diagnosis. Women with ectopic pregnancies should be evalu
past), low-dose vaginal hormone therapy should be given only ated immediately by an obstetrician fbr consideration of'
with the approval ofthe treating oncologist. Vaginal dehydro surgical management versus medical management with
epiandrosterone and the selective estrogen receptor modulator methotrexate. The treatment ol PID is covered in MKSAP 19
ospemifene are both FDA approved fbr management of dys Inlectious Disease. Gynecologr should be involved in the man
pareunia associated with GSM; most experts consider these agement of ovarian cysts, particularly fbr a ruptured hemor
treatment modalities second line therapies because of their rhagic cyst in a patient with unstable vital signs and ovarian
side effects and limited safety experience. torsion, which is considered a surgical emergency. Appendicitis
requires surgical consultation.
I(EY POIilT5
HVC . Routine laboratory testing for the diagnosis of meno Chronic Pelvic Pain
pause is not recommended; patients with possible early Chronic pelvic pain (CPP) refers to nonmenstn-tal pelvic pain
menopause should have pregnancy excluded and of at least 6 months'duration that is severe enough to result in
undergo measurement of follicle-stimulating hormone, tunctional disability requiring medical care. Evaluation and
thyroid-stimulating hormone, and prolactin. diagnosis of CPP are challenging because it is often rnultifacto
. In the perimenopausal period, pregnancy is possible rial and involves both physiologic and psychological compo
even if menstrual cycles are irregular, and contraception nents. Risk flactors ftrr CPP include physical, sexual, and/or
should be discussed ifpregnancy is not desired. enrotional abusel PID; history of abdominopelvic surgery;
. Estrogen is the most effective therapy for vasomotor chronic pain syndromes; and psychological conditions, such
symptoms of menopause. as anxiety or depression.
Common causes o{ CPP can be classilied as grnecologic,
. Nonhormonal options for vasomotor symptoms of
gastrointestinal, urologic, and musculoskeletalrneurologic and
menopause include antidepressant agents, gabapentin,
are outlined in Table 53.
pregabalin, and clonidine.
Evaluation includes a detailed history to determine char
o Management of genitourinary sl,ndrome of menopause acteristics of the pain, including association with menstrual
includes topical nonhormonal and hormonal preparations cycle, urination, or bowel movement, as well as assessment fbr
to achieve symptom relieL risk factors and known causes of CPP Physical examination
includes detailed abdominal and grnecologic examinations.
Laboratory testing is of limited value unless specific disorders
Pelvic Pain are suggested by the history and clinical examination (such as
Acute Pelvic Pain urinalysis for urinary tract infection or STI testing).
Although the differential diagnosis of acute pelvic pain is Transvaginal ultrasonography is used to identify anatomic
broad, clinicians should consider several critical diagnoses, pathologr, such as a pelvic mass. In the absence of an abnor
including ectopic pregnancy, pelvic inflammatory disease mality noted on clinical examination or ultrasound, iaparos
(PID), ruptured ovarian cyst, ovarian torsion, and appendicitis, copy may be helpful fbr the evaluation of severe symptoms of'
when evaluating a woman with acute pelvic pain. unclear cause to assess for conditions such as endometriosis.
History should elicit the characteristics of the pain, Treatment is aimed at the underlying cause and may be
including location, radiation, severity, timing, duration, and challenging if no clear cause is identified. Menstrual suppres
alleviating and aggravating factors. N4edical history should sion with hormonal contraception can help many patients.
note previous abdominal or gznecologic diagnoses or surger Pelvic floor physical therapy, cognitive behavioral therapy. and
ies. A sexual history should be obtained for all patients. The sex therapy are recommended. Serotonin-norepinephrine
physical examination should focus on vital signs, abdominal reuptake inhibitors, gabapentin, and pregabalin may be help
examination, and pelvic examination, specifically a bimanual ful in patients with neuropathic pain. Trigger point injections
examination to assess for fundal and adnexal tenderness or should be considered in myofascial chronic pelvic pain.
MASSES. Acupuncture and yoga may be helpful tbr some patients.
A pregnancy test should be obtained for all women of NSAIDs may be helpful for patients with moderate CPP.
reproductive age presenting with acute pelvic pain. Any Routine surgical lysis ofadhesions is not indicated. Peripheral
woman with a positive pregnancy test should have ectopic nerve block or surgical release can also be used fbr suspected
pregnancy excluded before other diagnoses are considered. nerve impingement or entrapment.
63
Women's Health
TABLE 53. Common Causes of Chronic Pelvic Pain include lack of or significant decrease in interest in sexual
Gynecologic Causes activit),. sexual,erotic thoughts or fantasies, excitement or
pleasure during sexual activiry and/or initiation ofsexual activ
Endometriosis
ity or responding to a parlner initiation. Female orgasmic disor-
Pelvic adhesions
der is characterized by a significant delay in. infrequency of, or
Chronic pelvic inflammatory disease
absence of orgasm, or significantly decreased intensity of
Adenomyosis orgasm during sexual activity. Genitopelvic pain penetration
Vulvodynia disorder is characterized by persistent or recurrent dilficultl
Ovarian cysts having sexual intercourse; experiencing vulvovaginal or pelvic
Uterine (fibroid) pain during intercourse; fear or an-xiety about urlvovaginal or
Urologic Causes pelvic pain; and/or tensing or tightening of pelvic floor muscles
lnterstitial cystitis/painful bladder syndrome during attempted vaginal penetration. Diagnosis of a sexual dis
Radiation cystitis
order requires that the symptoms of each disorder occur more
than 75'){, oFthe time for at least 6 months and cause clinicallv
Urethral syndrome
significant distress.
Urinary calculi
Beyond the three categories of female sex'ual dysfunction.
Gastrointestinal Causes
\\''omen may erperience sexual dysfunction u,ith and after preg
lrritable bowel syndrome nancy. Perinatal events. such as cesarean or instrument delivery
lnftammatory bowel disease or perineal tears, may cause pelvic pain. Estrogen levels are lo\\er
Chronic constipation during breastfeeding, which may result in vaginal dryness.
Diverticulitis Postpartum depression may contribute to decreased sexual desire
Celiac disease or frequency. Menopause and GSM may also affect sexual func
Musculoskeletal/Neurologic Causes tion because vaginal dryness may contribute to pain $,ith vaginal
intercourse.
Myofascial pain syndromes
When assessing sexual function. it is important to realize
Fibromyalgia
that sexual functioning involves multiple components. includ
Chronic coccygeal pain
ing emotions, relationships, past experiences. physiologic
Chronic low back pain
responses, overall health, and personal beliefs; therefore, a
Piriformis syndrome comprehensive approach is needed to evaluate and treat the
Neuralgia condition.
Assessment of female sexual dysfunction includes detailed
f,EY POI ]III questions regarding the patient's sexual and gender identitlr
. Women presenting with acute peMc pain should be symptom characteristics. including duration. acute or gradual
evaluated for critical diagnoses, including ectopic preg- onset, and ifassociated with a specific partner or a generalized
nancy, pelvic inflammatory disease, ruptured ovarian concern; association with painl precipitating events; and level
cyst, ovarian torsion, and appendicitis. of distress caused by the symptoms. [n addition. assessment
. should include current lif'e stressors: medical and surgical his
A pregnancy test should be obtained in all women of
tory; medications; and history of physical, emotional. andior
reproductive age presenting with acute pelvic pain.
sexual abuse. Screening lbr concurrent depression is indicated.
o Risk factors for chronic pelvic pain include physical, The Female Sexual Function Index or the Female Sexual
sexual, and/or emotional abuse; pelvic inflammatory
Distress Scale can be used during the assessment. Pelvic exam-
disease; history of abdominopeMc surgery; chronic
ination is aimed at assessing for specific sites of pain or tender
pain syndromes; Bnd psychological conditions, such as
ness. vaginal dryness. or atrophy,. Laboratorl' testing is of
anxiety or depression.
limited value and is performed only if an underlying cause is
suspected.
Female Sexual Dysfunction Therapy is aimed at the underlying cause of female sex-
Female sexual dysfunction is characterized by persistent or ual dysfunction, which is often multifactorial. There are two
recurrent distressing sexual concerns or difficulties and is FDA approved medications for female sexual dysfunction.
likely underappreciated. According to the DSM-S, female sex Flibanserin, a serotonin receptor agonist,antagonist, is
ual dysfunction is divided into three categories: female sexual approved fbr premenopausal women with female sexual
interest/arousal disorder, female orgasmic disorder, and gen interestlarousal disorder without depression. Its use is limited
ito-pelvic pain/penetration disorder. secondary to side effects (dizziness, syncope, hypotension,
Female sexual interest/arousal disorder is characterized somnolence); incidence of side effects is increased when com
by a lack of or signiflcant decrease in sexual activity. This may bined with alcohol or certain medications (antidepressants).
64
!
L
I Women's Health
i
1
65
Women's Health
"Safe in pregnancy.
bUncomplicated vulvovaginal candidiasis: Candida albicans, mild to moderate symptom severity, healthy nonpregnant women, four or fewer episodes per year.
mellitus, or immunosuppression. Only topical azo e therapres, applied for 7 days, are recommended for use among pregnant women.
TreatmentregimensfromtheCDCSexuallyTransmitted n{ectionsGuidelinesathttps://www.cdc.gov/std/tg2015/default.htm.AccessedJone17,2021.
65
Eye Disorders
l( EY P 011{ TS konttnucd)
o Diagnostic testing for vulvovaginal candidiasis involves
l0'2, potassium hydroxide wet mount showing yeast,
hyphae, or pseudohlphae.
. Nucleic acid amplification testing (NAAT) and antigen
testing are routinely available to diagnose bacterial vagi-
nosis and'"ulvovaginal candidiasis; NAAT is the preferred
diagnostic test for trichomoniasis.
Eye Disorders
t IGU R E 6.
Candida vaginitis showing buddin g yeast (b/u e arrow\,spores lblack
Eye Emergencies
arrow\, and elongated spores appearing as pseudohyphae ( red arrow)with lnternists must accurately diagnose conditions that can cause
potassium hydroxide.
permanent vision loss and require immediate ophthalmologic
evaluation (Table 55).
infection with a non-albicans Condidc species requiring dif Patients with periocular trauma and potential globe
ferent treatment. injury should undergo visual acuity assessment and anterior
Asymptomatic patients should not receive treatment. See eye segment evaluation with either a penlight or slit lamp.
Table 5,1 for treatment options for symptomatic uncomplicated Findings that suggest globe rupture include pupil deviation
and complicated vulvovaginal candidiasis. Recurrent symp- toward the laceration site, anterior chamber depth loss and
toms after treatment warrant an evaluation for underlying blood collection (hyphema) (Figure 7), and subconjunctival
predisposing Factors. l-remorrhage completely surrounding the cornea. Patients
witl.r suspected globe injury should not undergo tonometry
Trichomoniasis or pupil dilation and should have an eye shield placed over
Trichomoniasis is a vaginal infection caused by Trichomonos the af'fected eye. When globe injury occurs or there is uncer
uoginolis. a flagellated protozoan, and is a sexually trans tainty about globe integrity, emergent referral to ophthalmol
mitted infection. It is associated with increased risk for ogr is indicated.
preterm labor in pregnancy as well as increased risk for HIV Chemical injury to the eyes causes decreased vision, eye
transmission. pair-r and burning, foreign body sensation, and tearing.
Women may be asymptomatic or present with copious Examination may reveal eyelid swelling and erythema, con
vaginal discharge that is malodorous; pale yellow, green, or gray; junctival injection, blepharospasm, and corneal defect. Eye
fiothy; and associated with vulval itching and burning. Vaginal irrigation with normal saline or lactated Ringer solution
pH is often elevated, but this is not helpful diagnostically.
Although microscopy with examination of a wet mount of the
vaginal fluid showing motile trichon-ronads can be diagnostic, TABLE 55. Selected Eye Conditions Requiring lmmediate
Ophthalmologic Evaluation
the CDC recommends nucleic acid amplification testing. When
Trichontonas is identifled, testing lor other STIs should be con Acute angle-closure glaucoma
sidered. See Table 54 for treatment of trichomoniasis. Women Cenlral retinal artery occlusion
should be retested within 3 months of treatment. Treatment of Central retinal vein occlusion
sexual partners is recommended to prevent reinfection. Chemical injury
l(EY POt l{rS Corneal ulcers
the common causes of vaginitis, and laboratory testing Hyperacute bacterial conjunctivitis
is necessary to establish a diagnosis in bacterial vaginitis Ke ratiti s
or trichomoniasis.
Optic neuritis
o Characteristic findings of bacterialvaginosis includes a Orbitalcellulitis
thin vaginal discharge, vaginal pH greater than 4.5, a
Retinal detachment
positive whiff test result in which application of 10%
Scleritis
potassium hydroxide to vaginal secretions results in a
fishy odor, and the presence ofclue cells. Trauma
(Continued) Uveitis
67
Eye Disorders
Conjunctivitis
Conjunctivitis is the most common cause of red eye and can be
due to infectious or noninfectious etiologies.
Viral conjunctivitis is commonly spread through direct
contact and often occurs concomitantly with upper respira
tory tract infection. Adenovirus, coxsackievirus, and enterovi
rus are frequently implicated pathogens. Symptoms typically
start in one eye before spreading bilaterally. Therapy is sup FI G U R E 8. Hyperacute bacterial conjunctivitis due to Neisseria gonorrhoeae
portive and includes cold compresses; antibiotics are not demonstrating copious purulent drainage and conjunctival swelling.
68
Eye Disorders
'Ciliary flush is a violet hue surrounding and circumferential to the cornea due to dilated episcleral and deep conjunctival vessels.
virus and adenovirus. Diminished vision and severe eye dis- systemic NSAIDs for symptom relief. Patients with recurrent
comfort with circumferential hyperemia are tlpical (Figure 9 episodes should be evaluated for underlying disease.
and Figure 10). Bacterial keratitis requires immediate ophthal- Scleritis is an inflammatory disorder of the sclera that
mologic evaluation. causes severe pain and, unlike episcleritis, can threaten vision
(Figure 12). Scleritis is often associated with underlying
Episcleritis and Scleritis inflammatory diseases, such as rheumatoid arthritis and sys-
Episcleritis is a self-limited inflammation of the episclera temic vasculitis. Diagnosis should prompt immediate referral
(layer between sclera and conjunctiva) that usually resolves to an ophthalmologisti systemic glucocorticoids or immuno
within several weeks (Figure l1). Treatment consists of suppressive agents are usually required for treatment.
69
Eye Disorders
70
Eye Disorders
Dry Eye
Dry eye (keratoconjunctivitis sicca) is a multifactorial inflam-
matory disorder and results from abnormal eye lubrication.
Normal tear film consists of an aqueous component formed by
lacrimal glands, lipids produced by meibomian glands, and
mucins produced by conjunctival goblet cells. Abnormalities
in any component of the film can lead to dry eyes. In most
cases, ocular surface inflammation additionallyplays a central
role. Systemic medications, such as p-blockers, antihista
mines, antipsychotics, and antineoplastics, can also cause
dry eye.
FIGURE 15. Subconjunctival hemonhageischaracterizedbyunilateral, localized, $rmptoms are uzually bilateral and include ocular pain, for-
and sharply circumscribed redness without discharge or pain. eigrr body sensation, light sensitivity, tearing, irritation/redness,
I
I
71
Eye Disorders
f,IY POITT
. Treatment of dry eye includes artificial tears and, occa-
sionally, punctal plugs; topical cyclosporine; warm
compresses; and lifestyle changes (such as reduced
screen time and increased fluid intake).
disc) and cupping identified by the disappearance of vessels over the edge of the
Corneal abrasions are superficial epithelial defects that result attenuated optic rim are characteristic of glaucoma.
72
Eye Disorders
Normal Abnormal
Cornea
lris
Lens
73
Eye Disorders
I
.i
Retinal Detachment
Retinal detachment involves separation of the retinal neurosen-
sory layer from underlying structures and is an ocular emergenry
requiring immediate referral to an ophthalmologist (Figure 21).
ffiptoms include unilateral flashing lights and floaters. Risk
factors include myopia, cataract surgery retinal lattice degenera
tion and retinalbreaks, and positive family history
't
Retinal Vascular Occlusion
Retinal Artery Occlusion
Central retinal artery occlusion (CRAO) presents as sudden,
painless severe monocular vision loss. Occlusion of smaller
branches (branch retinal artery occlusion) leads to less pro :
found visual field defects. CRAO is most commonly due to
FIGURE 20. Atrophic(dry)age-related maculardegeneration,showing distinct carotid artery atherosclerosis but can also be due to cardiogenic
yellow-white lesions called drusen (arows) surrounding the macular area and areas
embolism, thrombophilia, and giant cell arteritis. Some affected
of pigment mottling.
patients have a history oftransient episodes ofpainless vision
loss. Fundoscopic examination may reveal retinal pallor, a
inhibitors are a cornerstone ofadvanced disease treatment and "cherry red" fovea, and intemrpted blood columns within reti
may improve vision or delay deterioration. nal vessels. CRAO is an ocular emergency requiring urgent
referral to an ophthalmologist.
fEY
'OtTI is the largest modifiable risk factor for
o Smoking age- Retinal Vein Occlusion
related macular degeneration. Central retinal vein occlusion (CRVO) is usually due to primary
thrombus formation, whereas branch retinal vein occlusion
(BRVO) is usually due to compression of a vein by adjacent
Optic Neuritis arterioles. CRVO typically presents as painless, blurred
Optic neuritis is a demyelinating inflammation of the optic monocular vision. BRVO may be asymptomatic or cause sco
nerve and is most commonly due to multiple sclerosis. It toma or visual field defect. Examination may reveal an afferent
manifests as painful acute monocular vision loss that develops pupillary defect (particularly in CRVO), retinal vein conges-
over a period of hours to days. Examination may reveal papil- tion, retinal hemorrhage, and cotton-wool spots (Figure 22).
litis (see MKSAP 19 Neurologz). An afferent pupillary defect is Risk factors for CRVO include advanced age, diabetes mellitus,
common, as is a central scotoma on visual field testing. and hypertension. Presence of CRVO should prompt immedi-
Diagnosis is clinical, although MRI may confirm diagnosis and ate referral to an ophthalmologist.
74
t
t
Ear, Nose, Mouth, and Throat Disorders
t
I
L
t
t
t
L
I I
t I *
L
t
t
L
FIGURE 22.Centralretinalveinocclusionischaracterizedbyopticdiscswelling,
L dilated and tortuous veins, and flame-shaped retinal hemorrhages (white arow)
and cotton'wool spols(yellow anow) ("blood and thunder').
t
L
L o Central retinal artery occlusion is an ocular emergency F IGUR E 2 3. Evidence of tympanic membrane retraction, perforation, and
that presents as sudden, painless severe monocular accumulation of squamous debris consistent with cholesteatoma.
t
vision loss and is most commonly due to carotid artery
L atherosclerosis. high-dose aspirin, and NSAIDs may cause hearing loss.
Although hearing loss is associated with loop diuretics, it is
o Central retinal vein occlusion typically presents as pain-
L often temporary in the absence of other ototoxic medications.
less blurred monocular vision, whereas branch retinal
I
Genetics is implicated in hearing loss and plays a role in 25%
L vein occlusion may be asyrnptomatic or present with
I to 50% of adult onset hearing loss.
scotoma or visual field defect.
t According to the U.S. Preventive Services Task Force rec-
I
ommendations, evidence is insufficient to assess the benefits
t
I
and harms ol screening for hearing loss in asymptomatic
i Ear, Nose, Mouth, and adults aged 50 years or older. Simple and effective tests, such
as evaluating whether a patient can hear a whispered voice or
5
Throat Disorders single question screening (for example, "Do you have diffi-
i culty with your hearing?"), can be performed in the office.
Hearing Loss Additional in office screening maneuvers are described in
t
An estimated 50'/. of adults aged 60 to 69 years in the United Table 59. In patients with identified hearing loss, physical
States have hearing loss that affects their day-to-day commu- examination should include otoscopic examination as well as
nication. Hearing loss is underrecognized by patients and the Weber and Rinne tests (T[ble 60) to differentiate conduc-
physicians and can negatively affect relationships and lead to tive from sensorineural hearing loss. Routine referral to an
social isolation and reduced quality of life. It is associated with audiologist is indicated when hearing loss is identified.
higher rates of unemployment, depression, dementia, hospi Imaging (typically MRI) is rarely required unless the hearing
talization, falls, functional decline, and death. Ioss is acute and unilateral or associated with other neurologic
Hearing loss is categorized according to the anatomic changes. Sudden-onset hearing loss (within
72 hours) requires
deficit: conductive (outer or middle ear), sensorineural (inner more urgent referral to an audiologist and MRI of the retroc-
ear), or mixed (Table 58). Conductive hearing loss is caused by ochlear structures.
obstruction or pathologr of the outer or middle ear (such as Treatment options for hearing loss include hearing-
cholesteatoma, Figure 23) that prevents sound transmission assistive devices, hearing aids, and cochlear implants. Many
and is more often associated with pain or ear drainage. varieties of hearing-assistive devices exist, and they can be
Sensorineural hearing loss is caused by disorders of the coch- linked with hearing aids. Examples include amplified tele-
lea, auditory nerve, and central nervous system and is more phones, notification systems, personal amplifiers, and televi-
often accompanied by tinnitus or vertigo. For discussion of sion streamers. Use of hearing aids is low owing to patient
vertigo, see MKSAP 19 General lnternal Medicine 1. Such medi barriers that include cost, stigma, cosmetic appearance, per-
cations as aminoglycosides, tetracycline, chemotherapeutic ceived ineffectiveness, and physical discomfort in wearing
agents (cisplatin, carboplatin, 5 fluorouracil, bleomycin), them. Treatment of sudden sensorineural hearing loss may
75
Ear, Nose, Mouth, and Throat Disorders
'Conductive hearing loss is inadequate mechanical transmission of sound through the tympanic membrane and ossicles of the middle ear
bSensorineural hearing loss is deficit or injury of the vestibulocochlear nerue.
LR = likelihood ratio.
uBased on 6 studies. bBased on 4 studies. .Based on I study. dBased on 2 studies.
76
Ear, Nose, Mouth, and Throat Disorders
TABLE 60. Distinguishing Between Conductive and persistent, bothersome tinnitus, providing relief from the
Sensorineural Hearing Loss With the Weber and Rinne Tests related stress and management of the perception of tinnitus.
Condition WeberTesf Rinne Testb Result Cognitive behavioral therapy is an eff'ective therapy fbr
Result tinnitus but may be limited by availability and lack of
Conductive Louder in the Decreased in the affected reimbursement.
hearing loss affected ear ear (bone conduction > air
conduction)
Sensori neural Louder in the As loud or louder in the Otitis Media and Otitis Externa
hearing loss unaffected ear affected ear (air conduction >
bone conduction)
Patients with acute otitis media (AOM) usually present with
unilateral ear pain and diminished hearing along with bulging
'A256 Hz vibrating tuning fork (although a 5 1 2-Hz tu ning fork may be used) is
; applied to the forehead or scalp at the midline, and the patient is asked if the or intense ery,thema of the tympanic membrane. Otitis media
sound is louder n one ear or the other; a normal test result shows no laterallzatton with effusion is often mistaken for AOM in adults (Figure 24);
r'A512-Hzvibratingtuningforkisappliedtothemastodprocessof theaffected it is characterized by the presence of fluid in the middle ear
ear until it is no longer heard. The fork is then repositioned outside of the external
auditory canal, and the patient is asked if he or she can again hear the tuning fork; without acute inflammation or signs of systemic illness. AOM
with a normal test result, air conduction is greater thao bone conduction, and the
tuning fork can be heard. is rare in adults, and although numerous guidelines exist for
treatment of AOM in children, management in adults is less
clear. The Choosing Wisely Canada initiative recommends
observation initially for uncomplicated AOM in adults and
include oral glucocorticoids within 2 weeks of onset, although
children. If antibiotics are prescribed, amoxicillin or amoxicillin
strong evidence ofefficacy is lacking.
clavulanic acid provides coverage fbr the most common organ
t(EY P0ta{Ts isms. Fever, posterior ear pain, and facial nerve paralysis are
HVC . Simple and effective tests for hearing loss, such as eval- rare but suggest complications, such as mastoiditis, and may
uating whether a patient can hear a whispered voice or require urgent imaging and surgical consultation. Patients
single question screening (for example, "Do you have with recurrent AOM or persistent hearing loss should be
77
Ear, Nose, Mouth, and Throat Disorders
Epistaxis
Epistaxis is a common problem affecting up to 60'7, of the U.S.
population, with 6'2, of affected patients requiring medical
attention. Ninety percent of epistaxis cases originate in the
anterior nasal septum in the Kiesselbach plexus (Figure 26).
Posterior bleeding, although less common, is more likely to
F IGUR E 2 5. Typical findings in the external auditory canal of a patient with result in significant hemorrhage. The history should include
otitis externa, including erythema and edema. assessment for timing and frequency of epistaxis, local trauma,
nose picking, intranasal medications and anticoagulant/anti
of AOE is tenderness with pushing on the tragus or pulling on platelet use, arid environments, infection, intranasal drug use
the pinna. (cocaine), family and personal history of bleeding disorders,
Topical treatments, including antibiotics, glucocorti and relevant comorbidities.
coids, antiseptics (acetic acid), and combination therapies, Anterior bleeding can be managed with compression of
are first line management for uncomplicated AOE; systemic the alae (lower one third of the nose) for at least 5 minutes.
antibiotics are ineffective and should be avoided. Quinolone If the bleeding site can be identified on anterior rhinoscopy,
drops are preferred in the case of a nonintact tympanic use of topical vasoconstrictors, such as oxymetazoline, and
membrane. Patients should be instructed on proper tech nasal cautery may be useful. If bleeding continues, nasal
nique for instillation of ear drops, including cleaning excess packing with an inflatable tamponade device or a foam
debris before instilling the drops and then lying down with
the affected ear facing upward, remaining in this position
for 3 to 5 minutes. lmmunocompromised patients require Anterior
€$ffridal arE y
topical and systemic therapy and are at increased risk for h*nrior
deeper infections, including bone involvement (malignant od{rmridal artery
Cerumen helps to protect, clean, and lubricate the external [,lodiiied with permission lromTunkel DE,Anne S, Payne SC, etal. Clinical practice guideline: nosebleed (epistaxis)
executrvesummaryotolaryngolHeadNeckSurg.2020;162:825.IPMID:31910122]d0i:10.1177l
auditory canal. Its normal expulsion, assisted by jaw movement, 0194599819889955
78
Ear, Nose, Mouth, and Throat Disorders
79
Ear, Nose, Mouth, and Throat Disorders
teria do not need to be tested. Patients with a Centor score of 1 wiki/File:Epiglottitis endoscopy.jpg.June20,20l3.AccessedJune26,2021.
80
Ear, Nose, Mouth, and Throat Disorders
production with warm compresses and sialagogues (sour Licensed under the Creative Commons Attribution ShareAlike 3.0 Unp0rted
(httpsJ/(reativecommons.org/licenses/by-sa/3.0/deed.en).
(CC 8Y'SA 3.0) lnternational License
81
Dermatologic Disorders
Dermatolog ic Disorders
General internists are called upon to evaluate dermatologic
problems on a regular basis. A thorough understanding
of the skin and its associated structures is required
(Figure 31). Of note, most topical therapies for dermato-
logic conditions are used off label because they are not
FDA approved for the specific condition in r,t'hich they are
t t G U R E 2 9. Red and swollen gingival tissue (b/ue arow) that typically bleeds commonly utilized.
with brushing or flossing is characteristic of gingivitis. Also seen is abundant yellow
dental plaque atthe gum line and between teeth (green anow).
Reproduced from Wikimedia Commons. Gingivitis before trealmenl. Dlgital image https://commons.wikimedia Approach to the Patient With
0rg/wiki/Category:Gingivitis#/media/File:Gingivitisbefore.JPG.AugustS,20l3 AccessedJune26,2021.
Dermatologic Disease
Morphology
Morphology of skin lesions can be categorized as primary
or secondary. Primary skin lesion morphology is the
appearance of a rash or growth in its initial or unaltered
state. Over time, with itching or rubbing, secondary skin
changes may occur. Table 62 lists primary and secondary
skin lesions. Other important characteristics of skin lesions
{ include color, distribution, and configuration. Dermatologic
s
I conditions can also be categorized as neoplastic or inflam
matory. Neoplasms manifest as localized areas of abnormal
skin, whereas inflammatory conditions are usually more
widespread.
A differential diagnosis should be initially developed on
the basis of morphologic features of the primary rash rather
t IGUR E 3 0. Necrotizing ulcerative gingivitis, previously known as Vincent than focusing on identification of specific rashes and their
angina (trench mouth), is an acute and painful oral conditi0n characterized by fetid characteristics.
breath, loss or diminution of the interdental papilla, and necrotic sloughing of the
gingiva. Necrotizing periodontal disease is a term that encompasses necrotizing
ulcerative gingivitis and necrotizing ulcerative periodontitis.
Physical Examination
No standardized method has been developed for how to best
perform a total-body skin examination, but experts agree that
and necrotic areas of the tonsils and gingiva with red irregular patients should be fully undressed. A systematic approach is
edges that may create a grayish pseudomembrane that bleeds recommended to decrease the possibility of overlooking any
if scraped (Figure 3O). Lymph nodes may be enlarged and ten area of the body during examination. One method is to start
der. This is usually seen in patients with poor oral hygiene or
malnutrition, orwho smoke. Treatment includes oral antibiot-
ics; twice-daily chlorhexidine rinses for 2 weeks: and, if severe, $rtt m fplll.rry
oatmll
cotnatlln
debridement.
Rcdculer
Dental fracture is an emergency because a tooth becomes drrmlr
nonviable t hour after fracture. Patients should handle the
tooth by the crown, wash it in cold water, reposition it in the
socket, and bite gently on a washcloth to help keep the tooth
in place until seen by a dental professional. If immediate repo
sitioning is not possible, the tooth should be placed in cold
pasteurized milk while seeking immediate care by a dental
professional.
TEY POITT
o Gingivitis is reversible with routine dental visits and
cleanings, brushing with fluoride toothpaste, flossing,
fluoride varnish anticavity treatment, and use of chlo-
rhexidine oral rinses. tIGURE 31. Structureoftheskinshowing thestratum corneum, basal cell layer,
papillary dermis, and reticular dermis.
a2
Dermatologic Disorders
Vesicle Small clear fluid-filled blister <1 cm Broad areas of exquisitely painful skin disproportionate to
clinical examination findings (necrotizing fasciitis)
Bulla Larger clear fluid-filled blister >1 cm
Angulated purpura of the distal extremities (sepsis,
Pustule Vesicle filled with purulent material autoimmune phenomena)
Atrophy Loss of one or more portions of the skin Palpable purpura (small-vessel vasculitis from infection
Burrow Serpiginous epidermal streaking/ medications, or autoimmune reactions)
disruption caused by scabies mite lmmunosuppressed patients (particularly those with neutropenia)
Comedo Noninflammatory lesion caused by keratin with skin lesions of unknown cause in the setting of fever,
debris within the sebaceous gland/hair particularly red or purple nodules (skin signs of infeaion in
follicle orifice; can be open or closed immunosuppressed hosts)
Telangiectasia Visibly dilated, but not palpable, blood Purple or necrotic skin lesions in immunosuppressed hosts
vessel in the epidermis (angioinvasive fungal infection)
Wheal/hive Tra nsient, edematous, e Rapidly growing lesions in immunosuppressed hosts (infection,
rythematous
papule or plaque malignancy)
Scale Thickened stratum corneum; often dry cancer, including persons with increased risk for skin cancer.
and white or gray The USPSTF does recommend counseling young adults with
Ulcer Full-thickness destruction of the fair skin types about minimizing exposure to ultraviolet (UV)
epidermis into the underlying dermis radiation to reduce their risk for skin cancer.
Diagnostic Tests
with the head and neck, working down to the torso, upper Diagnostic tests and tools that may augment the physical
extremities, and lower extremities. The routine skin examina examination include microscopic examination of skin scrap
tion may also be incorporated into the physical examination ings, examination with Wood lamp, and skin biopsy. Skin
by assessing the trunk when auscultating the heart and lungs, scrapings should be viewed on a microscope slide with potas
the arms when checking blood pressure and pulse, and the sium hydroxide if tinea is suspected or mineral oil if scabies is
legs when checking pulses and joints. a consideration.
Rashes in persons with darker skin color may be more A Wood lamp is an inexpensive UV light that can be used
difficult to detect because the erythema may be more to evaluate the depigmentation seen in vitiligo and the fluores-
subtle. Areas of active inflammation develop hyper or cence seen in erythrasma.
83
Dermatologic Disorders
84
Dermatologic Disorders
dependent on the chemical structure, concentration, and TABLE 66. Frequently Used Topical Dermatologic
delivery vehicle of the medication as well as skin hydration Medications
and thickness.
Medication lndications
In general, ointments have a higher potency than creams,
Topical glucocorticoids lnflammatory dermatoses
which in turn have a higher potency than solutions or suspen-
sions. The various vehicles available for topical medications Topicalantifungals Cutaneous fungal infections
are listed in Table 65. Patients tend to favor creams because Terbinafine, nystatin,
econazole, miconazole,
they rub in, leaving minimal residue. An occlusive vehicle, ciclopirox, clotrimazole
such as an ointment, increases penetration, but adherence
Topical retinoids
may be low because of its greasy feel. The alcohol content
Tretinoin, adapalene Acne
and drying nature of gels can cause burning and stinging on
damaged skin. Tazarotene Psoriasis
Topicalantibiotics
Mupirocin, neomycin, Superficial bacterial
TABLE65. Vehicles forTopical Dermatologic Medications
bacitracin infections
Vehicle Characteristics
Erythromycin, clindamycin Acne
Cream Equal mixture of oil and watel with a smooth
Metronidazole Rosacea
texture and whitish in color
Topical chemotherapy
Easily rubs in and leaves minimal residue
5-Fluorouracil Actinic keratosis
Highly prefened by patients because of
aesthetic feel and appearance Topicalantiparasitic
Foam Preparation that merges a gas (propane or Permethrin Scabies
butane) with the medication
Vitamin D analogues
Allows the medication to be dispersed within a
field of minute bubbles Calcipotriene Psoriasis
85
Dermatologic Disorders
, Potency and Exarnples of Topical bacitracin) are used frequently. The most frequent side effect is
Glucocorticoids allergic contact dermatitis, which limits their utility.
Potency Glucocorticoid
Weak Hydrocortisone 1 .0% or 2.5% o Topical glucocorticoids can avoid the adverse effects of
Low Desonide 0.05% systemic glucocorticoid therapy but can cause striae,
Triamcinolone acetonide 0.025a/" easy bruising, atrophy, and telangiectasia.
Fluocinolone acetonide 0.01 % . There is a generic equivalent in each class oftopical glu- HVC
Fluocinolone acetonide 0.025% o Combined topical glucocorticoids and antifungal agents HVC
Topical Retinoids
Topical retinoids are used frequently in the treatment of acne
and psoriasis. They are vitamin A analogues that affect
keratinocyte proliferation and differentiation. Use of these
medications should be avoided in pregnancy owing to the risk
for teratogenicity.
Topical Antibiotics
Clindamycin and erythromycin are topical antibiotics used in
the treatment of acne. They are well tolerated and have few to
no side effects, although bacterial resistance with monotherapy
has been inereasing over the past several decades. Mupirocin is
a topical antibiotic ointment commonly used to treat skin
infections caused by Stophylococcus oureus, such as impetigo. tIGURE 34. Atopicdermatitis,showingskinthickening(lichenification)dueto
Over-the-counter combination antibiotics (neomycin and scratching and dry scaling in the popliteal flexure.
86 .t
Dermatologic Disorders
Contact Dermatitis
Pathophysiolory and Evaluation
There are two types of contact dermatitis: allergic and irritant.
Allergic contact dermatitis is a tlpe IV delayed hypersensitivity
reaction that results in an eczematous eruption at the site of
contact. Irritant contact dermatitis is more common than
allergic contact dermatitis and results from direct damage to
the skin caused by chemicals, soaps, or detergents; it is not
immune mediated.
Discovering the cause of contact dermatitis is essential to
its treatment and prevention. A detailed history can help iden t IGUR E 3 5. Epicutaneous patch testing is commonly applied to the back (top) to
tiff the cause, but often epicutaneous patch testing is needed evaluate for allerg ic contact dermatitis. The erythematous patch identified 48 hours
after epicutaneous patch testing (bottom) indicates a positive reaction to a specific
to identiSz the source (Figure 35). Because allergies can develop
allergen.
over time, a lengthy history of using a skin product without a
reaction does not exclude that product as a potential trigger.
Common contact allergens are described in Table 68.
Urushiol, the allergen found in plants, such as poison ivy, T&mi..il &6. Common ContactAllergens
poison oak, and poison sumac, is a common cause of allergic Contact Allergens Occupations/Exposures
contact dermatitis. Clinically, it presents with intensely pru Benzophenones Sunscreens (contact and
ritic, often linear, vesicular papules, plaques, and vesicles photocontact dermatitis)
(Figure 36). In sensitized patients, lesions can present at differ- Balsam of Peru Common fragrance in personal
ent locations at different times for up to 14 days after exposure. care products and foods
Fluid from blisters is not antigenic. After exposure, patients Methacrylate Dental flllings, cement for joint
should remove contaminated clothing and gently wash the replacement (orthopedic
surgeons and dentists)
skin with soap and water.
Nickel is another common allergen and is found in every- Methylisothiazolinone Baby wipes, personal care
products
day items, such as jewelry zippers, and cell phones.
Paraphenylenediamine Permanent hair dye
Eczematous dermatitis of the lower abdomen is a common
presentation of a nickel allergr caused by a clothing snap or Formaldehyde and Cosmetics and personal care
formaldehyde-releasing products
belt buckle. Patients may use a commercially available nickel preservatives
test kit to identifiz items that contain nickel. (quaternium-15)
Neomycin and bacitracin are commonly used for wound Rubber accelerators Shoe and glove dermatitis;
care. They can cause an allergic contact derrnatitis that mimics (carba mix, thiurams, common in health care workers
a wound infection. Given the prevalence of sensitivity to these benzothiazoles)
products, patients should use plain petrolatum in place of Cocamidopropyl Personal care products
betaine (shampoos)
topical antibiotics to aid healing of clean wounds.
87
Dermatologic Disorders
-*\r. * ',!
Fl GU RE 39. Seborrheic dermatitis, with fine, oily scale around the medial
eyebrows.
Nummular Dermatitis
Nummular dermatitis is characterized by coin-shaped, pruritic,
scaly plaques commonly found on the extremities (Figure O).
It is more common in men and elderly persons. The round
plaques may be mistaken for other conditions, such as tinea
corporis, psoriasis, or contact dermatitis. Potent topical gluco-
corticoids and emollients are the preferred treatment.
lntertrigo
Intertrigo is dermatitis involving adjacent skin folds (axillary
infoamammarfr abdominal, and inguinaD Gigure 41). Predisposing
conditions include obesity, friction, occlusion, and factors that
interfere with immune response, such as diabetes mellitus.
Intertrigo presents with erythematous, macerated plaques.
Secondary infection with Candido is common and may be sus
pected by the presence of multiple small red papules and pus
tules that surround the main rash (satellite pustules).
Treatment of intertrigo consists of drying the area and use
of agents (such as drying powder) to reduce moisture and
U R E 4 1 . lntertrigo, a chronic recurrent skin condition that is often seen in
maceration and to prevent secondary yeast infection. Short F IG
patients with obesity, is caused by moist conditions in skin fold areas and is
courses of low-potency topical glucocorticoids may be added
worsened by heat and exercise. The rash is confined to the intertriginous areas and
to treat the associated inflammation. If secondary Candida does not extend beyond these boundaries. Secondary infection with Candida may
infection is suspected, concomitant treatment with topical occur, as in this patient. lndicators ol Candida infection include small red papules
89
Dermatologic Disorders
Urticaria
Urticaria (hives) represent localized edema caused by mast cell
degranulation releasing histamine, leukotrienes, complement,
and prostaglandins. Urticaria are characterized by well
demarcated 1- to B cm plaques that appear quickly and resolve
in a matter of hours . Individual lesions may be pink or white,
often surrounded by a red flare, and may manifest as round,
oval, annular, arciform, or serpiginous. Urticaria are extremely
pruritic and tend to appear on body sites that receive repetitive
FIGU RE 42. Venous stasis dermatitis, characterized by edema, bilateral pressure or rubbing, such as the waistline and posterior neck.
erythematous patches, weeping vesicles, ankle varic0sities, and hyperpigmentation.
Although individual urticaria resolve in less than 24 hours,
recurrent crops of hives may last for weeks. In most cases,
Venous Stasis Dermatitis urticaria resolve spontaneously and the cause is never deter
mined. If the episodes last for longer than 6 weeks, the condi
Venous stasis dermatitis is common in patients with chronic
lower extremig edema, most commonly secondary to venous
tion is classified as chronic. Ifthe individual urticarial lesions
persist longer lhan 24 hours, or if they are accompanied by
stasis (Figure 42). Venous stasis dermatitis can be very pruritic
and erythematous, and symptoms are often bilateral with systemic symptoms (joint pain, fever), urticarial vasculitis (a
medial distal leg predominance. Dependent edema, hyperpig form of small vessel vasculitis) must be considered.
Food allergz, viral infection, and medication reaction are
mentation, and dilated veins are common associated findings.
Symptoms can be managed topically with glucocorticoids and the most common causes of acute urticaria (Figure 43). Food
emollients, but the condition will not significantly improve induced allergic reactions most commonly result from shell
until the edema is addressed with leg elevation and compres- fish, peanuts, and tree nuts. Infrequent but important causes
sion. Because of a similar clinical presentation, venous stasis of urticaria include autoimmune thyroid disease and malig-
dermatitis may be misdiagnosed as cellulitis; however, celluli nancies, particularly lymphoma. Physical urticaria are induced
tis is usually unilateral, more acute in onset, and often associ by physical stimuli, such as sunlight, sweating, physical pres-
ated with pain, leukocl.tosis, and occasionally fever. sure, vibration, water, or cold temperature.
Urticaria is a clinical diagnosis. Physical maneuvers that
(tr Polf,It can help with diagnosis include circling lesions with a marker
. Allergic contact dermatitis is a type IV delayed hyper- to see whether they resolve within 24 hours and gently
sensitivity reaction and is best diagnosed with epicuta- scratching uninvolved skin with the end ofa tongue depressor
neous patch testing. to see whether urticaria (not just redness) can be induced
o Hand dermatitis is characterized by pruritic, erythema-
tous plaques on the hands and results from a combina-
tion ofcauses, such as excessive hand washing, contact
dermatitis, atopic dermatitis, or dyshidrotic eczema.
HVC o Inexpensive treatment of seborrheic dermatitis includes
over-the counter medications, such as selenium sulfide
or zinc pyrithione shampoos.
o Treatment of intertrigo (dermatitis involving adjacent
skin folds) consists of drying the affected area, using
drying powder to reduce moisture, and in some cases
using short courses of low potency topical glucocorti-
coids to treat the associated inflammation; concomitant
treatment with topical antifungal agents can be added if
secondary candidainfection is suspected.
(continued) FIG U RE 43, Urticarial lesions on the waist, characterized by small and large
edematous and erythematous plaques of various shapes.
90
I
Dermatologic Disorders
Exanthematous (Morbilliform)
Exanthematous, or morbilliform (measles-like), drug erup
tions are the most common drug reactions, most likely repre-
senting a type IV delayed hypersensitivity reaction. As such,
the rash appears during the first or second week after drug
exposure, although subsequent exposures can produce a reac-
tion much more quickly. Patients develop erythematous pap
ules and macules that coalesce symmetrically to form plaques.
tl GU RE 44. Tongue'blade-stroke-induced dermographism (dermatographism). The papules are often dense and monomorphic and accompa-
A wheal appears within a few minutes in the area subjected to a light scratch. nied by varying degrees of pruritus, beginning on the trunk
and progressing distally across the limbs (Figure 45). Palms
(dermatographism) (Figure 44). The cause of urticaria is and soles are usually spared. This type of reaction is classically
induced with the use of ampicillin or amoxicillin in patients
investigated primarily by history and physical examination.
Diagnostic evaluation for urticaria is not recommended with acute Epstein Barr virus infection.
Treatment involves cessation of the causative agent, and
unless the history suggests a specific cause. If symptoms per-
the use of systemic and/or topical glucocorticoids and oral H,
sist, laboratory tests, including a complete blood count with
antihistamines. Patients should be counseled to contact their
differential, urinalysis, erythrocyte sedimentation rate or
clinician ifthey develop fever, skin pain, blisters, pustules, or
C reactive protein, thyroid stimulating hormone, and liver
mucous membrane involvement, indicating a more serious
chemistry tests, can be considered. If associated with systemic
and potentially Iife-threating condition.
symptoms or suspicion of urticarial vasculitis, skin biopsy is
helpful. Nonsedating, long-acting antihistamines are the
treatment of choice for urticaria; topical antihistamines have Fixed Drug Eruption
not been found to be effective and may lead to allergic contact Fixed drug eruptions recur at the same sites on the skin with
dermatitis. each exposure. Pink-to-purple circinate plaques with central
Angioedema, both acquired and hereditary is discussed dusky discoloration or vesiculation appear most commonly on
in MKSAP 19 Pulmonary and Critical Care Medicine. the lips, face, fingers, and genitals (Figure 46). The first expo
sure typically generates one plaque that recurs in the same
TIY POIilTS
location with each repeated exposure. Additional plaques
HVC . Urticaria is characterized by well-demarcated, erythe- may develop with subsequent exposures. Lesions typically
matous, pruritic plaques that appear quickly and resolve with postinflammatory hyperpigmentation. Treatment
resolve in a matter of hours; diagnostic evaluation for involves cessation of the causative agent, and, if symptoms are
urticaria is not recommended unless the history sug- significant, systemic and/or topical glucocorticoids and oral H,
gests a specific cause. antihistamines.
HVC o Nonsedating, long-acting antihistamines are the treat-
ment of choice for urticaria; topical antihistamines have Drug-lnduced Hypersensitivity
not been found to be effective and may lead to allergic Syndrome (DRESS Syndrome)
contact dermatitis. Drug-induced hypersensitivity syndrome (DIHS) and drug
reaction with eosinophilia and systemic symptoms (DRESS)
are synonymous and represent a severe life-threatening medi-
Drug Eruptions cation reaction. DRESS is increasingly being referred to as
Drug reactions are an important consideration in the differen- DIHS to emphasize the fact that eosinophilia is not always
tial diagnosis fbr any rash because medications can produce present.
almost any pattern of skin disease. The most common types of The most common medications that cause DIHS include
cutaneous adverse reactions are exanthematous (morbilli sulfonamide antibiotics, allopurinol, and anticonvulsants, but
form), urticarial, fixed drug, photosensitivity, drug-induced many other medications have been implicated (Table 7o).
hypersensitivity syndrome, hypersensitivity vasculitis, acute Unlike other medication reactions, the onset of symp
se'uere
generalized exanthematous pustulosis, and the severe toms is delayed, often 2 to 6 weeks after exposure. Symptoms
91
Dermatologic Disorders
lmmunologic Penicillin, anesthetic agents, ACE Minutes to a few hours Most chronic urticaria cases are not
inhibitors drug related, but aspirin may cause or
exacerbate chronic cases of urticaria
Nonimmunologic Vancomycin, rifampicin, acetylcysteine, Minutes to a few hours Red person syndrome is related to the
opioids, radiocontrast agents infusion rate of vancomycin
Photosensitivity
Phototoxic Voriconazole, demeclorycline, Hours Voriconazole phototoxicity can cause
doxycycline, NSAlDs, skin cancer within a few years of
fluoroquinolones, amiodarone treatment
May result in fingernail shedding
Photoallergic Hydrochlorothiazide, sulfonamide 7-1 0 d for first exposure; May persist after ultraviolet light
antibiotics; sulfonylureas; tricyclic hours for subsequent exposure (fall or winter months)
antidepressants; sunscreens; exPosures
fragrances containing
{
6-methylcou mari n, musk am brette,
or sandalwood oil
Pigmentation Minocycl ine, amiodarone, Variable Most appear on sun-exposed areas
hydroxychloroquine, antiretroviral
Bleomycin may cause flagellate
therapy, bleomycin, gold salts
pigmented macules on the back
Zidovudine may cause pig mentatron
or pigmented streaking of the
fingernails
Minocycline may cause
hyperpigmentation on sun-exposed
areas and on acne scars
Fixed d rug Sulfonamides, NSAlDs, antiepileptic 1-2 wk after initial Pseudoephedrine-related reactions
agents, tetracyclines exposure;24hfor may not pigment
subsequent exposures
Va scu liti NSAlDs, antibiotics, 1-3 wk, within 3 d of Levamisole is used to add bulk and
hyd roch lorothiazide, fu rosem ide, rechallenge weight to cocaine; drug-induced
minocycline, TNF inhibitors, vasculitis is often ANCA positive, with
propylthiouracil, G M-CSF, levamisole both p- and c-ANCA patterns
Stevens-Johnson Allopurinol aminopenicillins 1-3 wk after exposure Systemic manifestations are common
syndrome/toxic (ampicillin, amoxicillin),
epidermal necrolysis carbamazepine, lamotrigine,
t
nevirapine, phenytoin,
sulfamethoxazole-trimethoprim,
sulfasalazine
begin with fever and flulike symptoms, quickly followed by lymphocytosis and elevated aminotransferase levels.
burning skin pain and rash. Patients typically develop a mor Hypotension, shock, and multisystem organ damage are pos
billiform exanthem that starts on the face and upper trunk sible. Death occurs in approximately 10'1, of patients with
and spreads distally (Figure 47). Eventually the patient devel- DIHS. Immediate discontinuation of the causative agent is
ops striking facial edema and redness, a hallmark of DIHS. mandatory and systemic glucocorticoids tapered over weeks
Oral mucosal involvement is common but is less severe than to months can be helpful. DIHS caused by an aromatic antiepi
Stevens Johnson syndrome/toxic epidermal necrolysis or ery leptic agent can also pose a serious threat owing to cross-
thema multiforme. reaction with other aromatic antiepileptics. For example, if
Systemic involvement is necessary for the diagnosis of phen).toin causes DIHS, the patient may experience cross-
DIHS and most commonly includes eosinophilia or an atypical reaction with phenobarbital or carbamazepine.
92
Dermatologic Disorders
Minocycline
Proton pump inhibitors
Sulfasalazine
93
Dermatologic Disorders
F IGUR E49 . Edema and vesicles on the lower lip and with erosions and mucosal
sloughing on the upper lip secondary to Stevens-Johnson syndrome.
TiaBLE 7''? . Comparison of Erythema Multiforme, Stevens-Johnson Syndrome, and Toxic Epidermal Necrolysis
Characteristic Eryrthema Multiforme Stevens-JohnsonSyndrome ToxicEpidermalNecrolysis
Morphology Typical three-zoned target (dark Atypical targets and confluent Extensive, confluent erythema
central area or blister; a red erythema with sloughing with sloughing
inflammatory ring; a ring of pale
edema); lesions with onlytwo
rings are common as well
Distribution Favors extremities Trunk and extremities; <10% Trunk and extremities; >30%
body surface area involvement" body surface area involvement"
Mucosal disease One or two sites Two or more sites Two or more sites
(oral, eye, genitourinary)
94
Dermatologic Disorders
TABLE 72. SCORTENToolValues Eighty percent of cases are caused by antibiotics. Most cases
are self limited and resolve within 2 weeks. Treatment consists
SCORTEN Features' Values Associated With
Poor Prognosis ofcessation ofthe causative agent and supportive carei topical
and systemic glucocorticoids may be helpful for symptomatic
Age ,40 y
relief. The condition carries a mortality rate of less than 5'/n.
Malignancy Present
Heart rate >120/min
r(rY P0rilTs
Body surface area >10"/"
. Exanthematous, or morbilliform, drug eruptions are the
most common drug reactions; treatment involves cessa-
Plasma glucose >252 mg/dL(1 3.98 mmol/L)
tion of the causative agent, and the use of systemic and/
Blood urea nitrogen >28 mg/dL(9.99 mmol/L)
or topical glucocorticoids and oral H, antihistamines.
Bicarbonate <20 mEq/L(20 mmol/L)
o Fixed drug reactions recur at the same site or sites with
SCORTEN = SCORe of Toxic Epidermal Necrosis. each exposure and consist of pink-to-purple circinate
"The likelihood of death increases with each feature. Five or more features is plaques with central dusky discoloration or vesicula-
associated wrth a 90o/o mortalrty rate.
tion, most commonly on the lips, face, fingers, and
Data from Bastuji Garin S, Fouchard N, Bertocchi M, et al. SCORTEN: a severity of
illness score for toxic epidermal necrolysis. J lnvest Dermatol. 2000;1 1 5:1 49 53. genitals.
lPlV lD: 1 09512291 d oi:101A46/j.1523 17 47 .2000.00061 .\
r Drug-induced hypersensitivity syndrome (also known HVC
as drug reaction with eosinophilia and systemic symp-
the most commonly used treatments, but neither is supported toms [DRESS]) develops 2 to 6 weeks after drug expo-
by strong evidence. Supportive care in an ICU with experi sure; discontinuation of the causative drug is the first
enced nursing staff is critical for wound care, and many step in treatment.
patients are transferred to a burn center. A low threshold is . Key features of Stevens-Johnson syndrome and toxic
recommended for performing cultures and initiation of epidermal necrolysis include full-thickness epidermal
empiric antibiotics; however, use of prophylactic antibiotics is necrosis with involvement of mucous membranes,
not recommended. Ophthalmologic and urologic consulta which is potentially fatal owing to secondary infection,
tions are mandatory if ocular or genital involvement is present transcutaneous fluid loss, or respiratory complications.
because destructive scarring may occur in these systems.
95
Dermatologic Disorders
q."
ra*
96
Dermatologic Disorders
comedones (blackheads); closed comedones (whiteheads); TABLE 74. Types of Acneiform Eruptions
inflammatory papules and pustules; and nodulocystic lesions
Acneiform Distinguishing Characteristics
on the face, neck, chest, shoulders, and back (Figure 54 and Eruption
Frgure 55,.
Perioral/ Papules and pustules around mouth,
Acneiform lesions are common, and the differential diag- periorificial eyelids, or nose; more common in
nosis is broad. Variations in distribution and primary lesions dermatitis women; topical glucocorticoids may be
may provide clues to help diflerentiate acne vulgaris from associated in some cases
other diagnoses (Table 74). Finally, onset of acne combined Rosacea Flushing, telangiectasias; lack of
with other signs of hyperandrogenism, such as hirsutism and comedones; pustules common on nose,
cheeks, forehead, and chin
oligomenorrhea, warrant consideration of polycystic ovary
Folliculitis Follicular-based papules and pustules;
syndrome, congenital adrenal hyperplasia, or an underlying
often on the scalp or trunk; most
adrenal or ovarian tumor. commonly caused by Staphylococcus
The pathogenesis is attributed to abnormal keratinization aureus
within the hair follicle, excess sebum production (stimulated Drug induced Monomorphic pustules and papules;
by androgens), colonization of the follicle by Cutibacterium common inciting medications include
glucocorticoids, lithium, anticonvulsants,
ocnes (formerly Propionibacterium acnes), and the release of and epidermal growth factor receptor
inhibitors
Chloracne Appears in the scrotal region and the
postauricular scalp; history of exposure to
halogenated aromatic compounds (dioxin)
Pseudofollicu litis Common in the beard area; skin-colored
barbae or erythematous papules at hair-bearing
sites; more common in Black men
97
Dermatologic Disorders
BP or topical retinoid
Add topical retinoid or BP (ii not on already)
-oR-
-oR-
Mild acne
Topical combination therapf
Consider alternate retinoid
BP + antibiotic or
-oR-
Retinoid + BP or
Consider topical dapsone
Retinoid + BP + antibiotic
FIGU R E 5 6. Treatment algorithm for the management of acne vulgaris in adolescents and young adults.
98
t
t Dermatologic Disorders
I
IL
is{f;i-r :r!, Managementof Rosacea
t
h lnterventions
Avoidance Sun protection (sunscreen or sun-protective
t clothing)
(
I Triggers: foods (spicy, warm), alcohol, warm
environments
L Topical Metronidazole, Q.7 5% or 1%
t Sodium sulfacetamide/sulfur
i Azelaic acid, 15%-20%
I
Calcineurin inhibitors (pimecrolimus,
t tacrolimus)
Permethrin
t Systemic Tetracycline antibiotics (doxycycline, 40 mg)
t Macrolide antibiotics (erythromyci n,
azithromycin, clarithromycin)
L
Lasel li ght, and Lasers (PDL, Nd:YAG, CO2, and others)
L surgica I
lntense pulsed light
L Electrosurgery
t
I Nd:YAG = neodymium-doped yttrium aluminum garnet; PDL = pulsed dye laser
Hidradenitis Suppurativa
Hidradenitis suppurativa is an inflammatory skin disorder of
the apocrine glands. This disease occurs more commonly in
women and typically begins after puberty. Areas frequently
affected are intertriginous areas, such as the axillae, groin, and
FIGU R E 5 9. Rhinophyma, characterized by hyperplastic sebaceous glands and underneath the breasts. The condition is characterized by
enlargement of the nose, in a patient with long*tanding, uncontrolled rosacea. comedones, painful nodules, abscesses, draining sinuses, and
scarring (Figure 6O). Risk factors include obesity, family his-
subtype (Figure 58). Most patients also have frequent flushing in tory and cigarette smoking. For this reason, weight loss and
response to triggers, such as stress, alcohol consumption, heat, smoking cessation are encouraged.
and excessive sunlight. Ocular rosacea may present with Hidradenitis suppurativa is extremely difficult to treat.
injected conjunctiva and "gritfy sensation" alone, or with asso- No single treatment has been effective for all patients; how-
ciated skin findings. In phymatous rosacea, severe sebaceous eveq several are options available (Table 76). Decolonization
hyperplasia and chronic inflammation lead to fibrous over- with dilute bleach baths and chlorhexidine washes may be
$owth of the skin, creating a nodular, tumor-like deformation used in addition to topical clindamycin. Common systemic
of the facial stmctures, most commonly the nose (rhinophyma) treatments include oral antibiotics, such as tetracyclines or
(Figure 59). the combination of clindamycin and rifampin. Adalimumab,
Treatment for rosacea is targeted toward the most promi- a tumor necrosis factor o inhibitor, is an FDA-approved
nent signs and symptoms in each patient (Table 75). Treatment treatment for moderate to severe hidradenitis suppurativa.
of erythrotelangiectatic rosacea focuses primarily on behavio- Moderate to severe disease is defined by the presence of
ral modiflcations, such as avoidance of identified triggers of multiple recurring nodules or abscesses and scarring.
flushing, proper use ofsun protection, and use ofgentle skin Surgery may be required for chronic hidradenitis suppu-
cleansers. Treatment for papulopustular rosacea includes rativa. This may involve incision and drainage or deroofing of
99
Dermatologi< Disorders
f EY POIXTS (ominued)
o Oral antibiotics are frequently used for moderate to
severe inflammatory acne and acne resistant to topical
treatment alone; courses of oral antibiotics should be
limited to 3 months to decrease the risk for antibiotic
resistance.
. Isotretinoin is indicated for severe nodulocystic and
recalcitrant acne; it is associated with severe birth
defects and must be administered through the federal I
XEY POIXII
HVC o Treatment of comedonal and inflammatory acne
includes topical retinoids, benzoyl peroxide, and
topical antibiotics; because of increased antibiotic
resistance, topical antibiotics should not be used as
monotherapy.
(Continued) ttGURE 61. Well-demarcated symmetric patches of depigmented skin with no
scale and accentuation of normal, sun-exposed skin, typical of vitiligo.
100
Dermatologic Disorders
Melasma
Melasma, also known as chloasma or the mask of pregnancy,
is a common acquired condition resulting in patchy hyperpig-
mentation of the skin on the face. It can also be seen on the
upper extremities. In this condition, melanocltes overproduce
melanin owing to a multitude of factors, including UV radia-
tion, genetic predisposition, and hormonal influence (preg-
nancy, use of oral contraceptives). Women and individuals
with darker skin types are more commonly predisposed.
Clinically, it presents as ill-defined, tan to dark brown reticu-
Iated patches, mostly on the sun-exposed areas of the face
(malar, mandibular, and centrofacial regions) (Figure 63). It
worsens with sun exposure. The main differential diagnoses
include postinflammatory hyperpigmentation, freckles, and
lentigines. Melasma often fades during the postparhrm period,
t IGUR E6 2. Vitiligo, with well-demarcated symmetric patches of depigmented with sun protection, and upon discontinuation oforal contra-
skin with no scale in a darkly pigmented person. ceptives. It may take months to years for pigmentation to
normalize.
Treatment can be challenging. The mainstay of treatment
commonly affected areas involve the extensor surfaces, such as is strict sun protection, avoidance or discontinuation of the
the dorsal hands and feet, elbows, and knees, and the perior- causative factors, and use of lightening agents.
ificial areas, such as around the mouth, eyes, rectum, and
genitals. It can be associated with other autoimmune condi-
tions, such as type 1 diabetes and thyroid disease.
It may be challenging to distinguish vitiligo from other
conditions that cause depigmentation, such as postinflam-
matory hypopigmentation, tinea versicolor, and pityriasis
alba. The diagnosis of postinflammatory hypopigmentation
is based on recognition of hypomelanotic or amelanotic
macules or patches that correspond to previous inflamma-
tory lesions. If the diagnosis is in doubt, a skin biopsy may
assist in identifying the responsible inflammatory lesion.
The diagnosis of tinea versicolor can be confirmed with a
potassium hydroxide preparation that shows hyphae and
yeast cells.
Pityriasis alba occurs in children and adolescents.
Defi ning characteristics include patches of hypopigmentation
on the face, neck, upper trunk, and proximal extremities.
Lesions typically manifest some border irregularity and are
associated with a fine scale.
History and physical examination, including a Wood
lamp examination to help accentuate the depigmentation and
the distribution ofthe lesions, helps confirm the diagnosis of
vitiligo. Lesions appear bright white and sharply delineated
when examined with a Wood lamp.
Treatment for depigmented skin can be challenging, pro-
longed, and suboptimal. Topical therapies include high-
potency glucocorticoids or immunomodulators (tacrolimus,
pimecrolimus). Phototherapy with UV light is also an option
and can be the treatment of choice for widespread involve-
ment. Repigmentation occurs first around hair follicles as they
are thought to be the reservoirs for melanocytes. As a result,
repigmented skin will initially have a speckled appearance.
Depigmentation therapy can also be considered if the vitiligo t tGU R E 63. lll-defined, reticulated hyperpigmented patches on the malar
is widespread (>50% BSA). cheeks, typical of melasma.
101
Dermatologic Disorders
T EY PO I IITS
o Vitiligo is an acquired autoimmune condition that
causes loss of melanocytes and subsequent depigmen-
tation of the skin; it can be associated with other auto-
immune conditions, such as gpe 1 diabetes and thyroid
disease.
. Treatment of depigmented skin includes topical thera
pies, such as high potency glucocorticoids or immu-
nomodulators, and phototherapy with ultraviolet light
for widespread involvement.
o Melasma, which results in patchy hyperpigmentation
on the face during pregnancy or with oral contraceptive
use, is treatedwith strict sun protection, avoidance or
discontinuation of causative factors, and lightening F IGUR E 6 5. Bullous pemphigoid is characterized by subepidermal bullae
blisters that are tense and do not rupture easily. lt predominantly involves
agents.
nonmucosal surfaces. Sites ol predilection are the lower abdomen, inner thighs
groin, axillae, and flexural aspects of the arms and legs.
Photo- or Light-lnduced
",* t Dermatoses
Photo and light-sensitive conditions may be secondary to
systemic diseases, such as systemic lupus erythematosus and
porphyria cutanea tarda, or conditions limited to the skin,
including phototoxic/photoallergic conditions and polymor
phous light eruption. Diagnosis of the disorders limited to
the skin is usually clinical, but skin biopsy is often per
formed to support the clinical diagnosis and rule out other
dermatoses.
102
Dermatologic Disorders
Bullous Tense blisters preceded by intense Subepidermal bullae without One of most common autoimmune
pemphigoid pruritus or urticarial lesions most acantholysis and with prominent blistering diseases, with up to 4.3
commonly seen in elderly persons eosinophils cases per 1 00,000 persons per year
on the trunk, limbs, and flexures;
DIF shows linear lgG deposition at
does not usually present with oral
lesions the basement membrane zone
Epidermolysis Bullae and erosions mostly on Subepidermal cleavage without Rare disease with unknown
bullosa acquisita extensor areas that may mimic acantholysis frequency
bullous pemphigoid, or heal with
scarring and milia DIF shows lgG deposition at the Can be associated with
basement membrane zone that inflammatory bowel disease
localizes to the base on salt-split skin
Cicatricial Presents with bullae, erosions, milia, Histology is similarto bullous Rare disease, with estimated
pemphigoid and scarring on the mucous pemphigoid incidence of 0.9 to 1.1 cases per
membranes and conjunctivae of million persons per year
middle-aged to elderly persons; DIF may reveal patterns similar to
oral mucosa is almost always
bullous pemphigoid, linear lgA lncreased risk for malignancy in
bullous dermatosis, or some patients
involved; conjunctival lesions are
also common
epidermolysis bullosa acquisita
Prompt treatment should be
initiated to avoid permanent ocular
and oral scarring
Dermatitis Severely pruritic grouped vesicles Histology shows neutrophilic Common blistering disease, with 10
herpetiformis or erosions on elbows, knees, back, infiltrate atthe tips of the dermal to 1 'l cases per 100,000 persons
scalp, and buttocks; lesions occur papillae causing subepidermal per year
in crops and are symmetrically separation
Nearly all patients with dermatitis
distributed; often the vesicles are
DIF shows granular lgA deposition herpetiformis will have celiac
not seen because the process is
disease
so itchy that they are almost
immediately broken
Linear lgA Pruritic, discrete, or clustered bullae Subepidermal bullae with ln adults, the estimated incidence is
bullous in a herpetiform pattern ("cluster of neutrophils 0.6 case per 1 00,000 persons per
dermatosis jewels"); annular or polycyclic year
DIF shows linear lgA deposition
lesions with vesicles and bullae at
Ocular involvement can occur
the periphery are common
DIF = direct immunofluorescence; llF = indirect immunofluorescence.
103
Dermatologic Disorders
104
5
I
b
I
t Dermatologic Disorders
I
t Fig
f
I Systemic St. John's wort
Voriconazole
t
Doxycycline
I
L Amiodarone
I
I Hydrochlorothiazide
I
Ouinolones
Photoallergic Causes
t Topical Sunscreens (benzophenones)
I
L Ketoprofen
I
Diclofenac
D
Fragrance (musk ambrette)
I
Chlorhexidine
Systemic Griseofulvin
Ouinolones
:
Ouinine
I
Sulfonamides
105
I
Dermatologic Disorders l
'1
I
']
I
classic findings of ecthyma. Ihey almost always occur on the legs or feet and are i
Dermatophlrtes
Tinea (dermatophytosis) is classified according to the body site
that is involved: tinea pedis (athlete's foot) (FigtrI€ 73), tinea
manuum (hand) , tinea cruris (perineum or jock itch) (ti8we 7a) ,
tinea capitis (scalp) (Figure 75), tinea faciei (face), and tinea
corporis (the body, excluding the genital area or inguinal folds)
(Figure 76). Three genera of fungi account for most dermato-
phytosis: Epidermophyton, Microsporum, and Trichophyton.
Trichophyton rubrum is the most common causative species.
Infection is more prevalent in hot and humid climates
and in patients who are immunocompromised, particularly
F IGUn E 7 0, Nonbullous impetigo is a superficial infection characterized by a those using chronic glucocorticoid therapy. Infection is
yellowish, crusted surface that may be caused by staphylococci or streptococci. acquired from other humans, animals, or fomites.
t06
t
t
t Dermatologic Disorders
t
t
t
t
I
I
t
L
I
t
t
I
I
t
L
t
t IGUR E 7 2. Erythrasma presents with sharply demarcated fine, pink-to-brown
L scaling patches that are typically found in skin fold areas.The rash will fluoresce a
i
coral red with Wood lamp illumination.
L
t
I
t
I
107
Dermatologic Disorders
Combined antifungals and topical glucocorticoids should be stratum corneum. Pityriasis versicolor is one of the most com
avoided because they can lead to increased recurrences and mon chronic superficial lungal infections and is typically
treatment failures. Dermatophytosis of hair-bearing skin, found in young adults, particularly in warm, humid environ
including tinea capitis, or those with recurrent or extensive ments. It presents as asymptomatic, oval to round, minimally
skin involvement should be treated with oral antifungal agents, scaly, hyper- or hypopigmented macules that can coalesce into
such as terbinafine or itraconazole. patches on the trunk and upper extremities (Figure 79).
The diagnosis of pityriasis versicolor can often be made by
Yeast clinical presentation but may be confirmed by visualization of
Candidiasis short rod shaped hyphae and round yeast ("spaghetti and meat-
Candida species, in particular C. albicons, can cause superfi- balls") on microscopic examination of skin scrapings using a
cial infections in moist occluded skin, such as the intertrigi potassium hydroxide preparation. Topical treatment using
nous areas, oropharyngeal mucosa, and genitals, as well as ketoconazole 2ol, shampoo or selenium sulfide suspension is
108
Dermatologic Disorders
I
FIGU R E 7 9. This patient exhibits classic findings of tinea versicolor, with tIGURE 81. Typical chickenpox(primaryvaricella-zostervirus infection) lesions
hypopigmented and faintly scaly macules coalescing into patches on the upper develop in crops, such that lesions in different slages of development (macules,
back. papules, vesicles, pustules, crusted erosions) are present simultaneously on any
one part of the body; lesions are most prominent on the trunk rather than lhe
effective. Either treatment should be applied from the upper extremities. Chickenpox is characterized by the simultaneous onset of fever and a
cutaneous eruption.
neck to the thighs and used to wash the scalp. Oral itraconazole
can be used fbr patients with extensive disease or frequent
recurrences. Because recurrences are frequent, weekly use of Diagnosis is based on clinical appearance. Molluscum
topical treatments can be preventive. contagiosum is usually self-limited. However, in patients with
severe cases or a protracted course, or in immunocompro
Viral Skin lnfections and Exanthems mised persons, destructive modalities (cryotherapy, curettage,
podophyllin) or topical medications (tretinoin, imiquimod)
Molluscum Contagiosum
Molluscum contagiosum is caused by a poxvirus. Infection can be used. Any of these treatments can be associated with
occurs norrnally in children, in adults as a sexually transmitted
scarring and pigment change.
disease, and in patients with AIDS. The lesions are white to,
yellow smooth papules with umbilicated centers (Figure 80). Varicella / Herpes Zoster
Varicella zoster virus is a DNA virus that causes varicella (chick
Many patients develop a surrounding eczematous host
response. Papules can be located anpvhere on the skin, but in enpox), an acute illness characterized by fever and an eruption
adults the genital area is typically involved. In patients with of vesicles on an ery.thematous base that is transmitted by res-
piratory droplets (Figure 81). After primary infection, the virus
AIDS, the most common area of infection is the face and the
lesions tend to be larger. Transmission is by direct contact or remains latent in the dorsal root or cranial ganglia. Reactivation
causes herpes zoster, or shingles (Figure 82). Prodromal symp-
by fomites.
toms, such as burning, stinging, or tingling, often occur in a
;#' dermatome, followed by eruption of grouped vesicles or pus
tules on an erythematous base. The most common dermatomes
109
Dermatologic Disorders
110
Dermatologic Disorders
occurs by close personal contact or by fomites, such as bed Scabiesb Topical 5% permethrin cream applied
from neck down and repeat in 7 days or
ding, clothing, or hairbrushes.
oral ivermectin" on days 1 and 8
Body lice live in the seams of clothing, not on the skin,
Wash and dry clothing, bedding, and
and are associated with poor hygiene. Excoriations result in linens in high heat; repeat in 1 wk
crusted papules or linear petechiae on the trunk, neck, and
Bed bugs Professional exterm ination
proximal arms. Body lice can transmit serious infections,
Wash and dry clothing, bedding, and
including trench fever (Bartonella quintana), louse-borne
linens in high heat; repeat in 1 wk
relapsing fever (Borrelia recurrentis), and epidemic typhus
uContraindicated in pregnant women.
(Rickettsio pro w azekii) .
blreat all members of the househoid.
Pubic lice (crab louse) are typically spread by sexual con-
tact but may attach to other sites, such as eyebrows or eye
lashes (Pediculosis cilioris). Screening for other sexually
transmitted infections should be considered in patients diag
nosed with pubic lice. Table 8o outlines treatment for lice.
Scabies
Scabies (Sarcoptes scobiei) infestation is characterized by
intensely pruritic, crusted papules, nodules, and burrows that
develop in the interdigital spaces, wrists, ankles, breasts, peri-
umbilical area, and genitals (Figure 85). In patients who are
immunocompromised, highly contagious crusted scabies
occurs, with thick concrete like scale. Scabies is transmitted
111
Dermatologic Disorders
TtY POIilIS
. Body lice cause excoriations that result in crusted pap-
ules or linear petechiae on the trunk, neck, and proxi
mal arms, and can transmit serious infections.
o Pruritus may persist for weeks after eradication of scabies HVC
and does not necessarily represent resistance or reinfec-
tion and does not require retreatment.
. Bed bug bites are characteristically Iinearly arranged
urticarial papules ("breakfast, lunch. and dinner' bites)
on exposed skin and are painless and pruritic.
Burns
Burns are the fourth most common type of traumatic injury
and may be caused by a multitude of agents. including chemi
cal. thermal, and electrical. as well as LIV light. Thermal burns
are assessed based on the amount of BSA involvement. depth
of burn, and specific area involved. Grading of burns is
described in Table 81. Estimating the BSA involved is based on
tlGU RE 86, Sarcoptesscableimite(arow) seen on mineral oil preparation of
skin scrapings.
the rule of 9s: The head and neck represent 9'X,, each arm is
9'X,. each leg is 18%, the anterior trunk is 18'),,. the posterior
pruritus" can be treated with antihistamines; topical glucocor trunk is 1B'7,, and the genital region is 1'X, BSA. Treatment of
ticoids; and, if severe, oral glucocorticoids. burns is based on the depth of the inlury Minor burns in areas
of low risk for infection with a small BSA involvement can be
Bed Bugs treated with gentle cleansing and application ofa sterile dress
Bed bugs (Cimex lectulorius) live in floorboards, walls, vents, ing with petrolatum. Wounds prone to developing infection
bedding, or furniture and crawl to the host to feed overnight. may be best treated lr,'ith a topical antibiotic in addition to the
They do not live on the human body. Bites are characteristi sterile dressing.
cally linearly arranged urticarial papules ("breakfast, lunch,
and dinner" bites) on exposed skin (Figure 87). The lesions are
painless and pruritic. See Table 80 for treatment. TABLE 81. Classification of Burns by Depth of lnjury
Depth Degree Structures Signs and
lnvolved Symptoms
S uperficial First Epidermis Red and
tender skin
Su perficial Second Epidermis Blisters,
pa rtia l-th ickn ess destroyed weeping,
partial sloughed
dermis skin, red;
painfulto air
and touch;
bla nches
112
Dermatologic Disorders
DPCP = d phenylcyclopropenone.
113
Dermatologic Disorders
114
Dermatologic Disorders
Nail Disorders
The nail complex is composed of the nail plate, hyponych-
ium, proximal and lateral nail folds, cuticle, nail bed, and
matrix (Figure qS). The hyponychium is the skin underneath
115
Dermatologic Disorders
the distal free edge of the nail plate. It seals the junction The distal nail plate becomes discolored (yellow white, or
between the distal nail plate and the nail bed. The nail fold brown) and thickened. Under the nail plate, there is an accu-
protects the nail matrix and produces the cuticle. The nail mulation of subungual debris, resulting in separation of the
bed is adherent to the bottom ofthe nail plate. Nail disorders nail plate from the nail bed (onycholysis) (Figure 96). Proximal
can be caused by various diseases, infections, trauma, and subungual onychomycosis appears similar except that it starts
drugs (Table 83). at the proximal nail fold. This pattem is rare and can be associ
ated with HIV and immunosuppression.
lnfection Onychomycosis is treated mostly for cosmetic reasons.
Onychomycosis Confirmation of infection with potassium hydroxide prepa
Onychomycosis is a dermatophytosis of the nail commonly ration, staining with periodic acid-Schifl or fungal culture
seen in elderly persons, especially those with comorbidities, should be done before treatment to rule out psoriasis or
such as diabetes mellitus, peripheral vascular disease, or traumatic nail changes. Topical antilungal agents are of
immunosuppression. Toenails are most commonly affected. limited efficacy; oral therapy with terbinafine or itraconazole
Distal subungual onychomycosis is the most common pattern. is more effective.
115
Dermatologic Disorders
tlG URE 96. Yellow hypertrophic nail plate and subungual debris resulting
from onychomycosis.
FIGUnE 98. Nail pitting in a patientwith psoriasis.
Paronychia
Paronychia is an infection or irritation of the nail fold. Infection
is causedbytrauma or chronic maceration leading to an incom-
petent cuticle. Acute paronychia is painful swelling of the nail
fold, most commonly caused by S. oureus. It typically affects
only one nail. Chronic paronychia tends to be more insidious
and involve multiple fingers. In adults, it is most often seen in
those with frequent hand immersion in water. There is tender
swelling in the nail folds with missing or dystrophic cuticles
(Figure 97). Chronic paronychia can cause ridgrng of the nail
plate. It is a multifactorial condition with several inciting fac-
tors, including water, irritants, and possibly Condido species.
Treatment of acute paronychia includes the use of warm
compresses; incision and drainage; and, in severe cases, oral
antibiotics. For chronic paronychia, minimizing inciting factors
is ke5z Treatment includes topical glucocorticoids and antifungal
agents to reduce inflammation and minimizing inciting factors.
lngrown Toenail
An ingrown toenail is the result of the nail plate growing into
the lateral nail fold, causing an inflammatory response.
Ingrown toenails of the great toe are most common, presenting
tI G U R E 9 7. Tender, pink swelling of the proximal and lateral nail fold due to as painful swelling of the lateral nail fold (Figure 1O2). There
chronic paronychia. Note the partially absent dystrophic cuticle (arows). may be some weeping granulation tissue.
117
Dermatologic Disorders
FIGURE 100. Characteristicfeaturesof nail psoriasisincludedistal onycholysis F IGUR E 10 2. Iender pink swelling of the lateral nail Iold of the great toe, with
(separation of the nail plate from the underlying nail bed) [arrows] and splinter some pink granulation tissue growing on the lateral distal end suggesting an
hemorrhages, which representthe Auspitz sign (punctuate bleeding) in the nail ingrown toenail.
bed. ln addition, some patients may display the "oil-stain" sign (clrcle), which is
118
Dermatologic Disorders
F IGUR E 1 04. Verruca plaque on the lateral nail Iold that looks like a wa(, but TABLE 84. Human Papillomavirus Subtypes
biopsy demonstrated squamous cell carcinoma. 5u btype Clinical Lesion (Location)
1 Verruca plantaris/palmaris (soles/palms)
Benign Nodules and Tumors )t Verruca vulgaris (fingers, knees, and elbows)
Seborrheic Keratosis 3, 10 Verruca plana (face and legs; associated with shaving)
Seborrheic keratoses are benign pigmented neoplasms of 6, 11 Condyloma acuminata
keratinocyte origin that are common in older adults. They are 16,18, Condyloma acuminata (associated with high risk
"stuck on" papules and plaques that occur anywhere on the 31, 33 for cervical, anal, and penile cancer)
body. They most commonly occur on the trunk and spare the
palms, soles, and mucous membranes. Seborrheic keratoses
range in size lrom a few millimeters to several centimeters.
Usually they are brorvn, but they can range in color lrom tan
to black (Figure f05). They may become irritated or excoriated.
Seborrheic keratosis can mimic melanoma and squamous cell
carcinoma, but skin biopsy confirms the diagnosis. When
seborrheic keratoses are symptomatic, they may be treated
with cryotherapy or shave removal. The sign of Leser 'l'rdlat is
the sudden eruption of numerous seborrheic keratosis and is
associated with an internal malignancy.
#;
.4:,1.'-. -
119
Dermatologic Disorders
120
Dermatologic Disorders
stand out from the patient's other nevi. Patients with multiple
dysplastic nevi are at risk for developing melanoma and
should be monitored closely (Figure 112). Some of these
patients have dysplastic nevus syndrome. Criteria for this
syndrome include a history of melanoma in one or more first-
or second-degree relatives, the presence ofmore than 50 nevi,
multiple nevi with atypical clinical features, and multiple
nevi with atypical histologic features. These patients are at
increased risk for melanoma and should have yearly total-
body skin examinations.
f IGURE I 1 1. Halo nevus,with a depigmented halosunounding a brown
papule.
. Warts are usually self-limited and treatment unneces- HVC
Halo nevi are central tan-to-brown macules or papules with a sary; however, in severe cases, including those with a
surrounding halo of hypopigmented or depigmented skin protracted course or in persons who are funmunocom-
(Figure 111) . They most frequently present on the back of teen- promised, destructive treatments can be used.
agers or young adults. Multiple halo nevi have been associated . Dysplastic nevi can display one or more of the identi$r-
with melanoma; therefore, these patients should have a com- ing characteristics of melanoma; patients with multiple
plete skin examination to assess for concurrent lesions.
dysplastic nevi are at increased risk for melanoma and
should be monitored closely.
Dysplastic Nevi r Excisional biopsy of dysplastic nevi is reserved for HVC
Dysplastic (or atypical) nevi are melanocytic nevi that are
lesions that display features suggesting melanoma, are
often asymmetric and irregularly bordered, have multiple
changing in size and color, are symptomatic, or stand
colors, and may be quite large in diameter. Dysplastic nevi are
out from the patient's other nevi.
most commonly found on the trunk and extremities. Many
have a "fried egg" appearance with an eccentric, dark brown
papular center and surrounding light brown ring. Dysplastic
nevi can display one or more of the identiffing characteristics
Premalignant and
of melanoma, making an accurate clinical diagnosis impossi- Malignant Tumors
ble (see Melanoma). In cases of uncertainty, biopsy of the Skin cancer is a major public health concem, and one in five
lesion can typically exclude melanoma; however, it is impera- people in the United States will develop it. Most cases are
tive to sample the entire lesion, including a 1- to 2-mm border caused by excessive UV light exposure. People ofall skin pig-
of normal surrounding skin. mentation can develop skin cancer, but fair-skinned persons
Although melanoma can arise within any melanocytic are more susceptible to the effects of UV light exposure.
nevi, including dysplastic nevi, the risk for this occurring
within any given lesion is quite small. Excisional biopsy is Actinic Keratosis
reserved for lesions that display features suggesting mela- Actinic keratoses are precancerous lesions of the epidermis.
noma, are changing in size and color, are symptomatic, or Approximately l% lo 5% of actinic keratoses will develop into
121
Dermatologic Disorders
.z$.
F IGUR E I I 3. Actinic keratoses are typically pink, thin papules or plaques that
occur on sun-exposed areas, particularly the face and dorsal arms.
Fl GUR E 11 5. Pink hyperkeratotic nodule on sun-exposed skin consistent with
squamous cell tarcinoma.
squamous cell skin cancers. Although the risk that an indi-
vidual lesion will develop into a squamous cell carcinoma is pigmented patients, those with skin of darker pigment more
low, many patients have multiple actinic keratoses, which
often develop SCC in regions with chronic inflammation or
increase their overall risk. Actinic keratoses present as ill-
scarring. SCC can be locally invasive and has the potential to
defined, pink, scaly papules or plaques, mostly in sun exposed
metastasize. Histologically, it can range from well differenti
areas (Figure 113). Individual lesions can be treated with cryo
ated SCC to the more aggressive poorly differentiated SCC.
therapy. [n patients with diffuse actinic damage, field treat
High risk features in SCC are tumor diameter greater than
ment with topical medications (5 fluorouracil, imiquimod, 2 cm, tumor thickness greater than 2 mm, poorly differenti-
ingenol mebutate) or photodynamic therapy can be used. ated subtypes, perineural invasion, and tumor location on the
Lesions that do not resolve with cryotherapy or are more indu
ear or nonglabrous lip (Figure 116).
rated will require biopsy to rule out an invasive neoplasm.
With SCC in situ (Bowen
disease), malignant keratino
cytes are confined to the epidermis. It appears as large scaly
Squamous Cell Carcinoma and Keratoacanthoma pink and tan plaques with well defined borders (Figure 117).
Squamous cell carcinoma (SCC), the second most common Surgical excision, including Mohs micrographic surgery
skin cancer, is a malignant neoplasm of keratinocytes. It pre is typically first line treatment lor high risk SCC given its
sents as pink, scaly indurated plaques, papules, or nodules potentially aggressive behavior and risk for metastasis. If sur-
that can ulcerate and bleed (Figure 114 and Figure 115). Risk gery is contraindicated. radiation therapy is an option. In cases
factors are UV light exposure, ionizing radiation, chemical of metastasis, chemotherapy can also be considered.
carcinogens (coal tar and arsenic), viruses (HPV), and immu Keratoacanthoma is considered to be a variant of SCC by
nosuppression (organ transplant). SCC occurs in areas of sun some and a benign tumor by others. Histologicallyl it can
exposure and areas of chronic injury (burn scars, irradiated resemble SCC. Keratoacanthoma has a distinct appearance and
sites, discoid lupus erythematosus). Compared with lightly
t IGUR E I 1 6. Squamous cell carcinoma lesions on the lower lip most commonly
F IGUR E I 1 4. Pink scaly plaque consistent with squamous cell carcinoma arise from sun damage, often in the setting o{ actinic cheilitis.
122
Dermatologic Disorders
123
Dermatologic Disorders
Melanoma
Melanoma is a malignant neoplasm of the melanocytes.
Although it is less common than BCC and SCC, it is histologi-
cally aggressive and has a much higher rate of metastasis. It is FIGUnE 121. Asuperficial spreading melanoma with prominentABCD:
responsible for most skin cancer deaths. In the United States, Asymmetry, inegular Borders, Color variation, and large size (Diameter).
124
Dermatologic Disorders
**
FIGURE 123.lll-delined,asymmetric,black-grayulceratedplaqueontheheel,
typical of acral lentiginous melanoma.
avoided in most pigmented lesions because there is risk fbr also arise from preexisting nevi, as did this melanoma. Nodular melanomas grow
vertically rather than horizontally.
transecting a melanoma and preventing accurate staging of
the lesion. A modified shave technique for pigmented lesions
is scoop biopsy, in which the deep dermis or subcutaneous Poor prognostic factors include male gender, increasing
tissue is removed to get underneath the lesion. In wide, ill- age, increased tumor thickness (Breslow depth), ulceration,
defined lesions, it may be impossible to remove the entire increased tumor mitotic rate, and head/neck/trunk locations.
lesion and thus an incisional biopsy is acceptable. Definitive Melanomas are staged with the tumor, nodal, and metastasis
treatment cannot be determined until histologic confirmation (TNM) system (https'//cancerstaging.org/references tools/
and final staging of the tumor is completed. quickreferences/Documents/ MelanomaLarge.pdf ).
125
Dermatologic Disorders
IE? POIIIS severe psoriasis also have a greater risk for cardiovascular dis-
t IGU R E I 26 . Psoriasis presents as well-demarcated, erythematous, tl GU RE 12 7. Lichen planus presents as purple, flat-topped papules, most
polymorphous plaques with silvery scale. commonly on the ankles and wrists.
126
Dermatologic Disorders
127
Dermatologic Disorders
'':'*Ell S+. Conditions ferrrr,rorrly.Associated With TAEI-E S7, Causes of Acute and Chronic Ervthrodernra
Erythenra Nodosum
Causes ofAcute Causes of Chronic
Streptococcal infection Erythroderma Erythroderma
Pregnancy Atopic dermatitis (flare) Atopic dermatitis
Oral contraceptive agent use Psoriasis (flare) Psoriasis
Hormone replacement therapy use Medication reaction Cutaneous T-cell lymphoma
lnflammatory bowel disease Autoimmune disease Graft-versus-host d isease
Erythroderma
Erythroderma is defined as diffuse erythema covering 80%
to 90o1, BSA that is commonly associated with pruritus,
peripheral edema, erosions, scaling, and lymphadenopathy
(Figure 131). Erythroderma may be acute or chronic in pres-
entation. Up to 40% of cases are idiopathic with exacerba-
tion of a preexisting rash and medication reaction being
other common etiologies (Table 87). Erythroderma second-
ary to drug reactions, staphylococcal scalded skin syn-
drome, and autoimmune bullous diseases often have an
acute onset without a long-standing history of preceding
dermatosis. Atopic dermatitis or psoriasis can flare to eryth-
roderma after injudicious use and abrupt cessation of sys- FIGURE I 32. Ihis patientwith cutaneousT-cell lymphoma hasclassicpatch'
temic glucocorticoids. Alopecia, nail dystrophy, and thick- stage mycosis {ungoides with well-demarcated patches and thin overlying scale on
the back. ln patients with skin o{ color, these patches are often hyperpigmented
ening of the palms and soles are indicative of a long standing
with less erythema and are usually pruritic.
cause, such as cutaneous T-cell lymphoma (Figure 132),
graft-versus-host disease, psoriasis, or pityriasis rubra pila-
Management of erythroderma involves treatment of
ris. Complications may result from heat and fluid loss across
the inflamed skin.
infection and managing fluid and electrolyte imbalance.
Emollients will help to restore the skin barrier, and topical
glucocorticoids and systemic antihistamines will improve
pruritus.
Sweet Syndrome
Sweet slmdrome, or acute febrile neutrophilic dermatosis, is
characterized by painful, edematous, red-to violaceous "juicy"
papules and plaques, most common on the face, neck, and
extremities (Figure 133). In addition to the skin emption,
patients typically have high fevers, leukocytosis with a left
shift, elevated inflammatory markers, and muscle or joint
pain. Histolory reveals a dense dermal infiltrate of neutrophils.
Most cases of Sweet syndrome Bpically develop after an upper
respiratory or gastrointestinal infection or in the setting of
FIGU R E 131 . Erythroderma with redness and scaling secondary to pityriasis hematologic abnormalities, particularly myelodysplastic syn-
rubra pilaris. drome or myelodysplastic syndrome transforming to acute
128
Bibliography
Bibliography
Routine Care ofthe Healthy Patient
American l)iabetes Association. 10. Cardio\ascr-rlar diseirse and risk manage
ment: standards ol medicll crre in diabetes 2019. I)iabetes (lare. 2019:
.12:S103 2rl. IPMID: l]05592:']61 doi:l0.2ll37idcl9 5010
t IGUR E 13 3. Ihe tender skin lesions of Sweet syndrome appear as "juicy"
indurated edematous red'purple plaques and nodules, sharply demarcated from Arnett DK, Blumenthal RS. Albcrt MA. et al. 2019 ACCIAIIA guicleline on thc
primary preventior] of carcliovascul,rr disease: a report of thc American
the adjacent skin.
College ofCardiologyiAmerican llexrt Associltion Trsk Force on Clinicll
Practice Cuidelines. Circulation. 2019r1.10:c596 646. IPNillD: 30879355]
tkri:10.1161 CIR.0000000000000678
Bibbins Donringo K, (;rossman I)C, Curry SJ. et xl: US Preventive Seruices 'lask
Acute myelogenous leukemia Force. Screening firr colorectal cancer: US Preventive Senices Task Firrce
rec{)mmendation strtement. JAN'IA. 2016;31 5 256,1-75. [PMI l): 27 301597 ]
BehEet syndromu doi: 10.1001 ijama.2016.5989
:
Chronic leukemia I
Bibbins Donringo K. Crossnran I)C. Curry SJ, et al; US Prerentir,e Senices'ljrsk
Ijorcc. Screening fi)r obstnlctive sleep apnea in adr.rlts: US Preveutive
lnflammatory browel disease Services Task Force recomnrend:rtion statement. JAMA. 2017:ill7:407 l.'1.
Multiple myeloma JPMID: 281 l846ll doi:10.10o1 ijama.20l6.20325
Cunninghanr ,\l-. Lrl II. Kovdc \,1. et al: ZOE 70 Stud),Croup. Ellicacy of the
Myelodysplastic syndrome herpes zoster subunit \accinr in adults 70 ycars of ilge or older N Engl J
Mcd. 2016:i175:1019 il2. IPMII): 276'26517] doi:10.1056,rNEJMo41603800
Myeloproliferative disorders
Curry SJ. Krist AH. Ou'ens I)K. et llr US Preventi\€ Senices'lask lirrcc.
Relapsing polychondritis Intenentions to prc\ent perinatal depression: US Prevcntive Seruices'lirsk
Rrrce reconrnrendation statement. JAMA. 2019;321:580 7. IPMll): :]0747971]l
Rheumatoid arthritis doj: 10.1001'jrma.2019.0007
Solid tumors (rare) Curry SJ, Krist AH. Owens [)K, et ll;
US Preventive Seruices'llsk l;trrcc.
Screening lirr cer-vical cancer: US I'reventive Services l-ask l,irrce recon]
Thyroid disease n]endation statement. J,\\,1A. 2018:320:671 86. ll,MID: 301.108tt.11
doi: 10.1001, jama.2018.10897
Curry SJ. Krist AH. Owens I)l(. et al: LIS Prevcntive Scrvices l'ask Forr,e.
Screeninll fi)r osteoporosis to pre\ent lii:lcturesi Prevcntive Srn ices Tlsk
myeloid Ieukenria (Table 88). Sweet syndrome has also been LIS
Iirrce reconrmendrtion strtement. r\MA. 2018:lll9:2521 25tll. LPMII):
associated with solid rnalignancies and medications (particu ')99167351 doi:10. lO0l 1iamx.2018.7498 I I
larl1, neutrophil stimulating n.redications, such as granulocyte Daridsorl K\di Barry NIJ. Mangione Cl!l. et rrlr []S Preventire Senices lirsk
Iirrce. Scrccning ltrr colorectal cancer: US Preventive Serviccs'lnsk Rrrce
colony stimulating factor and all trans retinoic acid). Rapid rccommenclation statement. JAMA. 2021;325:I965 1977. IP]\{lD: 3.10032I8|
rcsponse to systemic glucocorticoids is characteristic. doi:10.1001'j4ma.2021.6238
Davidson KW Barry l\,IJ, N{rngione CM, et al; IJS Preventi\€ Services 'Lrsk
KEY POITTS Iror-ce. Scrccning ftrr prediabeles and type 2 diabetes: US Pre\'€ntive Services
'lirsk Force recomrncndalion statemcnt. IAMA. 2021::126:736 713. [PN,llD:
. Cardiovascular disease is the leading cause ofdeath 3,1.1 )7591l doi:10. I 00I jamir.2021.125i11
among patients wilh severe psoriasis. Diaz(lranados CA, Dunning Al. Kimmcl M, et al. I.ifficacy ot high dose versr-ts
. Topical glucocorticoids are the mainstay of treatment
stirndard ckrseinfluenzaraccineinolderadults.NEnglJl\,led.20llr371:6i15
.15. I PMID: 25 119609] doi:1 0. I 056i NIIJ M oa13t s7 27
for localized psoriasis; however, in patients with exten Ferkct BS. Grootenboer N. Colkesen EB. et al. Systerratic review ol guidelinrs
sive involvement, psoriatic arthritis, psoriasis recalci on lbdominll aortic LlneurJ/srn screening. J \hsc Surg.2012;55:1296 3O'1.
ll'}MID: 2132.16301 doi:10.1016, j.jvs.2010.10.118
trant to topical treatments, or psoriasis in locations Fontham El'll. Woll AMD. Churcl-r TR. et al. (lervical cilrlccr scrcening lirr
such as the scalp or groin, phototherapy, methotrexate, inclividuals at averrge risk: 2020 guirleline update from thc .\merican
Crncer SocieRi CA Cancer J Clin. 2020. IPMID: 32729638] ckri:10.31122
acitretin, and cyclosporine should be considered. cilrc.21628
o Lichen planus is treated with potent topical glucocorti Freedman \,1S. Bernstein H. Ault KA. Recomnrended ir(lult imnrllnizirtiorl
schedule. tlnited Stirtes. 2021. Ann Intern Mecl. 202l:171:371 8.1. LPMII):
coids, whereas systemic glucocorticoids, oral retinoids,
3:].5710111 doi:10.7326,1M20 8080
sulfasaiazine, and phototherapy are used for severe Girnguli I. Souza l. Mc\Villi.rnrs .lM. el rl. Associrtion of \'ledicirre s annurrl
cutaneous or persistent oral disease. rvcllness visit s,ith crncer screening, ref'errals. utilizxtion. and spencling.
Continued) lle.rlth Ati (Millwrnd). 2019r38:1927 :15. IPMID: 31682.513] doi:10.13771
h11 haff.201 9.0030'1
129
Bibliography
Grossman DC, Curry SJ, Owens DK, et al; US Preventive Services Task Force. Owens DK. Davidson KW Krist AH, et al; US Preventive Services Task Force.
Screening for prostate cancer: US Preventive Services Task Force recom Screening for HIV infection: US Preventive Services Task Force recommen
mendation statement. JAMA. 2018;319:1901-13. [pMID: 29801017] dation statement. JAMA. 2019;321:2326-36. IPMID: 311847011
doi:10.1001/jama.2018.3710
Piercy KL, Troiano RP, Ballard RM, et al. The physical activity guidelines for
Krist AH, Davidson KW, Mangione CM, et al; US Preventive Services Task Americans. JAMA. 2018;320:2O20 8. [PMID: 30,118471] doi:10.10O1
Force. Behavioral counseling interventions to promote a healthy diet and
physical activity for cardiovascular disease prevention in adults with car
iama.2018.14854 I
Qaseem A, Crandall CJ, Mustafa RA, et al; Clinical Guidelines Commiftee of the
diovascular risk factors: US Preventive Services Task Force recommenda
American College of Physicians. Screening for colorectal cancer in asymp-
tion statement. JAMA. 2020;324:2069 -75. [PMID: 332316701 doi:10.1001/ tomatic arerage risk adults: a guidance statement from the American
jama.2O2O.21749
College ofPhysicians. Ann Intern Med. 2019;171:643-654. IPMID: 31683290]
Krist AH, Davidson KW' Mangione CM, et al; US Preventive Services Task doi:10.7326rM19 0642
Force. lnterventions for tobacco smoking cessation in adults, including
Qaseem A, Lin JS, Mustafa RA, et al; Clinical Guidelines Committee of the
pregnant persons: US Preventive Services Task Force recommendation American College ofPhysicians. Screening for breast cancer in average-risk
statement. JAMA. 2021;325:265 79. [PMID: 33464343] doi:10.1001/jama. women: a guidance statement from the American College of Physicians.
2020.2so19 Ann Intem Med. 2019;17O:547-6O. [PMID: 3O959525] doi:10.7326lMl8 2147
Krist AH, Davidson KW Mangione CM, et all US Preventive Services Task Schillie S, Harris A, Link Gelles R, et al. Recommendations of the Advisory
Force. Screening for asymptomatic carotid artery stenosis: US Preventive Committee on Immunization Practices for use of a hepatitis B vaccine with
Services Task Force recommendation statement. JAMA. 202L325:476-81. a novel adjuvant. MMWR Morb Mortal Wkly Rep. 2018;67:455-8. [PMID:
IPMID: 335285421 doi:10.1001 /jama.2020.26988 29 67 247 2l doi :10. 1 5585 /mmwr.mm6715a5
Krist AH, Davidson KW, Mangione CM, et al; US Preventive Services Task Schillie Vellozzi C, Reingold A, et al. Prevention ofhepatitis B virus infection
S,
Force. Screening for hepatitis B virus infection in adolescents and adults: in the United States: recommendations of the Advisory Committee on
US Preventive Services Task Force recommendation statement. IAMA. lmmunization Practices. MMWR Recomm Rep. 2018:67:1-31. [PMID:
2O2O ;324 :2415 22. IPMID: 33320230] doi :1 0. 1001 /iama .2020.22980 299399801 doi:10.15585/mmwr.116701a1
Krist AH, Davidson KW Mangione CM, et al; US Preventive Services Task Schillie S, Wester C, Osborne M, et al. CDC recommendations for hepatitis C
Force. Screening for hypertension in adults: US Preventive Services Task screening among adults-United States, 2020. MMWR Recomm Rep.
Force reaffirmation recommendation statement. IAMA. 2021;325:1650-6. 2O2O :69 :l-17. IPMID : 322717231 doi:10.15585 /mmwrr.69D2al
IPMID: 33904861J doi:10.1001/jam a.2O21.49a7 Siu AL; US Preventive Services Task Force. Screening for abnormal blood glu- \
Krist AH, Davidson KW Mangione CM, et al; US Preventive Services Task cose and tlpe 2 diabetes mellitus: U.S. Preventive Services Task Force
Force. Screening for lung cancer: US Preventive Services Task Force recom recommendation statement. Ann Intern Med. 2015;163:861-8. [PMID:
mendation statement. JAMA. 2021;325:962-70. IPMID: 3368747O] 265015131 doi:10.7326lM15 2345
doi:10.1001ijama.2021.1177 Siu AL; U.S. Preventive Services Task Force. Screening for breast cancer: U.S.
Krist AH, Davidson KW, Mangione CM, et al; US Preventive Services Task Preventive Services Task Force recommendation statement. Ann Intern
Force. Screening for unhealthy drug use: US Preventive Services Task Force Med. 2016:161:279-96. [PMID; 267571701 doi:10.73261M15 2886
recommendation statement. IAMA. 2020;323:2301 9. [PMID: 32515821] Siu AL; U.S. Preventive Sewices Task Force. Screening for high blood pressure
doi:10.1001/jama.2020.8020 in adults: U.S. Preventive Services Task Force recommendation statement.
Krogsboll LT, Jorgensen KJ, Gotzsche PC. General health checks in adults for Ann Intem Med. 2015 ;163 :778 86. [PM I D: 26458 1231 doi:LO.7 326 / Ml 5 2223
reducing morbidity and mortality from disease. Cochrane Database Syst Rev. Siu AL, Bibbins-Domingo K, Grossman DC, et al; US Preventive Services Task
2019;1:CD009009. IPMID: 30699470] doi:10.1002/14651858.CD009O09.pub3 Force. Screening for depression in adults: US Preventive Services Task Force
LeFevre ML; U.S. Preventive Services Task Force. Screening for abdominal aortic recommendation statement. IAMA. 2016;315:380-7. [PMID: 268132111
aneurysm: U.S. Preventive Services Task Force recommendation statement. doi:10.1001 /jama.2O15.18392
Ann Intem Med. 2 Ol4 ;167 :281 90. [PMl D: 24957320] doi:10.7326lMl4 l2O4 Sosa LE, Njie GJ, Lobato MN, et al. Tuberculosis Screening, Testing, and
Manson JE, Bassuk SS. Vitamin and mineral supplements: what clinicians Treatment of U.S. Health Care Personnel: Recommendations from the
National Tuberculosis Controllers Association and CDC, 2019. MMWR
need to know. JAMA. 2018;319:859-60. [PMID: 29404568] doi:10.10011
jama.2017.2lol2 Morb Mortal Wkly Rep. 2019;68:439 43. [PMID, 31099768]
Zheng SL, Roddick Al. Association ofaspirin use for primary prevention with
Marin M, Marlow M, Moore KL, et al. Recommendation of the Advisory
cardiovascular events and bleeding events: a systematic review and meta
Committee on Immunization Practices for use of a third dose of mumps
analysis. JAMA. 2019 ;327:277 -87. [PMID: 30667501] doi:l0.l001rjama.2O18.
virus containing vaccine in persons at increased risk for mumps during an
20578
outbreak. MMWR Morb Mortal Wkly Rep. 2018;67:33-38. [PMID: 2932a728]
doi:10. 1 5585 /mmwr.mm67 01 a7 Zulman DM, Haverfield MC, Shaw JG, et al. Practices to foster physician pres-
ence and connection with patients in the clinical encounter. JAMA.
Mbaeyi SA, Bozio CH, Duffy J, et al. Meningococcal vaccination: recommenda
2020:323:70-81. IPMID: 31910284] doi:10.1001/iama.2019.19003
tions of the Advisory Committee on Immunization Practices, United States,
2020. MMWR Recomm Rep. 2020;69(No. RR-9):l-41. DOI: http://dx.doi. Patient Safety and Quality Impmvement
org/10.15585/mmwr.rr6909a1
Campagna V, Nelson SA, Krsnak J. Improving care transitions to drive patient
Meites E, Szilag/i PG, Chesson HW, et al. Human papillomavirus vaccination outcomes: the Triple Aim meets the Four Pillars. Prof Case Manag.
for adults: updated recommendations of the Advisory Committee on 201921:297 305. IPMID: 315802961 doi:l0.7097 /NCM.0o00000000000387
Immunization Practices. MMWR Morb Mortal Wkly Rep. 2019;68:698-702.
Clarke GM, Conti S, Wolters AT, et al. Evaluating the impact of healthcare
IPMID: 31415491] doi:10.15585/mmwr.mm6832a3
interventions using routine data. BMJ. 2ol9;365:12239. [PMID, 31221675]
Nelson NE Weng MK, Hofmeister MG, et al. Prevention of hepatitis A virus doi:10. ll36/bmj.12239
infection in the United States: recommendations of the Advisory Commiftee
Heisler M. Do patient centered medical homes enhance patients' medication
on Immunization Practices, 2020. MMWR Recomm Rep. 2020;69:1 38.
adherence? Findings on the next frontier in quality improvement. Ann
IPMID: 326148111 doi:10.15585/mmwr.rr69O5a1
Intern Med. 2017 ;166:146-7. IPMID: 27842385]
Oefhnger KC, Fontham ET, Etzioni R, et al; American Cancer Society. Breast
Kini V, Ho PM. lnterventions to improve medication adherence: A review [Letter].
cancer screening for women at average risk: 2015 guideline update from the
JAMA. 2018:320:2461-2473. IPMID: 30561486] doi:10.1001/jama.2018.19277
American Cancer Society. JAMA. 2015;314:1599 6t4. [PMID: 26501536]
doir10.1001 /jama.2015.12783 The Joint Commission. 2020 National Patient Safety Goals. www.jointcommis
sion.org/standards/national-patient-safety-goals. Accessed July 24, 2020.
Owens DK, Davidson KW Krist AH, et al; US Preventive Services Task Force.
Risk assessment, genetic counseling, and genetic testing for BRCA related The Joint Commission. Patient safety systems. Comprehensive Accreditation
cancer: US Preventive Services Task Force recommendation statement. Manual for Hospitals. Update 2. January 2016. www.iointcommission.org
JAMA. 2019;322:6s2-6s. [PMID: 31429903] assets/l/18/Psc for Web.pdf. Accessed June 10, 2020.
Owens DK, Davidson KW Krist AH, et al; US Preventive Services Task Force. The Joint Cornmission. Performance measurement. www.jointcommission.
Screening for hepatitis C virus infection in adolescents and adults: US org/en/measurement/. AcceSsed May 14, 2O2O-
Prcventive Services Task Force recommendation statement. JAMA. 2020. Kamermayer AK, Leasure AR, Anderson L. The effectiveness oftransitions-of
IPMID: 321190761 doi:10.1001/jama.2O2O1723 care interventions in reducing hospital readmissions and mortality: a
130
Bibliography
systematic review. Dimens Crit Care Nurs. 2ou;36:311-6. IPMID: 28976480] Chung F, Memtsoudis SG, Ramachandran SK, et al. Society of anesthesia and
doi:1o.1097/DCC.0000000000000266 sleep medicine guidelines on preoperative screening and assessment of
adult patients with obstructive sleep apnea. Anesth Analg.2016;123:452 73.
Kini V Ho PM. Interventions to improve medication adherence, a review
2o18;320:2461 73. IPMID, 30561486] doi:10.1001/iama. IPMID : 27442772) doi:10.1213/ANE.0000000000001416
lletterl. JAMA.
2018.t9271 Cohn SL. Preoperative evaiuation for noncardiac surgery. Ann lntern Med.
Shah A. Using data for improvement. BMJ. 2o19;364:1189. IPMID: 30770353] 2016r165:lTC81-96. IPMID, 279190971 doi:10.7326/AITC201612060
doi:10.1136/bmj.l189 Doherty JU, Cluckman Tl, Hucker WJ, et al.2017 ACC expert consensus deci-
sion pathway for periprocedural management ofanticoagulation in patients
Silver SA, Harel Z, McQuillan R, et al. How to begin a quality improvement
project. Clin J Am Soc Nephrol. 2016;11:893 90o. IPMID: 27016497) with nonvalvular Atrial Fibrillation: a report of the American College of
doi:10.221 5/C]N.11491015
Cardiologr Clinical Expert Consensus Document Task Force. I Am Coll
Cardiol. 2017;69:871 98. [PMID: 28081965] doi:10.1016/j.jacc.2016.11.024
Yoon J, Chang E, Rubenstein LV et al. lmpact of primary care intensive man
agement on high risk veterans' costs and utilization: a randomized quality Duggan EW Carlson K, Umpierrez GE. Perioperative hyperglycemia manage
improvement trial. Ann lntern Med. 2018;168:846 54. [PMID: 29868706] ment: an update. Anesthesiolory. 2017;126:547-60. [PMID: 28121636]
doi:10.7326lM17 3039 doi:10.1097/ALN.0000o00000001515
U.S. Department of Health and Human Services. Centers for Medicare and Fleisher LA, Fleischmann KE, Auerbach AD, et al; American College of
Medicaid Services. CMS core measures. March 27, 2020. www.cms.gov/ Cardiologr. 2014 ACC/AHA guideline on perioperative cardiovascular eval
Medicare/Quality lnitiatives Patient Assessment-lnstruments/ uation and management of patients undergoing noncardiac surgery: a
report of the American College of Cardiolos//American Heart Association
QualityMeasures/Core Measures. Accessed July 24, 2020.
Task Force on Practice Guidelines. I Am Coll Cardiol.2o14;64:e77 137.
U.S. Department of Health & Human Services, Health Resources and Services IPMID: 2s0915441 doi:10.1016/j.jacc.2014.07.9 44
Administration. Quality improvement. April 2011. www.hrsa.gov/quality/
toolbox/508pdfs/qualio/improvement.pdf. Accessed June 10, 2020. Gialdini G, Nearing K, Bhave PD, et al. Perioperative atrial fibrillation and the
long-term risk of ischemic stroke. JAMA. 2014;312:616 22. IPMID: 25117130]
U.S. Department of Health & Human Services, Office of Disease Prevention doi : 10. 1001 /jama.2014.9743
and Health Promotion. National action plan to improve health literacy. May
2010. https://health.gov/communication/hlactionplan/pdf/Health Gould MK, Garcia DA, Wren SM, et al. Prevention of VTE in nonorthopedic surgi-
Literacy_Action Plan.pdf. Accessed June 10, 2020. cal patients: antithrombotic therapy and prevention ofthrombosis, 9th ed:
American College ofchest Physicians evidence-based clinical practice guide-
Hospital Medicine Principles lines. Chest. 2Ol2;141:e2275 7S. [PMID:22315263] doi:10.1378/chest.11-2297
Destino LA, Dixit A, Pantaleoni JL, et al. Improving communication with pri- Graham MM, Sessler D[, Parlow JL, et al. Aspirin in patients with previous
mary care physicians at the time of hospital discharge. Jt Comm i Qual percutaneous coronary intervention undergoing noncardiac surgery. Ann
Patient Saf. 2017;43:80-8. IPMID: 283345661 doi:10.1016/j.jcjq.2016.11.00s Intern Med. 2Ol8 ;168 237 44. [PMID: 29132159] doi:LO.7 326 / Mt7 -23 41
Fang M, Greysen SR. The role of the medical consultant in 2018: putting it January CI, Wann LS, Calkins H, et al. 2019 AHA/ACC/HRS focused update of
all together [Editorial]. J Hosp Med. 2018;13:81 2. [PMID: 30534637] the 2014 AHA/ACC/HRS guideline for the management of patients with
doi:10.12788/jhm.3l23 atrial fibrillation: a report of the American College of Cardiolory/American
Heart Association Task Force on Clinical Practice Guidelines and the Heart
Gerotziafas GT, Papageorgiou L, Salta
S, et al. Updated clinical models for VTE
prediction in hospitalized medical patients. Thromb Res. 2O18i164 Suppl Rhythm Society. J Am Coll Cardiol. 2019;74:104-32. [PMID: 30703431]
doi:1O.10i6/j.jacc.2O19.01.011
1 :S62-569. IPMID: 29703486] doi:10.1016ij.thromres.2018.02.004
Levine GN, Bates ER, Bittl JA, et al. 2016 ACC/AHA guideline focused update on
Marcantonio ER. Delirium in hospitalized older adults. N Engl J Med.
duration of dual antiplatelet therapy in patients with coronary artery dis
2Ol7 ;377 :1456 1466. [PMID: 29o2o5791 doi:10.10s6/NEJMcp1605501
ease: a report of the American College of Cardiolos//American Heart
Nichani S, Fitterman N, Lukela M, et al; Society of Hospital Medicine. The core Association Task Force on Clinical Practice Guidelines. I Am Coll Cardiol.
competencies in hospital medicine 2017 revision. Section 3: healthcare sys- 2016;68:1082 115. IPMID: 27036918] doi:10.1016/j.jacc.2016.03.513
tems. J Hosp Med. 2017;12:S55 S82. [PMID: 28411301] doi:10.12788/ jhm.2729
Lin MH, Kamel H, Singer DE, et al. Perioperative/postoperative atrial fibrilla
Rohatgi N, Loftus P. Grujic O, et al. Surgical comanagement by hospitalists tion and risk ofsubsequent stroke and/or mortality. Stroke. 2019;50:1364-
improves patient outcomes: a propensity score analysis. Ann Surg. 71. IPMID: 31o43r48] doi:10.1161/STROKEAHA.118.o23921
2Ot6;264:275 82. IPMID: 26764873] doi:10.1097/SLA.0000000000001629
Liu MM, Reidy AB, Saatee S, et al. Perioperative steroid management:
Schtinemann [I], Cushman M, Bumett AE, et al. American Societyof Hematolos/ approaches based on current evidence. Anesthesiolos/. 2017;727:166 72.
2018 guidelines for management of venous thromboembolism: prophylaxis IPMID: 284528061 doi:10.1o97lALN.0000000000oo16s9
for hospitalized and nonhospitalized medical patients. Blood Adv.
Mohanty S, Rosenthal RA, Russell MM, et al. Optimal perioperative management
2018;2:3198 225. IPMID: 304827631 doilrO.1182 lbloodadvances.20t8o22g54
ofthe geriatric patient: a best practices guideline from the American College
Scotten M, Manos EL, Malicoat A, et al. Minding the gap: Interprofessional com of Surgeons NSQIP and the American Geriatrics Sociery J Am Coll Surg.
munication during inpatient and post discharge chasm care. Patient Educ 2016;222:930 47. [PMID: 270497831 doi: 10.1016/j.jamcollsurg.2015.12.026
Couns. 2015;98:895-900. [PMID: 25862470) doi:10.1016/j.pec.2015.03.009
Northup PG, Friedman LS, Kamath PS. AGA clinical practice update on surgi
Wang ES, Moreland C, Shoffeitt M, et al. Who consults us and why? An evalua cal risk assessment and perioperative management in cirrhosis: expert
tion of medicine consult,/comanagement services at academic medicalcent review. Clin Gastroenterol Hepatol. 2019;17:595 606. [PMID: 30273751]
ers. J Hosp Med. 2018r13:840-3. [PMID, 301565821 doi:10.12788/jhm.2996 doi :10.1016/j.cgh.2018.09.043
Perioperative Medicine Odor PM, Bampoe S, Gilhooly D, et al. Perioperative interventions forprevention
of postop€rative pulmonary complications: systematic review and meta
American Diabetes Association. 15. Diabetes Care in the Hospital: Standards of analysis. BMJ. 2020;368:m54O. [PMID; 32161042] doi:10.1136/bmj.m540
Medical Care in Diabetes 2021. Diabetes Carc.2021 Jan;44(Suppl 1):5211-
20. [PMID: 33298426] doi:10.2337ldc21 S015. Palace MR. Perioperative management of thyroid dysfunction. Health Serv
lnsights. 2Ol7:10:1178632916689677. [PMID: 284694541 doiiO.llTT I
Anderson DR, Morgano GII Bennett C, et al. American Society of Hematolos/ 11786329t6689677
2019 guidelines for management of venous thromboembolism: prevention
of venous thromboembolism in surgical hospitalized patients. Blood Adv Pannucci CJ, Swistun L, MacDonald JK, et al. Individualized venous thrombo
2019;3:3898 944. IPMID: 377946021 doi:10.1182/bloodadvances.2O19000975 embolism risk stratification using the 2005 Caprini score to identily the
benefits and harms of chemoprophylaxis in surgical patients: a meta
Benesch C, Glance LG, Derdeyn CP, et al; American Heart Association Stroke analysis. Ann Surg. 2077;265:1094 1O3. [PMID: 28106607) doi:10.1097/
Council; Council on Arteriosclerosis, Thrombosis and Vascular Biolory; sLA.0000000000002126
Council on Cardiovascular and Stroke Nursing; Council on Clinical
Cardiologl: and Council on Epidemiolos/ and Prevention. Perioperative Simha V Shah P Perioperative glucose control in patients with diabetes under
neurological evaluation and management to lower the risk of acute stroke going elective surgery JAMA. 20191321:399-400. IPMID: 3061s0311
in patients undergoing noncardiac, nonneurological surgery: a scientific doi 10.1001 /jama.2018.20922
:
statement from the American Heart Association/American Stroke Weimann A, Braga M, Carli E et al. ESPEN guideline: clinical nutrition in surgery
Association. Circulation. 2O21:CIRO000000000000968. IPMID: 33827230] Clin Nutr. 2017 ;36:623 50. [PMID: 2838547] doi:10.1O16/j.c]nu.20U.02.O13
doi:1O.1161 /CIR.000OO0000O000968
Wijeysundera DN, Beattie WS, Hillis GS, et al; Measurement of Exercise Tolerance
Burton BN, A Court AM, Brovman EY et al. Optimizing preoperative anemia to before Surgery Study Investigators. Integration of the Duke Activity Status
improve patient outcomes. Anesthesiol Clin. 2o18;361701 13. [PMID: Index into preoperative risk evaluation: a multicentre prospective cohort study
303907891 doi:10.1016/j.anclin.2018.07.017 BrJ Anaesth.2020;724:261 70. [PMID:31864719] doi:10.1016/j.bja.2019.11.o25
131
Bibliography
Ob€sity Mulhall lB Trost LW Brannigan RE, et al. Evaluation and management of tes-
Arterbum DE, Telem DA, Kushner RFl Courcoulas AP Benefits and risks of tosterone deficiency: AUA guideline. J Urol. 2018:200:423 32. [PMID:
bariatric surgery in adults: a review. JAMA. 2O20:324:879-87. [pMID: 296019231 doi:10.1016/i.iuro.2018.03.115
328703011 doi:10.1001 /jama.2O2O.12567 Pirola GM, Verdacchi T, Rosadi S, et al. Chronic prostatitis: current treatment
lensen MD, Ryan DH, Apovian CM, et al; American College of Cardiolory/ options. Res Rep Urol. 2019;11:165 74. IPMID: 31240202) doi:10.2147lRRU.
American Heart Association Task Force on Practice Guidelines. 2013 AHA/ stg4679 j
ACC/TOS guidetine for the management of overweight and obesity in adults: Qaseem A, Horwitch CA, Vijan S, et al; Clinical Guidelines Committee of the
a report ofthe American College ofCardiolos//American Heart Association American College of Physicians. Testosterone treatment in adult men with
Task Force on Practice Guidelines and The Obesity Society. Circulation. age related low testosterone: a clinical guideline from the American College
2014;129:S102 38. [PMID: 24222017] doi:10.1161/01 .CX.OOOO437739.77477.ee of Physicians. Ann lntern Med. 2020;172:126 33. [PMID: 31905405]
Johnston BC, Kanters S, Bandayrel K, et al. Comparison ofweight loss among doi:1O.7326rM19 0882
named diet programs in overweight and obese adults, a meta analysis. Sharlip ID, Belker AM, Honig S, et al; American Urological Association.
JAMA. 2014r312:923 33. IPMID: 25182101] doi:10.1001/jama.2014.10397 Vasectomy: AUA guideline. J Urol. 2012;188:2482-9r. [PMID: 23098786]
Khera R, Murad MH, Chandar AK, et al. Association ofpharmacological treat- doi:10.1016/j.juro.2012.09.080
ments for obesity with weight loss and adverse events: a Systematic review
Women's Health
and meta-analysis. JAMA. 2016;315 :2424-34. [PMID: 27299618] doi:10.1001/
jama.2ol6.7602 American College of Obstetricians and Gynecologists committee opinion no.
LeBlanc ES, Patnode CD, Webber EM, et al. Behavioral and pharmacotherapy
736. Optimizing postpartum care. Obstet Gynecol. 2018;131:e140-50.
weight loss interventions to prevent obesity-related morbidity and mortal- IPMID: 296839111 doi:10.1097/AOG.0000000000002633
ity in adults: updated evidence report and systematic review for the US American College of Obstetricians and Gynecologists committee opinion no.
Preventive Services Task Force. JAMA. 2018;320:1172-91. [PMID: 30326501] 762 summary Prepregnancy counseling. Obstet Gynecol. 2019].133:228 30.
doi:10.1001/iama.2018.7777 IPMID: 305756721 doi:10.1097/AOG.0000000000003014
Marcotte E, Chand B. Management and prevention ofsurgical and nutritional American College of Obstetricians and Gynecologists practice bulletin sum
complications after bariatric surgery Surg Clin North Am. 2016;96:843-56. mary no. 213. Female sexual dysfunction. Obstet Gynecol. 2019;134:203-5.
IPMID: 27473805] doi:10.1016/j.suc.2016.03.006 IPMID: 312415951 doi:10.1097/AOC.0000000000003325
Mechanick Jl, Apovian C, Brethauer S, et al. Clinical practice guidelines for the American Psychiatric Association. Diognostic ond Stotisticol Manual of
perioperative nutrition, metabolic, and nonsurgical support of patients Mentol Disorders. 5th ed. American Psychiatric Association; 2013.
undergoing bariatric procedure-2Olg update: cosponsored by American Curtis KM, Peipert JE Long-acting reversible contraception. N Engl I Med.
Association ofClinical Endocrinologists/American College ofEndocrinolog/, 2Ol7 ;376:461 8. IPMID: 28146650] doi:10.1056/NEJMcp1608736
The Obesity Society, American Society for Metabolic & Bariatric Surgery
Obesity Medicine Association, and American Society of Anesthesiologists Holbrook AI, Moy L, Akin EA, et al; Expert Panel on Breast Imaging: ACR
Executive Summary Endocr Pract. 2019;2511346-59. [PMID: 31682518] appropriateness criteria' breast pain. J Am Coll Radiol. 2018;15:3276-82.
doi:10.4158/GL 2019 0406 IPMID: 30392596J doi:10.1o16/j.jacr.2018.09.014
Moyer VA; U.S. Preventive Services Task Force. Screening for and management Moy L, Heller SL, Bailey L, et al; Expert Panel on Breast Imaging:. ACR appro
of obesity in adults: U.S. Preventive Services Task Force recommendation priateness criteria' palpable Breast Masses. I Am Coll Radiol. 2017;14:S203-
statement. Ann Intern Med. 2012;157:373-8. [PMID: 22733087] doi: 24. [PMID: 28473077) doi:to.1016/i.iacr.20u.02.033
10.732610003 4819 157 5 201209040 00475 The NAMS 2017 Hormone Therapy Position Statement Advisory Panel. The
Puzziferri N, Roshek TB 3rd, Mayo HG, et al. Long-term follow-up after bari- 2017 hormone therapy position statement of the North American
atric surgery: a systematic review JAM A.2074;312:934-42. [PMID: 25182102] Menopause Society. Menopause. 2ol7;24:728-53. [PMID: 286508691
doi:10.1001/jama.2Ol4.10706 doi:10.1097/GM8.00000o0000000921
Tsai AG, Bessesen DH. Obesity. Ann Intern Med. 2019;170:lTC33-48. [PMID: Paladine HL, Blenning CE, Strangas Y. Postpartum care: an approach to the
30831s931 doi:10.7326lAITC201903050 fourth trimester. Am Fam Physician. 2019;100:485-91. [PMID, 31613576]
Tucker J, Fischer T, Upjohn L, et al. Unapproved pharmaceutical in$edients Paladine HL, Desai UA. Vaginitis: diagnosis and treatment. Am Fam Physician.
included in dietary supplements associated with US Food and Drug 2018;97:321 9. [PMID: 296715761
Administration warnings. IAMA Netlv Open. 2018;1:e183337. [PMID: Salzman B, Collins E, Hersh L. Common breast problems. Am Fam Physician.
306462381 doi:10.1001/jamanetlvorkopen.2018.3337 2O19:99:505-14. [PMID: 30990294]
Vidal J, Corcelles R, Jim6nez A, et al. Metabolic and bariatric surgery for obe Stuenkel CA, Davis SR, Gompel A, et al. Treatment ofsymptoms of the meno
sity. Gastroenterolosi. 2017;152:1780-90. [PMID: 28193516] doi:10.1053/j. pause: an Endocrine Society clinical practice guideline. J Clin Endocrinol
9astro.2017.01.051 Metab. 2015;100:3975- 4011. IPMID: 264449941 doi:10.121Orjc.2015-2236
Men's Health Woodhams EJ, Gilliam M. Contraception. Ann Intern Med. 2019;170:lTCl8-
ITC32. IPMID: 30716758] doi:10.7326/AITC201902050
Barry MJ, Fowler FJ lr, O'Leary ME et al; Measurement Committee of the
American Urological Association. The American Urological Association Wouk N, Helton M. Abnormal uterine bleeding in premenopausal women. Am
symptom index for benign prostatic hyperplasia. J Urol. 2017;197:5189-97. Fam Physician. 2ol9 :99 t 435- 4 43. I PM I D: 30932a48]
IPMID: 280127471 doi:10.1016/j.juro.2016.10.071
Ele Disorders
Burnett AL, Nehra A, Breau RH, et al. Erectile dysfunction: AUA guideline. J
Urol. 2018;20O:633 41. IPMID: 29746858) doi:10.1016/j.juro.2018.05.004 Akpek EK, Amescua G, Farid M, et al: American Academy of Ophthalmolory
Preferred Practice Pattern Cornea and Exlernal Disease Panel. Dry eye
Gill BC, Shoskes DA. Bacterial prostatitis. Curr Opin Infect Dis. 2016;29:86-91.
syndrome preferred practice pattern" . Ophthalmolory. 2019;126:P286 334.
IPMID: 26555o381 doi:10.1o97lQCO.0000000000000222
IPMID:303667981
Li X, Li J, Li X, et al. Effectiveness and safety offluoxetine for premature ejacu-
Amescua G, Ak"ek EK, Farid M, et al; American Academy of Ophthalmolog/
Iation; Protocol for a systematic review. Medicine (Baltimore). 2019;
Preferred Practice Paftem C-omea and Lxtemal Disease Panel. Blepharitis pre-
98:e14481. lPMlDt 3O7 6277 2l doi:10.1097/MD.0000000000014481
fened practice pattem". Ophthalmologr. 2019;126:PS6 93. [PMID: 30366800]
Liu H, Zhang M, Huang M, et al. Comparative emcacy and safety of drug treatment
Austin A, Lietman T, Rose Nussbaumer J. Update on the management of infec-
for premature ejaculation: a systemic review and Bayesian network meta
tious keratitis. Ophthalmologr. 2017:124:1678-89. [PMID: 28942073]
anab,sis. Andrologia. 2020:e13806. [PMtD: 32892379] doi:10.1111 /and.13806
doi:10.1016/j.ophtha.2017.05.012
Lorenzo L, Rogel R, Sanchez Gonzalez JV et al. Evaluation ofadult acute scro
Dattilo M, Biousse V Newman NJ. Update on the management of central reti
tum in the emergency room: clinical characteristics, diagnosis, manage-
ment, and costs. Urolory. 2016:94:36-41. [PMID; 27210570] doi:10.1016/j. nal artery occlusion. Neurol Clin. 2017;35:83 i00. [PMID: 278868971
doi:10.1016/j.nc1.2016.08.013
urolos/.2016.05.018
Martin C, Nolen H, Podolnick J, et al. Current and emerging therapies in pre-
lonas JB, Aung T, Bourne RR, et al. Glaucoma. Lancet. 2017;390:2183 93.
mature eiaculation: where we are coming from, where we are going. Int l [PMID: 28577860] doi:10.1016/5014o 6736(17)31469 t
Urol. 2Ol7 :24:40 -50. IPMID: 277046321 doll}.11U/iju.13202 Liu YC, Wilkins M, Kim T, et al. Cataracts. [-ancet. 2017;39o:600 12. [PMID:
Montgomery J, Dimick JB, Telem DA. Management of groin hernias in 282421111 doi:10.1016/s0140-6736(17)30s44 5
adults-2018. JAMA. 2018;320:1029-30. [PMID, gOtZgSOg] doi:10.1001/ Pflipsen M, Massaquoi M, Wolf S. Evaluation of the paintul eye. Am Fam
jama.2018.10680emprici Physician. 2016;93:991 -8. IPMID: 27304768]
132
Bibliography
Rosenbaum JT, Dick AD. The eyes have it: a rheumatologist,s view of uveitis. guidelines. J Allergz Clin Immunol. 2017;139:S49-57. [PMID: 28390477]
Arthritis Rheumatol. 2018;70:1533 43. [pMID: 29290291] doi:10.1002/ doi:10.1016/j.jaci.2017.01.009
art.40568
Fine LM, Bemstein JA Guideline of chronic urticaria beyond. Aller$/ Asthma
Stein JD, Khawaja AI Weizer JS. Glaucoma in adults-screening, diagnosis, and Immunol Res. 2016;8:396 403. IPMID:27334227]doi:tO.4168/aair.2016.8.5.396
management: areview IAMA. 2021;325:164-174. [PMID; 33433580] doi:10.1001/
jama.2o2o.21899 Goldburg SR, Strober BE, Payefte MJ. Hidradenitis suppurativa: current and
emerging treatments. J Am Acad Dermatol. 2020;82:1067 82. IpMID:
Tarff A, Behrens A. Ocular emergencies: red eye. Med Clin North Am. 31604i001 doi:10.1016/j.jaad.2019.08.089
2017; 101 :615-39. IPMID | 2837 2717) doi:10.1016/j.mcn a.2Ot6.t2.Ot3
Goldburg SR, Strober BE, Payette MJ. Hidradenitis suppurativa: epidemiolo$/,
Varu DM, Rhee MK, Akpek EK, et al; American Academy of Ophthalmologr clinical presentation, andpathogenesis. J Am Acad Dermatol. 2020;82:104S-
preferred practice pattern cornea and extemal disease panel. Conjunctivitis 58. [PMIDT 31604104] doi:10.1016/j.jaad.2O19.08.o9o
preferred practice pattern R. Ophthalmologr. 2Ol9;126:p94 169. [PMID;
Goydos JS, Shoen SL. Acral lentiginous melanoma. Cancer Treat Res.
303667971
2076;767:321-9. [PMID: 26601870] doi:10.1007/978 -3 3tg 22539-5 t4
Ear, Nose, Mouth, and Throat Disorders Greenhalgh DG. Management of burns. N Engl J Med. 2019;380:2349-59.
Bauer CA. Tinnitus. N Engl J Med. 2018;378:1224 31. [PMID: 296OL2SS]
IPMID: 311890381 doi:10.10s6/NEJM ra78o7442
doi:10.1056/NEJMcp1506631 Gupta AK, Foley KA, Mays RR, et al. Monotherapy for toenail onychomycosis:
Chandrasekhar SS, Tsai Do BS, Schwartz SR, et al. Clinical practice guideline: a systematic review and network meta-analysis. Br J Dermatol. 2020;
sudden hearing loss (update). Otolaryngol Head Neck Surg. 2019;161(1 782:287 99. [PMID: 31120134] doi:10.1111 /bjd.181ss
suppl) :51 45. [PMID: 31369359] doi:7o.7177 I Ol9 459981985988s Higgins JC, Maher MH, Douglas MS. Diagnosing common benign skin tumors.
Cunningham LL, Tucci DL. Hearing loss in adults. N Engl J Med. 2017;377:2465- Am Fam Physician. 2O15;92:601-7. lPMlDt 26447443)
73. IPMID: 29262274] doi;10.1056/NEJMra1616601 Hinshaw MA, Rubin A. Inflammatory diseases of the nail unit. Semin Cutan
Med Surg. 2015;34:109 16. [PMID: 26176289] doi:10.12788/j.sder.2o15.0132
Harris AM, Hicks LA, Qaseem A; High Value Care Task Force ofthe American
College of Physicians and for the Centers for Disease Control and Huerth KA, Hassan S, Callender VD. Therapeutic insights in melasma and
Prevention. Appropriate antibiotic use for acute respiratory tract infection hyperpigmentation management. J Drugs Dermatol. 2019;18:718 29.
in adults: advice for high value care from the American College of IPMID:37424704)
Physicians and the Centers for Disease Control and Prevention. Ann Intern Kaushik N, Pujalte GG, Reese ST Superficial fungal infections. Prim Care.
Med. 20t6 ;16 4 : 425 34. [PMID: 267 85 4O2] doi: 10. 7326 /M15 1840 2Ol5 ;42:507-16. [PMID: 26612371] doi:10.1016/j.pop.2015.08.004
Klein MR. Infections of the oropharynx. Emerg Med Clin North Am. Kim JY, Kozlow JH, Mittal B, et al; Work Group. Guidelines of care for the
2Ol9 ;37 :69 -8O. IPMID: 304547811 doi:10.1016/j.emc.2018.09.002 management ofbasal cell carcinoma. J Am Acad Dermatol. 2018;78:540-59.
Nieman CL, Oh ES. Hearing loss. Ann Intern Med. 2020;173:ITC81 96. IPMID: IPMID, 293313851 doi:10.1016/j.jaad.2017.10.006
332536101 doi:10.7326 I AITC2O2O72OIO Kim IY, Kozlow lH, Mittal B, et al; Work Croup. Guidelines of care for the
Ogle OE. Salivary gland diseases. Dent Clin North Am. 2020;64:87-104. [PMID: management ofcutaneous squamous cell carcinoma. J Am Acad Dermatol.
317352351 doi:10.1016/j.cden.2019.08.007 2018;78:560-78. IPMID: 29331386] doi:10.1016/j.jaad.2017.10.007
Rosenfeld RM, Schwartz SR, Cannon CR, et al. Clinical practice guideline: Lebwohl M. Psoriasis. Ann Intern Med. 2018;168:lTC49 64. IPMID: 29610923)
acute otitis externa. Otolaryngol Head Neck Surg. 2014;150:51-24. [PMID: doi:1O.7326lAITC20180403O
244913101 doi:10.7177 I O19 4599813517083 Marson JW, Baldwin HE. Rosacea: a wholistic review and update from patho
Schwartz SR, Magit AE, Rosenfeld RM, et al. Clinical practice guideline genesis to diagnosis and therapy. Int J Dermatol. 2020;59:e175-82. [PMID:
(update): earwax (cerumen impaction). Otolaryngol Head Neck Surg. 3188o3271 doi:10.1111/ijd.14757
2017;156:51-29. IPMID: 28045591] doi:10.1177l0194599a1667t497 Metin A, Dilek N, Demirseven DD. Fungal infections of the folds (intertrigi-
Stephens MB, Wiedemer JP, Kushner GM. Dental problems in primary care. nous areas). Clin Dermatol. 2015 ;33 : 437 47. IPMID : 26051058J doi : 10.1 01 6 /j.
Am Fam Physician. 2018;98:654 60. IPMID: 30485039] clindermatol.2015.04.005
Tunkel DE, Anne S, Payne SC, et al. Clinical Practice Guideline: nosebleed Monteiro AF, Rato M, Martins C. Drug-induced photosensitivity: photoallergic
(epista-xis) executive summary. Otolaryngol Head Neck Surg. 2020;162:8- and phototoxic reactions. Clin Dermatol. 2016 Sep-Oct;34:571-81. [PMID:
27 6384351 doi: 10.1016 /j.clindermatol.2016. 05. 006
2s. IPMID: 3l9tot221 doi:to.lU7 I 0194599819889955
Mubki T, Rudnicka L, Olszewska M, et al. Evaluation and diagnosis ofthe hair
Dermatologic Disorders loss patient: part I. History and clinical examination. J Am Acad Dermatol.
Antonov D, Schliemann S, Elsner P Hand dermatitis: a review of clinical fea- 2074;71:475.e7-415.el5. IPMID: 25128118] doi:10.1016/j.jaad .2014.O4.07O
tures, prevention and treatment. Am I Clin Dermatol. 2015;76:257 70. Mubki T, Rudnicka L, Olszewska M, et al. Evaluation and diagnosis ofthe hair
IPMID: 25920436] doi:10.1007/s40257 01s 0130-z loss patient: part Il. Trichoscopic and laboratory evaluations. J Am Acad
Atzmony L, Reiter O, Hodak E, et a1. Treatments for cutaneous lichen planus: Dermatol. 2014;77:431.e1-431.e11. [PMID: 25128119] doi:10.1016/j.jaad.
a systematic review and meta analysis. Am J Clin Dermatol. 2O76;L7:77 22. 2014.05.008
IPMID: 26507510] doi:10.1007/s40257 015 0160 6 Pedrosa AF, Lisboa C, Gonqalves Rodrigues A. Malassezia infections: a medical
Bibbins Domingo K, Grossman DC, Curry Sl, et al; US Preventive Services conundrum. J Am Acad Dermatol. 2014;77:170-6. [PMID: 24569116]
Task Force. Screening for skin cancer: US Preventive Services Task Force doi: 1o. 1016/j. jaad .2013.12.022
recommendation statement. IAMA. 2016;316:429-35. [PMID: 274589481 Rodrigues M, Ezzedine K, Hamzavi l, et al; Vitiligo Working Group. Current
doi:10.1001/jama.2016.8465 and emerging treatments for vitiligo. J Am Acad Dermatol. 2Ol7;77:17-29.
Bissell DM, Anderson KE, Bonkovsky HL. Porphyria. N Engl j Med. IPMID: 286195571 doi:10.1o16/j.jaad.2016.11.010
:862 72. IPMID: 28854095] doi:10.1056/NEJMra1608634
2Ol7 ;377 Shevchenko A, Valdes Rodriguez R, Yosipovitch G. Causes, pathophysiolory,
Borda U, Wikramanayake TC. Seborrheic dermatitis and dandruff: a compre and treatment of pruritus in the mature patient. Clin Dermatol.
hensive review. J Clin Investig Dermatol. 2015;3. IPMID: 271485601 2018;36:140-51. [PMID; 29566918] doi:10.1016/j.clindermatol.2017.10.005
doi:10.13i88/2373 1044.1000019 Siegel JA, Korgavkar K, Weinstock MA. Current perspective on actinic kerato
Chen X, Anstey AV Bugert lJ. Molluscum contagiosum virus infection. Lancet sis: a review. Br J Dermatol. 2ol7;777:35o 8. IPMID: 27500794] doi:10.1111/
lnfect Dis. 201313:877 88. [PMID: 23972567] doi:10.1016/51473 bid.148s2
3099(13)70109-9 Spindler V Waschke J. Pemphigus a disease of desmosome dysfunction
Coates SJ, Thomas C, Chosidow O, et al. Ectoparasites: pediculosis and tungia caused by multiple mechanisms. Front Immunol. 2018;9:136. [PMID:
29 449846) doi:10.3389 /fi mmu. 2018. 0013
sis. J Am Acad Dermatol. 2020;82:551-69. IPMIDT 313067291 doi:10.1016/j.
jaad.2019.05.110 Swetter SM, Tsao H, Bichakjian CK, et al. Guidelines ofcare for the manage-
t Duong TA, Valeyrie Allanore L, Wolkenstein B et al. Severe cutaneous adverse ment of primary cutaneous melanoma. J Am Acad Dermatol. 2019;80:208
reactions to drugs. Lancet. 2017;390:1996 2017. IPMID: 284762871 50. [PMID: 303927ss] doi:10.1016/j.jaad.2018.08.055
l
doi:10.1016/S0140 6736(t6)3O378-6 Thomas C, Coates SJ, Engelman D, et al. Ectoparasites: scabies. J Am Acad
Dermatol. 2020;82:533-48. [PMID: 3131O840] doi:10.1016/i.jaad.2019.05.109
Eichenheld LF, Ahluwalia l, Waldman A, et al. Current guidelines for the
\ evaluation and management ofatopic dermatitis: a comparison ofthe Joint Zaenglein AL. Acne vulgaris. N Engl J Med. 2o18;379:1343 52. [PMID:
; Task Force Practice Parameter and American Academy of Dermatolosi 302819821 doi:10.1056/NElMcp77 02493
t
133
I
1
I
t
I
i
I
!t
I
I
I
\
I
t
I
tI
I
I
I
I
'l
I
I
1
I
I
l
I
'l
I
I
I
I
I
I
'l
,l
1
t
I
\
I
I
1
I
'1
']
I
I
I
I
1
I
I
I
t
I
'l
l
1
v!
General lnternal Medicin e 2 o,
The American College of Physicians designates MKSAP 19 General Internal Medicine 2 for a maximum of 31.5 AMA
PRA CategorA 7 Credits'M. Physicians should claim only the credit commensurate with the extent of their participa-
tion in the activity.
Successful completion of the CME activity, which includes participation in the evaluation component, enables
the participant to earn up to 31.5 medical knowledge MOC points in the American Board of Internal Medicine,s
Maintenance of Certiflcation (MOC) program. It is the CME activity provider's responsibility to submit participant
completion information to ACCME for the purpose of granting MOC credit.
MKSAP 19 Subscribers can enter their self-assessment question answers and submit for CME/MOC in two ways:
1. Users of MKSAP 19 Complete who prefer to use their print books and a paper answer sheet to study and
record their answers can use the printed answer sheet at the back of this book to record their answers. The
corresponding online answer sheets, which are available on the MKSAP 19 Resource Page, may be used
to transcribe answers onto the online answer sheets. Users may then submit their answers to qualify for
CME credits or MOC points (see below for information on Opting in for MOC). Users who prefer to record
their answers on a paper answer sheet should save their answer sheet for future use. Users who study
with MKSAP 19 print can also submit their answers directly within MKSAP 19 Digital by accessing the
self-assessment questions dashboard and selecting the preferred subspecialty section to begin answering
questions.
2. Users of MKSAP 19 Digital can enter their answers within the digital program by accessing the self-
assessment questions dashboard and selecting the preferred subspecialty section to begin answering ques-
tions and clicking the Submit CME button once they qualify for CME and are ready to submit. Learners
should keep in mind their yearly CME and MOC deadlines when determining the appropriate time to submit.
Learners' CME/MOC submission progress will be shown on the MKSAP 19 Digital CME/MOC/CPD page.
135
I
I
1
II
I
I
'l
I
1
l
1
I
I.I
I
1
I
I
1
I
'1
t
I
l
1
I
.I
I
'l
I
1
i
I'l
I
I
I.I
l
L
t
t
Directions
t
I Each of the numbered I'terns is followed by lettered answers. Se/ect th e ONE lettered answer that is BESI in each case. vt
L o,
I
L o
F
t Item 1 Behavioral counseling for smoking cessation and phar- vt
l,I
I
A S8-year-old woman is evaluated for worsening vaginal macologic treatment are offered. the patient agrees to see a o
vt
I
nutritionist for weight loss counseling. ut
L
d4mess and itching over the past year. She now avoids
I
i sexual intercourse, which has become painful. She has no
t
vaginal bleeding or discharge or urinary symptoms. Her last Which of the following is the most appropriate additional ttc,
I
period was 6 years ago. Medical history is otherwise unre- treatment?
t
: markable, and she takes no medications. (A)
: Acitretin
I Genital examination is notable for dry and pale vaginal
mucosa with absent rugae. The cervix is normal appearing.
(B) Adalimumab
I Bimanual examination is normal. (C) Methotrexate
Vaginal pH is 5.5. Wet mount is normal. There are no (D) Narrow-band ultraviolet B phototherapy
hyphae on potassium hydroxide preparation.
Item 4
A 62 year old woman is seen for a preoperative evaluation
tr
before total knee arthroplasty. Her only medical problem is
rheumatoid arthritis treated with etanercept; methotrexate;
and prednisone, 6 mg/d, for the past 12 months. During a
hip replacement 8 months ago, she experienced signs and
symptoms of adrenal insulficiency.
On physical examination, vital signs are normal. BMI
is 19. Other than changes associated with rheumatoid
arthritis, the physical examination is normal.
137
Self-Assessment Test
t/r I
.D
Item 5 On physical examination, the average of three blood :
pressure measurements is 129/78 mm Hg; other vital signs
vr A 50-year-old man is evaluated for a lesion on the right
ul are normal. BMI is 30. The remainder of the physical exam- I
(D
t^ side of his face that has changed in color over the past
ination is normal.
UI 4 months and grown in size, now measuring 7 mm in I
(D diameter. Medical history is unremarkable, and he takes
no medications. Which of the following is the most appropriate next step in
Skin flndings are shown. management?
.D
t,,t
(A) Initiateantihypertensivemedication
(B) Obtainechocardiography
(C) Repeat blood pressure measurement annually
(D) Repeat blood pressure measurement in 3 to 5 years
Item 8
A 76-year-old man is evaluated for a year-long history of
urinary frequency and hesitancy and a sensation of incom-
plete bladder emptying. He has no other symptoms. He has
hypertension treated with lisinopril.
Digital rectal examination reveals a uniformly enlarged
and nontender prostate with no bogginess or nodules. The
remainder of the examination is normal.
Urinalysis is normal.
Bladder ultrasonography reveals 35 mL of postvoid
residual urine volume.
Item 6
A 64-year-old man is evaluated during a routine well- Item 9
ness visit. He received a tetanus toxoid, reduced diph- A 48-year-old man is evaluated during a follow-up visit for
theria toxoid, and acellular pertussis vaccine 5 years obesity. He has been unsuccessful in losing weight despite
ago; the live attenuated herpes zoster vaccine 4 years several weight loss diets, including structured meal plans,
ago; and the influenza and COVID-l9 vaccines during nutritionist referral, and weight loss medications. He walks
the most recent influenza season. He had an episode of 15 minutes daily but finds exercise difficult because of
herpes zoster 2 years ago. Medical history is otherwise joint pain. Medical history is significant for type 2 diabetes
unremarkable. He takes no medications and does not mellitus, hypertension, obstructive sleep apnea, and osteo-
smoke cigarettes. arthritis. Medications are metformin, empagliflozin, lira-
glutide, lisinopril, hydrochlorothiazide, topical diclofenac,
Which of the following is the most appropriate vaccine to and acetaminophen. He uses continuous positive airway
administer to this patient? pressure ventilation at night.
On physical examination, blood pressure is 148/80 mm
(A) Quadrivalentmeningococcalconjugate vaccine
Hg, and pulse rate is 95/min; other vital signs are normal.
(B) Recombinant (inactivated) herpes zoster vaccine BMI is 40. Crepitus in his knees bilaterally is noted, along
(C) 23-Valentpneumococcalpolysaccharidevaccine with tenderness of the medial joint line. The remainder of
(D) No vaccines are indicated the examination is normal.
138
Self-Assessment Test
UI
t-6,
tr Item 10
A 25 year-old man is evaluated fbr a 2-day history of
no active joint symptoms; her rheumatoid arthritis is
remission.
On physical examination. vital signs are normal.
it.t
(u
a rapidly spreading pruritic rash on his chest, back, 'lhere is chronic synovial hypertrophy of the metacarpo E
t
upper arms and legs. Medical history is signif)cant for UI
acne vuigaris treated with tretinoin cream. Hc started
phalangeal joints. Hand joints are not warm or sr.t,ollen. o
I,l
sulfamethoxazole trimethoprim ftrr his acne 9 days there is medial joint line tenderness and crepitus in right t
knee.
ago.
'Ihe rash on his back is shou,n. o
vt
Which of the following is the most appropriate
perioperative medication management?
(A) Continue methotrexate and adalimumab
(B) Continue methotrexate and hold adalimurnab
(C) Hold methotrexate and adalimumab
(D) Hold methotrexate and continue adalimunrab
Item 12
A 78-year-old man is evaluated for a 3-year history of ery-
thema and flaking on the scalp and face.
On physical examination, ill-deflned greasy, yellow
to erythematous patches with slight scale are noted on the
eyebrows, nose, and medial aspects of the cheeks. Similar
changes on the scalp are shown.
Item 13
tr Item 11
A 62 year old woman is seen for a preoperative evaluation
A 34-year-old man is evaluated for an B month history of
erectile dysfunction and decreased libido. He reports an
beftrre right knee arthroplasty. She has osteoarthritis of absence of early-morning erections. He reports no depres
the knee and also has a history of rheumatoid arthritis. sive or anxiety-related symptoms. Medical history is signif-
Medications are topical diclofenac, me thotrexate, {blic acid, icant for a single episode of depression. His only medication
and adalimumab. Other than the right knee, she reports is sertraline, initiated 9 months ago.
139
Self-Assessment Test
tn
(D
Physical examination flndings, including vital signs,
tn are normal.
t^
(D Laboratory studies show an early morning serum total
(n
l,I testosterone level of 560 ngldl (19.4 nmol/L).
.D
Which of the following is the most appropriate
management?
.D
UI
(A) Sex therapy counseling
(B) Startintramusculartestosterone
(C) Start oral tadalafll
(D) Taper sertraline
Item 14
A 27-year old woman is evaluated for contraceptive coun-
seling. She is nulliparous. Medical history is signiflcant for
migraine with aura. She is sexually active and has a history
of multiple male partners. She is a nonsmoker. She takes no Conjunctivae are red and injected. There are erosions and
medications. flaccid bullae in the oropharynx. The rash involves 20olo of her
body surface area. Carbamazepine has been discontinued.
Which ofthe following is the most appropriate contractptive
option for this patient? Which of the following is the most likely diagnosis?
(A) Combined hormonal oral contraceptive pills (A) Acute generalized er1rthematous pustulosis
(B) Combined hormonal vaginal ring (B) Drug-inducedhypersensitivityreaction
(C) Levonorgestrel-releasing intrauterine device (C) Erythroderma
(D) Male condoms (D) Stevens-Johnson/toxic epidermal necrolysis overlap
syndrome
Item 15
A 4S-year-old man is evaluated during a follow-up visit Item 17
for type 2 diabetes mellitus. Over the past 3 months, his A 36-year-old man is evaluated for a 2-day history of right-
hemoglobin A," has remained at 8.3%. Medications are eye redness with a yellow discharge. When he awoke this
metformin and liraglutide. The patient is not following a morning, his right eye was matted with a small amount of
speciflc diet and does not exercise. He states he is not fully yellow crust. He reports no pain or change in visual acuity.
satisfled with his Iifestyle but is ambivalent about change. He does not wear contact lenses. He has no other symptoms
or medical problems, and he takes no medications.
Which of the following is the most appropriate initial On physical examination, the right eye shows diffuse,
management step? circumferential conjunctival erythema with yellow crust-
ing on the eyelashes. Visual acuity is 20/20 bilaterally. The
(A) Add pioglitazone remainder of the examination is normal.
(B) Motivational interviewing After a discussion of the risks and benefits of treatment,
(C) Refer to a dietitian the patient prefers pharmacologic treatment.
(D) Refer to a supervised exercise program
Which of the following is the most appropriate topical
treatment?
tr Item 16
A 32 year-old woman is evaluated in the emergency depart-
(A) Ofloxacin
(B) Olopatadine
ment for a rapidly spreading rash involving her trunk, (C) Prednisolone
extremities, and face that started 48 hours ago. She has
intense skin pain, ulcerations in her mouth and vagina, (D) Trimethoprim-polymyxin
gritty sensation in her eyes, and fever. She began taking
carbamazepine 12 days ago for trigeminal neuralgia. Her
medical history is otherwise unremarkable, and she takes
no other medications.
Item 18
A 24-year-old person is evaluated as an add on patient for
tr
On physical examination, the patient appears ill and a right wrist growth. He describes a small soft mass on the
in pain. Temperature is 40 "C (104 'F), blood pressure is ventral aspect of the wrist that he flrst noticed 6 months
110/65 mm Hg, pulse rate is 110/min, and respiration rate is ago. It has not enlarged, and it does not cause any symptoms
l8/min. BMI is 33. Skin flndings are shown (see top of next or limitations. 'lhe patient is new to the practice and has
column). indicated on the intake form that his gender is transgender
140
Self-Assessment Test
UI
o,
tr
CONI
male. He responds to questioning that he would prefer to be
addressed as'he" or'him."
On physical examination, vital signs are normal. A soft,
during the most recent influenza season. She received two
doses ofthe recombinant herpes zoster vaccine 3 years ago.
Medical history is significant for stage 4 chronic kidney dis-
(l,
tr
nontender 0.5-cm mass is present on the ventral aspect of ease and type 2 diabetes mellitus.
r/t
the wrist, consistent with a ganglion cyst. The wrist has full .u
vt
range of motion, and there is no erythema or induration. Which of the following is the most appropriate vaccine to vt
adminisftr to this patient?
Which of the following is the most appropriate next step in (l,
(A) Tetanus and diphtheria toxoids booster r/t
patient management?
(B) 13-Valent pneumococcal conjugate vaccine
(A) Obtain a hormonal and surgical history (C) 23-Valent pneumococcal polysaccharide vaccine
(B) Obtain a social and sexual history (D) No vaccines are indicated
(C) Perform a genital examination
(D) Screen for sexually transmitted infections
(E) Expectantmanagement
Item 21
An B1-year old man is seen for a preoperative evaluation tr
before total right hip arthroplasty. Medical history is signif-
Item 19 icant for mild cognitive impairment, COPD, chronic kidney
A 7S-year-old woman is evaluated for a rapidly growing, disease, diabetes mellitus, and hypertension. Medications
painful nodule on the right dorsal hand that appeared are topical NSAIDs, albuterol and tiotropium inhalers, met
6 weeks ago. Medical history is unremarkable, and she takes formin, and lisinopril.
no medications. On physical examination, vital signs are normal. Oxy-
gen saturation is 95'2, breathing ambient air. BMI is 24. The
Skin flndings are shown.
patient is alert and oriented. Decreased right hip range
of motion, especially internal rotation, is present. Cardiac
examination is normal. Pulmonary examination reveals
distant breath sounds without wheezing.
Item 22
A 4S-year-old man is evaluated during a routine visit. He
reports no symptoms, and his medical and family history
are unremarkable. He does not smoke cigarettes, and he
engages in moderate exercise for 150 minutes per week and
consumes a Mediterranean diet. He takes no medications.
On physical examination, blood pressure is 120/78 mm
Hg; other vital signs are normal. BMI is 25. The remainder of
the physical examination is normal.
Which of the following is the most likely diagnosis?
(A) Which of the following is the most appropriate
Actinic keratosis
cardiovascular disease assessment?
(B) Basal cell carcinoma
(C) Keratoacanthoma
(A) Calculating cardiovascular disease risk
(D) (B) Carotidarteryultrasonography
Bodermagangrenosum
(E) Sporotrichosis
(C) ECG
(D) Exercise stress test
(E) No cardiovascular disease assessment
Item 20
A 65-year-old woman is evaluated during a routine health
examination. She received the 23-valent pneumococcal
polysaccharide vaccine 4 years ago; the tetanus toxoid,
Item 23
A 65-year-old man is seen for a preoperative medical
tr
reduced diphtheria toxoid, and acellular pertussis vaccine evaluation 2 weeks before total left shoulder arthroplasty.
2 years ago; and the influenza and COVID 19 vaccines Medical history is signiflcant for COPD. He was hospitalized
141
Self-Assessment Test
atl
o
lrll 6 months ago for COPD exacerbation. He reports no changes Oral mucosa is normal.
ta E in cough or sputum production. He has no shortness of l ''
UI cONI 5r.r15 or t'ever. Screening for obstructive sleep apnea (OSA)
(D Which of the following is the most likely diagnosis? I
UI
Ut with the STOP-BANG questionnaire categorizes the patient :
as low risk lor OSA. He has a 30 pack-year history of ciga (A) Drug induced hypersensitivity syndrome
.D
rette smoking but quit smoking 6 years ago. Medications are (B) Erythroderma -1
{
.D
inhaled tiotropium. beclomethasone, and albuterol. (C) Lichen planus
(a On physical examination, vital signs are normal. Oxy- (D) Pemphigus vulgaris :
gen saturation is 94'X, breathing ambient air. There are
(E) Stevens Johnson syndrome ;
decreased breath sounds throughout and an increased expi
1
ratory phase. Left shoulder range of motion is painful and ;
restricted. The remainder of the examination is normal.
Item 25 I
Which of the following is the most appropriate
A 23-year old woman is evaluated for a 1 week history of
j
perioperative pulmonary assessment?
vaginal discharge and odor. The patient is sexually active I
with men, and her last sexual encounter was 2 weeks ago.
(A) Arterial blood gas analysis She takes an oral contraceptive pill for contraception. Her last
l
(B) Chest radiography menstrual period was 3 weeks ago. Medical history is other
(c) wise unremarkable. and she takes no other medications.
l
Spirometry
(D) No further testing
The external genitalia examination is normal. Pelvic li
examination shows a thin, gray homogenous discharge
with a mild odor. The cervix is normal appearing, without
discharge. Bimanual examination is unremarkable.
tr Item 24
A 62 year-old man is evaluated in the emergency depart-
Vaginal pH is 5.0. Wet mount shows 307, clue cells. There
are no hyphae on a potassium hydroxide preparation. Whiff
ment for a rash that developed over his entire body test result is positive. A pregnancy test result is negative.
during the past week. He w'as discharged from the hos-
pital 15 days ago lor a COPD exacerbation. He received Which ofthe following is the most appropriate management?
prednisone for 5 days and levofloxacin for 7 days. Medical
history is signiflcant for chronic plaque psoriasis that he (A) Nucleic acid ampliflcation test for bacterial vaginosis
does not treat. Medications are tiotropium and salmeterol (B) Oral azithromycin
fluticasone. (C) Oral fluconazole
On physical examination, the patient appears ill and (D) Oral metronidazole
is bundled in blankets and shivering. Temperature is 37'C
(98.6 'F)l other vital signs are normal. The rash covers 90'7,
ofbody surface area. Representative skin and nail flndings Item 26
are shown.
A 4S-year-old woman is evaluated for 4 month history
**&.'
of persistent rhinorrhea, frontal headache, loss of smell,
and malaise. She has self-treated with saline irrigation and
antihistamine/decongestants.
On physical examination, vital signs are normal. Nasal
turbinates are swollen, and thick yellow nasal discharge is
noted. Maxillae and forehead are tender to palpation. the
remainder of the examination is normal.
llem 27
A 42 year old man is evaluated for premature ejaculation.
He reports that during sexual intercourse, he ejaculates
within 30 seconds after vaginal penetration, which causes
him signiflcant psychological distress. He has no decrease in
libido or difficulty maintaining an erection. Medical history
is positive for depressive symptoms predating the onset of
premature ejaculation. He takes no medications.
142
Self-Assessment Test
vt
Which of the following is the most appropriate treatment? history is signiflcant for hypertension, degenerative joint Fc,
disease, and hypothyroidism. There has been no recent q,
(A) Clomipramine change in her clinical status or medications. Review of sys-
(B) Paroxetine tems is negative. Medications are levothyroxine, chlorthali- Ut
UI
(C) Squeeze technique done, and lisinopril. (u
vt
(D) On physical examination, vital signs are normal. BMI is ut
Start-stoptechnique
(E) 22.There is restricted and painful range of right hip motion.
Topical anesthetic (l,
The remainder of the physical examination is normal.
t/t
Two months ago, thyroid-stimulating hormone mea
Item 28 surement and serum electrolytes were normal. No other
Iaboratory flndings are available.
A S4-year-old asymptomatic woman is evaluated during a
wellness visit. She has a 35 pack-year smoking history but
quit smoking 12 years ago. She has no medical problems and Which of the following is the most appropriate
preoperative laboratory testing for this patient?
takes no medications.
Physical examination, including vital signs, is unre (A) Coagulation studies and platelet count
markable. (B) Hemoglobin and serum creatinine
(C) Thyroid-stimulating hormone and free thyroxine
Which of the following is the most appropriate lung cancer
(D) Urinalysis
screening stratery for this patient?
(A) Annualchestradiography
(B) Annual low-dose CT of the chest Item 31
(C) One-time low-dose CT of the chest A 39-year-old woman comes to the offlce to discuss bariatric
(D) No screening is indicated surgery. She has been unable to lose weight with diet and
exercise, including supervised weight loss programs. She has
tried naltrexone bupropion and phentermine-topiramate
Item 29 with modest success, but the weight returned after med-
A 32 year old man is evaluated for a 2 year history of nail ication discontinuation. Medical history is signiflcant for
changes. Medical history is otherwise unremarkable. Fam- type 2 diabetes mellitus. Medications are metformin and
ily history is signiflcant for plaque psoriasis in his brother. liraglutide.
He takes no medications. On physical examination, vital signs are normal. BMI is
Nine of 10 flngernails are dystrophic. Nail flndings are 4.5. Theremainder of the examination is normal.
shown.
Which of the following is the most appropriate treatment
for this patient?
(A) Gastric banding
(B) Intragastricballoon
(C) Orlistat
(D) Sleeve gastrectomy
Item 32
A 76-year old woman is seen for a preoperative medical tr
evaluation before elective total ankle arthroplasty. She has
heart failure with reduced ejection fraction and COPD. She
has no chest pain, presyncope, or change in weight. Medi
Toenails are not involved. The remainder of the skin
cations are tiotropium, budesonide, and albuterol inhalers;
and hair examination is normal.
furosemide; metoprolol; lisinopril; and 2 L/min of oxygen
via nasal cannula during exertion.
Which of the following is the most likely diagnosis? On physical examination, blood pressure is 145/80 mm
(A) Chronic paronychia Hg, pulse rate is 109/min, respiration rate is 24lmin, and
(B) Lichen planus oxygen saturation is 90% breathing ambient air. Jugular
venous pressure is elevated. There is an Sa and agrade 216
(C) Nail psoriasis
systolic murmur at the lower left sternal border. Breath
(D) Onychomycosis sounds are distant without crackles.
A recent transthoracic echocardiogram showed an
ejection fraction of 4Ook, moderate to severe right ventricle
tr Item 3O
A 69-year old woman is seen for a preoperative medical
dysfunction, pulmonary artery systolic pressure of 65 mm
Hg, and tricuspid regurgitation. ECG shows right bundle
evaluation before total right hip arthroplasty. Medical branch block.
143
Self-Assessment Test
vt
.D
UI
ut
tr
CONT,
Which of the following is the most appropriate
preoperative management of this patient?
the buccal mucous and the hard and soft palates. There is n<r
conjunctir,al. urethral, or perineal involr.ement.
.D
l^ (A) Cancel surgery
gt Which of the following is the most likely diagnosis?
(B) lncreasefurosemicle
.D (C) lnitiatc tadalafil (A) Bullous pemphigoid
{ (D) Start continuous oxygen therapy (lJ) Dermatilis herpclilormis
.D (C) Pemphigus vulgaris
UI
(D) Stevens Johnson sl,ndrome
Item 33 (E) Toric epidern.ral necrolysis
A 33 year old woman is evaluated during a preconception
counseling visit. She is nulliparous. Her last menstrual
period was 2 weeks ago. She is asymptomatic, without Item 35
medical problems. She received her measles, mumps, and A S2-year-old man is evaluated for decreased hearing and a
rubella vaccine during childhood and an influenza vaccine 4 year history of tinnitus without vertigo. The patient does
during the most recent influenza season. She received the not notice hearing loss but has come for evaluation based
tetanus toxoid, reduced diphtheria toxoid, and acellular on his wife's recommendation. He has no history of noise
pertussis (Tdap) vaccine 6 years ago. She is unsure whether
exposures. He takes no medications.
she had chickenpox. She is sexually active with her hus On physical examination, the whispered voice test
band only. Her last Pap smear was obtained 1 year ago and suggests left hearing loss. Otoscopy reveals no cerumen
was normall human papillomavirus testing was negative. impaction or other blockage ofthe external auditory canal.
She does not smoke cigarettes or drink alcohol. Her only Tympanic membranes appear normal. Placement of a
medication is a prenatal vitamin. 256 Hz vibrating tuning fork midline on the forehead results
A pregnancy test result is negative. in a louder sound in the right ear. For each ear, the tuning
lork sound is louder when placed in front of the external
Which of the following is the most appropriate ear canal compared with placement on the mastoid process.
management?
(A) Administer the Tdap vaccine Which of the following is the most likely cause of hearing
(B) loss in this patient?
Obtain nucleic acid ampliflcation test for chlamydia
(C) Obtain Pap smear (A) Conductive hearing loss, left ear
(D) Obtain varicella antibody titer (B) Conductive hearing loss, right ear
(C) Sensorineural hearing loss, left ear
(D) Sensorineural hearing loss, right ear
tr Item 34
A 51 year old man is evah.rated in the emergencJ' depart
ment tbr a 2 clay history of sloughing skin in his mouth and
on his facc, ncck. trunk. :u.td lrnrs. His eyes are una{fected,
and he reports no dysuria. l{e has no other symptoms.
Item 36
A 57-year-old man is evaluated fbr a 2 day history ol mild
tr
to moderate burning left flank pain rn,ith an associated rash
Medical history is significant ltrr rosacea treated lt,ith doxy that appeared shortly after pain tinset.
cycline for the past 6 n.ronths. Skin findings are shorvn.
On physical examination. the patient is not toxic
appearing. Vital signs are norrnal. The rash is shou'n.
;E--'
tIlF
144
Self-Assessment Test
vt
6'
tr
CONT.
Which of the following is the most appropriate
treatment? E
(l,
tr
(A) Topical acyclovir tt
,tl
(B) Valacyclovir €,
vt
(C) Valacyclovir and gabapentin t
(D) Valacyclovir and prednisone
€,
vt
tr Item 37
A 66-year old woman is evaluated in the emergency
department for severe aching pain and visual blurring
in the right eye that began 2 hours ago. She describes
seeing halos around lights, accompanied by a throb-
bing, periocular, right sided headache and nausea. She
reports no other symptoms. Medical history is significant
for depression and hyperopia. iler only medication is
paroxetine.
On physical examination. blood pressure is
160194 mm IIg and pulse rate is 102/min. The right pupil Which of the following is the most likely diagnosis?
is 4 mm in diameter and responds minimally to direct (A) Hemangioma
light. There is diff'use conjunctival injection, and the cornea
appears cloudy. Visual acuity is 20/60. The globe is tense
(B) Keratoacanthoma
and tender to palpation. There is no erythema or edema of (C) Nodular melanoma
eyelids or surrouncling eye structures. Funduscopic exam (D) Pigmented basal cell carcinoma
ination shows cupping of the optic disc. Examination of the
left eye is unremarkable; visual acuity is 20/30.
Item 40
Which of the following is the most likely diagnosis? A 2S-year-old woman is seen during a routine visit. She is
(A) Acute angle-closure glaucoma
in the third trimester of a normal pregnancy. She is up to
date on all of her immunizations and received the tetanus
(B) Bacterial keratitis
toxoid, reduced diphtheria toxoid, and acellular pertussis-
(C) Orbital cellulitis vaccine during her flrst pregnancy 24 months ago and the
(D) Scleritis inactivated influenza vaccine just before the start of the
current pregnancy. She has no medical problems. She takes
a multivitamin and folic acid but no other medications or
Item 38 supplements.
145
Self-Assessment Test
vr
(D
ffi
g
murmur at the right upper sternal border that radiates to initiated ledipasvir-sofosbuvir. Her medical history is
UI the right carotid artery. The lower extremities are cool, with otherwise unremarkable, and she takes no other medi
U)
(D
coNl no palpable pulses. cations.
t/t Transthoracic echocardiography findings are compat
t^ Skin flndings on the low back are shown.
ible with severe calcific aortic stenosis with left ventricular
.D
ejection fraction of 45oL, meeting the criteria for val'",ular
intervention.
o
UI
Which of the following is the most appropriate
recommendation regarding surgery?
(A) Aortic valve replacement prior to aortofemoral
bypass
(B) Aortic valve replacement plus aortofemoral bypass
(C) Percutaneous balloon valvuloplasty prior to aorto-
femoral bypass
(D) Percutaneous balloon valvuloplasty plus aortofemoral
bypass
Item 42
A 20 year old woman is evaluated for a 2-day history of sore
throat, nonproductive cough, nasal congestion, headache, The volar wrists have lesions similar to those on the
and malaise. She has no other medical problems and does back. There is no lymphadenopathy.
not smoke.
On physical examination, vital signs are norrnal. The Which of the following is the most likely diagnosis?
nasal mucosa is boggy and inflamed, with clear discharge.
The pharynx is erythematous. Tonsils are normal appear- (A) Lichen planus
ing. Tympanic membranes are clear. The remainder of the (B) Pityriasis rosea
examination is unremarkable. (C) Psoriasis
A SARS-CoV-2 rapid antigen test result is negative. (D) Secondary syphilis
(A) Hepatitis C antibody assay Which of the following is the most appropriate
management?
(B) Nucleic acid ampliflcation test for chlamydia and gon-
orrhea (A) Obtain a follicle stimulating hormone level
(C) Syphilis enzyme immunoassay test (B) Obtain a Pap smear
(D) No testing is indicated (C) Obtain an endometrial biopsy
(D) Start oral contraceptive pills
Item 44
A S8-year-old woman is evaluated for a 5-week history of Item 46
an itchy rash on the low back. She was recently diagnosed A 56-year old man is evaluated during a follow-up visit
with hepatitis C virus infection, for which she has just for diabetes mellitus, hypertension, and hyperlipidemia.
145
Self-Assessment Test
(a
(u
His father had a myocardial infarction before the age of Skin findings on the volar aspect of the wrist are
55 years, as did his younger brother. Has a 3S-pack-year shown. E
(u
smoking history but quit smoking 3 months ago. He is
sedentary. He has no other medical problems, including t
tt
no history of gastrointestinal bleeding, upper gastroin- (l,
(
testinal pain, uncontrolled hypertension, chronic kidney tl
disease, NSAID use, or thrombocytopenia. Medications are
varenicline, metformin, lisinopril, hydrochlorothiazide, =
(
(u
and atorvastatin.
On physical examination, vital signs are normal.
BMI is 28. The remainder of the physical examination is
normal.
His 10-year atherosclerotic cardiovascular disease risk
is 25% using the Pooled Cohort Equations.'
The patient receives counseling on therapeutic life-
style changes, including diet, exercise, and weight con-
trol and encouragement regarding continued smoking
cessation.
Item 49
Item 47 A 28-year-old woman is evaluated for a painless breast
A 46-year-old woman is seen for a routine evaluation lump she noticed 1 week ago. Medical history is otherwise
after Roux-en-Y gastric bypass surgery 2 months ago. unremarkable. Her grandmother was diagnosed with breast
She is doing well and has lost 13.6 kC (30 lb). She recently cancer at age72 years. Her only medication is a combined
initiated an exercise program. She has type 2 diabetes hormonal contraceptive pill.
mellitus and hypertension. Her current medications are Breast examination reveals a 1 cm, nontender, smooth,
metformin, low dose insulin glargine, chlorthalidone, mobile, round mass on the medial aspect of the right breast.
lisinopril, atorvastatin, vitamin Brr, vitamin D, thiamine, The overlying skin is normal. There is no lymphadenopathy.
calcium, and a multivitamin with iron and folate. The left breast is normal.
On physical examination, blood pressure is 132/
78 mm Hg; the remainder of the physical examination is Which of the following is the most appropriate
normal. management?
Home glucose monitoring data for the previous (A) Breastultrasonography
7 days show average fasting blood glucose of 108 mg/dl
(6.0 mmol/L). She has experienced no episodes of hypo-
(B) Mammography
glycemia. (C) Repeat clinical breast examination in 6 weeks
(D) No further evaluation is indicated
Which ofthe following medications should be discontinued
for this patient?
Item 50
(A) Atorvastatin
A 27-year old woman is evaluated for a 2-week history of
(B) Insulin glargine hives that burn and sting. Individual wheals resolve with
(C) Lisinopril darkening of the skin in 1 to 2 days. She also has associated
(D) Metformin fever and joint pain. Medical history is otherwise unremark-
(E) Vitamin D and calcium able. Other than acetaminophen for fever and joint pain, she
takes no medications.
On physical examination, temperature is 38.3 "C
(101 'F); other vital signs are normal. There are 2- to 3-cm
Item 48 wheals with pale red flares on the arms, Iegs, upper chest,
A 44-year-old woman is evaluated for a 4-week history of and back. There is faint hyperpigmentation in areas where
worsening rash. She used clobetasol cream for 1week, but wheals have resolved. The remainder of the examination
the rash worsened. is normal.
147
Self-Assessment Test
yr
.D
Which of the following is the most likely diagnoeis? accommodation. Visual acuity is 20160 in the left eye and
vl 2Ol20 in the right eye. Nondilated funduscopic examination
vt (A) Chronic idiopathic urticaria
(D is normal.
vt (B) Hereditaryangioedema
lrr
(C) Urticarial drug reaction Which of the following is the most likely diagnosis?
(D
(D) Urticarialvasculitis
(A) Age-related macular degeneration
CD
vr
(B) Central retinal artery occlusion
Item 51 (C) Central retinal vein occlusion
A 56-year-old man is evaluated for erectile dysfunction that (D) Retinal detachment
began insidiously l year ago. He reports that his libido and
mood are good. He is in a monogamous relationship with
his wife. He runs on a treadmill for 30 to 45 minutes three to Item 54
four times weekly, and he has no cardiovascular symptoms. A 36-year-old wbman is evaluated for a 3-month history
The patient's medical history is also signiflcant for coronary of white patches around the eyes, legs, and feet. Medical
artery bypass graft surgery 4 years ago, hyperlipidemia, and history is otherwise unremarkable, and she takes no
hypertension. Medications are metoprolol, losartan, aspirin, medications.
and atorvastatin. On physical examination, vital signs are normal. Skin
On physical examination, vital signs are normal. BMI findings are shown.
is 24. The remainder of the examination is unremarkable.
Laboratory studies show an early moming serum total
testosterone level of 380 ng/dl (13.18 nmol/L).
Item 52
A 65-year-old man is evaluated during a follow-up visit
for hypertension. He reports that he eats fruits, nuts, and
vegetables daily and red meat once weekly. He drinks artifi-
cially sweetened soda daily and one alcoholic beverage most
evenings with dinner. He does not smoke cigarettes. He
lives alone and does not maintain social relationships and
describes himself as lonely. Depression screening is negative.
tr Item 53
A 68-year-old woman is evaluated for reduced vision and
flashing lights in her left eye that began this morning. This
was preceded by floaters in her left eye, flrst appearing
several days ago and now with increasing frequency. She Which of the fo[o\Ming tests should be obtained?
has no other symptoms. Medical history is signiflcant for (A) Antinuclearantibody
myopia. She takes no medications.
On physical examination, she has an inferior visual (B) Hepatitis C antibody
fleld defect in her left eye. Visual fields in the right eye are (C) HIV testing
normal. Pupils are equally round and reactive to light and (D) Thyroid-stimulatinghormone
148
Self-Assessment Test
t^
(C) Pregnancytest o,
Item 55
A 60-year-old woman is evaluated for bone health during a (D) Transvaginal ultrasonography
o,
routine wellness visit. Her mother was treated for a hip frac- F
t!
ture after a fall at the age of65 years. the patient is sedentary tt
and has a 3O-pack-year history of cigarette smoking but Item 58 o
v!
tt
quit smoking 2 months ago. She has no medical problems A 26-year-old woman is evaluated for a L-week history of a
and takes no medications. pruritic rash on the chin. She has no other medical problems
Physical examination flndings, including vital signs, and takes no medications. =(u
t/t
are normal. Her weight is 56 kg (123 lb); BMI is 21. Skin findings are shown.
tr Item 56
A 76 year-old woman is seen tbr a preoperative medical
evaluation before elective cholecystectomy. Medical his-
tory is signiflcant for nonvalvular atrial flbrillation, hyper-
tension, and diabetes mellitus. Medications are warfarin,
metoprolol. and met formin.
tr Item 57
A 32-year old woman is evaluated for acute onset of right
Two weeks ago, he underwent surgical resection for stage
III colon cancer. He will begin a 3-month course of adjuvant
chemotherapy in 3 weeks. He is doing well postswgically.
lower quadrant abdominal pain that began 1 day ago. She
All other immunizations are up to date. He has no other
reports some vaginal spotting but no discharge, fever, nau-
medical problems and takes no medications.
sea, or vomiting. She is sexually active with her husband
and uses a levonorgestrel releasing intrauterine device for
contraception. Medical history is otherwise unremarkable, Which of the following is the preferred influenza
and she takes no medications. immunization stratery?
On physical examination, blood pressureislO2lT4 mn (A) Defervaccination until completion of chemotherapy
I Ig and pulse rate is 95/min; other vital signs are normal. (B) High-dose vaccine
Abdominal examination reveals tenderness in the right
(C) Nasalsprayvaccine
lower quadrant with no guarding. Pelvic examination
reveals a normal appearing cervix with a small amount (D) Standard-dose inactivated vaccine
ofblood in the vault and tenderness in the right adnexa.
Complete blood count is normal.
149
Self-Assessment Test
t/t
o
lll and atorvastatin. The patient also takes p-carotene, fish oil, significant for atrial flbrillation. hypertension, and peripheral
a,t
E vitamin E, and large doses of biotin. The patient expresses no vascular disease. Medications are rivaroxaban, metoprolol,
ur coNT 6.ri.. to quit smoking at this time.
o
ur
losartan, and atorvastatin.
tr On physical examination, vital signs are normal. The
Which ofthe following is the most appropriate physical examination documents osteoarthritis of the kree
(D
recommendation regarding this patient's supplements? and an irregular heart rhythm.
Laboratory studies show a serum creatirrine level of
o (A) Stop biotin 1.0 mg/dl (es.+ pmol/L) and an estimated glomerular filtra
ut
(B) Stop B carotene tion rate greater than 60 ml/min/1.73 m2.
(C) Stop flsh oil
(D) Stop vitamin E Which ofthe following is the most appropriate perioperative
(E) Stop all supplements management of the patient's rivamxaban?
(A) Continue rivaroxaban until day of surgery
(B) Withhold rivaroxaban 3 days before surgery
Item 61 (C) Withhold rivaroxaban 3 days before surgery; bridge
A 29-year-old woman is evaluated for a 6-month history of with low-molecular weight heparin
acne that worsens during menses. She has been using topi-
(D) Withhold rivaroxaban 7 days before surgery
cal benzoyl peroxide wash and topical tretinoin cream, but
there has been no improvement. She reports no hirsutism or (E) Withhold rivaroxaban 7 days before surgery; bridge
virilization symptoms. Menses are regular. She has no other with low molecular-weight heparin
medical problems and takes no additional medications.
Skin flndings are shown.
Item 53
A 37-year-old man is evaluated during a pre employment
examination. He is starting a new job as a medical assistant
in a primary care offlce. Within the past 2 years, he com-
pleted his hepatitis B vaccination and the measles, mumps,
and rubella vaccine series and received the tetanus and
diphtheria toxoids and COVID-l9 vaccines. He received the
influenza vaccine during the most recent influenza season.
He has no medical conditions and takes no medications.
laboratory studies are positive for the hepatitis B sur
face antibody and rubella and varicella IgG antibody.
Item 64
No features ofhyperandrogenism are noted. A 32-year old woman telephones the office to request emer-
gency contraception after an episode ofunprotected sexual
Which of the following is the most appropriate intercourse 4 days ago. She reports no syrnptoms. Medical
management? history is unremarkable, and she takes no medications.
She isout of town and is unable to come into the office for
(A) Hormonemeasurements several days.
(B) Pelvicultrasonography Her last recorded BMI is 28, and she reports no change
(C) Progesterone eluting intrauterine device in weight since her last visit 6 months ago.
(D) Spironolactone
(E) Topical metronidazole cream Which of the following is the most appropriate emergency
contraceptive option for this patient?
(A) Combined hormonal oral contraceptive pill
tr Item 62
A 66-year-oldwoman is seen for a preoperative medical eval
(B)
(C)
Levonorgestrel oral contraceptive pill
Subdermalcontraceptiveimplant
uation before total right knee arthroplasty. Medical history is (D) Ulipristal acetate oral contraceptive pill
150
Self-Assessment Test
vt
o,
tr Item 55
A 28-year-old woman is evaluated in the emergency depart
cancer. She is willing to undergo colorectal cancer screening
but does not want colonoscopy.
(u
ment 3 hours after sustaining a burn of the Ieft arm while
burning leaves in her backyard. Which of the following is the most appropriate colorectal v!
v!
screening strategl for this patient? q,
On physical examination, vital signs are normal. Skin t!
Ut
flndings on the left arrn are shown. (A) Fecal immunochemical test every 1to 2 years
(B) CTcolonographyeverylOyears o
(C) Serum circulating methylated SEPTg DNA measurement ttt
everylto2years
(D) Sigmoidoscopy every 2 years
Item 68
A 48-year-old woman is evaluated during a routine exam-
ination. She reports that her last menstrual period was
more than 12 months ago. She states she has occasional hot
flushes. She has no genitourinary symptoms. Medical his-
tory is unremarkable, and she takes no medications.
Physical examination flndings, including vital signs,
are normal.
A pregnancy test result is negative.
151
Self-Assessment Test
vt
(D
Item 70 (C) Delal.surgery for 9 months
t^ A 24-year-old man is evaluated during a routine health (D) Proceed with surgery nolt
U!
(D examination. He received the tetanus toxoid, reduced diph-
t/t
lr! theria toxoid, and acellular pertussis vaccine and one dose
J
(D of the quadrivalent meningococcal conjugate vaccine when Item 73
he was 18 years old, before starting college. He received the A 3S-year-old woman is evaluated for a 3 week history of
o influenza and COVID-l9 vaccines during the most recent pruritic rash on several areas, including the hands. There is
UI influenza season. He has not received the human papillo- no facial involvement.
mavirus vaccine series. He does not smoke cigarettes. Skin flndings are shown.
tr Item 71
An 1B-year old man is evaluated in the emergency depart
ment fbr severe right sided scrotal pain that started sud
denly 4 hours ago and has progressed. He is nauseated but
reports no other symptoms. He has no history of trauma. He
has no other medical problems and takes no medications.
On physical examination. the patient is in distress.
Temperature is 36.9 'C (98.4 "F). blood pressure is 1461
92 mm Hg, pulse rate is 102rmin. and respiration rate is
16/min. The right testis is elevated, and the overlying scrotal
skin is erythematous and warm. The epididymis cannot be
distinguished from the testis owing to severe scrotal srtell
ing and pain. Cremasteric reflex is absent on the right and
intact on the left.
152
Self-Assessment Test
ut
€,
lJl t-aboratorv studies: Which of the following is the most appropriate
bl uemogo6in 11 g/dl (110 g/L) treatment? E
(u
C0Nl p1^1.1.1 90,000/pL (90 x 10e/L)
"orrr, (A) Ketoconazole cream E
INR 1.3 vt
Totalbilirubin 2.0 mg/dl (34.2 pmol/L) (B) Mupirocin ointment ra
o
.,I
Creatinine 1.2 mg/dl (106.1 pmol/L) (C) Prednisone vt
(D) Triamcinolonecream
Which of the following is the most appropriate E
perioperative management? t
(A) Counsel patient on increased surgical risk due to cirrhosis llem 77
(B) Frozen plasma prior to surgery A 65-year-old man is evaluated during a follow-up visit for
(C) Livertransplantation evaluation type 2 diabetes mellitus. He feels well. He reports no chest
pain or shortness of breath. Medical history is also signifl-
(D) No additional testing or intervention
cant for atrial flbrillation, dyslipidemia, and hypertension.
Medications are metformin, Iiraglutide, lisinopril, meto-
prolol, rivaroxaban, and atorvastatin.
Item 75
Physical examination flndings, including vital signs,
A 3S-year-old woman is seen in the office to discuss phar- are normal.
macotherapy for obesity. She has not lost signiflcant weight His 1O-year atherosclerotic cardiovascular disease risk
over the past 6 months despite dietary counseling, commer- is 24% using the Pooled Cohort Equations.
cial weight loss programs, and exercise. Medical history is
significant for nephrolithiasis, insomnia, hypertension, and
Which of the following is the best advice regarding aspirin
irritable bowel syndrome with diarrhea. Medications are therapy for this patient?
hydrochlorothiazide and loperamide.
On physical examination, vital signs are normal. BMI is (A) Aspirin, 81mg/d
39. The remainder of the examination is normal. (B) Aspirin, 81 mg/d, and omeprazole
(C) Aspirin,325 mg/d
Which of the following is the most appropriate (D) No aspirin
management?
(A) Liraglutide
(B)
(C)
Naltrexone-bupropion
Orlistat
Item 78
A 50 year-old man is evaluated during a follow-up visit tr
(D) Phentermine-topiramate
I month after being hospitalized for deep venous throm-
bosis of the right leg. He is no longer taking apixaban
because he received only a 14-day supply after Ieaving
Item 76 the hospital and his symptoms had improved after fln-
ishing the supply. He thought he had flnished a course of
A 56-year-old man is evaluated for worsening erythema therapy.
and pruritus that began 3 days ago. Ten days ago, bacitra- On physical examination, vital signs and the remainder
cin ointment was used following excision of a dysplastic of the physical examination are normal.
nevus on his upper back; it was stopped 3 days ago. Two
weeks ago, he was treated with amoxicillin for community-
Which of the following steps should be taken now to
acquired pneumonia. Medical history is otherwise unre- promote medication adherence?
markable, and he takes no medications.
Skin findings are shown. (A) Patienteducation
)
(B) Recommend a pill box
t (C) Screen for low health literacy
(D) Select an anticoagulant with the lowest copayment
t
Item 79
A 62-year-old woman is evaluated in the hospital for venous
tr
I thromboembolism (WE) prophylaxis. The patient has locally
I
invasive ovarian cancer and will undergo total abdominal
I hysterectomy and bilateral salpingo oophorectomy. She has
i no personal or family history of WE. She is otherwise well
L and takes no medication.
On physical examination, vital signs are normal. BMI
L is 33. Ascites is present. The remainder of the physical
examination is unremarkable.
L
Intermittent pneumatic compression is initiated.
I
t
153
L
I
Self-Assessment Test
yt
.D
ut
UI
tr
CONT,
Which of the following is the most appropriate additional
WE prophylaxis for this patient?
Item 82
A 34-year-old woman is evaluated for preconception coun-
.D
UI (A) Insertion of an inferior vena cava filter seling. She received two doses of the human papillomavirus
UI
(B) Low-molecular weight heparin (LMWH) for 7 days (HPV) vaccine before 15 years of age; the tetanus toxoid,
(D reduced diphtheria toxoid, and acellular pertussis (Tdap)
(C) LMWH for 30 days
vaccine 5 years ago; and the influenza and COVID-19 vac-
(D) No additional prophylaxis cines during the most recent influenza season. She has no
(D
UI medical problems and takes no medications.
Laboratory studies reveal a negative rubella antibody
tr Item 8O
A 66-year-old man is evaluated in the emergency depart
titer and a positive varicella antibody titer. A pregnancy test
result is negative.
ment for sudden onset of perineal pain that began 1 day
ago. He also reports urinary frequency, urgency. and hes Which of the following is the most appropriate vaccination
itancy, and fever and chills. He is not sexually active. strategr for this patient?
Medical history is otherw,ise unremarkable, and he takes (A) Additional dose of HPV vaccine
no medications. (B) Measles, mumps, and rubella vaccine
On physical examination, temperature is 38.7 'C
(tOt.S 'F); other vital signs are normal. On digital rectal (C) Tdapvaccine
examination, the prostate is tender, enlarged, and boggy. (D) No vaccines are indicated at this time
The remainder of the examination is unremarkable.
Item 81
A 64-year old man is evaluated for a 1 month history of
a pruritic and irritated wart like growth on his back. The
warty lesion has been present for over a year, but the irrita-
tion and redness are new.
There are scattered seborrheic keratoses throughout
trunk. Skin flndings are shown.
154
Self-Assessment Test
t
o,
tr Item 84
A 4S-year old man is seen for a preoperative medical eval-
Item 87
An 18 year-old man is evaluated during a preparticipation E
(l,
uation before undergoing an elective hernia repair. He is physical examination for his college soccer team. He is
otherwise well. History and physical examination are per- asymptomatic. He describes no limitation in his exercise ut
tt
(u
formed. Complete blood count, coagulation studies, chest capacity, palpitations, syncope, presyncope, dyspnea, or l,t
ut
radiography, and electrocardiography are not obtained. chest discomfort. He has exercise-induced bronchospasm.
His only medication is abudesonide-formoterol inhalerthat
(u
This preoperative evaluation stratery is most in keeping he uses before exercise. He does not smoke, drink alcohol, vt
with which of the following? or use any drugs. There is no family history of cardiac dis
ease or sudden death.
(A) Defensive medicine Physical examination flndings, including vital signs
(B) High value care and cardiac auscultation, are normal.
(C) Lean quality improvement model
(D) Medical rationing Which of the following is the most appropriate test to
perform?
(A) Chest radiography
Item 85 (B) Echocardiography
A 46-year-old woman is evaluated for a 2-week history of (C) Spirometry
vaginal discharge and itching. She reports no other vaginal
or urinary symptoms. She is sexually active with one male
(D) No testing is indicated
partner. She has a levonorgestrel-releasing intrauterine
device for contraception. Medical history is otherwise unre- Item 88
markable, and she takes no medications.
Speculum examination shows gray frothy discharge. A 72 year-old man is evaluated for a 6-month history of a
The cervix is normal appearing, without purulent discharge.
painful lesion on the left ear that is increasing in size. Medi-
Bimanual examination is unremarkable. cal history is otherwise unremarkable.
Laboratory studies reveal a vaginal pH of5.0. There are Skin flndings are shown.
no hyphae on potassium hydroxide preparation. Tricho-
monad nucleic acid amplification test result is positive. A
pregnancy test result is negative.
Item 86
EI An 82-year-old woman is seen for a preoperative medical
patient
evaluaiion before undergoing cataract surgery' The
of breath
is wtreetctrair bound. Shi reports stable shortness
Which of the following is the most lftely diagnosis?
with minimal to moderate exertion' Medical history is sig
nificant for hypertension, chronic kidney disease' COPD' (A) Basal cell carcinoma
joint disease. Medications are albuterol' (B) Bowen disease
".Ja.g"".."iive
ipotio-pi"*, and beclomethasone inhalers; lisinopril; (C) Seborrheickeratosis
metoprolol; and oxygen by nasal cannula, 2 L/min contin-
uously.
(D) Squamous cell carcinoma
6n physical examination, blood pressure is 150/85 mm
i"t. it gzl-in, respiration rate is.24lmin' and
Hg,
,rir'.
pulse
o*'yg".t trtrrt"tion is 947, on 2 L/min of oxygen by nasal
gteath tounds are distant, with an occasional
can-
wheeze'
Item 89
A l9-year-old woman depart-
is evaluated in the emergency
tr
menifor a 3-day history ofprogressively severe sore throat'
has drooling'
She cannot t*"llo* Iiquids due to pain and
Which of the following is the most appropriate
preoperative testing for this patient? On physical examination, temperature is 38'5 "C
(tot.g "p), Uiood pressure is 110/70 mm Hg, pulse rate is 110/
min, and respiration rate is 25lmin' There is no stridor'
(A) Chest radiograPhY She is
t
\
Self-Assessment Test
t/l
.D in (C) Nystatin cream
Which of the following is the most appropriate next step
vt
UI
tr
CONT.
management?
(D) Oral itraconazole
(E) Terbinaflnecream
,
.D
vl postoperative ileus following a hysterectoml' Sfe reports :
iloit""i"g nausea and abdominal fullness' Her last bowel \
Item 90 -or".n..tlt was on the morning of surgery'
She has been
l
A 33-year-old woman is evaluated for a 6-month history of
is
using hydrocodone for pain. Her only other medication
bilateral breast pain that begins before her menstrual period prophylactic
' 'O; physicallow -molecular-weight heparin'
and resolves after the onset of menstruation' She reports examination, vital signs are normal' There
no other breast symptoms, masses, or nipple discharge' is a healinghysterectomy incision. The abdomen is diflusely
Medical history is otherwise unremarkable. Family history tender without guarding; bowel sounds are decreased'
is signiflcant for breast cancer in her grandmother at age Comprehensive metabolic profile is normal'
oz yiars.Her only medication is a combined hormonal oral Plain abdominal radiograph shows mildly dilated !
Ioops of small and large bowel with air fluid levels and air
I
contraceptive Pill.
On physical examination, vital signs are normal' Bilat- throughout the colon. I !
(C) Methylnaltrexone I
(A) BreastultrasonograPhY i
(D) Nasogastric tube Placement t
(B) Initiate danazol I
(E) Neostigmine 1
(C) Mammography !
(D) Patient education and reassurance I
Item 91
Item 93
A randomized controlled screening trial for prostate cancer
tr 1
1
t
of mildly pruritic rash on her feet, with thickening and antigen blood test were compared with men who were not
discoloration of several toenails. She states that she is not screened. Results were analyzed according to their original I
1
concerned about the toenails because polish covers them. group assignment. Approximately 1.3 deaths from prostate
:
She has no other medical problems. cancer were prevented over 13 years per 1000 men screened.
!
Skin flndings are shown. Overall mortality was not reduced by screening. Men in the
screened population were more likely to undergo repeated
l
)
(A) Classifizing outcomes by overall mortality i
Upon removing the toenail polish, several nails are dys- radiating, right-sided scrotal pain and swelling. The pain l
trophic, with subungual keratosis and discoloration. began insidiously and has slowly worsened in intensity. He I
also reports dysuria and frequency but no fever or hema- I
Which of the following is the most appropriate treatment turia. He has no previous history of scrotal pain, trauma, \ I
for this patient's rash? or urologic procedures. He is sexually active only with his
wife and has no history of sexually transmitted infections.
(A) Adalimumabinjection Medical history is otherwise unremarkable, and he takes no
I
(B) Clobetasolcream \
medications. 1
i
156 i
\ l
Self-Assessment Test
ta
Physical examination, including pelvic examination, o,
se..,;'*"ll,f
CONr 5a.u1r1 pain
i::l::il:l#;#11',,:;?ill,?ilil;ff il is normal.
ts
is relievcd with elevationof,the scr<ltur.n. Cre- A pregnancy test result and chlamydia screening are
E
g,
masteric reflex is nonnal. negative. E
tl
UI
(l,
Which of the following is the most appropriate Which of the following is the most appropriate UI
tl
management?
(A) Dopplerultrasonographyolthe scrotum
management?
gq,
(A) Acetaminophen rrt
(B) [ntramuscular ceftriaxone and oral doxycycline (B) Combined hormonal oral contraceptive pill
(C) Intramuscular ceftriaxone and oral levofloxacin (C) Copper intrauterine device placement
(D) Oral levofloxacin (D) Transvaginal ultrasonography
:
Item 95 Item 97
An 18 year old woman is evaluated lor sunburn of the dor A 65-year old man is evaluated during a wellness visit.
sal hands and dorsal f'eet with burning and pruritus that He has a 50 pack year smoking history but quit smoking
began 6 hours after walking on the beach. Medical history is 7 years ago. He is up to date on colorectal and lung cancer
signiflcant for acne. Medications are drospirenone ethinyl screening. He prefers not to be screened for prostate cancer.
estradiol oral contraceptive, doxycycline, and tretinoin He has not been screened for abdominal aortic aneurysm.
cream. On physical examination, vital signs are normal. BMI is 35.
Skin flndings on the feet are shown.
I
Which of the following is the most appropriate screening test?
i
(A) Abdominal auscultation and palpation
(B) Abdominal CT
(C) Abdominal ultrasonography
(D) Magnetic resonance angiography of the abdomen
:
Item 98
A 4O-year old man is evaluated for a 5-day history of sore
throat and fever. He has no cough or nasal congestion. Med-
ical history is otherwise unremarkable. He does not smoke.
On physical examination, temperature is 38.5'C (101.3.F).
The oropha4mx is erythematous, with enlarged tonsils with
purulent exudate. The anterior cervical lymph nodes are ten-
der and enlarged. The remainder of the physical examination
is normal.
157
Self-Assessment Test
vlt
.D
Item 101
D
ui A 56 year old woman is evaluated for a 2-year history of
UI
(D worsening hot flashes and night sweats. She reports that
U)
(,t the night sweats awaken her several times nightly, result-
(D
ing in fatigue and missed work. History is significant for
hypertension and hysterectomy at age 48 years for uterine
flbroids. She has no history of venous thromboembolism.
(D
(,I Findings on screening mammography 1 year ago were neg-
ative. Family history is signiflcant for breast cancer in her
grandmother, diagnosed at age 75 years. The patient's only
medication is amlodipine.
On physical examination, blood pressure is 134/82 mm
Hg; other vital signs are normal. The remainder of the exam-
ination is unremarkable.
ao
Which of the following is the most appropriate treatment?
(A) Oral estrogen and progesterone
(B) Oral gabapentin
(C) Transdermalestrogen
(D) Transdermal estrogen and oral progesterone
Item 1O2
A 44-year old woman is evaluated for redness, tearing, and
irritation ofthe right eye that began 1 day ago. She reports
no eye pain, photophobia, or change in vision. Her left eye
is uninvolved. She has no other symptoms. Medical history
is otherwise unremarkable, and she takes no medications.
On physical examination, visual acuity is20l2O in both
ln Tlte rash is notecl on her face. neck, back. trunk. and eyes. Pupils are equally round and reactive to light and
llJ proxinral legs. involving 25'){, of her body surface area. accommodation. The right eye is shown.
coNT lhg abscess site on the left leg is healing without signs of
infection.
tr Item 1OO
A 72 year-old woman is seen fbr a preoperative medical eval There is no preauricular lymphadenopathy
uation prior to a henticolectomy fbr newly diagnosed stage II
colon cancer. Five months ago. she underwent drug-eluting
Which of the following is the most likely diagnosis?
coronary artery stent placement for stable ischemic heart
disease and is now asymptomatic. I\4edications are mcto- (A) Episcleritis
prolol, aspirin. clopidogrel. benazepril. and aton,astatin. (B) Herpes simplex keratitis
Physical cxamination findings, including vital signs, (C) Scleritis
are normal.
(D) Uveitis
Which of the following is the most appropriate preoperative
(E) Viralconjunctivitis
management of this patient's dual antiplatelet therapy?
(A) Continue aspirin lnd clopidogrel Item 1O3
(B) Continue aspirin and r,t,ithhold clopidogrel A 44 year old man is evaluated for a 6 week history of a
mildly itchy rash in his armpits.
(C) Withholcl aspirin and clopiclogrel The rash is conflned to the axillae. There is some flne
(D) Withhold aspirin and continue clopidogrel scale. Skin findings are shown (see top ofnext page).
158
Self-Assessment Test
v!
Which of the following is the most appropriate management? 6'
F
(A) Behavioralcounseling q,
(B) Review benefits of a healthy diet E
U!
(C) Review benefits of exercise Ut
o
UI
(D) Review consequences of hypertension vl
o
t/t
Item 106
A 50 year-old man is evaluated during a routine wellness
visit. He has no medical problems and takes no medications.
the patient is engaged in a discussion about the benefits and
harms of prostate cancer screening, after wl-rich he expresses
no pref'erence for or against prostate cancer screening.
tr Item 104
A 68 -v-etrr old man undergoes l preoperativc meclicirl
evaluation befirre total knee arthropllsty. Mcdical his
tory is significant lirr hvpertension. l-rl,perlipidenria,
degenerative joint disease. ancl psoriasis. Meclications are
amlodipine, rnetoprolol, methotrexdte. folic acid. ancl
atorvilstatir-1.
Or-r 1rl-rysical exarnination. r,ital signs are nonnal. Small.
scattered psoriatic plac;ues are noted on the elbows, knees.
and sacrum. Iixilminatirin of'the left knee sl-runs linritecl
range of'motion. Jhe renrainder of the physical erlr.nir.rution
is non-nal.
Item 105
A 28 year-old man is evaluated during a fbllow up visit fbr
hypertension and diabetes mellitus. He works two jobs and
eats fast fbod fbr most meals. He reports that he does not have
time to exercise. He states that he is unhappy with his life
style. Medications are amlodipine and hydrochlorothiazide.
159
Self-Assessment Test
vr
.D
Which of the following is the most appropriate
D
UI treatment?
UI
.D
U!
(A) Oral chloroquine
UT
(B) Topical clobetasol cream
.D (C) Topical metronidazole cream
{
.D
(D) Topical tacrolimus cream
UI
Item 108
A 52 year-old woman is evaluated during a follow up
visit for type 2 diabetes mellitus. She reports that she is
unable to lose weight. She has implemented a healthier
diet and exercise. Medical history is also signiflcant for
depression and epilepsy, both well controlled. Medi-
cations are metformin, glimepiride, bupropion, and Which of the following is the most likely diagnosis?
topiramate. (A) Dysplastic nevus
On physical examination, vital signs are normal. BMI
(B) Pigmented actinic keratosis
is 36.
(C) Pigmented basal cell carcinoma
Which of the following medication changes is most likely (D) Seborrheickeratosis
to promote weight loss in this patient?
(A) Switch bupropion to paroxetine
(B) bupropion to venlafaxine
Item 1 11
Switch
(C) Switch glimepiride to liraglutide A 64 year old man is seen for a preoperative evaluation
before a complex open ventral hernia repair. Medical his
(D) Switch glimepiride to sitagliptin
tory is significant for COPD. He stoppecl smoking 9 months
(E) Switch topiramate to carbamazepine ago. He has dyspnea when walking rapidly on a flat surface.
His symptoms are currently stable, with minimal cough and
sputum production. Medications include inhaled tiotro
Item 109 pium and as nceded albuterol. Spironretry results from
A 26-year old woman is evaluated during a routine exam- 4 months ago slrow an FVC/FEV, rati<-r of 0.5 and an FEV,
ination. She had gestational diabetes during a pregnancy of 60'X, of predicted. Screening for obstructive sleep apnea
4 years ago, but her 12-week postpartum oral glucose tol is negative.
erance test was normal. A copper intrauterine device (lUD) On physical examination, vital signs are normal. Oxy-
for contraception was placed at that time. A Pap smear gen saturation is 94% breatl.ring ambient air. Chest shows
obtained I year ago was normal. She received the human diminished breath sounds r.r,ithout wheezes. Other than a
papillomavirus (HPV) vaccine as a child. She is sexually Iarge ventral hernia, the remainder of the physical exam
active with her husband and has no sexual or vaginal con- ination is normal.
cerns. Medical history is otherwise unremarkable, and she
takes no medications. Which of the following is mostly likely to reduce
On physical examination, vital signs are normal. postoperative pulmonary complications in this
BMI is 20. The remainder of the examination is unre patient?
markable. (A) Perioperative prophylactic respiratory physio
therapy
Which of the following is the most appropriate (B) Postoperativeincentivespirometry
management?
(C) Postoperative nasogastric tube decompression
(A) Cervical cltology and HPV testing (D) Preoperative chlorhexidine oral washes
(B) Hemoglobin A," test
(C) Nucleic acid amplification test for chlamydia
(D) Replace copper IUD Item 1 12
A 23-year old man is evaluated at an urgent care center for
a 5-day history of right ear pain associated with otorrhea,
Item 1 10 pruritus, and diminished hearing. He reports no rash or
A 55 year old man is evaluated for a painless spot on his other symptoms. He has no other medical problems and
back that his wife noticed. He is unaware how long the spot takes no medications.
has been there. Medical history is unremarkable, and he On physical examination, vital signs are normal. There
takes no medications. is discomfort when pulling on the right ear pinna. Findings
Skin flndings are shown (see top of next column). on otoscopy are shown (see top ofnext page).
160
Self-Assessment Test
t
6'
Physical examination flndings, including vital signs, F
are normal. E
o
Which of the following supplements can be recommended UI
It
as most likely to help the patient meet her therapeutic goal? o
UI
UI
(A) VitaminA
(B) Vitamin C (u
(C)
lrt
Vitamin D with calcium
(D) Vitamin E
(E) No vitamin or supplement
Item 1 15
A 24 year old woman is evaluated for a 5 year history of
atopic dermatitis. She continues to have mild flares every
few weeks on her arms. Her only medication is topical clo-
Using an otoscope and a wire ear loop, debris is betasol cream.
removed from the canal.
Skin flndings are shown.
Item I 13
A 58 year old man is evaluated for decreased libido and
erectile dysfunction that began insidiously over the past
6 months. He is not having nocturnal or early morning
erections. His medical history is otherwise unremarkable.
On physical examination, vital signs are normal. BMI
is 24. There is no gynecomastia. Distribution of pubic hair is
normal. Testes are normal sized.
Laboratory studies show an B:00 eru serum total
testosterone level of 200 ng/dl (O.g+ nmol/L) and a
thyroid-stimulating hormone level of 3.6 pU/mL (3.6 mUiL).
161
Self-Assessment Test
t/r
(D
UI
(a
(D
tr
CONT,
the midcalf to the left medial malleolus and dorsomedial
aspect of the right fbot. There is no fluctuance or drainage.
Doppler ultrasound olthe right lower extremity shows
time of discharge, physical examination shows no evidence
of volume overload and laboratory studies are normal.
t^
(a no evidence of deep venous thrombosis or fluid collection. Which of the following factors is fundamental to a
Intravenous cefazolin is initiated. successful transition of care for this patient?
o
Which of the following is the most appropriate management
(A) Discharge planning by the hospital physician
.D
tt for preventing venous thromboembolism? (B) Explicit communication with the primary care physician
(C) Medication reconciliation by the daughter
(A) Direct oral anticoagulant
(D) Scheduling of follow up laboratory evaluation
(B) lntermittent pneumatic compression (lPC)
(C) IPC and direct oral anticoagulant
(D) IPC and lorv molecular weight heparin (LMWH) Item 120
(E) LMWH A 62 year old woman is evaluated for a 6 month history
of intermittent, painful sores on the back of her hands that
worsen with sun exposure. She has no other medical prob-
Item 11 7 lems and takes no medications.
Skin findings are shown.
A 30 year-old man is evaluated during a routine wellness :
visit. He is asymptomatic, has no ongoing medical prob
lems, and does not use tobacco products. Previous screening
for hepatitis C virus and HIV infections were negative. His
immunizations are up to date.
On physical examination, blood pressure is 120/70 mm
Hg. The remainder of the examination is normal.
162
L
L
Self-Assessment Test
t
L UI
(C) 13 Valent pneumococcal conjugate vaccine Which of the following is the most appropriate
o
L (D) 23 Valent pneumococcal polysaccharide vaccine management?
o
L (A) Chest CT angiography E
t!
I
Item 122 (B) Elevate head of'the bed and limit sedating medications UI
(l,
L UI
(C) Start albuterol vt
A 72-year-old man is evaluated lor recurrent urinary tract
L infections (UTIs). He is currently asymptomatic. Seven days (D) Start bilevel positive airway pressure ventilation
ago, he presented with dysuria, urgency, and suprapubic (E) Start empiric antibiotics =(l,
t pain. A urine culture was obtained, and he was started on
t/t
t
t empiric trimethoprim-sulfamethoxazole. This is his third
UTI in 12 months. All previous urine cultures grew Esch- Item 125
I
5 erichia coli sensitive to trimethoprim sulfamethoxazole. A 26-year-old man is evaluated for redness of the left eye
t All episodes were treated with a 7-day course of antibiotics, accompanied by yellow discharge and crusting on the eye
L with rapid resolution of symptoms. Medical history is oth lids for the past 2 days. The symptoms are most noticeable
erwise unremarkable, and he takes no other medications. upon awakening. He reports some visual blurring that clears
t I
On physical examination, vital signs are normal. Digital with blinking. The right eye is uninvolved. He reports no
t rectal examination reveals a nontender prostate with no bog-
giness. The remainder of the examination is unremarkable.
other symptoms. He does not wear contact lenses. Medical
history is unremarkable, and he takes no medications.
t Urine culture shows growth of E. coli sensitive to On physical examination, there is diffuse circumferen
t
trimethoprim sulflamethoxazole. tial conjunctival erythema of the left eye that is most notice
I able peripherally. A small amount of dried, yellow discharge
Which of the following is the most appropriate management? is noted on the superior and inferior eyelashes. Visual acuity
t is 20/30 in both eyes. The remainder of the examination is
I (A) CToftheprostate
I
5 unremarkable.
(B) Prostatic massage followed by urine culture
I
L (C) Six-week course of trimethoprim sulfamethoxazole Which of the following is the most likely diagnosis?
(D) No additional treatment
L (A) Acutebacterial conjunctivitis
I (B) Blepharitis
t Item 123 (C) Hyperacutebacterialconjunctivitis
L A 40 year-old woman is evaluated during a routine well- (D) Scleritis
I
ness visit. She asks about breast cancer screening. She has (E) Viralconjunctivitis
I no medical problems or concerning family history, includ,
L
ing no history ofbreast cancer, and has no risk factors for
breast cancer. Item 126
t
I
On physical examination, breast examination is normal. A 65-year-old woman is evaluated for a mole beneath the
right eye that has been present for several months but
L
Which of the following is the most reasonable breast appears to be growing.
cancer screening management stratery for this patient? Skin flndings are shown.
L
(A) Breast MRI
(B) Digital breast tomosynthesis
: (C) Mammography
(D) Shared decision making
tr Item 124
A 66 year old man is evaluated in the postanesthesia care
unit after undergoing elective lumbar laminectomy. He
has snoring and oxygen desaturations. Medical history is
signiflcant for obesity, hypertension, and diabetes mellitus.
Medications are metoprolol, amlodipine, metfbrmin (with-
held preoperatively), and hydromorphone.
On physical examination, temperature is 37'C (98.7'1"),
blood pressure is 149188 mrn Hg, pulse rate is B9/min, and
respiration rate is 16/min. While awake and upright, oxygen Which of the following is the most likely diagnosis?
saturations are 95')(, on 2 L of oxygen and 93'l,, breathing (A) Malignant melanoma
room air; while sleeping supine, oxygen saturation inter
mittently dips to 8B'2, on 2 L of oxygen. BMI is 42. His neck
(B) Pigmented basal cell carcinoma
is large. On pulmonary examination, lungs are clear. lhe (C) Seborrheickeratosis
abdomen is soft and protuberant, with a large pannus. (D) Squamous cell carcinoma
163
Answers and Critiques
Item 1 Answer: D XEY POl lll5 (continued)
Educational Objective: Treat genitourinary syndrome o If nonhormonal treatments are not effective in the
of menopause. treatment of genitourinary syndrome of menopause,
initial treatment is daily topical vaginal guidelines recommend the use of low dose topical
The most appropriate
moisturizer (Option D). This patient has genitourinary syn vaginal estrogen therapy rather than systemic estro-
drome of menopause (GSM). Vaginal symptoms are common gen therapy for patients whose only symptoms are
in menopause and can include sexual symptoms, such as related to vaginal atrophy. a
(l,
dyspareunia; vaginal dryness, burning, and itching; and uri ET
nary symptoms, such as dysuria and frequent urinary tract Bibliography
infections. The North American Menopause Society recom The NAMS 2017 hormone therapy position statement advisory panel. The
2017 hormone therapy position statement of the North American
t,
mends initiating nonhormonal therapies as first line treat Menopause Society. Menopatse. 2017:24:728-53. [PMID: 28650869] =l
E
ment lbr GSM betbre considering other therapies, including doi: lo. I 097i GME. 00000000OO000921 .E
UI
topical vaginal estrogen therapy. Nonhormonal approaches o
include as needed vaginal lubricants for intercourse and vag
inal moisturizers that can alleviate vaginal dryness and irrita Item 2 Answer: B cta=
tion when used regularly. A daily topical vaginal moisturizer
Educational Objective: Treat hidradenitis suppurativa.
may improve this patient's symptoms of vaginal dryness and
itching and pain with sexual intercourse. Adalimumab (Option B), a tumor necrosis factor-a inhibitor,
lf nonhormonal treatments are not ell'ective, guidelines is the most appropriate additional treatment. It is the only
recommend the use of low dose topical vaginal estrogen FDA approved therapy for hidradenitis suppurativa and is
therapy rather than systemic estrogen therapy for patients indicated lor use in patients with moderate to severe dis
whose only symptoms are related to vaginal atrophy. Sys ease. Moderate to severe disease is defined by the presence of
temic estrogen therapy (Option A) is associated with higher multiple recurring nodules or the development of scarring.
risk fbr complications, such as venous thromboembolism, Hidradenitis suppurativa is an inflammatory skin disorder
compared with low dose topical vaginal estrogen therapy. of the apocrine glands. This disease tends to occur more
Ospemifene (Option B) is a selective estrogen receptor commonll,in women and tlpically begins after puberty. The
modulator that is FDA approved tbr the treatment of moder pathogenesis of hidradenitis begins with follicular occlusion,
ate to severe GSM. However, its use is limited by its side efl'ect but not infection or inflammation of the apocrine glands.
profile, which includes hot flashes and an increased risk lbr After occlusion, secretions build up in the follicular duct and
venous thromboembolism. A daily topical vaginal moistur result in rupture and a subsequent inflammatory reaction that
izer is a more appropriate flrst line therapy. resembles a bacterial abscess. An acute inflammatory reaction
'lbpical vaginal estrogen therapy (Option C) is a treatment is then triggered in the surrounding tissue. The role ofbacteria
option fbr GSM, but the North American Menop:ruse Society rec is controversial and is likely a secondary colonization, because
ommends the use olnonhonnonal therapies as initial treatment. lesions are initially sterile and antibiotics are not entirely
Women with urinary slmptoms ol GSM, which this patient efI'ective in preventing new lesions. The chronic and recurrent
does not have, may benefit more fiom topical estrogen therapy nature of hidradenitis suppurativa helps distinguish it from
because there is some evidence that topical estrogen therapy can other inlectious causes. Areas frequently aflected are inter
help reduce urinary slmptoms, including frequent urinary tract triginous areas, such as the axilla, groin, and underneath the
infections. Topical vaginal estrogen is available in three prepara breasts. The condition is characterizrd by comedones, painful
tions (cream, tablet, or ring) and is not associated with the same nodules, abscesses, draining sinuses, and scarring. Risk fac
risks as systemic estrogen therapy. l,brwomen with breast cancer tors include obesity, metabolic syndrome, cigarette smoking,
(past or current), topical vaginal estrogen therapy should only be and family history. For this reason, weight loss and smoking
oflbred in consultation with their or.rcologist. cessation are an essential part of the treatment plan. Hi
dradenitis suppurativa is extremely difficult to treat. No single
I(EY POITIS
treatment has been eflective for all patients; however, several
. The North American Menopause Society recommends options are available. Decolonization with dilute bleach baths
initiating nonhormonal therapies, such as daily vagi and chlorhexidine washes may be used in addition to topical
nal moisturizer and vaginal lubricants, as first line clindamycin as initial therapy. For more extensive or resistant
treatments fbr genitourinary syndrome of menopause disease, common systemic treatments include oral antibiotics,
before considering topical vaginal estrogen therapy. such as tetracyclines or the combination of clindamycin and
(Continued) rifampin. This patient with several active lesions and scarring
155
Answers and Critiques
is experiencing moderate to severe disease. and treatment require dosage adjustn-rent in patients with advanced kid
with adalimumab is indicated. ney disease because decreased kidney clearance of these
An oral retinoid, such as acitretin (Option A), is an medications may be associated with adverse events. The
excellent treatment option for hidradenitis suppurativa CDS system r.rotifled the physician of the contraindication
that fails to respond to initial therapy with topical or oral to use full dose rivaroxaban ir.r this patient and prevented
antibiotics and other adjunctive therapies. Nonadherence the potential harn-r.
is common due to side effects. However, coadministration Electronic transmission ol prescriptions (Option B)
with doxycycline poses an unacceptable high risk for the has reduced delays in obtaining prescriptions and elimi-
patient to develop idiopathic intracranial hypertension and nates errors due to prescription legibilitf issues. This process
is therefore not the best choice fbr additional therapy. improles the likelihood that a r.nedication is disper.rsed as
Methotrexate (Option C) has not been shown to be intended but is unlikely to result in a change in the rnedica
efficacious in hidradenitis suppurativa and should not be tion prescribed.
prescribed. When applied to medication prescritring, best practice
t^ Narrow band ultraviolet B phototherapy (Option D) is alerts or advisories typically provide infbrmation on how
o a powerful tool in the treatment of psoriasis vulgaris and best to institute and monitor specific rnedications. The1,
=
tt other conditions. such as mycosis fungoides, atopic derma rnal' provide guidance on appropriate lab<,rratory ntonitoring
o, titis, and vitiligo, but has no role in treatment of hidradenitis (Option C) or duration of therapy. They usually are distinct
CL suppurativa. fiom a contraindicaticln alert.
n Patient inlbrmation fact sheets (Option D) are t1'picalll'
TEY POIIIIT
provided by the pharmacy upolr medication dispensing and
lt o Hidradenitis suppurativa is associated with smoking, provide information regarding medication administration.
.D
t/l obesity, and metabolic syndrome, and these condi- erpected medication effects. and potential adverse events.
tions should be addressed. Their provision typically $'ould not result in a change in
o Adalimumab is FDA approved for moderate to severe rnedication.
hidradenitis, defined by the presence ofdiffuse nodules IEY POIilI
or scar formation.
o Computerized clinical decision support can high-
light potential contraindications to diagnostic tests,
Bibliography
specify dose recommendations. identify potential
Coldburg SR. Strober B[. Payette MJ. Hidradenitis suppurativa: current and
emerging treatments. J Am Acad Dermatol. 2020:82:lo6l 82. LPMII): drug interactions, and suggest modifications to
316041001 doi:10.101 6rj.jaad.2019.08.089 drug dosage in patients with kidney or liver
dysfunction.
tr Item 3 Answer: A
Educational Objective: Use clinical decision support to
Bibliography
Billstein Leber M, Carrilk) CJ. Cassano ,\1. et al. .\SIIP guidelines on pre
improve medication safety. Venting medication er()rs in hospitrls. Am J tlerlth S).st Pharnt. 201tt:
75:1,193 517. IPMTD:302578-1-11 doi:10.21 16 aihplT08ll
Computerized clir.rical decision support (CDS) (Option A)
is the feature ot the electronic health record (EHR) that
rnost likely accounts for tl.re physician's decisiort to mod
ify the patient's prescription. The EIIR is a computer-
ized system that encompasses patient record reviert, and
Item 4 Answer: A
Educational Objective: Manage perioperative
tr
glucocorticoids.
treatment ordering lunctionality fbr clinicians. The EilR
enables the timely sharing of patient information by mul The most appropriate preoperative gluc<-rcorticoid manage
tiple users, resulting in in-rproved communication and care ment is to plan for administration of stress-dose gluco
efficiency. Computerized physician order entry (CPOF.) is corticoids perioperatively (Option A) to avoid an adrenal
a system by w'hich clinicians electronically enter medica crisis. Patients receiving long term glucocorticoid therapy
tion, radiology, and iaboratory orders, thereby reducing may be at risk fbr secclndary adrenal insulficiency (Al) and
errors and delays related to illegible handu'ritir.rg. Comput perioperative adrenal crisis. Historicall\l increased gluco
erized CDS is the use of intbrmation technology to facilitate corticoid doses (stress dosing) n'ere administered liberally
clinical decision making. When integrated into a CPOE in the perioperative period to mimic the natural response
system, CDS can highlight potential contraindications to of'the adrenal glands t<-r phy'siologic stress. However. studies
diagnostic tests, specify dose recommendations. identily have shorvn that stress dosing n.ray not be required in n.ran1'
potential drug interactit-rns, and suggest modifications to patients receiving low dose and short course glucocorti
drug dosage in patients r,,r,ith kidney or liver dysfunction. coid therapy. When managing a patient u'ith glucocorticoid
In this case. the physician u'as going to prescribe this patient exposure rvithin the past year, considerations include the
lull d<lse rivaroxaban to reduce the risk for stroke caused by type of surgery (low intermediate, or high risk) and the
atrial librillation. However. many direct oral anticoagulants dose and duration of glucocorticoid therapy. Patients on
156
Answers and Critiques
tr
CONI,
high doses ol glucocorticoids fbr cxtended periods (equiva
lent of >20 nrgid prednisone fbr >3 lreeks) arc at high risk
for AI and should receive stress dosing u,hen undergoing
lclrenal insufficiency: Guidelines Irorn the Association of
Anaesthetists, the Royal College of Physicians and the Society fbr
lrndocrinologlr UK. Anaesthesia. 2O2O :75:654 63. IPM] D: 32017012|r
doi: l 0. l111, anae. l,l96:l
intermediate to high-risk surgery. These patients may not
need glucocorticoid support fbr a low risk proccclure, such
as cataract surgery. Patients at low risk fbr AI (those taking
lor,r, doses ol glucocorticoicls lirr shorter periods) can gen-
Item 5 Answer: B
erally proceed to surgery ol any risk lerel r.t,ithout stress- Educational Objective: Diagnose malignant
dose glucocorticoids. Patients at moderate risk fttr AI may melanoma with an excisional biopsy.
require testing to assess firr the presence of adrcnal insuffi- The most appropriate management is excisional biopsy
ciency belore determining best pcrioperative management, (Option B) in this patient who most likely has a malignant
although newer guidelines suggest treating empirically with melanoma. Melanoma is the leading cause of skin cancer
stress dose glucocorticoids ftrr a large proportion of this related deaths, and early detection and diagnosis are t/l
(l,
intermediate risk group. this patient is at moclerate risk for imperative. Skin biopsy is required fbr definitive diagno ET
AI because she receired rnorc than 5 mgrfl ol prcdnisone sis. An excisional biopsy, a technique in which the entire
during thc past year: previously required supplemcntal glu lesion is removed using 1- to 2 mm peripheral margins. (J
cocorticoids during another nlajor operation; and is under- is the most appropriate biopsy technique. For localized
going total l<nce arthroplasty. which is an intermediate risk
=,
melanomas, prognosis is related to the depth of invasion, |g
procedure. Continuing her baseline glucocorticoids alone UI
either by Clark level (l to V) or by Breslow depth. A high
(Option B) r.r,ould not be suflicient to avoid a potential o
mitotic rate, lymphovascular invasion, and the presence
t
adrenal crisis. of bleeding or ulceration are poor prognostic signs. As =
Nlornir.rg measurement of serum cortisol (Option C) the depth of invasion increases, the risk for nodal and
can be usecl to screen for adrenal suppression in patients ultimately distant metastasis increases. Lesion depth also
with less clear risk. If the B:00 ,rv morning corlisol level infbrms the nature of definitive excision. A margin of I cm
is less than ll pg/dl (82.8 nnrol/1.) in the absence of glu is acceptable for lesions that are less than 1 mm in thick
cocorticoid rcplacement for 24 hours, the aclrenal gland ness. Melanomas between 1 mm and 2 mm in thickness
is supprcssccl and perioperative glucocorticoid stress dos should be resected with a 2 cm margin, provided that
ing is indicatcd. An B:00 ,rvr cortisol level greater than a skin graft is not required for closure. Lesions that are
15 pgrdt- (+t+ nmolll) indicates normal adrenal function greater than 2 mm in thickness should also be resected
and no nccd fbr stress dosing. Values of 3 to la pg/dl with 2-cm margins.
(83-386 nrnol/L) require additional evaluation, such as a Cryotherapy (Option A) or other destructive therapies,
corticotropin (ACTH) stimulatior-r test or glucocorticoid such as electrodesiccation and curettage, should not be used
stress dosir.rg, according the physician's best juclgment. to treat pigmented lesions that are suspicious for malig
Prior eviclence of adrenal insufliciency and glucocorti nant melanoma. Destructive therapies of this llpe remove
coid use of greater than 5 mgld make adrenal axis testing the ability to determine the depth of the lesion, which is
unnecessary in this patient. impofiant for determining prognosis and planning surgical
Because of the risk fbr adrenal suppression and dis- excision. ln addition, destroying the tumor removes the
ease flare. prcdnisone should not be u,ithheld (Option D). ability assess for genetic mutations that may guide f'uture
Studies clemonstrate that only glucocorlicoid doscs greater therapy.
than l5 mgrcl are associated with signiflcant increases in the Survival in malignant melanoma is dependent on early
risk for post arthroplasty inlection in patients with rheu diagnosis. This lesion exhibits all of the ABCDE criteria fbr
matoid arthritis. Continuing low daily glucocorticoicl doses diagnosing melanoma: Asymmetry, Border irregularity,
throughout the perioperative period is unlikely to result in Color variation, Diameter greater than 6 mm, and Evolution
complications in this patient. or change over time. Because of the importance of early
detection and diagnosis of malignant melanoma, reevalua
XEV POII{IS
tion in 3 months (Option C) is not appropriate.
. Perioperative glucocorticoid stress dosing may not be Superflcial shave biopsies (Option D) should be
required in many patients receiving low dose and avoided in most pigmented lesions because there is risk
short-course glucocorticoid therapy. fbr transecting a melanoma and preventing true stag
. Patients at moderate risk for adrenal insufficiency ing of the lesion. A deep shave (scoop or saucerization)
may require testing to determine perioperative man- biopsy to a depth below the lesion is also adequate but
agement, although treating empirically with stress is best performed by dermatologists, who have more
doses of glucocorticoids may be reasonable. experience with this procedure. However, an excisional
biopsy is most appropriate because it enables the entire
lesion to be examined by the pathologist fbr an accurate
Bibliography
\ bodcock T. Barker P, Daniel Breslow depth measurement, the most important factor in
S, et al. Guidelines for the ntrnrgement of
glucocorticoids during the peri operative period ti)r patients \\'ith determining prognosis.
167
Answers and Critiques
XEY POIXT aged 50 years or older should receive the recombinant (inac
tivated) herpes zoster vaccine.
o For suspected malignant melanoma, excisional biopsy,
a technique in which the entire lesion is removed TEY POIXIS
using 1 to 2-mm peripheral margins, is used for both . Adults aged 50 years or older should receive the
diagnosis and determination of Breslow depth. recombinant herpes zoster vaccine regardless of a his-
tory of herpes zoster vaccination with the live attenu
Bibliography ated vaccine or a previous episode of herpes zoster.
Swetter SM,'Isao H, Bichakjian CK. et al. Guidelines of care for the manage
ment of primary cutaneous melanoma. J Am Acad Dermatol. 2019:
. The Advisory Committee on Immunization Practices
80:208 50. IPMID: 30392755J doi:10.1016rj.jaad.2018.08.055 provides no recommendation regarding use of the
recombinant herpes zoster vaccine in immunocom
promised patients or patients with HIV infection.
D Item 6 Answer: B
(a
(D
Ed u cati o na I Objective : Vaccinate with recombinant Bibliography
= (inactivated) herpes zoster vaccine. I.'reedman \lS. Bernsteir.l tl. Ault KA. Reconrmended adult immunization
Ut schedule. United Strtes, 2021. Ann lntern l\'led. 2021. [PI!{lD: 33571011]
q, doi:10.71126 \12O SOUO
The most appropriate vaccine to administer to this patient is
EL the recombinant (inactivated) herpes zoster vaccine (Option
r.|
B). Adults aged 50 years or older should receive the recom- Item 7 Answer: C
lt binant (inactivated) herpes zoster vaccine to reduce the inci
Educational Objective: Screen and monitor for
(D dence ofzoster and postherpetic neuralgia. The recombinant
UI hypertension in a young adult.
vaccine is administered intramuscularly in two doses, with
an interval of 2 to 6 months between doses. This patient's The most appropriate next step in management is to repeat
previous episode of herpes zoster does not provide immunity blood pressure measurement annually (Option C). This
against future episodes and should not preclude additional patient has received appropriate screening for hypertension
zoster vaccination. In addition, persons who received the at a routine wellness examination. She has no signs or symp
live attenuated herpes zoster vaccine, such as this patient, toms ofcardiovascular disease and her blood pressure at this
should be oflered the recombinant herpes zoster vaccine visit is in the high normal range, according to the U.S. Pre
given its increased efficacy; the recombinant vaccine has ventive Services Task Force (USPSTF). The USPSTF supports
shown 97"/,, efficacy in persons aged 50 to 69 years and 91"/,, screening all adults beginning at age 18 years for hyperten
efficacy in persons aged 70 years or older. Patients who have sion (grade A). Screening should occur annually in adults
received the live attenuated vaccine should wait at least aged 40 years or older and in younger adults at increased
8 weeks before receiving the recombinant vaccine. The Advi risk, including patients with high normal blood pressure.
sory Committee on Immunization Practices has not made a patients who are overweight or obese, and Black patients.
recommendation regarding the recombinant herpes zoster On the basis ol this patient's blood pressure reading today
vaccine in immunocompromised patients or patients with (129178 mm Hg) and her having obesity (BMI of 30), she
HIV infection, regardless of CD4 cell count. should have repeated blood pressure measurement annually.
The quadrivalent meningococcal conjugate vac patients who are overweight or obese. and Black patients.
cine (Option A) is recommended to be given at age 11 to On the basis of this patient's blood pressure reading today
12 years, with a booster dose at age 16 years. Adults who are (130/85 mm Hg) and her having obesity (BMI of 30), she
at increased risk for meningococcal disease should undergo should have repeated blood pressure measurement annually.
primary vaccination if they never received age-appropriate The 2017 American College of Cardiolog,iAmerican Heart
vaccination. Increased risk is deflned as persistent comple Association blood pressure guideline recommends repeated
ment component deflciencies or patients taking eculizumab; annual blood pressure measurement fbr adults with a nor
functional or anatomic asplenia (including sickle cell dis mal pressure reading.
ease); HIV infection; microbiologists routinely exposed to This patient does not meet criteria for initiation of
Neisserio meningitidis; a meningococcal disease outbreak; antihypertensive medication (Option A). Before treatment
and travel or residence in countries with hyperendemic or is initiated, when possible and in the absence of severe
epidemic meningococcal disease. This patient has no indica- hypertension or end organ damage, the diagnosis should
tion for the quadrivalent meningococcal conjugate vaccine. be conflrmed with blood pressure measurements outside of
For healthy patients who are nonsmokers, the 23-valent the clinical setting by using ambulatory or home blood pres
pneumococcal polysaccharide vaccine (Option C) is recom sure monitoring. This patient with high normal blood pres
mended at age 65 years. This 64 year-old patient who is a sure reading can be reassessed in 1 year without initiating
nonsmoker with no significant medical history does not treatment.
need to receive the vaccine early. Left ventricular hypertrophy (LVH) is commonly mea
Providing no vaccines is not the best strateS/ (Option sured by ECG andror echocardiography (Option B). Assess
D). In the absence ofspecific contraindications, all patients ment for LVH is not universally recommended during
158
Answers and Critiques
evaluation and management of hypertension. Assessment a clinical beneflt. S-ARIs can be initiated when cr-blocker
for LVH is most useful in adults with conflrmed hyperten- therapy alone does not control symptoms or if a patient is
sion who are young (<18 years of age) or have evidence of intolerant of maximum dose cx blocker therapy.
secondary hypertension, chronic uncontrolled hyperten- Scheduled or timed voiding (Option B) is a technique
sion, or a history of symptoms of heart failure. Echocardi to manage urge urinary incontinence, not LUTS associ-
ography is not indicated in this patient with normal physical ated with BPH. This behavioral strateg/ includes scheduled
examination findings and a high-normal blood pressure. voiding attempts at intervals shorter than the usual time
According to the USPSTF, for patients younger than between incontinence episodes, regardless of the urge to
40 years with normal initial blood pressure and no risk void, with a gradual increase in the time between voids.
factors for atherosclerotic cardiovascular disease, screen ln patients with cognitive impairment related functional
ing should occur at 3 to 5 year intervals (Option D). This urinary incontinence, timed voiding with prompting by the
patient is at increased risk for hypertension, and annual caregiver may also be useful. This patient does not report
blood pressure measurement is recommended. incontinence. ,a
{l,
Urodynamic testing (Option D) is not indicated for
KEY POIilTS E
patients who have not yet attempted pharmacotherapy and
. The U.S. Preventive Services Task Force supports have no evidence of urinary retention or urinary inconti- L,,
screening all adults for hypertension beginning at age nence, as is the case for this patient. -E,
18 years. .E
f,EY POIilT5 UI
. Hypertension screening should occur annually in
. ttt
cr-Blockers, such as tamsulosin, are first line therapy
adults aged 40 years or older and in younger adults at ,h
for lower urinary tract symptoms secondary to benign =
increased risk, including patients with high-normal
prostatic hyperplasia; symptom improvement can be 4
blood pressure (130 to 139/85 to 89 mm Hg), patients
seen within 48 hours of initiation.
who are overweight or obese, and Black patients.
o So-Reductase inhibitors, such as finasteride, are
Bibliography second line therapy for lower urinary tract symptoms
Siu AL; U.S. Preventi\e Services Task Force. Screening for high blood pres secondary to benign prostatic hyperplasia; clinical
srrre in adults: U.S. Preventive Services Task Force recommendation
statement. Ann Intern Med. 2015;163:778 86. [PMID: 26458123] doi:]0.
improvement is not seen for several months.
7:t26 M15-2223
Bibliography
Item 8 Answer: C Bortnick E, Brown C, Simma Chiang V Kaplan SA. Modern best practice
in the management of benign prostatic hyperplasia in the elderly. Ther
Educational Objective: Treat lower urinary tract A<lv Urol. 2020;12:7756287220929 486. IPMID : 32547 6421 doi: 10.1177l
17562A7220929486
symptoms secondary to benigrr prostatic hyperplasia.
not have psychological conditions that impede adherence to inrpractical. ancl slstemic glucocorticoid therapl' mav be
these requirements. required.
Regular exercise (Option B) has many important health I)rug inducecl lty persensitivitl sl'ndrome (DlllS)
beneflts. However. exercise alone or combined r.t,ith calorie (Option A). fbrnrerll cirlled dmg rcilction rlith eosinophilia
restriction results in additional weight loss of no more than lncl systemic s)'nlplonts (DRESS). is a severe lile threatening
3 kg (o.o lb). This will not result in the weight loss required rleclicatior-r reaction. S1'mptoms begin r'vith f'ever and flu
by this patient, and it is unlikely that he can comply owing lil<c symptoms. n'hich are quickll follou'ed b1' burning
to his hip and knee osteoarthritis. skir.r pain irnd rlsh. The rash is t1 picllll' a morbillifirrnt
Of all the weight loss medications. phentermine- exanthem that strrts on the tace and upper trllnk xnd
topiramate (Option C) has the highest probability of achiev- spreads distalll: lrventuallli patie nts clcve lop striking facill
ing a 5'1, weight loss. Howeveq contraindications include edema and redness. a l-rallmark o{ this condition. as shou'n.
nephrolithiasis, uncontrolled hypertension, or resting tachy
D cardia. A 5'1, weight loss will not be sufficient to treat this
ar! patient's many comorbid conditions. Finally, he is already
taking a weight loss medication (liraglutide) and has resting
=
.D
gr tachycardia; therefore, phentermine topiramate is contra-
q,
indicated.
a Recommending a very-low-calorie diet (Option D) for
f.l
this patient is unrealistic because he reports having not
lt had success with other dietary interventions. lt is unlikely
(D
that diet therapy alone will result in the weight loss that is
UI needed to address his obesity related complications.
XEY POITT
o Bariatric surgery is indicated for motivated individu-
with BMI of 40 or greater or BMI of 35 or greater
als
with obesity associated comorbid complications
Orll mucosal invohernent is cornntor-r. Ur-rlike other severe
who have not responded to behavioral treatment or
meclicltion reactions. the orlset ot svnrptoms in DIIIS is
pharmacotherapy.
cicllled. often 2 to 6 uleeks irtler erposure. The rash onsct
lltcr 9 clal's and l:rcl< of systernic synrptorns in this patient
Bibtiography irre inconsistent n'ith DItlS.
Tsai AC, Bessesen DH. Obesit_ri Ann Intern Med. 20191170:lTC33 .18. [PN{ID:
308315931 doi: 10.7326'AlTC20l9O3050
,\ flxed dmg emption (Option C) presents \\'ith one
to several duskl purple pink nracules or plirques. rlost
cor.ttrt.tonlv on tlie lips, tace. {ingers. ancl genitals. Thel rnl1,
1O
tr Item Answer: B
Educational Objective: Diagnose exanthematous
occur u,ithin the tirst 2.,.r,eeks of stirrting a causativc nrccl
icltion but are locirlized. inclucling u,hen tl.re1' recur Llpon
re exposure. The clif luse rash in this prrtier-rt is inconsistent
(morbilliform) drug eruption.
u'ith a fired drug rcaction.
The rnost likell, diagnosis is erirnthematous clrug eruption H]'persensilivit) \'asculitis (Option D) also occurs
(Option B). 'lhis patiellt preseltts u'ith a rrorbillifbrrn rash 'nvithin 2 u'eeks ol clrug erposure but rnirnilests as palpirble
9 days after stirrting sultamethorazole trinrethoprim, a purpura and not thc clifluse rash seen in this patient.
frequent ciruse of medication reactions. Eranthentatous or
morbillifirrm (n.reasles like) emptions are the most com XEY POIIIIS
rnon fbrm ot cutaneous drug reactions. most likely rep o Exanthematous or morbillifbrm (measles-like) erup-
resenting a t),pe IV delalecl hYpersensitivitr.relction. The tions are the most common form of cutaneous drug
rasl-r lr1-tpclrs during the first or secor-rd r,r'eek after drug reactions, usually appearing within the first or second
exposure. rrlthough subsequent exposures can produce a week of drug exposure. I
reactior.r ntuch more quicl<ly piitients develop erythem
o An exanthematous rash begins as erythematous pap
atous paltules and macules that coalesce svntmetricallv
to fornr plaques. beginning on the trunk :rncl ltrogressing ules and macules that coalesce symmetrically to form
plaques. beginning on the trunk and progressing dis-
distalll across the lirnbs irnd usualll' sparing the pahns
and soles. '[he papules arc olten dense irncl ntonontor tally across the limbs and usually sparing the palms
phic and arc accompaniecl I-r_\, r,iirr,ing clegrees of pruritus. and soles.
l-ymph:rdenopathy is ltot uncontmon. Treatmcnt involves
cessation of the causative agent. use ofpotent topical glu Bibliography
cocorticoids. and oral FI, antil-ristamines. A u,iclespread Kroshins\'D. Adverse cutaneous reactions to medications IEditorial]. Clin
Dermatol. 2020:38:605 6. IPMID: 333.11194] doi:10.1016/j.clinderniltol.
rash may rnake the topical irpplication of glucocorticoids 2(020.06.o17
170
Answers and Criti que:
Jhe most appropriatc periopcrative rnnnagenlcnt of this The most appropriate treatment is zinc pyrithione shampoo
patient's antirheunratic reginrcr-r is to cor-ttinue nrethcl (Option E). This patient has a classic presentation of seb
trexilte ancl hold lclalinrunrab lirr 2 rveeks belbre and after orrheic dermatitis, a common condition characterized by
surgery (Option B). O'ur,ing to both their cliseitse and its greasy, scaly, yellow to erythematous patches in seborrheic
trertment, patients rvith rlteutnrttologic disease undcr areas (scalp, face, ears, upper chest, axillae, and inguinal
going elective surgerv rrrc rrt highcr risk tirr pcrioperrtive folds). On the face, speciflc areas of involvement include
inlectious conrplicittions. it.tcltttling risk firr peri the eyebrows, medial aspects of the cheeks, inter eyebrow
prosthetic joint inlcctiou in totitl joir.rt rtrthroplasties ol the region, and nasal ala. Topical antifungal treatments are first
hip and knee. Appropriatc lxrrrlilgenrerrt ol lntirheunratic line treatment. Over the counter medications, such as zinc ut
(l,
rnedication in the perioperrltive periocl rnitigates this risk. pyrithione and selenium sulflde shampoos, are the most
ET
Elective surgery shoukl bc dclayed until the undcrly cost eflective treatment options to use first. Ketoconazole
ing disease is controllecl irncl preterably in renrission, shampoo and cream are also effective. Patients should apply TJ
because this is associated with rccluccd inf'ection risk and the shampoo on the skin and allow it to sit for .5 minutes T'
improved oLitconles. All nonbiukrgic cliscase nrodilyittg before rinsing it off. Seborrheic dermatitis is believed to be
E
(g
antirheurtatic drugs (l)MAItl)s) should bc continued caused by heightened sensitivity to yeasts, such as Molos lrl
througlrout the periolrerative period in luticnts under sezia. lt is more common in patients with neurologic dis
o
B
vt
going elective j<linl relrlacenrent. l,lvidence sllggests that orders. such as Parkinson disease and Alzheimer dementia.
continuing t)l\4ARI)s is r.rssociatecl with a krwer risk ltrr Low potency topical glucocorticoids can be used in combi
infections vcrsus discorltinuing l)MAltl)s at the tinre of nation with antifungal treatments when severe inflamma
surgery. Diseasc flrres itrc also less fiec;uent rlter surgcry tion is present. Often, a low-potency topical glucocorticoid
in patients who continue I)MAltl)s periopcrativc'ly. Bio' is added for flares.
logic agents. such irs lclalimunrrtb (Option A), should be Clindamycin lotion (Option A) is often used to treat
withhelil fbr one closing cycle bcftrrc the surgery, witl.t acne and folliculitis, which are characterized by inflam
surgery pertbrnrecl at the cncl of thc clrclc. Jarrus kinase matory papules and pusfules; these are not present in this
inhibitors. such as tofacitinib. shoulcl bc withheld ftlr at patient. Clindamycin is not efl'ective for seborrheic derma
least 7 clays bclilrc sul-gcry. Once thc' opcralive wound titis.
shows evidcncc of hellirrg (typicxlly arour.tcl 14 days), Clobetasol ointment (Option B) is an ultra high
agellts that hirve been u,ithhel(l sl'roulcl bc rcsurned irt the potency topical glucocorticoid and would not be appropri
previously' establishccl dosc. ate treatment for this patient's seborrheic dermatitis. Fur
this patient has rvell controllcd disease. and procced thermore, use of ultra high-potency topical glucocorticoids
'lhe
ing u,ith elective knee replacenrcrlt surgery is acceptable. increases the risk for skin atrophy and should not be used
DMARI) lrethotrexate (Options C, D) shoulcl be contiuued on the face. A low potency topical glucocorticoid, such as
throughout the periopcrativc pc'riod, atrd adalinturnab, a hydrocortisone or desonide, can be used in combination
biologic usually dosetl every 2 wccl<s, should be ltcld fbr with topical antifungal agents for treatment of seborrheic
2 rveeks befbre the surgery itncl resut.tred rtpproximately dermatitis flares.
2 rvceks after sulger),. 5 Fluorouracil cream (Option C) is used to treat
actinic keratosis and is not appropriate treatment fbr this
f,EY POIf,IS
patient. 5 Fluorouracil cream often exacerbates seborrheic
. All nonbiologicdisease-modiffingantirheumatic dermatitis.
drugs should be continued throughout the periopera- Oral ketoconazole (Option D) is not an appropriate
tive period in patients with rheumatologic disease treatment for seborrheic dermatitis or any superlicial fun
undergoing elective arthroplasty. gal infection. In 2016, the FDA issued a boxed warning lor
. Biologic agents should be withheld for one dosing ketoconazole due to risk of liver and adrenal toxicity and
cycle preoperatively, before arthroplasty, with surgery interactions with other medications. The use of systemic
performed at the end of the cycle and resumed when ketoconazole is limited to the treatment of susceptible
systemic fungal infections (blastomycosis, histoplasmosis,
the wound shows evidence of healing.
paracoccidioidomycosis, coccidioidomycosis, and chro
momycosis) in patients in whom other, saf'er antifungal
Bibliography therapies have failed or who are intolerant to such ther-
(i, et al. 2017 American College of
Goodman SM, Springer B, Guyatt apies. Rarely, in refractory cases of seborrheic dermatitis,
Rheumatolops//American Association of Hip and Knee Surgeons guide
line fbr the perioperative mrnagement of antirheumatic medication in oral antifungals can be considered; in these cases, flu
patients witlr rheumatic diseases undergoing elective-total hip or total conazole would be preferred because of the lower risk for
inee arthroplasry Arthritis Rheumatol. 2oi7;69:1538 51. [PMID: 286209481
doi:10.1O02/art.40149 side effects.
171
Answers and Critiques
XEY POITTS for treating ED, tadalafil therapy should not be initiated
o in this patient without flrst tapering sertraline, a possible
Cost effective treatment of seborrheic dermatitis, a
causative agent, because there are no contraindications to
common condition characterized by greasy, yellow
doing so.
scaly, yellow to erythematous patches in seborrheic
areas, includes zinc pyrithione or selenium sulfide r(EY POtl{TS
shampoos. . All patients with erectile dysfunction (ED) must have
r Oral ketoconazole is not an appropriate treatment for a thorough medication history performed to deter-
any superficial fungal infection. mine whether any medications may be contributing
to or causing the ED.
l
172
Answers and Critiques
discarded. A new vaginal ring is inserted I week later. The interviewing can be particularly effective in establishing a
vaginal ring is as eflective as combined oral contraceptive partnership to create lifestyle changes and may be highly
pills in preventing pregnancy. The combined hormonal relevant in this patient, who is currently not motivated to
(estrogen progesterone) vaginal ring is not an appropriate make lifestyle changes.
option because estrogen is contraindicated in this patient. It may be desirable to increase this patient's medica
Although barrier contraceptive methods, such as male tions for diabetes in an attempt to lower the hemoglobin A,.
condoms (Option D), are safe fbr women who cannot take level. However, addressing only the pharmacologic aspects
estrogen and can provide protection fiom sexually transmit of this patient's care will not resolve the underlying issue
ted infections, when used alone they are considered one of of an unhealthy diet, lack of exercise, and ambivalence to
the least effective lorms of contraception. change. ln addition, pioglitazone (Option A), a drug that
increases the peripheral uptake of glucose, is associated
rEY POII{TS
with weight gain and is not the best choice for this patient.
o Contraindications to estrogen-containing contracep- Behavioral counseling with motivational interviewing and tt
€,
tives include migraine with aura, history of breast increasing the dose of metformin or liraglutide, if possible,
ET
cancer, venous thromboembolic disease, uncontrolled are better choices.
hypertension, or smoking more than 15 cigarettes a Using a multidisciplinary team of health care profes (J
day for women older than 35 years. sionals (dietitians, nurses, and psychologists) can be help ?r
r Non estrogen hormonal contraception options are ful when available. However, in the absence of motivation |E
to change, referring the patient to a dietitian or supervised vt
levonorgestrel-releasing intrauterine devices (lUDs), (l,
subdermal progestin implant, depot medroxlproges exercise program (Options C, D) is unlikely to be success
ful in achieving a healthier lif'estyle in this ambivalent vt
=
terone acetate injection, and progesterone only oral
patient.
contraceptive pills; a copper IUD is a nonhormonal
option. f,EY POttrs
o Motivational interviewing is a counseling approach
Bibliography that uses directive, patient-centered techniques; its
Woodhams EJ, Gilliam M. Contraceptbn. Ann lntern Med.2019;170:lTCl8
32. I PMID, 307167581 doi:10.7326lAl'l'C201902050
overall goal is to assist patients in addressing and
changing unhealthy behaviors.
o Foundational to motivational interviewing is an
Item 15 Answer: B attempt by the physician to understand the patient's
perspectives, followed by having the patient reflect on
Ed u catio n a I O bj ective: Use motivational interviewing
to help patients make lifestyle changes. the need for change by highlighting the discrepancy
between the patient's current and desired behavior.
The most appropriate initial management step is behav
ioral counseling that includes motivational interviewing Bibliography
(Option B). The basic elements in motivational interview Mifsud lL, Galea J, Garside J, et al. Motivational interviewing to support
ing include engagement, focusing, evoking, and planning. modifiable risk factor change in individuals at increased risk of cardio
vascular disease: a systematic review and meta analysis. PLoS One.
Motivational interviewing is a counseling approach that 202O;15:e0241193. [PMID: 331758491 doi:10.1371/journal.pone.0241l93
uses directive, patient centered techniques. Its overall goal
is to assist patients in addressing and changing unhealthy
16
behaviors. Motivational interviewing capitalizes on the
patient's intrinsic motivation to change and identifles rea-
sons fbr ambivalence to change and discusses them without
Item
Ed
Answer: D
ucational Objective: Diagnose Stevens-Johnson
tr
syndrome toxic epidermal necrofsis overlap syndrome.
directly confronting the patient. Foundational to motiva-
tional interviewing is the physician's attempt to understand Stcvcns Johnson synclrorne (S.lS) and toxic epidcnnril
the patient's perspectives, followed by having the patient necrolysis (TEN) (Option D) are thc rnost sL'vere ar-rcl deaclly
reflect on the need for change by highlighting the discrep of'the cutaneous adverse clrug rcactions. SJS/TEN overlup
ar.rcy between the patient's current and desired behavior. sl,nclronre rcfers to patients r,t,itlt lo'X, to 30'li, body surlircc
The physician attempts to build rapport and engage the arca aflected. Symptonrs bcgirt usr-rally lvithin 1 to 3 r,r'ccks
patient with a supportive, nonjudgmental attitude. The phy of erposure to iln inciting rgenl. such its carbantitzepitte.
sician elicits ideas and feelings about current behaviors u'ith fever and r.nalaise ftrllou'ed [.l1 skin pain. grittiness or
and how they match the patient's desires and values, and sirrrd like irritatioll of thc eyes. anci rtcll'nopl-ragia. Shortll'
has the patient reflect on the possibility of better options thereatter. p.rtients devclop rcd or purple dus\'macules on
compared with current behaviors. The conversation allows the trunk that progress to vcsicles. erosiort. and ulcerirti<tn.
identification of speciflc changes important to the patient. l'}ainlul erosions delelop in thc tnouth. e),es. or genitirls itt as
ln a process known as change talk, the physician elicits the nttny as 95')1, of patients. Mortllity is high, rvith 5')1, to 10'){, of'
patient's own reasons and rationale for change. Motivational SJS cases being fatal.
173
Answers and Critiques
tr
CONI,
Acute generalizecl exanthemiltoLls pustulosis (ACEI))
(Option A) refers to the rapicl onset ol a pustular rash af ter
a nreclication exposurc. C)r'rsct nray be as soon as 1 day alicr
Treatment may reduce transmissibility. 1he choice of
antibiotic is usually empiric. Because a course of a broad-
spectrum topical antibiotic is usually effective, the most con
cxl)osllre to the nrcdication rlr a fcw clays at most. Patients venient or least expensive option can be selected; there is no
present with fever and crythcnril and eventually devekrp clinical evidence suggesting the superiority of any particular
dense non {blliculocentric pustules, primarily in skin firkls antibiotic. Although there are no data supporting the cost-
and on the trunk. lhe onsct ol this patient's rash 12 days eflectiveness of using antibiotics in mild bacterial conjunctivitis,
illier initiation of carbirmazcpinc irnd lack of pustules arguc the shortened morbidity associated with their use makes
against ACEP choice of therapy an individual decision. Trimethoprim
I)rug induced hvpcrscnsitivity' syndrome (l)lllS) polymyxin ophthalmic solution is an appropriate antibi
(Option B) is also a severc litc thrcatening medication rclc otic option for patients, such as this one, who do not wear
tion. Unlike other scvcrc rnedication rcaclions. the onsct contact lenses. Other reasonable options are erythromy
D ol symptoms is delayed. oltcn 2 to 6 wecks after exllosurc. cin or bacitracin-polymyxin ophthalmic ointments. Lack
UI Symptoms begin with fcvcr and flulikc synrptoms. cluickly of symptom improvement or symptom worsening within
{
.D filllowecl by burning skin pain arrcl rash. Palienls typically several days of initiation of an appropriate topical antimi
UI clcvelop a morbillilbrnr exanthenr that starts on thc facc crobial should prompt ophthalmology referral.
9, ancl upper trunk and sprcads <listally. l'iventually. the patient Because of concern about antibiotic resistance and
CL clcvelops striking facial eclenrir ancl redness, a hallmark of' higher cost, ofloxacin ophthalmic solution (Option A)
rl I)lllS. Oral nrucosal involvenrerrt is common.'[he onset ol should be reserved for patients who wear contact lenses,
ll this pirtient's rash l2 da-vs lller initiation of carbamazepinc which increases the risk for pseudomonal infection.
and lack of a morbilliftrrnr rash ancl fircial edema trrgue Topical antihistamines, such as olopatadine (Option B),
o
UI agrrinst DIHS. have no role in the treatment of acute bacterial conjunctivi
lrrytl.rroderma (Option C) is clcfined as diffuse erythenra tis. Olopatadine is commonly used in patients with allergic
covcring B0')1, to 90'7, bocly surllce arca and is comnronly conjunctivitis, who typically present with bilateral eye red
associated with pruritus, pcripheral eclema, erosions, scal ness, pruritus, watery discharge, and eyelid swelling.
ing, and lymphadenoprrthy. 'lhe lrost common causes flrc Topical ophthalmologic glucocorticoids, such as pred
idiopathic (up to 40'X,). exacerbrrtion of a preeristing rash. or nisolone alone (Option C) or in combination with an anti
mcdicatior.r rcaction. liris 1'uticnt's illr.ress is not manifcstccl biotic. do not have a role in the treatment of acute bacterial
l.ry widespread erythcma. conjunctivitis. When given to patients with either bacterial
or viral keratitis, they can lead to corneal damage, including
f,EY POIilI
perloration and scarring.
o Stevens-Johnson syndrome/toxic epidermal necrolysis
overlap syndrome begins 1 to 3 weeks after exposure t(tY PoltTS
to an inciting agent and is characterized by a wide r Mild bacterial conjunctivitis is typically a self-limited
spread mucocutaneous reaction that progresses to condition, but topical antibiotic treatment may
vesicles, erosions, and ulcerations. shorten the duration of symptoms and transmissibil-
ity and is a reasonable option in some patients.
Bibliography o The choice of a topical broad spectrum antibiotic for
Noe Mll, Micheletti RG. Diagnosis and management of Stevens Johnson
bacterial conjunctivitis is usually empiric; the most
syndrome/toxic epidermal necrolysis. Clin Dermatol. 2020)8:607 12.
I PMID: 333411951 doi:10.lol6lj.clindermak)1.2020.06.016 convenient or least expensive option can be selected.
Bibliography
Item 17 Answer: D Varu DM, Rhee MK, Akpek EK, et al;American Academyof Ophthalmologr
Preferred Practice Pattern Cornea and External Disease Panel.
Educational Objective: Treat acute bacterial Conjunctivitis Preferred Practice Pattern . Ophthalmology. 2019;
conjunctivitis with a topical antibiotic. 126:P94 169. [PMID: 303667971 doi:l0.l0l 6/j.ophtha.2018.1O.O2O
174
Answers and Critiques
l'll gender related history (hormonal, surgical, social, andsexual) Actinic keratosis (Option A) is a precancerous lesion
(Ontions A, B) and perfgrming a genital examinarion
lff
'"''' (Option C) are unnecessary. ln acldition, these interventions
that can potentially develop into squamous cell carcinoma.
It presents as a thin papule or plaque with scale that is often
may make this patient feel uncomfortable and potentially more easily felt than seen. This patient's rapidly evolving
dissuade him from returning fbr important ongoing health papule with a keratin-fllled center is not compatible with
care. A comprehensive history of a transgender person is actinic keratosis.
usually not possible to obtain in one visit; it is best obtained Basal cell carcinoma (Option B) is a malignant neo
over time in order to build rapport with the patient. In plasm arising from the basal layer of the epidermis. There
general, history taking lor transgender persons is the same are different histologic subtypes, including superflcial,
as for all patients and includes fantily, reproductive, sexual, pigmented, sclerotic, and nodular, that result in varying
psychiatric, and social histories. Elernents of the history that clinical appearance. Nodular basal cell carcinoma is the most
are unique to the transgender population are hormonal and common type, classically appearing as pearly papules with
surgical therapies related to gender transition. rolled borders and arborizing telangiectasias. Frequently, a ur
Recommendations fbr sexually transmitted infection central ulceration may appear that is similar in appearance
o
(STl) screening (Option D) are the same fbr transgender to the crater seen in keratoacanthoma (as shown). ET
patients as fbr all patients. Screening should account fbr
the patient's anatomy and sexual history. Just as it would be
U
!,
inappropriate to perform STI screening in a nontransgender .g
patient during a first time visit fbr unrelated episodic care. tt
it would also be inappropriate to screen this patient today. (l,
STI screening is important ancl shouid be performed; how v!
=
E
ever, it can wait until patient rapport has been established
and a more detailed history has been obtained to guide
screening.
Many online resources are available fbr learning about
transgender persons and providing culturally sensitive
medical care. The University of' Califbrnia, San Francisco,
has published guidelines fbr primary and gender-affirming
I care of transgender and gender nonbinary persons at http://
transhealth.ucsfledu/protocols. ln addition, the National
Lesbian, Gay, Bisexual, and 1'ransgender (LGBT) Health
However, basal cell carcinomas do not have the keratin-
Eclucation Center, a program of the Fenway Institute, pro
filled center and typically do not develop as rapidly as this
vides learning modules at wwwlgbthealtheducation.org/
patient's lesion.
lgbt educationiiearning modules/.
Pyoderma gangrenosum (Option D) presents with
I an exquisitely tender papule, pustule, or nodule. The area
o Physicians should provide care for all patients in a quickly enlarges and begins to ulcerate in a cribriform pat-
sensitive, respectful, and affirming manner. tern, with intervening strands of epithelium. There is a char-
t acteristic violaceous border with an overhanging epithelium
Bibliography around the central exudative ulcer, which is often described
I
cell carcinoma. After rapid growth, some keratoacanthomas o Keratoacanthomas present as a pink nodule with
tend to involute. Because keratoacanthomas are difficult to a keratin-filled center ("volcaniform" appearance),
i differentiate from cutaneous squamous cell carcinoma, they are rapidly growing and frequently painful, and
I are often treated with surgical excision. For this patient with
are considered a variant of squamous cell
a lesion on the dorsal right hand, Mohs micrographic surgery
carcinoma.
would most likely be performed.
175
l
L
Answers and Critiques
176
Answers and Critiques
[l lrreoperatirr cvlluatior.r. ir.rcluding in pltients rt,ith COp]) recommends against screening low-risk and asymptomatic
lfl (,rs is the cirsc lirr this pltient). I,'urthermorc, eviclence adults with resting or exercise ECG.
CoNI d1;er not support
:i spirontetric thresholcl below which the Cardiovascular disease is the leading cause of morbidity
risk of surgery is unacccptable ar.rcl spironletrv is rl poor and mortality in the United States, accounting for 1 of every
predictor of ntorl)idit), or ntortalilri Spirometry ntight be 3 deaths among adults. The rationale for calculating cardio
helpful in patierrts with oltstructive airways disease il there vascular risk is that some asymptomatic individuals between
is suspicion that the patient is not optimally trcated; the 40 and 75 years of age without a history of cardiovascular
presence of reversible airfkx,r,obstruction suggests tlte need disease may have undetected atherosclerotic changes and
ftrr nrore aggressive therapv may be candidates for preventive interventions. Assessing
XEY POIilTS cardiovascular risk to identi$r persons that will beneflt from
prevention strategies is reasonable (Option E).
. Frailty is a multifactorial state of decreased physio-
logic reserves and increased vulnerability to stressors. f,tY P0rtTs
(^
r Frailty is associated with perioperative complications, . The U.S. Preventive Services Task Force recommends o
prolonged hospital stays, and increased mortality. calculating the cardiovascular disease risk in adults ET
aged 40 to 75 years using the Pooled Cohort
rr,
Bibliography Equations. !,
Alvarez Nebreda ML, tsentov N, Urman RD, et .rl. Recommendations for o The U.S. Preventive Services Task Force does not rec- |g
preoperative mtnagement ol tiailty from the Society fbr perioperative tt
Assessment and Quality lmprovement (SpAel). J Clin Anesth. 2018;47: ommend screening for coronary artery disease with (l,
il:l 42. IPMtD: 29550619] doi: 10. l0l6/j.jc1inane.2018.02.011 either resting or exercise ECG in asymptomatic patients
t!
=
at low risk. tr
177
Answers and Critiques
screening chest radiography lead to changes in perioper- to the deep red color with fine lamellar scale seen in this
]!t
El 21ivg management. ancl abnormal findings can usually be patient. Although lichen planus can afi'ect the nail, it tends to
coNl preclicted on the basis of clinical examination. lt rnay be cause thinner, atrophic nails with red streaks or pterygiunl
reasonable. but not necessary. to obtain chest radiographs unguis (dorsal pterygiurn). wl.ricl.r mirnifests as a triangular
in patients older than 70 vears rt'ith chronic cardiopulmo- plate d1'strophl:
nary disease if they have not had chest radiograplly in the Pemphigus vulgirris (Option D), an autoimmune blis
previons 6 months. Preoperative chest radiography may be tering disease. presents with erosions rather than the scaly
indicated in patients unclergoing thoracic or nlediastinal plaques seen in this patient. In acldition. pemphigus vulgaris
surgery which this patlent is not. aln'ays involves the or:.r1 mucosil: this patient's nlucosa is
Although spirometry (Option C) may help delineate normal.
progression of airflow obstruction due to worsening COPD. Stevens-Johnson syndrome/toxic epidennal necrolysis
results do not change perioperative nlanagement in patients (SlS TEN) (Option E) preser-rts as purple duskl' macules on
such as this <,rne. There is no absolute value of FEV, or FEV, the trunk that progress to vesicles, erosions. ar.rd ulceration.
ut t VC that precludes surgery. nor do these values help predict Painlul nrucosal erosions develop in as nlanl' as 95'1, of
€
.D
perioperative pulmonary complications. patients. This patient's diltuse erythroderma is inconsistent
t \\'ith SJS ]'EN.
o, TEY
EL . 'OIXII
Patients with COPD should be screened preopera-
TEY POIilIS
n tively for signs and symptoms of COPD exacerbation. . Erlthroderma describes the condition of erythema
. covering greater than 80% body surface area.
.lt Chest radiography, arterial blood gas analysis, or
E
(D spirometry is not routinely indicated in the preopera . Up to 40% of cases of erythroderma are idiopathic;
UI
tive evaluation of patients with chronic lung disease. exacerbation of a preexisting rash (e.g., psoriasis,
atopic dermatitis) and medication reaction are other
Bibliography common etiologies.
Misk<.rvic A. Lunrb AB. Postoperative pulmonary complications. Br J
178
Answers and Criti ques
179
Answers and Critiques
effective treatment for premature ejaculation but is associ- low dose CT scan for persons aged 50 years to 80 years with
ated with more side ellects than monotherapies. at least a 2O-pack-year smoking history and who are still
Clomipramine (Option A), a tricyclic antidepressant smoking or who quit within the past 15 years. Screening
(TCA) with serotonergic activity, is effective for premature should be discontinued once a person has not smoked for
ejaculation. It is considered second line therapy, however, 15 years or develops a health problem that substantially
because TCAs have more side effects than SSRIs. Clomip limits life expectancy or the ability or willingness to have
ramine can be used when SSRIs are either ineffective or not curative lung surgery. Shared decision making, including
tolerated, but it would be inappropriate to initiate clomip providing patients with information about radiation expo
ramine in this patient before a trial of paroxetine. sure with CT, risk for false positive diagnoses, and anxiety
The squeeze or start stop technique (Options C, D) related to surveillance of slow growing lung nodules, is a
should not be considered in place of starting fluoxetine. crucial element in making the decision to screen. Screening
The squeeze technique involves squeezing the glans of alone cannot prevent most lung cancer related deaths, and
the penis when nearing orgasm until the urge to ejac- smoking cessation is essential. This patient has a significant
vt ulate subsides. Developed in the 1950s, the start-stop smoking history within the past 15 years and should be
E technique involves stimulating the penis until the patient ollered screening.
.D
t feels the urge to ejaculate and then stopping until the urge Chest radiography (Option A) is not an effective means
q,
subsides. This process is repeated multiple times until of screening for lung cancer. Multiple studies have investi
CL ejaculation. Although initial studies suggested that both gated chest radiography screening in patients at risk for lung
ft methods were effective, subsequent evidence has been cancer, and none has demonstrated a mortality benefit.
less convincing. One time screening with low dose CT (Option C) is not
lt
C
.D
A topical anesthetic agent (Option E) made of 2.5't the optimal strate$/ because continued annual screening
Ut prilocaine and lidocaine is an eII'ective therapy, resolving is associated with the discovery ol cancers that were not
premature ejaculation more quickly than an SSRI, and can detected on the initial screen. Most guidelines recommend
be used on an as needed basis for patients who do not want annual screening until at least the age of 74 years or until
to take a daily oral medication. However, in this patient with at least 15 years has passed since the patient quit smoking.
depressive symptoms, paroxetine is a better choice. Not screening this patient for lung cancer is not the best
t(tY Pot ltTs strates/ (Option D). Annual screening with low-dose CT in
patients with a significant past or current smoking history
o Premature ejaculation is defined as ejaculation with results in a 20'1, to 24'l. reduction in lung cancer mortality.
minimal stimulation that occurs earlier than desired:
treatment is indicated if associated with psychological f,EY POIIITI
distress. . The U.S. Preventive Services Task Force recommends
o Paroxetine monotherapy or combined with a phos- lung cancer screening with an annual low-dose CT of
phodiesterase 5 inhibitor is considered first-line the chest for persons aged 50 years to 8O years with at
treatment for premature ejaculation; complete thera least a 20-pack-year smoking history who are still
peutic effect is reached within 2 to 3 weeks of use. smoking or who quit within the past lS years. t
o Screening alone cannot prevent most lung cancer-
Bibliography related deaths, and smoking cessation is essential. l
Liu H, Zhang M, tJuang M, et al. Comparative efficacy and safety of drug
treatment for premature ejaculation: a systemic review and-Bayesian
network meta-analysis. Andrologia. 2020:52:el3806. [pMtO, ffegZaZq] Bibliography
doi: 10.1 111/and. 13806 Krist AH, Davidson KW Mangione CM, et al: US preventive Services Task
Force. Screening for lung cancer: US preventive Services Task Force rec
ommendation statement. JAMA. 2021:32S:qOZ ZO. [pMIO, 336g7470]
doi:10.1001/jama .2O2t.1|7
Item 28 Answer: B
Educational Objective: Screen for lung cancer in at_risk
patients. Item 29 Answer: C
The most appropriate lung cancer screening strates/ for Educational Objective: Diagnose nail psoriasis.
this patient is annual low dose CT of the chest (Option B). The most likely diagnosis is nail psoriasis (Option C). Nail
Smoking is the most important risk factor for the devel_ psoriasis is characterized by dystrophic nails with
opment of lung cancer, and risk increases with additional vellow_red
(oil stain) discoloration caused by inflammation of the nail
years oftobacco exposure and age. Lung cancer has a poor
bed; pitting caused by inflammation of the nail matrix: and
prognosis, and nearly 90%, ofpersons with lung cancer
die distal onycholysis (separation of nail plate from the nail
of the disease. However, early-stage non-small cell lung can_
bed). Other findings include nail plate thickening, distal nail
cer has a better prognosis and can be treated with surgical plate crumbling, and splinter hemorrhages. Nail psoriasis
resection. The U.S. preventive Services Task Force guide
more commonly affects the fingernails than the toenails.
lines recommend lung cancer screening with an annual
Nail psoriasis typically occurs in patients with psoriasis, but
180
Answers and Critiques
occasionally the nail changes may precede the skin changes. Item 30 Answer: B
A family history of psoriasis can be helpful, as can a through
physical examination looking for "hidden" areas of plaque
Educational Objective: Obtain appropriate preoperative EI
laboratory testing.
psoriasis in the gluteal cleft, periumbilically, in the concha of
the ear, and on the scalp. Nail psoriasis is strongly associated Ihe most appropriate preoperative laboratory testing fbr this
with psoriatic arthritis. Patients with psoriasis, particularly patient is hemitglobin and serum creatinine lreasurement
involving the nails, should be screened for psoriatic arthritis (Option B). Preoperative laboratury testing is primarity driven
at diagnosis and follow up. by patient related factors. Overtesting in tl-re preoperative
Paronychia (Option A) is an infection of the nail fold. period is comnton and leads to higher costs and additional
Acute paronychia is characterized by painful swelling of the downstream testing, which can cause harm. Preoperative
nail fold, most commonly caused by Staphylococcus eureus. hemoglobin testing is indicated in r patient with signs or
It typically affects only one nail. Chronic paronychia tends to symptoms suggestive ot' :rnemia, recent blood loss, use of'
be more insidious and involve multiple flngers. In adults, it is medications tl.rat could aflbct hematopoiesis, or anticipated rtt
q,
most often seen in those with frequent hand immersion in significant blood loss. Patient relatecl f'actors that may drive
water. There is tender swelling in the nail folds, with missing preoperilti\e semm creatinine testing include hypertension,
or dystrophic cuticles. Chronic paronychia can cause ridg- chronic kidney disease, diabetes, cardiac disease. ancl mecl
ing of the nail plate. These flndings are not present in this ications that afl'ect kidney function. Total hip arthroplasty
t,
T'
patient, making chronic paronychia an unlikely diagnosis. is associated with significant btood loss. and establishing r!
Lichen planus (Option B) can affect the nails in about the patient's baseline hentoglobin level will be helpful. This tn
10'2, ofcases. It causes nail plate dystrophy, including longi- patient has hypertension and is taking lisinopril, both ol {t
tudinal roughness and ridging, nail thinning, red streaking, which can alter kidney function, and serum creatinine mea rn
=
and pterygium unguis formation (scarring of the proximal suremellt is indicated.
nail fold and matrix), as shown. ln all patients undergoing surgery, a careiul preopera
tive bleeding history. including a farnily history. should be
obtained. Coagulation studies (Option A) are indicated in
patients taking anticoagulants, those with known or sus
pected liver dysfunction, or those with a personal or family
history suggestive of abnormal bleeding. A platelet count
is reasonable in patients with a history olbleecling cliathe
sis, those with a myeloprolilerative disorder, those taking
myclotoxic medications. or those with cirrhosis.'Ihis patient
does not have indications fbr coagulation studies or platelet
count.
This patient has been receiving thyroid hormone
replacement, ar-rd her thyroid-stimulating hormone (TSH)
level was normal 2 months ago. Patients whose test results in
the past 4 months were normal and have a stable clinical sta
tus do not need repeated testing befbre surgeryi a fiee serum
This patient's findings are inconsistent with nail lichen thyroxine level is Llnnecessary in a patient with a normal
planus. TStl level and on a stable dose of levothynrxine (Option C).
In patients with onychomycosis (Option D), the toenails There is little evidence that treatment of asymptonl
are more commonly affected than fingernails. This patient's atic bacteriuria will prevent postsurgical prosthetic joit.tt
toenails are not involved. Nail pitting is not seen in onycho infection, and it is not indicated. Inappropriate treatment of
mycosis but is present in this patient's nails. At times, it can asymptomatic bacteriuria is a ntajor driver of antimicrobial
be difficult to exclude onychomycosis by physical examina resistance, particularly in health care facilities. 'l'reatment o1
tion alone. In such cases, potassium hydroxide preparation, asymptonlatic bacteriuria is, however, indicated in patients
periodic acid-Schiffstaining, or f'ungal culture is indicated. scheduled to undergo an invasive procedure inv<tlving the
urinary tract. Urinalysis (Option D) in this asymptomatic
f,EY POIilT
patient undergoing orthopedic surgery is unnecessary.
. Nail psoriasis, which more commonly affects the fin
gernails than toenails, is characterized by dystrophic f,tY POlf,rs
nails with yellow-red (oil stain) discoloration, pitting, . Urinalysis is not indicated before joint arthroplasty in
and distal onycholysis.
asymptomatic patients.
Bibliography
r Patientswhose laboratorytest results in thepast 4 months
were normal and whose clinical status is stable do not
Hinshaw MA, Rubin A. Inflammatory diseases of the nail unit. Semin Cutan
Med Surg. 2015;34:109 16. [PMlD, 261762891 doi:10.12788/i.sder. need repeat testing before surgery.
2015.0132
181
Answers and Critiques
Bibliography Bibliography
Edwards AF. R)rest DJ. Preoperative laboratory testing. Anesthesiol Clin. Arterburn D. \\rellman R. Emiliano A, et al: PCORneI Bariatric Stud)
2018::16:493 507. IPMID: 303907711 doi:10.1016/i.anclin.20l8.07.002 Collaboratire. Comparative effectiveness and salety of bariatric proce
dures fbr $eight loss: a PCORnet cohort stud): Ann Intern l!4ed' 2018:
169 :7 41 50. [PM D : 301]831391 doi: Io.7 326 t Ml7 27 86
I
Item 3! Answer: D
Educational Objective: Treat obesity with sleeve
gastrectomy.
Item 32 Answer: A
Educational Objective: Evaluate a patient with
tr
pulmonary hypertension prior to surgery.
The most appropriate treatment for this patient is sleeve
gastrectomy (Option D). Guidelines recommend bariatric The most appropriate preoperative management of this
surgery for patients with a BMI of 40 or greater or for those patient is to cancel surgery (Option A) and refer the patient
with a BMI ol 35 or greater who have at least one serious for eraiuation by a pulmonary hypertension (PH) specialist.
weight related comorbid condition, such as type 2 diabetes In the perioperative period. PH. defined as pulmonary artery
UI
E mellitus, obstructive sleep apnea, or knee or hip osteoar pressure (PAP) greater than 25 mm Hg, is associated uith
(D
(/r
thritis. Sleeve gastrectomy is accomplished by excising the mortaliry rates of 4')1, to 2,1')1, and significant morbidi[.in up
o, part of the stomach along the greater curvature, creating to 42"/.' ol patients. Potential complications ol PH include
CL an approximately 85')(, reduction in size of the stomach. It m1'ocardial infarction. arrhl'thmias. right ventricular (R\')
r.l results in restriction of caloric intake via early satiety with failure. venous thromboembolism. and pulmonary failure.
a smaller stomach and hormonal (glucagon like peptide I Patients rvith high risk PH features. including group 1 PH
st and related hormones) appetite suppression. The smaller (pulmonary arterial hypertension), PAP greater than 70 mn.t
(D gastric surface area als<-r results in less production of ghrelin,
t^ Hg. and moderate or selere RV dysfunction. should undergo
an appetite stimulant. Roux-en Y gastric bypass (RYGB) a thorough preoperative risk assessment by a PH specialist.
surgery has the most efficacy data and remains superior to Surgery should generally be avoided in patients n'ith severe
other procedures for weight loss. but it also has the highest PH. This patient has severe COPD and heart failure nith
rate of early postoperative complications. RYGB consists of reduced eiection fraction with evidence of PH and moder-
creating a small proximal gastric pouch separate from the ate to severe right ventricular dystunction. She should be
distal stomach and creating a biliopancreatic limb that con referred to a PH specialist for evaluation prior to the electire
nects the Roux limb to the gastrojejunostomy. The weight procedure.
loss with sleeve gastrectomy is less than RYGB surgery. Although increasing this patient's furosernide (Option
Sleeve gastrectomy, however, is associated with f'ewer major B) may be indicated for heart iailure and volume overload.
surgical complications at 30 days than other fbrms of bar changes to the medication regimen should be made as part
iatric surgery. After 5 years, the two bariatric procedures do of an overall treatment strate$,' and not as part of a preop
not differ in regard to health related quality of lif'e or major erative evaluation. Decisions regarding medication manage
complications. ment in the perioperative period in this patient should be
Gastric banding (Option A) is often complicated by made in conjunction with a Pt{ specialist.
band erosion and slippage. Due to complications and poor Most cases of PH are caused by left-sided heart disease
efficacy, sleeve gastrectomy is preferred. and hypoxic respiratory disorders. Treatment is directed
The FDA has approved intragastric balloons (Option B) toward the underlying disorder. Patients n'ith group 1 PH
and vagal blockade devices, which resulted in modest weight (u'hich includes idiopathic and heritable pulmonary arterial
loss in trials. However, long-term safety and elficacy data hlpertension, and disease related to drugs and toxins. connec
are lacking, and most insurance companies classiff these tive tissue diseases. HIV infection, schistosomiasis. and portal
devices as i nvest igationill. hypertension) may be treated n,ith a calcium channel blocker
This patient is taking liraglutide. which promotes or \asodilator therapli such as tadalafil (Option C) tbllou'ing
weight loss, and has tried two different weight loss medi \asoreactivity testing. Starting tadalafil is not indicated.
cations with modest initial success but experienced weight Chronic hypoxia $,ith resultant pulmonary \asocon
gain upon their discontinuation. 'this is common fbr patients striction may be a major contributor to a patient's PH. but
who take weight loss medications. Orlistat (Option C) is less further evaluation is necessary prior to instituting continu
eflective in achieving target weight loss than the weight loss ous orygen therapl' (Option D).
medications that this patient has already tried. Provided that
this patient has no contraindications to bariatric surgery TEY POITI
sleeve gastrectomy is an appropriate choice. . Patients with high-risk pulmonary hypertension
undergoing noncardiac surgery have high risk for
I(EY POITT
complications, including myocardial ischemia, venous
o Sleeve gastrectomy is associated with less weight loss thromboembolism, cardiogenic shock, and dysrhyth
compared with Roux-en Y gastric bypass but fewer mias, and should be evaluated by a pulmonary hyper-
30-day postoperative complications. tension specialist before undergoing surgery.
182
Answers and Critiques
Bibliography Bibliography
Aguirre MA, Lynch I, Hardman B. Perioperative management of pulmonary American College of Obstetricians and Gynecologists. ACOC crrmmittee opin
hypertension and right ventricular failure during noncardiac surgery Adv ion no. 762 summary: prepregnancy counseling. Obstet Gynecol. 2019;
Anesth. 2018;36:201 30. IPMID: 30414638] doi:10.1016/j.aan.2018.07.011 133:228 30. IPMID: 30s7s672] doi:10.1097/AOC.0000000000003014
Item 33 Answer: D
Educational Objective: Provide preconception care for
Item 34 Answer: C
Educational Obiective: Diagnose pemphigus vulgaris.
tr
an average-risk woman. The most likely diagnosis is pemphigus vulgaris (Option C),
an autoimmune bullous disease (ABD). ABDs are caused by
The most appropriate management is to obtain a varicella
antibodies interfering with cohesion between keratinorytes
antibody titer (Option D). All women considering pregnancy
of the epidermis (desmosomes) or between the epidermis
should be assessed for immunity to varicella and rubella.
and dermis (basement membrane zone). ABDs can be sub- v!
This patient reports receiving the measles, mumps, and (u
divided into intraepidermal and subepidermal disorders.
rubella vaccine during childhood but is unsure whether she ET
Intraepidermal ABDs present with flaccid vesicles that rup
had varicella (chickenpox). She should have a varicella anti- .=
ture easily, whereas subepidermal ABDs show intact, tense rr,
body titer test. If she is not immune to varicella, she should
bullae. Pemphigus vulgaris is the most common intraepi
receive the varicella vaccine and be instructed to wait 4 weeks =t
dermal ABD, and its incidence increases with age. It presents lE
before attempting to conceive. If an already pregnant woman UI
with flaccid oral or other mucosal bullae that rupture easily
is not immune to either varicella or rubella, these vaccines (u
and leave erosions. Lesions heal with pigment change but
should be administered after delivery but before hospital la
otherwise do not scar. This patient has an acute onsct of a =
discharge. Immunizations to be avoided during pregnancy
blistering disorder. Pemphigus vulgaris always involves the
include Iive vaccines (varicella, rubella, measles, mumps,
oral mucosa and may affect the skin in varying degrees.
live attenuated influenza, live attenuated herpes zoster) and
Bullous pemphigoid (Option A) is an ABD that f'eatures
human papillomavirus (HPV) vaccine.
tense bullae. Although bullae may rupture and result in
All pregnant women should receive a tetanus toxoid,
erosions, typically several bullae are present on examina-
reduced diphtheria toxoid, and acellular pertussis (Tdap)
vaccine (Option A) between 27 weeks' and 36 weeks' gesta
tion (as shown). Pruritus is the predominant symptom in
bullous pemphigoid.
tion with each pregnancy, helping to ensure immunity to the
child upon birth. Administering Tdap during pregnancy is
recommended regardless of when a patient received her last
tetanus vaccination. This patient is currently not pregnant
and should not receive the Tdap vaccine at this visit because
she is otherwise current with this vaccination.
A nucleic acid amplification test for chlamydia (Option B)
is not indicated for this patient. Chlamydia screening of all sex
ually active women younger than 25 years is recommended.
Women aged 25 years or older should be screened if they have
sexually transmitted infection (STI) risk factors, such as new
or multiple sex partners, sex partners who have multiple other
partners, no or inconsistent condom use (outside a monoga
mous partnership), previous STI, current STI, or exchanging
sex for money or drugs. This patient is not high risk, and
although screening for STIs is considered routine during preg
nancy, it is not necessary during preconception care.
A Pap smear is not indicated (Option C). the patient's Dermatitis herpetiformis (Option B) presents with
last Pap smear was obtained 1 year ago and was normal; HPV clustered fragile vesicles that break down quickly, leaving
testing was negative. Women aged 30 to 39 years who have small erosions on the elbows, knees, or buttocks. It intensely
a normal Pap smear and a negative HPV test result do not pruritic and does not involve the oral mucosa.
require another Pap smear for 5 years. Although Stevens Johnson syndrome (SJS) (Option D)
and toxic epidermal necrolysis (TEN) (Option E) should
IEY be considered in any patient with desquamative lbaturcs
r AII'OIilIs
women considering pregnancy should be assessed in the acute setting, several aspects ol this patient's pre
for immunit5r to varicella and rubella. sentation make these life threatening conditions unlikely.
o If an already pregnant woman Patients with SIS and TFIN present with significant skin
is not immune to either
pain (even in unaffected areas), malaise, and fever and typ
varicella or rubella, these vaccines should be adminis-
ically appear toxic. Furthermore, patients with SJS and l'EN
tered after delivery and before hospital discharge.
present with two mucosal surfhces involved, whereas this
183
Answers and Critiques
fiE prrticnt has onll'one involr,ed mucosal surface: the mouth. The Weber test, because it bypasses the tympanic
515 or TEN occurs secondary to medication or infectior.r. membrane and ossicles, tests sensorineural hearing. In this
coNr
typically within 2 weeks ol adrninistration, usually shorter. patient, the Weber test lateralized to the right ear (sound was
Although this patient has a history <tf doxycycline use, it louder in the right ear); this ear has normal sensorineural
wirs started 6 months ago. not rt,ithin the 2 week wir.rdow function (Option D). The sound was heard less well in the
typical fbr SJS and TEN. ear with impaired sensorineural flunction, the left ear.
lt
Item 36 Answer:
Educational Obiective: Treat herpes
B
zoster.
tr
(E
vf
Item 35 lnswer: C
-lhe
most appropriate treatnrent is val:rcyclovir (Option B) for
Ed u cationa I Objective : Diagnose sensorineural hearing
this patient lvith herpes zoster infbction. lnitiation oi systen-ric
loss.
antiviral therapy (acyclovir. valacyclovir, or famciclovir) can
The most likely cause of hearing loss in this patient is senso help attenuate the duration ol rash and symptoms during a
rineural hearing loss of the left ear (Option C). Hearing Ioss herpes zoster outbreak and reduce the risk for postherpetic
is categorized according to the anatomic deflcit: conductive, ncuralgia if administered n,ithin the first 72 hours after onset
sensorineural, or mixed. Conductive hearing loss is more of symptoms. Valacyclovir and famciclovir are often preti:rred
often associated with pain or ear drainage, whereas senso- because of possible increased efficacy and decreased fre
rineural hearing loss is more often accompanied by tinnitus quency of dosing compared with acyclovir. This eflbct is best
or vertigo. In patients with hearing loss, physical examina established in patients older than .50 years. Varicella zoster
tion should include otoscopic examination as well as the vims is :r DNA virus that causes varicella (cl-rickenpox). r,r'hich
Weber and Rinne tests. The Weber test assesses sensorineural is irn ncute illness u,ith fbver and an eruption of 'nesicles on an
hearing. In the Weber test, a 256 Hz vibrating tuning fork is crythematous base that is transmittecl by respiratory droplets.
applied to the midline forehead, bridge of the nose, or teeth. Alier primary infection. the vims remains latent in the dorsal
A normal result is no lateralization of loudness of the sound root or cranial ganglia. Reactivatir-rn causes hetpes zoster.
(sound loudness equal in both ears). In the presence ofsen Proclromal symptoms, such as trurning. stinging, or tingling,
sorineural hearing loss, the sound lateralizes (is louder) in the ollen occur in a localized regior.r, firllowed by a dermatonral
uninvolved ear. In this patient, the sound was louder in the emption of'grouped vesicles or pustules on an erythenlatous
right ear; thus, the left ear is impaired. The Rinne test assesses birse. The most common dernrator-nes afl'ected are in the tho
conductive hearing. In this test, the patient is asked to identiff racic region. With inohenrer.rt of'the first division of the tri
whether the tuning fork sound is louder when placed on the gen.rinal nen'e (forehead extending or,er upper elelid, or n:rsal
mastoid process (bone conduction) or when placed in front of tip invol'uement). ophthalnrologic evaluation is mandatory
the extemal auditory canal (air conduction). A normal test is beciruse herpes zoster opl.rthalmicus and possible blir.rdness
associated with air conduction louder than bone conduction. can result. If vesicles are noted in the e.xternal ear canal.
This patient, with a Weber test that lateralizes to the right ear evaluation by an otolaryngologist may be required because
(the "good ear") and normal conduction in the left ear (air peripheral facial paralysis and auclitoryivestibular symptonrs
conduction greater than bone conduction) has sensorineural ciln occur (Ramsay Hunt syndrome).
hearing loss of the Ieft ear. The most concerning condition Whereas systemic acvclovir is an eff'ective option fbr
causing chronic unilateral sensorineural hearing loss with shortening symptoms ancl decreasing the likelihood ot
tinnitus is an acoustic neuroma. The flrst diagnostic step is postherpetic neuralgia. topical ircyclovir (Option A) or
confirmation of the hearing loss with audiometry followed by topical penciclovir is ineflective and should not be used.
MRI of the internal auditory meatus. Gabapentin may be useful in alleviating the pain associ
This patient has normal conductive hearing in both the atcd lvith post-herpetic neuralgia once it has become estab
right and left ears (Options A, B) as confirmed by the bilat lished, but there is no evidence that gabapentin plus an
erally normal Rinne test (air conduction greater than bone antiviral decreases (Option C) the likelihood of developing
conduction). postherpetic neuralgia. As is the casc with glucocorticoicls,
184
Answers and Critiques
tr
CONI.
gabaper-rtin may be used as an adjunctive therapy in patients
with severe pain with acute neuritis.
Although commonly prescribed in the past. evidence is
Attacks ol acute ACG more commonly occur in the
evening. lthen lower light Ievels cause mydriasis and fold
ing back ofthe iris. blocking the narrow angle. Risk factors
insulflcient to support the addition ol glucocorticoids. such for acute ACG include older ager lemale sex; hyperopirr:
as prednisone (Option D), to antiviral therapy to reduce the Asian ethnicity; and receipt ofcertain drugs, such as selec
synrptonls of herpes zoster or the incidence of postherpetic tive serotonin reuptake inhibitors. Common examination
neuralgia. Postherpetic neuralgia is defined as chronic neunr findings include a mid dilated, minimally responsive
pathic pain that persists for 3 months or more after tl.re initial pupil; conjunctivaI injection; decreased visual acuity; and
onset of herpes zoster. Glucocorticclids mly be consiclered as globe tenderness kr palpation. l.'unduscopic examination
adjunctivc therapy in patients with severe acnte neuritis. but reveals an increascd cup to disc ratio if chronic ACG is
this is not necessary in this patient with mild to nroderate pain. also present. Ophthalmology referral within 1 hour ol
presentation is necessirrl, 1o minirnize risk for permanent
TEY POIXIS
visual loss. U!
q,
r A dermatomal eruption of grouped vesicles or pus- Bacterial keratitis (Option B) is rnost commonly seen
tules on an erythematous base is consistent with the ET
in patients ruho r-rsc contilct lcnses and u,ear their lenses
diagnosis of herpes zoster. overnight or improperly nash the lenses with nonsterile |.,
o Oral acyclovir, valaryclovir, or famciclovir is effective water. Patients with bacterill kcratitis have eye pain. red E
for treating herpes zoster if initiated within 72 hours of ness, floreign body scnsation. :rnd difficulty keeping the t!
aflected eye open. lhey do not see halos around lights or vt
presentation and can shorten the disease course as well (l,
I as decrease the likelihood ofpostherpetic neuralgia. have nausea. vomiting. or periorbital heiidache, as seen in
t
this patient. =
Bibliography Orbital cellulitis (Option C) is an infection of the
Schmader K. Herpes zoster. Ann Intern Med. 2018;169:l'ICl9 31. IPMID
globe, ocular fat, ;rncl ocular muscles of the orbit that can
3008371 8l doi:10.7326lAITC201808070 be both vision' and life threatening because of potential
spread into tl-rc brain. Pllicnts with orbital cellulitis fre
: quently have ophtl.ralmoplcgia and proptosis and not the
! Item 37 Answer: A clouded cornea and irupillary changes seen in this patient.
Orbital cellulitis warrunts hospitalization for intravenous
\ Educational Objective: Diagnose acute angle closure
antibiotics.
glaucoma.
Patients with scleritis (Option D) present n,ith a painful
'lhe nrost lil<ely diagnosis is acute trngle closurc glaucoma red eye accompanied by visual disturbance and photopho
(ACG) (Option A). Acute ACG presents with unilaterirl visual bia. The cardinal sign of scleritis is edema of the sclera, o{ten
:
disturbancc and seeing halos around lights. often accompanied associated with a violaceous discoloration of the underlying
t by scvcrc eye pain, eye redness, nausea, vontiting, and head. sclera and intensc dilatior.r of episcleral blood vessels. Ten
ache. Patientswith ACG have narrow anterior chamber angles, derness is invariably present.'lhese findings are not present
which inhibit drainage of the aqueous fluid; see imagc shown. in this patient.
Normal Abnormal
Cornea
Iris
Lens
185
i
Answers and Critiques
186
Answers and Critiques
criterion), particularly vertical growth, and are more aggres l9 years or older who did not receive the Tdap vaccine at
sive than other subtlpes. Because of the increased thick age 11 years or older should receive one dose of the Tdap
ness of the melanoma at diagnosis and eggressive behavior.
vaccine. All adults should receive a tetanus and diphthe_
prompt identification, biopsy, and treatment are imperative. ria toxoids (Td) or Tdap booster every 10 years. pregnant
All suspicious pigmented lesions must be biopsied. The pre women should receive at least one dose of the Tdap vaccine
ferred method to biopsy a pigmented lesion is an excisional between 27 weeks' and 36 weeks' gestation with every
biopsy with a t to 3-mm margin to obtain the entire lesion pregnancy. Pregnant women who had not previously been
and prevent sampling error. fully vaccinated against tetanus and diphtheria should also
A hemangioma (Option A) is a fypically red or pink receive a Td series.
,
benign vascular tumor. Int'antile hemangiomas are com Patients should be administered the human papilloma
mon but resolve before adulthood. Cherry hemangiomas are virus (HPV) vaccine series (Option A) at age 11 or 12 years
I common in adults and typically increase in number with or between the ages of 13 and 26 years if not given previ
age. Aithough hemangiomas can be blue or purple, they ously. In unvaccinated patients aged 27 to 45 years, vacci Ut
(l,
would not have the black appearance or rapid growth of this nation can be considered on the basis of risk using a shared
patient's nodular melanoma. ET
decision making process. Vaccination is not recommended
A keratoacanthoma (Option B) has a distinct appear. during pregnancy, although no harmful effects have been (J
ance and clinical course. It appears rapidly (within noted when inadvertently given to pregnant women and
4-6 weeks) as a round pink nodule with a central, kera q
=,
pregnancy testing is not necessary before vaccination. This .E
tin filled crater, giving it a "volcaniform" appearance. After patient is up to date on her immunizations, and repeating UI
(t,
its rapid growth, some keratoacanthomas tend to involute in HPV vaccination during pregnancy is neither recommended
6 months. Because keratoacanthomas are difficult to differ nor necessary. UI
=
entiate from squamous cell carcinoma, they are often treated Influenza revaccination (Option B) is necessary each
with surgical excision. The lack of a keratin filled center and year, owing to frequent genetic changes in the influenza
uniform black pigmentation of this patient's skin lesion dif: virus (antigenic drift). Annual vaccination is recommended
ferentiates it from a keratoacanthoma. for all individuals aged 6 months or older. All women who
A pigmented basal cell carcinoma (BCC) (Option D) are pregnant or might be pregnant during the influenza sea
typically has features ofa nodular BCC, such as rolled, pearly, son should receive standard dose influenza vaccination as
translucent borders and arborizing telangiectasias. Use of soon as it becomes available, regardless ofthe stage ofpreg
dermoscopy can help distinguish pigmented BCC from mel- nancy. Revaccinating this patient who received the vaccine
anoma clinically, and skin biopsy conflrms the diagnosis. just before conception is not necessary.
This patient's lesion is pigmented but has no other features No vaccination (Option D) fbr this patient is incorrect.
ofa nodular BCC. Routine vaccinations for pregnant women include 'ldap
vaccination and influenza vaccination during influenza
t(EY P0r1{TS
season.
r Nodular melanomas do not always fulfill the ABCDE
I(EY POITTS
diagnostic criteria; they may be uniform in color (or
amelanotic) and have regular, symmetric borders, . Pregnant women should receive one dose of the teta-
making them difficult to diagnose until a later stage. nus toxoid, reduced diphtheria toxoid, and acellular
. All suspicious pigmented lesions must be biopsied.
pertussis vaccine befween 27 weeks' and 36 weeks'
gestation with every pregnancy.
187
Answers and Critiques I
I
cough, headache. nasal congestion, and malaise in the
tr
CONT,
lleart Association guideline fbr the managenlent of patiellts
rvith v:rlvr-rlar heart disease recommends that uhen valvu
lar [.reart ciisease is diagnosecl in paticllts being considered
absence of fever, as well as the findings on physical exam-
ination. are consistent with an acute upper respiratory tract
fbr elcctive noncardiac stlrgery. the first step is to review infection (URI), or the common cold. URIs are caused by a
the standard criteria fbr intervention of the specific valve large variety of viruses, most commonly rhinovims' Virus
lesion. If the patient meets standard criteria for intenen transmission is via inhalation or direct contact with secre-
tiol1. it is prudent to deler the elective tloncarcliac proce tions infected with viral particles. The overproduction of :
dure and proceed to valvc intervention instead. ln patients mucosal secretions is caused by infection of epithelial cells
r,r,ith significant asynlptomatic valve disease who clo not responsible for the mucociliary layer within the respiratory i
meet stancl:rrd criteria for inten'ention. the risk of the non tract and oropharynx. Symptoms of an uncomplicated URI l
cardiirc procedure can be tninintized by l) har:ing al-l accu may persist for up to 2 weeks after initial infection, and
rate diagnosis of the tlrpe and severitl' ol valve d1'sfunction. cough may persist even after resolution of the acute illness. l
D 2) choosing an anesthetic approach appropriate to the valve Treatment for the common cold focuses on symptom man
UI lesion. and 3) er.rsuring a l.rigl.rer level of intri]operative agement, including reducing severity of discomfort caused :
Bibliography Bibliography
Fleisher LA, Fleischmann KE, Auerbach AD, et al; American College of Harris AM, Hicks LA. Qaseem A: High \hlue Care Task Force of the American
Cardiologr. 20i4 ACC/AHA guideline on perioperati\€ cardiovascular College of Physicians and for the Centers for Disease Control and
evaluation and management of patients undergoing noncardiac surgery: Prevention. Appropriate antibiotic use for acute respiratory tract infec-
a report of the American College of Cardiologr/American Heart tion in adults: advice for high lalue care from the American College of
Association Task Force on practice guidelines. I Am Coll Cardiol. 2014; Physicians and the Centers fbr Disease Control and prevention. Ann
64:e77 737. [PMID: 25091544] doi:10.1016/j.jacc.2}t4.O7.944 Intern Med. 2016;764:425-34. IPMID:26785a02] doi:10.7326 MtS 1840
188
Answers and Critiques
Patients at highest risk for HCV include individuals who Item 44 Answer: A
engage in high risk sexual behavior (such as having multi- Educational Obiective: Diagnose lichen planus.
ple partners), practice unhealthy drug use, have been incar
cerated, or are receiving hemodialysis. One time screening The most likely diagnosis is lichen planus (Option A). Some,
for HCV should be completed in all patients aged 18 to but not all, studies have shown an increased prevalence of
79 years. The U.S. Preventive Services Task Force (USPSTF) hepatitis C virus infection in patients with lichen planus.
does not promote screening in patients older than 79 years, Lichen planus is a relatively common T cell-mediated dis-
although the CDC does not specify an upper age limit. ease presenting with pruritic purple papules that are often
Patients with risk factors should also be retested regard- flat topped and have a polygonal appearance. Papules char-
Iess of age. Screening is accomplished by testing for HCV acteristically erupt on the low back, volar wrists, elbows,
antibody, followed by HCV RNA quantification if results ofl knees, and ankles. Othervariants include nail, genital, bul-
antibody testing are positive. Conventional Iiver chemistry lous, atrophic, and hypertrophic Iichen planus. Mucosal
testing is not sufficiently sensitive to exclude chronic HCV lesions have lacy white streaks (Wickham striae), as shown r^
(l,
infection. in this patient, or erosions and ulcerations. 3
ET
According to the USPSTF, screening for chlamydia and
gonorrhea (Option B) should be performed in all sexually t,
active women aged 24 years or younger because ofincreased T'
prevalence in this population. The USPSTF believes repeat ru
screening is reasonable in patients whose sexual history la
(l,
reveals new or persistent risk factors since the last negative
t^
test result. There is no indication to screen this patient for =
sexually transmitted infections.
The USPSTF recommends screening for syphi
lis (Option C) in persons who are at increased risk for
infection. Men who have sex with men (MSM) and men
and women living with HIV have the highest risk for
syphilis; 67% of cases of primary and secondary syphilis
occurred among MSM, and approximately one half of all
MSM diagnosed with syphilis were also coinfected with
HIV. Other factors associated with increased risk that
clinicians should consider include history of incarcera
tion, history of commercial sex work, and well-described Lichen planus also exhibits the Koebner phenome-
regional variations. non, which describes its occurrence in areas of trauma,
Not screening for HCV infection is inappropriate such as those caused by scratching. Lichen planus is typi
(Option D). HCV is the most common chronic bloodborne cally treated with topical glucocorticoids or ultraviolet light
pathogen in the United States and is a leading cause of com- therapy. Lichen planus tends to resolve over the course of
plications from chronic liver disease. HCV infection is asso, 1 to 2 years, although oral and nail lichen planus are more
ciated with more deaths than the top 60 other reportable persistent.
infectious diseases combined, including HIV Effective and Pityriasis rosea (Option B) begins as a single annu-
safe treatment is available for patients with HCV infection, lar patch or plaque with fine scaling (the herald patch),
and screening for HCV should be performed and treatment typically on the trunk, followed by numerous smaller
provided to prevent complications of chronic liver disease skin-colored to pink papules and plaques erupting along
and premature death. skin cleavage lines. As a result of the Koebner phenome
I(EY POIXIS non, scratching the lichen planus on the lower back can
result in a linear-appearing rash, mimicking the appearance
. One-time screening for hepatitis C virus should be
of pityriasis rosea. However, Iichen planus is signiflcantly
completed in all patients aged 18 to 79 years. more purple in color than pityriasis rosea and tends to
. Patients with risk factors for hepatitis C, such as indi erupt on the lower back, whereas pityriasis rosea tends to
viduals who engage in high-risk sexual behavior, erupt on the upper and mid back. This patient also has rash
practice unhealthy drug use, have been incarcerated, on her volar wrists, which would be an unusual flnding in
or are receiving hemodialysis, should be retested pityriasis rosea.
regardless of age. Psoriasis (Option C) has many clinical presentations,
the most common being plaque psoriasis. Plaque psoriasis
Bibliography typicaly presents with thick, well-demarcated erythem-
Owens DK, Davidson KW Krist AH, et al; US Preventive Services Task Force. atous plaques with overlying silvery scale. Nail psoriasis
Screening lbr hepatitis C virus infection in adolescents and adults: US
Preventive Services Task Force recommendation statement. JAMA.
presents with nail pitting, onycholysis (separation of the
2O2O:323:97O 5. [PMID: 327190761 doi:10.1001 /jama.2O2O.1123 nail plate from the nail bed), and "oil spots." Other less
189
Answers and Critigues
common subtypes include guttate psoriasis (commonly seen This patient's changes in menstrual bleeding may be related
in pediatric patients), palmoplantar psoriasis (frequently to perimenopausal hormone changes. However. FSH levels
with pustules as the primary lesion). and inverse psoriasis vary during the menstrual cycle in premenopausal women
(often seen without scale in the intertriginous areas). This and may not be indicative of perimenopause.
patient's purple papules do not resemble the red plaques Obtaining a Pap smear (Option B) is not appropriate
with thick scale of psoriasis. management. Women older than 30 years rt,ith a normal
The rash of secondary syphilis (Option D) is more Pap smear and a negative human papillomavirus (HPV) test
brownish red in color compared with the purple papules result require screening every 5 years. This patient's last Pap
of lichen planus. Secondary syphilis also typically involves smear was obtained 3 year ago and was normal: HPV testing
the trunk and extremities, often including the palms and was negative.
soles. Involvement of the palms and soles and general, Although oral contraceptive pills (Option D) are an
ized lymphadenopathy favor the diagnosis of secondary option to manage abnormal uterine bleeding, this patient
D syphilis. requires further assessment and the exclusion of endome
tt trial cancer before initiating treatment.
E TEY POIilT
o o Lichen planus TEY POtilIS
t/t is a T cell-mediated disease classically
o, presenting with pruritic purple papules that are often o Risk factors for endometrial cancer include obesity
CL flat topped and characteristically erupt on the low back, (BMI >30); a history of unopposed estrogen (e.g., poly
n volar wrists, elbows, knees, and ankles. cystic ovary syndrome, obesity)l or genetic syndromes,
such as Lynch or Cowden syndrome.
lt
o Bibliography . When presenting with abnormal uterine bleeding.
t^ Ioannides D. Vakirlis E. Kemen!'l-. et al. European Sl guidelines on the women aged 45 years and older or those younger than
management of lichen planus: a cooperation of the l.]uropean
Dermatolory Forum u,ith the Iiuropean Academy of Dermatolog'and 45 years who are at increased risk for endometrial
Venereolopy. J Eur Acad Dermatol Venereol. 202o::l{:1.10:l l-1. IPMID:
cancer should undergo endometrial biopsy.
326785131 doi:10.1111, jdv16.16.1
Bibliography
Item 45 Answer: C Anrerican Collcge of Obstetricians and G)necologists. ACOC committee
opinion n{). 557: Management of acute ;rbnormirl uterine bleeding in
Educational Objective: Evaluate a patient for nonpregn:rnt reprotluctile-aged nomen. Obstet G!'necol. 201:l;121:891 6.
endometrial cancer. lPl!{lD: 2:16:ls7o6l doi:10.1097 01.AOG.0000128616.67925.9^
190
Answers and Critiques
ln the secondary prevention of ASCVD events, clopi has diabetes, she should continue therapy with atorvastatin
dogrel is recommended fbr patients who are aspirin intol (Option A) regardless ol weight loss.
erant. 'lhere is no indicatior.r that this patient is irspirin 'l'he American Diabetes Associatior-r hypertension
intolerant. and there is no recommendation to substitute treatment goal for most persons rvith diabetes is less
clopidogrel (Option B) tbr aspirin for the primary preven than 140/90 mm Hg. 'l'hose persons with known ASCVD
tion of'ASCVD. or at high risk (10-year ASCVD risk >15'1,) should l.rave a
Anticoagulation has no role in the prirnary prevention blood pressure target less than 130/80 mm Hg. Guide
of ASCVD. ln patients with peripheral artery disease. the use lines f rom the American College of Cardiologyi'American
of low dose rivaroxaban (Option D) plus aspirin compared Heart Association :rdvocate fbr a treatment target below
with aspirin alone has been associated with a significant 130/80 n.rm Hg for most patients with diabetes. l'his
decrease in cardiovascular mortality (t.Z')(, vs. ).2"/,,) and patient is asymptomatic and, although her blood pressure
stroke (0.7'X, vs. 1.,1'){,) but a higher incidence of gastrointes should be carelully r.r.ronitored, she can continue lisino
tinal bleeding. This patient does not have an indicatior.r ft-rr pril (Option C) because her blood pressure remains well Ut
low dose rivaroxaban.
o
controlled on current medications.
ET
Not recommending lurther management is inct-rr Metfbrmin (Option D) does not cause hypoglycemia
rect (Option E). Adding low dose aspirin to his medica and sl.rould be continued, particularly if insulin glargine is (J
tion regimen is an appropriate choice fbr this patient in to be discor.rtinued.
'lhe primary sites ol vitamin D absorption are the jeju
-,E
light of his high risk fbr ASCVD and acceptable bleeding r!
risk. num and ileum. and deficiencies ol vitamin D and other vt
(u
fat soluble vitamins occur regularly after RYGB surgery. Cal 3
I(EV POI ilT
. Aspirin may be considered in adults aged 40 to
cium absorption is dependent on the presence of vitamin D. tUI
Patients require lif'elong vitamin D and calcium supplemen
70 years who are at higher atherosclerotic cardiovas tation (Option E) alter most bariatric surgical procedures.
cular disease risk and are not at increased bleeding and they sl.rouid not be discontinued.
risk.
I(EY POIilT5
191
Answers and Critiques
Bibliography
\\'insettFT. Prtcl SCi. Kelll B(i. Bedside clirgnostics li)r intections: .l guide for
tlermaro)ogisrs. .\m J Clin l)ermalol. 2020:21:697 709. IP\lll): 3]5622011
tkri:10.i007 slO257 020 00526-v
o Item 49 Answer: A
Educational Obiective: Evaluate a breast mass in a
woman younger than 30 years.
192
Answers and Critiques
f,EY POaXTS (wdnwdl vasculitis, which burn and sting and with individual wheals
. persisting longer than 24 hours.
All women presenting with a breast mass should be
Angioedema (Option B) is a transient, Iocalized subcu
evaluated by imaging even if their clinical breast
taneous or submucosal form ofurticaria caused by extrav-
examination is without concerning features.
asation of fluid into interstitial tissues. Angioedema may
occur with or without urticaria and can be a component
Bibliography
of anaphylaxis. The margins of the affected areas are often
Moy L, Heller SL, Bailey L, et al; Expert Panel on Breast Imaging. ACR
Appropriateness Criteria- palpable breast masses. J Am Coll Radiol. indistinct, and the skin may be a normal hue or faintly pink.
2017;14:S203 24. [PMID: 28473077] doi:10.1016/j.jacr.2017.02.033 The localized swelling, indistinct margins, and normal or
faint color of angioedema are distinct from the flndings in
urticaria.
Item 50 Answer: D Urticarial drug reaction (Option C) is an unlikely diag
Ed ucationa I O bjective : Diagnose urticarial vasculitis. nosis, given that this patient has no history of new medi- t,t
o
cations with the exception of acetaminophen, which was
The most likely diagnosis is urticarial vasculitis (Option D), EF
initiated after she developed systemic symptoms from the
which is characterized by individual urticarial lesions lasting
urticarial vasculitis. frt
longer than 24 hours; symptoms of burning and stinging
tE,
more common than itch; wheals that resolve with hyperpig TEY POITIS
.E
mentation (as shown); and associated systemic symptoms, o Urticarial vasculitis urticarial
is characterized by UI
such as fever and joint pain. lesions that last longer than 24 hours and that burn c,
and sting rather than itch; wheals that resolve with tt
=
hyperpigmentation; and associated systemic symp-
toms, such as fever and joint pain.
. If urticarial vasculitis is suspected, skin biopsy is
helpful diagnostical ly.
Bibliography
Davis MD, van der Hilst JC. Mimickers of urticaria: urticarial vasculitis
and autoinflammatory diseases. J Allerry Clin Immunol Pract. 2018;
6:1762-70. IPMID: 298717971 doi:10.1016/j.jaip.2o18.05.006
Item 51 Answer: D
Educational Objective: Treat erectile dysfunction in a
patient with coronary artery disease.
When urticarial vasculitis is accompanied by
decreased semm complement (C3 or C4), it is classifled as The most appropriate management is oral sildenafll (Option
hypocomplementemic urticarial vasculitis; these cases are D). Ihis patient with erectile dysfunction (ED) has a history
strongly associated with systemic lupus erythematosus and of previously revascularized coronary artery disease. He is
glomerulonephritis. The cause of urticaria is investigated currently without cardiovascular symptoms and exercises
primarily by history and physical examination. Diagnostic regularly. According to the Third Princeton Consensus Con-
evaluation for urticaria is not recommended unless history ference guidelines on the safety of ED treatment in patients
suggests a speciflc cause. If symptoms persist, laboratory with cardiovascular disease, this patient is low risk and can
tests, including a complete blood count with differential, be treated pharmacologically. Phosphodiesterase-S (PDE 5)
urinalysis, ery.throcyte sedimentation rate or C reactive inhibitors, such as sildenafll, tadalafll, and vardenafll, are
protein, thyroid-stimulating hormone, and liver chem- flrst-line pharmacotherapy for ED. All PDE-S inhibitors have
istry tests, can be considered. If associated with systemic similar efficacy and are FDA approved for on-demand use.
symptoms or suspicion of urticarial vasculitis, skin biopsy Tadalafll, which has a long halfJife, is also FDA approved for
is helpful. daily use. It is essential to instruct patients who are prescribed
Typical urticaria lesions itch and are transient, and PDE 5 inhibitors on proper use: The medication should be
wheals appear and disappear within minutes to a few hours. taken 30 to 60 minutes before sexual activity, and efficacy
Although individual urticarial lesions should resolve in less may be decreased if taken after consumption of a high-fat
than 24 hours, recurrent crops of hives may last for weeks. meal. PDE-S inhibitors should not be prescribed to patients
Most cases of urticaria resolve spontaneously, and the cause on nitrates because of the risk for hypotension. Similarly, they
is never determined (idiopathic). If the episodes last longer should be used with caution in the setting of concomitant
than 6 weeks, the condition is classifled as chronic (Option cr-blocker therapy. All patients with ED should also be coun
A). the features ofchronic idiopathic urticaria are inconsis- seled on exercising regularly, minimizing stress, losing weight
tent with this patient's urticarial vasculitis, a small vessel if overweight, and smoking cessation.
193
Answers and Critiques
According to the Third Princeton Consensus Conference to cardiovascular risk. The eIlect ol artificial sweeteners
guidelines, cardiac testing, such as an ECG (Option A) or an on health is uncertain. Some studies indicate that artificial
exercise stress test (Option B), is not required for patients sweeteners may cause weight gain and have effects on bowel
at low cardiovascular risk befbre initiating pharmacologic health and the gut n.ricrobiome. Although artificial sweet
therapy fbr ED. This patient has Iow cardiovascular risk, and eners have been associated with increased risk for cancer
cardiac testing is therefbre not warranted. in animal models. the association has not been fbund in
Acc<-rrding to the 2018 American Urological Associa humans. Artiflcially sweetened beverages are often recom
tion guideline statement, all patients with ED should have mended as a transition from sugar sweetened beverages to
an early morning serum total testosterone measurement water and can also decrease dental caries in these cases.
(Option C). This patient's early morning testosterone level is this patient's once weekly consumption of red n.reat
normal. There is no role fbr testosterone supplementation in (Option C) is unlikely to be a significant contributor to
patients with ED who are not androgen deficient. cardiovascular risk. Many studies have shown that liequent
D XEY POIXIS
consumption of red meat results in a modest increased risk
ta for cancer; diabetes; cardiovascular disease: and. in some
E
(D
. Phosphodiesterase-S inhibitors, such as sildenafil, studies, mortality. Efl'ect sizes are generally small and incon
Ut tadalafil, and vardenafil, are considered first-line sistent, and no randomized controlled trial data conflrm this.
o, pharmacotherapy fbr erectile dysfunction and can be This patient's red meat consumption is quite minimal and is
CL used by patients with known cardiovascular disease unlikely to contribute to cardiovascular risk.
n after assessment of cardiovascular risk.
XEV POITT
lt . Phosphodiesterase-s inhibitors should not be pre-
o Evidence supports that loneliness and social isolation
o
la
scribed to patients on nitrates because ofthe risk fbr
are significant risk factors for cardiovascular disease
hypotension and should be used with caution in the
and death.
setting of concomitant a-blocker therapy.
Bibliography
Bibliography
Hakulinen C. Pulkki Raback L, Virtanen M, et xl. Social isolation and loneli
Burnett AL, Nchra A, Breau RI l. ct al. Erectile dysfunction: AUA guidrline. J
ness ls risk lactors lbr nryocardial infarction, stroke and mortality: UK
Urol. 2018:200:633 41. I PM l l): 29746858] doi: l0. l0l6ij.juro.20l8.0s.oo4 tsiobank cohort study o1 ,179 05.1 men and women. I leart. 2018:104:1536
42. I l'}M ll): 295883291 doi: 10.1136, heartjnl 2017 l]12663
Item 52 Answer: D
Educational Objective: Evaluate social isolation
factor for cardiovascular disease.
as risk Item 53 Answer:
Educational Objective: Diagnose retinal detachment.
D
tr
In addition to the patient's age, sex, and hypertension, a The nrost likely diagr.ursis is retinal cletachment (Option D).
contributing Iactor fbr tuture cardiovascular disease ir.r this which is causing a painless visual fielcl defbct. Hcr syntlt
patient is scicial isolation (Option D). Tl.rere is increasing evi toms ol visual loss. Ilashing lights, and floaters should raise
dence supporting loneliness and specifically social isolation suspicion fbr this diagnosis, which can be conflrmecl by ln
as risk lactors for cardiovascular disease and death. A study ophthalmologist using indirect ophthalmoscopy or rvide field
of over 450,000 patients demonstrated that social isolation retinal photography Indirect ophthalnroscopy using a cor.t
was associated with a 1.43 (957, CI, 1.3 1.55) increased risk densing lens provides the ophthahnologist a u,ide field ot
for incident acute myocardial infarction (AMI) and 1.39 (95'1, vision ot the retina and three'dimensional viert's showing ele'
CI, 1.25 1.54) increased risk for stroke. Some, but not all, of valirin ancl depth. Bccluse retinal detachn.rent typically occurs
these eflbcts were attenuated by correction for traditional risk f'ar into the periphery. oftice-based direct ophthalmoscopy
factors lbr cardiovascular disease and stroke. Social isolation lvill likely be normal or ilt least nondiagnostic. Floaters ma1'
was also associated with a 25'7, increase in n-rortality after AMI be clescribed by patients as a -cobr,rcb" and represent accon.r
and a 32'X, increase in nrortality after stroke. par.rying posterior vitreous detachment (PVD). This patient
this patient's level of'alcohol consumption (Option A) likely developed a PVt) in the days preceding her presen
is considered lower risk. The 2020 Dietary Guidelines indi tation, which led to a retinal tear. I,lashing lights represent
cate that irdults should limit consumption of alcohol to no vitreoretinal attachments stimulating retinal depolarization
more than one drink per day for nonpregnant women and as they pull anay'fionr the retina. l-xrger retinal detachnlents
two drinks per day fbr men. Studies have shown that this causc rrore severe visual disturbances. Risk factors ftrr retinal
amount of alcohol consumption may reduce the risk fbr Ml. detachnrent include increasing age, nryopia. and recent cat
However, it is generally agreed that individuals who prefer aract surgery. This patient requires imrnediate ophthalnrol-
not to drink alcohol should not initiate alcohol consumption ogr referral for diagnosis and treatment to prevent thrthcr
for the purposes of cardiovascular risk reduction. vision krss.
This patient's consumption ot' artificially sweetened Age related macular degeneration (AMD) (Option A) is
soda (Option B) is unlikely to be a significant contributor often lsymptomatic in its earliest stages. I)ry AMD typically
194
L
t
l
Answers and Critiques
L
t
t tr
CONT.
presents with gradual onset of vision loss that is most notice-
able when driving or with a scotoma (blind spot). Wet AMD
can present acutely with change or loss in central vision
show absence of melanocytes. Treatment can be challeng-
ing and prolonged, with suboptimal results. Potent topical
glucocorticoids, topical calcineurin inhibitors (tacrolimus or
t resulting from subretinal swelling and hemorrhage. Neither pimecrolimus), and phototherapy are common treatments.
t is typically associatedwith flashing lights or floaters. Repigmentation often occurs first in perifollicular areas (as
Patients with central retinal artery occlusion (CRAO) shown).
t (Option B) present with abrupt, painless vision loss in one
f eye. The vision loss is profound, and afiected patients can
typically visualize only gross movements, although there
L may be a small area of vision in the temporal field. A com-
t plete or relative afferent pupillary defbct is present, and
L
funduscopic examination will reveal retinal paleness due to
t ischemic changes. These lindings are absent in this patient. tl(l,
CRAO is most commonly associated with ipsilateral carotid
t artery atherosclerotic disease.
ET
195
Answers and Critiques
Universal screening for HIV infection is recommended for in healthy populations; however, testing for deficiency is
adolescents and adults aged 15 to 65 years. appropriate in groups at high risk or in patients presenting
with low bone mass, fractures, hypocalcemia, or hyper
f,tY POtllT parathyroidism. This patient has not met an indication for
o Vitiligo, an autoimmune skin condition characterized vitamin D measurement (Option D).
by depigmented patches, is associated with autoim-
I(EY POITI
mune thyroid disease; thyroid-stimulating hormone
measurement should be performed at time of diagnosis. . The U.S. Preventive Services Task Force recommends
screening for osteoporosis in all women aged 65 years
Bibliography or older and in women younger than 65 years who are
'Iaieb A, Alomar A, Bohm M, et al; Vitiligo European Task Force (VETF). at increased risk for osteoporosis, as determined by a
Guidelines for the management of vitiligo: the European Dermatolory formal clinical risk assessment tool.
Forum consensus. Br J Dermatol. 2013;168:5 19. [PMID: 228606211
D doi:10.1111/j.1365 2133.2012.11197.x
Ut Bibliography
€
(D Curry SJ, Krist AH. Owens DK, et al; US Preventive Services'lhsk Force.
Screening fbr osteoporosis to prevent I'ractures: US Preventive Services
l,t Item 55 Answer: A Task Force recommendation statement. JAMA. 2018:319:2521 31. IPMID:
o, 299 467 351 doi:10. I 001 /iama.20l 8.7498
TL Educational Objective: Assess fracture risk in a woman
n at high risk for osteoporosis.
lr
.D
UI
The most appropriate next step is to assess fracture risk
a clinical assessment tool (Option A). The U.S. Preventive
Services Task Force (USPSTF) recommends screening for oste-
with Item 56
Ed u catio na I O
Answer:
bjective: Manage
C
warfarin anticoagulation
tr
in the perioperative period.
oporosis in all women aged 65 years or older and in women
younger than 65 years who are at increased risk for osteopo The most appropriate perioperative management of this
rosis, as determined by a formal clinical risk assessment tool. patient's warlarin is to withhold warfarin .5 days prior to
This patient presents with several risk factors for osteoporosis suryery anci to restart in 12 to 2.1 hours aftcr suryery (Option C).
and, most important, bone fracture. This patient's risk factors Wartarin shoulcl be withheld a ntinitnum <tf 5 days befbre
include low body weight (BMI of 21), cigarette smoking, and surgery to nornulize the patient's INR. I he timing rll narlarin
family history of hip fracture. The Fracture Risk Assessment discontinuatirin is based on its 116' to 42 hour hal{'lifb and
(FRAX) is a commonly used clinical risk assessment tool clinicai studies docuntenting the time lbr thc INR to return
(available at ww w.shef.ac.uk/FRAX). Women with a 10 year to normal tbllowing cliscontinuation. In general, 2 to 3 days
FRAX risk for major osteoporotic fracture equal to or higher are required fbr the INR kr clrop belou'2 ancl .4 to 6 days fbr
than that of a 65 year old White woman without additional the INR to nomrirlizc. Somc physicians obtain an INR on clay
risk factors (10 year risk of 8.4'1,) should undergo screen- .1and intcn'ene u,ith vitanrir-r K supplementation if necessary
ing for osteoporosis. Screening can be accomplished with for persistentlv elevated INRs. Postoperative re initiation of
bone mineral density (BMD) measurement, most commonly anticoagulation shoukl be guidecl by'the patient's thrombotic
with dual-energr x ray absorptiometry of the hip and lumbar and bleeding risk and cleterrninecl in collaboratirm with the
spine. surgical team. Owing to i1s delayed eftbct. the first dose of
BMD measurement (Option B) is not indicated in this r,rartlrin is typically rrdn.rinistered 12 ttl')4 hours alter surgery.
patient unless a fracture risk assessment with a formal risk pencling adequate henrostasis ancl barring any surgical con.t
assessment tool (e.g., FRAX) indicates an increased risk for plicutions that rnay increase the risk firr bleedir-rg. It usually
fracture. takcs 5 to 6 days to achieve a therapeutic INR.
The U.S. National Osteoporosis Foundation recommends Withholding wartarin f'ur only 2 days prior to surgery
pharmacologic treatment for patients with osteoporosis- (Option A) increascs the risk firr surgical bleedittg or,r'ittg
related hip or spine fracturesl those with a BMD T-score of to the long hall life of wirrfarin. Many paticnts r,vould har,e
-2.5 or less; and those with a BMD T score between -1 and an INR ol lpprorimltely 2 lbllou,ing only 2 clal's ol warfarin
2.5 with a 10 year risk of 3'l" or greater for hip fracture or discontinuation. Starting wart'arin 2 or 5 days after surgery
20'1, risk or greater for major osteoporosis-related fracture (Option I)) is unneccssarily delaycd, considering 5 to 6 duys
as estimated by the FRAX tool. This patient has yet to meet are requireci lbr fLll rrnticoagulation.
an indication for treatment with a bisphosphonate, such as Withholding wurfarin lbr .5 days ar.rcl bridging n,ith
alendronate (Option C). lorv molecular g,eigfit heparin (Option B) is not ir.rdicated
The National Academy of Medicine recommends cal because this may increase bleeding risk r,rdtl.rout of icring any
cium intake of 1000 to 1200 mg/d, ideally from dietary adclitional lntithr-ombotic bcnefits in the perioperatire period.
sources. A calcium supplement may be used for patients This ilaticnt has a CI IATDS, VASc score of 5 (fi:nrale, hyperten-
whose diets are insufficient but should not be recommended sion. diabctes mellitus. age) '"r,ithout adclitional risk lactors lor
independent of dietary assessment and intervention. Rou- bleeding and is :rt moderatc to high risk firr stroke with atrial
tine screening for vitamin D deflciency is not recommended flbrillation o\er the long tenn (l .'1"t, ttl"t, annually). Bridging
196
t
i ffi cln be consiclerecl fbr patients at l.righest risk titr dev,astirt Although transvaginal ultrasonogrlrphl, (Option D) can be
lll 111g thromlloembolic cliscirse (rccent pulnronary embolism, valuable in thc assessment of acute pelvic 1rain. a pregnancy
coNT
t priu. stroke. high thrornbotic risk mechanical heart valve. test should be obtaincd first. Transvaginal ultrasonography can
CtIA,t)S2 \ASc score >7). ilssess fbr ectopic pregnanc'!! orarian cyst. ovarian torsion. and
1
197
Answers and Critiques
of washing with soap and water and removal of the crust. patients undergoing chemotherapy, the vaccine should ide
Topical antibiotic treatment with mupirocin or retapamulin ally be administered at least 2 weeks before initiation of
is effective in most cases of impetigo. chemotherapy or 1 week after administration of chemother-
Topical antibiotics are as effective as oral antibiotics in apy if befween cycles. However, some studies ha've shown
the treatment of impetigo. There is no evidence to support no dillerence in immunogenic response at different time
the combined use of a topical and an oral antibiotic for impe points during chemotherapeutic cycles.
tigo. Systemic antibiotics. such as cephalexin (Option B) and Def'erring influenza vaccination until completion of
doxycycline (Option C), are helpful in cases of widespread chemotherapy (Option A) is not an appropriate option.
bullous impetigo or when methicillin resistant S. oureus is Immunocompromised pctients are especially prone to
suspected or confirmed. However, this patient has localized severe influenza infection, and a priority must be placed on
nonbullous impetigo, and topical antibiotics, such as mupi preventing inf'ections, if possible. Although chemotherapy
rocin oinlment. are eflective. may potentially blunt the immune response to vaccination,
The classic presentation of cutaneous herpes simplex is the vaccine should still be administered.
UI a group of painful, small vesicles on an erythematous base. High dose quadrivalent influenza vaccine (Option B)
€
(D transitioning to pustules and subsequent crusting of' the became a'uailable in the 2020-2021 influenza season and is
ut lesior-rs over time. The diagnosis is typically made on clinical an option for people aged 65 years or older. The previous
o, grounds. Oral antiviral agents (acyclovir valacyclovir, or f'am high-dose trivalent vaccine was 24'2, more efl'ective than its
CL ciclovir) (Option D) car.r be used to treat primary infbctions standard dose counterpart. Estinlates of relative efficacy or
n and episodic or secondary recurrences, and as suppression or eflectiveness of the quadrivalent high dose vaccine com
prophylaxis fbr patients with six or more recurrences perlear. pared with standarcl dose quadrivalent vaccine are not avail-
lt
Topical glucocorticoids, such as triamcinolone cream able. This patient is younger than 65 years and would not
o
t^ (Option E), will most likely cause tl.ris localized inf'ection beneflt fiom the high-dose fornrulation.
to enlarge. Topicirl glucocorticoids :rre used for their anti The nasal spray influenza vaccine (Option C) is a live
inflammatory efl'ects arrd are most commonly indicated in attenuated vaccine and should not be given to patients with
patients with eczematous dermatosis; they should not be active cancer or who are undergoing chemotherapy. The live
applied to patients with bacterial, viral. or fungal infections. attenuated influenza vaccine is contraindicated in immu
nocompromised patients and additionally should be used
XEY PO I ]IT
with caution in patients with significant medical conditions,
r Nonbullous impetigo, characterized by eroded erythe- including cardiovascular, pulmonary. or liver disease; dialysis-
matous papules or plaques with honey colored crust, dependent end stage kidney disease; or diabetes mellitus.
can be treated with topical antibiotics, such as mupi lhis live attenuated vaccine should not be given to members of
rocin or retapamulin. patient's housel.rold. The nasal spray vaccination is other-
tl.ris
wise an option for some individuals aged 2 through 49 years.
Bibliography
t(EY POiltTS
Kosar L. l.aubscher'1. l\4anagement of impetigo ancl cellulitis: simple consid
errtions for promoting appropriate antibiotic use in skin infections. Can o In patients with active cancer, annual vaccination
Fam Physician. 2017:63:615 8. LPMID: 288079581
with the standard dose inactivated influenza vaccine
should be administered at least 2 weeks before initia
tion of chemotherapy or 1 week after administration
Item 59 Answer: D
of chemotherapy if between cycles.
Educational Objective: Prevent influenza in patients
o The live attenuated influenza vaccine is contraindi-
who are immunocompromised.
cated in immunocompromised patients, including
The most appropriate vaccination strategv for this patient those with active cancer or who are undergoing
is to administer the standard-dose inactivated influenza chemotherapy.
vaccine (Option D). Annual influenza vaccination is rec
ommended for all individuals aged 6 months or older. The
Bibliography
influenza vaccine should be administered as soon as it l.reedmrrr.r MS. Ilernstein H. Ault KA. Recommended ildult immunization
becomes available, pref'erably by October. but can be given schedule. United States, 2021. 1\nn Intern Merl. ')02l:171:37.1 8.1. [PMID:
335710111 ckri:10.7326rM20 8080
at any time during the influenza season. lmmunocompro-
mised patients, including patients with cancer and those
undergoing chemotherapy, are at risk lbr influenza and
may be more likely to develop complications. Successful
immunization requires a competent immune system to
Item 60 Answer: E
Educational Objective: Prevent harm in patients using
tr
generate antibodies; althougl.r this response may be atten
dietary supplements.
uated in immunocompromised patients, vaccination is still lhe most appropriate recommendation is to stop all dietary
indicated. Members of this patient's immediate household supplements (Option E). Dietary supplcments. including vita
should also receive the inactivated influenza vaccine. In nrins, minerals, botanicals, herbals, metabolites. and amino
198
Alswers and Critiques
tr
CONT,
aci(ls, are categorized es tbods by thc FDA. Thercfbre, man
ufacturers are not requircd to demonstrate e{Icacy or safety
of their products unless the supplerner.rt ir-rcludcs ingredients
Item 61
Educational
Answer:
Objective:
D
Manage acne in an adult woman.
thirt were introduced aftcr 199.1. As ir result. the cflicacl' ol The most appropriate management is to start spironolac
nrrist supplemcnts has not been subject to rigorous evalua tone (Option D). This patient has adult female type acne,
tion. ln addition to qucstionable ef liclcl,i supplcrncnt use is characterized by papules and nodules on the lower half of
:lssociilted \\,ith risk fbr both direct ancl indirect hru'nrs. Dircct the face with marked presence on the jawline. Adult acne is
harms include side eflbctsr interactions r,r,ith other dmgsr more common in women and, when present, results from
ancl hanns related to inclusion ol unadvertiscd additives. an abnormal response to physiologic levels of androgens. It
cornpounds. or toxins. lndirect harms occur rthen dietar] typically presents along the jawline and often flares with the
supplement use replaces or dela1's strrndard trcrtnlents that menstrual cycle. Although considered second line therapy,
have clemonstrated efficacy: When counscling paticnts usir.rg oral contraceptives or spironolactone (aldosterone receptor
supplements, thc physician's role is to in[brm these patients of' blocker with antiandrogen activity) can be added to other v,
(u
potential sidc eftbcts ancl benefits, if l<nown. treatments in women with moderate to severe acne. Spi-
ET
Biotin (Option A), a common over-the counter sup ronolactone is an excellent choice for adult female acne
plcment, has no kno'"r,n toric eft'ects. Hovvever. pitients tak for which topical therapies have failed. Spironolactone is a (,
ing thyroid rcplacement xnd also tll<ing morc than 5 to pregnancy category C (risk cannot be ruled out), and con !,
l0 mgrd ol biotin should discontinuc it entirely or at least 2 traception is recommended when used in premenopausal tg
to 5 days befbre thyroid firnction testing. l]iotin interf'erencc women for the treatment of conditions such as hirsutism .A
(l,
caLrses falsely high levels ol fiee thyroxine (T r). fie c triiodo and acne.
thyronine (T,), total T.,, and total 'l'., and a lirlsely low thy Obtaining hormone measurements (Option A), such =
a
roicl stimulatir.tg hon.none level, mirnicking thyrotoxicosis. as follicle stimulating hormone, luteinizing hormone, and
l]iotin also interferes u'ith serum tl-oponin testing. resultilltj free testosterone levels, is not indicated. The patient has no
in falsely lor,r, lcvels. which conlcl leird to difficulty in the features that suggest hyperandrogenism, such as hirsutism,
dirrgnosis of acutc coronary syndronre. l)epencling on the muscle mass gain, deepening of the voice, or clitoromegaly.
assay used. biotin can also interfere u,ith the nreasuremeut Rapid onset of acne combined with other signs of hyperan-
of digoxin. IL'rritin, testoslcrorle. brlin natriuretic pepticle. drogenism warrants consideration of polycystic ovary syn-
ancl progesterone. drome, congenital adrenal hyperplasia, or an underlying
l'}atients w]ro are snrokers and those exposcd to asbestos adrenal or ovarian tumor.
should avoid p carotene (Option B) because eviclcnce has Pelvic ultrasonography (Option B) might be indicated
linkeci B carotene'uvith increased risk tbr lung cancer (but in patients with findings suspicious for polycystic ovary
not rt,ith other cancers) in these p<;pu lations. F-or this patient syndrome. However, this patient has no features of polycystic
who continues 1o smokc, discontinuation ol B carotene is ovary syndrome, such as hirsutism or oligo /anovulation,
recommended. and pelvic ultrasonography is not indicated.
Both fish oil andvitamin E (Options C, D) nrrrl increirsc Progesterone eluting intrauterine devices (Option C), as
thc risk fbr blccding in patients tirking anticoagular.rts. This well as any other progesterone based hormonal contracep
patient shoulcl be adviscd to stop taking fish oil ancl vitanrin tion, can exacerbate acne and would not be recommended.
E because he is taking rivaroxabln fbr atrial tibrillation. Combined estrogen and progesterone oral contraceptives are
Beciiuse this patient has contraindic:rtions fur [i carotenc, effective treatments for inflammatory acne in women with-
f ish oil, and vitamin E ancl biotin nlay compromise the abil out evidence of hyperandrogenism and would be an option
ity to monitor his thyroid disease, all supplenrer.rts shoulcl be for this patient.
discontinuecl. Topical antibiotics are helpful in inflammatory acne
because they target Cutibacterium ocnes and have anti
KEY POIIIIS inflammatory effects. It is recommended that topical anti
o Dietary supplements have questionable efficacy in biotics be combined with topical benzoyl peroxide in
improving health, and their use is associated with risk treatment for mild. moderate. or severe acne to avoid
for both direct and indirect harms; the physician's antibiotic resistance. The common topical antibiotics clin-
role is to inform patients of potential side effects. damycin and erythromycin are pregnancy category B,
. whereas topical benzoyl peroxide is pregnancy category
In patients taking anticoagulants, vitamin E and fish
C. Topical metronidazole cream (Option E) is a flrst line
oil increase the risk for bleeding; p-carotene increases
treatment for rosacea but not for acne.
the risk for lung cancer in cigarette smokers; and biotin
interferes with several biological assays. XEY POIilT5
. Spironolactone andcombined oral contraceptives can
Bibliography be used for adult female acne in patients in whom
Incze M. Vitamins and nutritional supplements: what do I need to know? topical therapies have failed.
JAMA Intern Med. 2019;179:460. [PMII): 306150201 tloi:10.1001r (Continued)
jamainternmed.2018.588O
199
Answers and Critiques
F
UI
tr Item
Ed
62 Answer: B
ucatio na I Obiective : Manage direct oral anticoagrrlant
short half-life; thus, bridging anticoagulation is not
needed.
€
o
therapy for an elective surgical procedure.
Bibliography
t/l The most appropriate perioperative managernent of the Lip GY, Banerjee A, Boriani G, et al. Antithrombotic therapy for atrial fibril-
o,
patient's ri'v'aroxaban is to withhold fbr 3 clays before surgery lation: CHEST guideline and expert panel report. Chest. 2018;15,1:1121-
EL (Option B). Befbre elective surgery,; management decisions 201. IPMID: 301.14419] doi:10.1016'j.chest.2018.07.040
n ab<lut anticoagulant therapy are made on the basis of'surgi
lt
E
cal bleeding risk, thrombotic risk ofl'anticoagulants, kidney Item 63 Answer: C
function, and type of dmg. In this case, tl.re patient is taking
.D
t^ Educational Objective: Manage vaccination of a health
rivaroxaban, a direct oral anticoagulant (DOAC), lcrr atrial
care worker.
fibrillation. She has a moderate high CHArDS.,,-\ASc score
of 4 (hypertension. age. vascular disease, fbmale) ancl normal The most appropriate vaccine to administer to this health
kidney function and is undergoing a surgery rvith moderate care worker is the tetanus toxoid, reduced diphtheria toxoid,
bleeding risk. Because ofthe bleeding risl<, she should not be and acellular pertussis (Tdap) vaccine (Option C). Health
anticoagulated at the time of surgcry. In patients with nor- care workers are at increased risk lor acquiring and trans
mal kidney function.l,r,ithholding a DOAC for four half lives mitting pertussis and should receive a dose of the Tdap vac-
(2 3 days) rcduces anticoagulant levels to approximately cine regardless ofwhen they received their last tetanus and
in near normal bleeding risk. Thus. with
6.25'){,, resulting diphtheria toxoids (Td) vaccine. Because this patient received
holding rivaroxaban 3 days befbre surgery in this patient is only a Td vaccine, a Tdap vaccine is required at this time.
rccommended. Health care workers are also at increased risk for infection
Although anticoagulation can be continued until the with influenza, hepatitis B, measles, mumps, rubella, and
day of surgery for select minor procedures (e.g., cataract or varicella viruses; this patient was previously vaccinated for
dental surgery), patients undergoing surgery with signifi these viruses and has documented immunity to hepatitis B,
cant blceding risk must have anticoagulation withheld long rubella, and varicella.
enough to ensure adequate drug clcarance and normaliza- Hepatitis A vaccination (Option A) is not indicated for
tion of coagulation. Continuing rivaroxaban until the day this patient because health care workers are not considered
of surgery (Option A) places this patient at an unacceptable to be at increased risk for hepatitis A virus infection unless
bleeding risk perioperatively. working in high-risk venues. Hepatitis A vaccination is rec
Patients u,ho are at especially' high risk fbr thrombo ommended for adults who request it or those at increased
sis r.thile ofl'anticoagulants may neecl a pre or postop risk for infection or complications of infections, including
erative anticoagulant bridge r,r,ith lolr,, molecular weight travelers to endemic areas, individuals with chronic liver
heparin to limit tirne ofI anticoagulation. This is most rel disease, men who have sex with men, users of illicit drugs,
evant lor drugs with a delaycd onset ol action and longer homeless persons, persons who conduct hepatitis A related
half'life. such as rvarfarin. t)OACs. hor,r,ever, have a rapid research, household or close contacts of children adopted
onset and short half lif'e; thus, bridging anlicoagulation from endemic areas, and those who work in settings of
is not needed. The BRII)GE trial demonstrated r.ro difler possible exposure (group homes, nonresidential day_care
ence in thrombotic events in patients on warfarin with facilities for developmentally disabled persons, and health
atrial fibrillation r,l,ho were bridged perioperatively but did care programs serving patients with unhealthy drug use).
observe a higher rate of bleeding in this group. Iherefore. Health care workers are not considered to be at increased
bridging anticoagulatior.r is rarely recommencled, except risk for meningococcal disease, and thus the quadrivalent
in the setting of very high throntbosis risk off r,r,arfarin meningococcal conjugate vaccine (MenACWy) (Option B) is
therapy, such as atrial fibrillation with a mechanical heart
not indicated for this patient. The MenACWy vaccine is recom_
valve. lhis patient is not at very high risk for thrombosis
mended for adults considered to be at increased risk, includ
and is not or.r warfarin: thereforc, bridging is not war ing first year college students living in dormitories; travelers
ranted (Options C, E).
to endemic areas; microbiologists with ongoing exposure to
200
t
Neisserio meningitidis; military recruits; those at increased routine use of oral contraceptives. This method also requires
risk because of an outbreak; and patients with anatomic or a repeat dose at 12 hours. Combined oral contraceptive pills
functional asplenia, complement deflciencies, or HIV infection. are less eflective and are associated with higher rates of side
Because health care workers are considered at effects compared with FDA approved emergency contra
increased risk for pertussis, not offering this the Tdap vac ceptive methods. Some experts will suggest this method
cine (Option D), regardless of when he received his last Td to patients when cost is an issue or when quick access to
vaccine, is not the best strategr. FDA approved emergency contraception is limited. Either
the copper IUD or oral ulipristal acetate would be recom-
f,EY POIilTS
mended as the most appropriate method for this patient.
. Health care workers are at increased risk for pertussis The levonorgestrel oral contraceptive pill (Option B) is
and should receive the tetanus toxoid, reduced diph available over the counter (without age restriction), and a
theria toxoid, and acellular perhlssis vaccine regard single dose is eflective for emergency contraception within
less ofwhen they received their last tetanus and diph- the 3 days after intercourse. Like ulipristal acetate, Ievo ut
(l,
theria toxoids vaccine. norgestrel delays ovulation. The effectiveness of levonorge-
CT
r In addition to pertussis, health care workers are also strel declines in women with a BMI greater than 26 and
at increased risk for influenza, hepatitis B, measles, would not be recommended for this patient. (J
mumps, rubella, and varicella viruses and should The subdermal contraceptive implant (Option C) is not
an eflective form of emergency contraception and would =,
receive the appropriate vaccinations. l9
therefore not be appropriate for this patient. |a
o
Bibliography XEY POIilIS 3
t/l
Freedman MS, Bernstein H, Ault KA. Recommended adult immunization
schedule, United States, 2021. Ann Intern Med. 2021. [pMIt): 33571011]
r Oral ulipristal acetate is an effective oral emergency
doi:10.71126/M20 8080 contraception option for women with a BMI greater
than 26.
Item 64 Answer: D . The copper intrauterine device is the most effective
Educational Objective: Provide emergency form ofemergency contraception option and can
contraception. reduce the risk for pregnancy by 99'/. if placed within
5 days of unprotected sexual intercourse.
The most appropriate emergency contraceptive option for this
patient is the ulipristal acetate oral contraceptive pill (Option
Bibliography
D). Emergency contraception refers to postcoital contracep Woodhams EJ, Gilliam M. Contraception. Ann lntern Med. 2019;l7O:lTCt8
tion using a device or medication to prevent pregnancy after 32. IPMID: 307167 s9l doi:10.7326 /AITC20l 902050
inadequately protected intercourse. All forms of emergency
contraception work by disrupting ovulation or preventing
fertilization of an egg. Emergency contraceptives are not
abortion inducing interventions. FDA approved options for
emergency contraception include oral ulipristal acetate, oral
Item 65 Answer: B
Educational Objective: Classiff the severity of a burn. tr
levonorgestrel, or a copper intrauterine device (lUD). New 'lhe classificrtion <;f this patient's burn is second clegree,
data suggest that the 52 mg levonorgestrel IUD is noninferior superficial partial thickness (Option B), which in'uolves the
to the copper IUD for emergency contraception, but it is not entire epidennis and penetrtrtes inb, bnt not through. thc
currently FDA approved for this indication. Ulipristal acetate clcrtnis. l]ecause thc epidermal layer is lost, the wound fbrrns
delays ovulation. It is the preferred oral contraceptive option a blister or n'eeps illterstitial f luid. The rn,ouncl blanchcs rvith
for women with a BMI greater than 26, such as this patient. pressurc and is painful to air. tenlperatrlre, and pressure
A single oral dose of ulipristal acetate is eft'ective for up to because thc vesscls ancl nerve of the dernris are still intact.
5 days after unprotected sexual intercourse. Although a pre- unlike the case with second degrec, cleep pafiial thickness
scription is required for oral ulipristal acetate, an in person burns lnd third clegree burns. Ilealing occurs through
medical visit is not required to provide this form of emer re epitlrclialization. Wound care lilr second clcgrce, par
gency contraception. The copper IUD is FDA approved for use tial thickr.ress burns includes cleaning the rtottt.tcl ruith soap
as emergency contraception and is the most effective fbrm and water and applying a topical antibiotic ointnlent. Spc
ofemergency contraception; it can reduce the risk for preg ciirlized wound dr-essit.tg will nraintlin a t.lloist environment
nancy by 99% if placed within 5 days of unprotected sexual and rnay renrain in place fbr several clays after thc injuryL
intercourse. Copper IUDs prevent fertilization by unknown Surgical excision and skir-r grafting shoulcl be consiclered ftrr
mechanisms and require a clinic visit fbr insertion. rtouncls that clo not re epithelialize after 2 kr 3 rt'eeks.
Emergency contraception can be provided with off A tirst degrec, ttr superficial, bttrn (Option A) dtles not
label use of combined oral contraceptive pills (Option A) penetrrte through the epiderrnis ancl presetlts rvith recl.
containing ethinyl estradiol or either norgestrel or levo- tender skin but no blistering or rt'eeping. Minimirl or no
norgestrel, but the dosing schedule is very different from the trcirtment is requircd.
201
Answers and Critiques
tr
CONI
A second-degree, deep partial thickness bum (Option
C) involves destruction of the epidermis and the dermis.
It presents as a wet or dry u,ound that is painful only to
patient given the degree of his symptoms. Topical agents in this
category include olopatadine, alcaftadine, bepotastine, azelas
tine hydrochloride, cetirizine, epinastine, ketotifen fumarate,
pressure. The wound may be any color most commonly and emedastine. Generic ketotifen fumarate does not require
white. yellor.l', or red. llealing is less likely to occur rvith re, a prescription. Although the full effect of these agents may take
epithelialization, and surgical evaluation may be necessary. days, some effect is seen immediately. An advantage of this
This patient's burn is painful rtithout pressure. so a deep class of drugs is that it is administered only once or twice daily,
partial-thickness burn is incorrect. depending on the speciflc formulation. The most common side
In a third degree, or full-thickness. burn (Option D), the eflect is buming and stinging with application, which may be
entire epidermis. dermis. and lat are destroyed. These wounds mitigated by using refrigerated agents.
are dry and often black in color (as shown) and cause minimal Topical antibiotics (Option A), such as trimethoprim-
pain because the nenes of the dermis have been destroyed. polymyxin B, have no role in the treatment of uncompli-
Excision and skin grafting are required for treatment. cated allergic conjunctivitis. This class of drug is effective for
vt treating acute bacterial conjunctivitis in patients who do not
€ {f-,q
(D wear contact lenses. Factors favoring bacterial conjunctivi-
vt tis include glued eyes in the morning, redness completely
o, obscuring the tarsal vessels, purulent discharge, and a red
ct eye observed at 20 feet. Absence of a purulent discharge
n makes bacterial conjunctivitis very unlikely.
Topical glucocorticoids (Option C), such as prednis
lt
(D
olone, are eflective for allergic conjunctivitis but are asso
vt ciated with serious potential side ellects, including vision
loss; glaucoma; corneal damage; and, with prolonged use,
cataracts. They should be reserved for patients who have
refractory symptoms and administered only under close
supervision of an ophthalmologist for short periods. They
would therefore be inappropriate to use in this patient.
A mast cell-stabilizing agent (Option D), such as cromo-
lyn, is eflective therapy for allergic conjunctivitis. However, it
needs to be used for up to 14 days before complete efficacy is
reached and requires administration four times daily, which
may decrease adherence. Therefore, use is reserved for those
o Second-degree, superficial partial-thickness burns
patients in whom other therapies have failed.
present as blisters or weeping wounds that blanch
Topical NSAIDs (Option E) function by blocking the
with pressure and are painful to air, temperature, and breakdown of thromboxane. Although topical NSAIDs have
pressure.
some efficacy in the treatment of allergic conjunctivitis com-
. Wound care for second-degree, partial-thickness pared with placebo, they are less effective than antihista-
burns includes cleaning the wound with soap and mines and are not a flrst-line treatment.
water and applying a topical antibiotic ointment fol-
lowed by an appropriate dressing.
rit'?otf,lt .:':, ',,'
o Perennial allergic conjunctivitis presents with bilat-
Bibliography eral eye redness, pruritus, and watery discharge.
Greenhalgh DG. Management ofburns. N Engl J Med. 2019;380:2349 59 o Topical antihistamines with mast cell-stabilizing
IPMID: 311890381 doi:10.1056/NEJM ral$O7 442
effects are first-line therapies for uncomplicated
allergic conjunctivitis.
Item 66 Answer: B
Ed ucationa I Obiective: Treat allergic conjunctivitis. Bibliography
Bielory L, Delgado L, Katelaris CH, et al. ICON: diagnosis and management
An antihistamine with mast cell-stabilizing properties is the of allergic conjunctivitis. Ann Allerry Asthma Immunol. 2020:124:ll8
34. IPMID: 31759180] doi:10.1016,,j.anai.2019.11.014
most appropriate topical treatment (Option B). This patient
presents with bilateral eye redness, pruritus, and watery dis-
charge, which are characteristic ofperennial allergic conjunc-
tivitis. Allergic conjunctMtis is due to IgE-mediated mast cell
Item 67 Answer: A G_
degranulation, with the resultant release of histamine and Educational Objective: Screen for colorectal cancer in a
other inflammatory mediators. In addition to basic eye care, person who does not prefer colonoscopy.
including cold compresses, and avoiding eye rubbing and An annual fecal immunochemical test (FIT) (Option A)
known allergens, topical pharmacotherapy is indicated for this is the most appropriate colorectal screening test for this
202
Answers and Critiques
203
Answers and Critiques
204
Answers and Critiques
tr Item 71 Answer: A
Educational Objective: Treat testicular torsion.
Item72 Answer: C
Educational Objective: Recommend timing of surgery
tr (J
?,
a!
following a stroke. UI
The most appropriate management is immediate surgical (l,
exploration (Option A). This patient has a classic prescnta 'lhe most appropriate recomnrendation is to dclay surgery
t,l
=
tion of testicular torsion, a twisting of the spermatic cord fbr at lcast 9 n.ronths fbllowing this patient's stroke (Option
contcnts resulting in testicular ischcrnia. Testicular torsion C). Prior stroke or transient ischemic attack (TIA) is a
is nlost comnron in children and your.rg adults and prcsents significant risk factor for recurrent stroke ir.r general and
as acute unilateral scrotal pain and swelling. Examination during the perioperative peririd and also increases the risk
reveirls elevation olthe im'olved scrotum ancl an absent cre- for a n.rajor adverse cardiac evcnt. Historically, it has beer-r
mirsteric ref lex (failure of the testis to elevate when the ipsi recomnrended to delxy elective noncardiac surgery fbr a
lateral inner thigh is stroked). I)ecreasecl bloocl flow seen on minimum of 3 montl.ts lbllowing stroke clr TIA owing to
!
Doppler ultrasound is typical. l'esticular torsiorr is a surgical unacceptably high risk fbr recurrent stroke and impaired
emergenc),,, ancl this patient should undergo immediatc sur- cerebral autoregulatior.r during that time franrc. However,
gical exploration. The time fiont symptom onset to surgical more recent data show that this excecdingly high risk fbr
t
exploration is the most important determining f:rctor of tes recurrent stroke persists longer than il montl.rs, graclually
ticular viability. If surgical exploratior.r is pertbrmed within returrring to a new baseline 9 to 12 months after the event,
6 hours of'sympton.r onset, the testis remains viable in 90'1, at r,rftich point it still remains elevated cclrnpared r.r,ith
to 100')1, of cases. Testicular viabilit_v drops to 50',r, if surgery persons who never had :r strokc or TIA. Although surgeries
is not performed until 12 hours after presentation and to less deen.rcd morc time sensitive or urgent may proceed af'ter
than 10',{, if pertbrn.red 2,1 hours or more afler presentation. 3 months, it is nrost prudent to rt,ait at least 6 nronths alter
I{ surgery is not reaclily available, an attempt to untwist the a priur stroke, and possibly as long as 9 months prior to
testicle manually is warranted. hven iI successtul, folktw up elective proccdures. It is also important to notc that sec
sulgery is typically required. ondary stroke prophylaxis with antiplatelet agents is ofler.r
Iiimpiric antimicrobial therapy with intran.ruscular ccttri held Ibr prokrnged periods befbre and after spine surgery
axor.rc and oral doxyclaline and scrotal eleration (Options B, owing to bleeding risks. rendering the patient at cven
C) are both appropriate management fbr epididymitis and greatcr risk for recurrcnt strokc.
epidiclymo orchitis in men younger thar.r 35 years, but not fbr Waiting 9 to 12 months for the risk for perioperative
a patient with suspected testicular torsicl.r. Epididymitis and stroke to reduce to baseline is ideal. although the risk tbr
epicliclymo rlrcl.ritis typically h:l,e a slou,er or.rsct and cause recurrent stroke remains elevated. Avoidir-rg surgery (Option
less severe painl on physical examinatior.r, there is an intact A) is r.rot absolutely neccssary and after the bencfits and risks
crenrasteric rellex ancl the involred testicle is not elevated. hare bcen weighed, surgery can be planned with appropri
Ilecause of'the very high likelihood of tcsticular tor ate risk mitigation after an appropriate interval after stroke
sion and tlre urgency fbr surgical exploratior.r in testicular or TlA.
torsion, sLlrgery should not be delayed to obtain irnrg Stroke or TIA witl-rin 30 days of planned surgery
ing studies. Although ultrasonography of'the scrotum (Options B, D) significantly increases the risk fbr perioper-
(Option D) is highly sensitive (89'1,) and highly specilic ative stroke; cluring this period. electivc surgeries should be
(99')1,) and hiis a false-negative rate of approximately 1'L avoidecl. Patients with stroke involving a large brain volume
fbr diagnosing testicular torsion, it is not mandatory or with a reccnt hemorrhagic stroke also are at risk for cere-
or nccessary in patients with a supportive l.ristory and bral hemorrhage if placed on cardiopulrnonary bypass and/
unequivclcal physical examination findings. iis exernpli or anticoagulation. If possible, nonemergency r.najor cardiac
fied by this patient. Scrotal ultrasonography is of most proccdures should be avoided.
205
Answers and Critiques
IEY POIilTS
. Patients at the highest risk for perioperative stroke
include those with a history of prior stroke or tran
sient ischemic attack.
o Elective surgery should ideally be delayed at least
6 months after a prior stroke, and possibly as long as
9 months following stroke or transient ischemic attack.
Bibliography
Benesch C, Glance LG. Derde),n CP. r,t rli American Heart .\ssociation Stroke
(iruncil: Council on i\rteriosclerosis. Throntbosis and \irscular Biolop5 r
(iruncil on Cardiovirscular and Stroke Nursing: Council on Clinical
Crrdiolo[X/l Council on lipidemiokr!l/ and Prevention. Perioperative neu
rological elaluation and management to los,er the risk ofacute stroke in
UI
prtients undergoing noncardiac. nonneurokrgical surgery': ir scientific
statement liom the Anrerican l{eart Association Americirn Stroke
=
.D
Association. (lirculation. 2021:CIR00000000OO0O0968. IPMID: 33827230]
UI
q, doi: I 0.11 6l rCI R.00OOO000OOOOO968
a
r.l
Item 73 Answer: D
.lt Educational Objective: Diagnose scabies.
tr Because of the lower mite burden in the typical presen
o
UI
The most likely diagnosis is scabies (Option D), an intensely tation of scabies, the scabies preparation may be negatire:
pruritic skin condition that presents with papules and bur diagnosis, however, can be made on the basis of the history
rows on the fingers and web spaces. Other common areas of (itch. close contact with individuals with similar lesions or
involvement are the breasts, wrists, axillae, waistline. geni- itch) and distribution of lesions. Treatment is topical perme
talia, and ankles. Scabies typically spares the face and scalp thrin or oral ivermectin.
in adults. Pruritus is usually worse at night. Scabies is caused Bed bug bites (Option A) present as erythematous pap-
by Sorcoptes scobiei. which lives in the human epidermis. ules, typically in a linear pattern, on exposed areas ofthe skin.
Skin to skin contact, including sexual transmission, is the Bites occur at night when the bed bug. Cimex lectulorius,
primary mode of transmission. Outbreaks are common in typically feeds. Bites have characteristically linearly arranged
families, institutional settings, and the homeless population urticarial papules ("breakfast. lunch. and dinner") and are
due to its highly contagious nature. Crusted scabies, a form usually fewer in number than seen in this patient. Bed bug
of scabies with a significantly higher mite load (thousands bites are not consistent with this patient's skin findings.
versus 5 15 mites in typical scabies), presents lvith wide Dyshidrosis (Option B). or dyshidrotic eczema. is a form
spread psoriasiform and hyperkeratotic lesions; it is more of hand dermatitis characterized by vesicles on the hands
common in immunocompromised patients. The skin find and fingers; it is also common on the feet. Dyshidrosis is
ings of crusted scabies are shown. often exacerbated by irritants, such as u,ater and detergents,
and is more common in patients with atopic clermatitis. It
does not present u,ith burrows, as seen in this patient.
lmpetigo (Option C) is a bacterial infection of the
skin most commonly caused by Stophylococcus oureus.
group A streptococci. or both. Impetigo presents as honey
colored crusted erosions or bullae, commonly on the face.
This patient's skin findings are not consistent with impetigo.
IEY POIlIII
o Scabies is a skin infestation with Sorcoptes scobiei
characterized by papules and burrows on the fingers
and web spaces, breasts, wrists, axillae, waistline,
genitalia, and ankles; it is transmitted by skin to skin
contact and can be treated with topical permethrin or
oral ivermectin.
o Crusted scabies is a form of scabies with a high mite
Diagnosis of scabies is conflrmed using :r mineral oil
load that presents with widespread psoriasiform and
preparation of skin scraping for microscopic identification
hyperkeratotic lesions; it is more common in immu-
of mites, eglls, or feces under the microscope. as shown (see
nocompromised patients.
top of next column).
206
Answers and Critiques
tr Item 74 Answer: A
Educational Objective: Assess risk in a patient with
medical management; a MELD score greater than
20 precludes all but the most urgent surgeries for life
threatening illness.
cirrhosis before elective surgery.
'lhe mt.rst appropriate perioperati\e
ntanagemeltt is to coun Bibliography
se1 the patient r-rn increasecl surgical risk as it relates k) his Northup l'(;. l.riedmln I-S. Kantath PS. AGA clinical practice updatc on
diagnosis of'cirrltosis (Option A). Dr-rring the perioperative surgical risk assessntent and perioperative nranagement in cirrhosis:
expert review. Clin (lastroentcRrl IIepatol. 2019;17:59.5 606. IPMII]:
period, prtients with cirrhosis arc rrt significar-rtly greater 302737Sll doi:10. l0l 6/j.cgh.20l t].09.043 t
risk fbr electrol_vte abnormllities. Iluid imbalance, deliriurn. q,
kidnel, failure. r.r'orsening liver dl,stunction. impaired clear ET
ance ol nlcdiciltiorls, ir.rf'ection. bleecling, clotting, and death. Item 75 Answer: A
'lhe likelihood of poor outcontes increases rt,ith
the extent of Educational Objective: Treat obesity with (J
lirer disease as rncasured by \,lodel tbr End stirge I-iver L)is pharmacotherapy. -,
.g
case (r\,1il.1)) or Child Turcotte-Pugh score. 'lhe MELD score 'lhe most appropriate management is liraglutide (Option A), UI
ir-rcorporates the INR. serunt total bilirubin level, and serun-r c,
a glucagon like peptide 1 (GLP-1) receptor agonist. Phar
creatinine lerels. A progressiYe increase in perioperative ntrtr UI
macotherapy is an option in the treatment ol obesity and =
E
talitl' lirr all proccdures is <tbsencd as MELD scores rise.
overweight for patients with BMI of 30 or greater or fbr
[)atients who have MEI.D scores below B to l0 generally do
those with BMI of 27 or greater and at least one obesity-
uell pcrioperativell, pr-oviciecl that their lir,.er clisease is stable
associated comorbid conditiorl (such as diabetes, hyper
iit-td contpensated, and it is reasonable to proceed rtith rlost
tension, or obstructive sleep apnea) who have not achieved
surgerics with optimal medical milnxgcment. When a MEI-t)
weight loss goals with a trial of at least 3 to 6 months of
score is greater than 20. mortalit!' rates increase to a dcgree
lilbstyle modiflcation. Pharmacotherapy is contraindicated
that prcclucles all but the most urgerlt surgerics fbr lite thrcat-
in pregnancy, and women of childbearing age should be
ening illness. A M ELD score ot 14 (as this patient has) inclicates
advised to avoid pregnancy while using these medications.
intermecliate risl< ftrr surgcry related complications. ancl the
The goal of pharmacotherapy is weight loss of 5'7, or more
risks and benefits of tl-re allticipated clecti'"e surgerl shoulcl be
during the first 3 to 6 months. If this target is not reached,
discussecl r,vith tl.rc patient al'ter the perioper:rtive evaluation.
use of the medication should be discontinued. Weight is
In patients r,r'ith cirrhosis ancl asymptom;itic clev:r
typically regained once pharmacotherapy is discontinued,
tior.t of the prothron.rbin tinre or INR, prophy'lactic trcat-
which underscores the importance of concomitant lifestyle
ment nrith either plasma or cryoprecipitate (Option B) is
changes. '[he potential weight loss beneflt of pharmacother
not indic'ated. 'lhcre is no cvidence of benefit with these
apy should be balanced against risk fbr adverse events and
intervclrtions and there is evidence of harnr. particularly
cost. Liraglutide in an injectable agent that acts to decrease
the risks fbr transfusion reactions xnd volunte overlotd.
gastric emptying, thereby increasing satiety, and has been
Patients n'ith cirrhosis ilnd platelet counts o1'50.0001pI-
associated with weight loss ot'5.2 kg (11.6 lb) compared with
(50 x 10" L) or grcater do not requirc platelet transfusior.rs
placebo. Contraindications include a history of medullary
prior to surger),.
thyroid carcinoma and family history of multiple endocrine
In the
abscnce of'dccompensatecl liver clisease (vari
neoplasia type 2, and it should be used cautiously in patients
ceal hcrnorrh:rgc. ascites, encephlkrpathl') ir NlELD score
with pancreatitis. Of the pharmacologic treatment options,
lcss than 15 clocs not mect criteria firr lir.er transplantation
liraglutide is the weight loss medication most likely to be
(Option C).
'lhis patient is at increilsecl risl< for srlrgery rclated well tolerated in this patient.
Naltrexone bupropion (Option B) is an opioid antag
corxplicxtions. Before proceeding r,r'ith surgery in such
onist and norepinephrine dopamine uptake inhibitor that
patients. it is impurtant to cliscuss w,ith theur the poterl
suppresses appetite. It has a higher discontinuation rate
tiiil risks and benefits of hip artl.rroplastl, (Option D). compared with other pharnlacologic options. The most com
inclucling alternative therapics that do not in\'ohre sLlrgical
mon adverse eflbcts include nausea, dizziness, changes in
intervclrtioll.
bowel habits, and insomnia. Contraindications include sei-
I(EY PO I XTI zure disorders. eating disorders, any current opiate use or
o A progressive increase in perioperative mortality for withdrawal, or any risky alcohol use. Because this patient
has insomnia, this drug combination would be a poor choice.
all procedures is observed as Model for End stage
Orlistat (Option C) is an intraluminal lipase inhibitor
Liver Disease scores rise.
(Continued) that induces weight loss by decreasing triglyceride absorp
tion. Although safb, it causes gastrointestinal side efl'ects,
207
Answers and Critiques
including oily stools, increased defecation, and fecal urgency infections have more signiflcant erythema, tenderness, and
or incontinence in as many as 30'/, of patients. Contraindi often exudative drainage. This patient has pruritus and a
cations include malabsorption syndromes and cholestasis. classic presentation of allergic contact dermatitis. and thus
This patient has irritable bowel syndrome with diarrhea, and mupirocin is not the best treatment option.
it is unlikely that she will tolerate the gastrointestinal side Prednisone (Option C) can be used for severe systemic
eflects of orlistat. drug reactions, such as a systemic reaction to an antibiotic,
Phentermine-topiramate (Option D) is a noradren but it would not be appropriate for treating this localized l
ergic y-aminobutyric acid receptor activator and AMPA contact dermatitis. Systemic drug reactions are typically
glutamate receptor inhibitor. Contraindications include morbilliform, widespread, and symmetric. This patient has
glaucoma, hyperthyroidism, and nephrolithiasis. Caution is no indication of a systemic drug reaction to the course
advised in patients with hypertension or resting tachycardia. of amoxicillin he had received, and prednisone is not
Because this patient has a history of nephrolithiasis, phen warranted.
D termine-topiramate is contraindicated.
TEY POIXIS
3
tt
€ XEY POIt{I . Contact dermatitis is an eczematous eruption that
.D
UI
o Pharmacotherapy is an option in the treatment of occurs at the site of allergen contact with the skin and
o, obesity and overweight for patients with BMI of 30 or is characterized by pruritic, erythematous patches or
EL greater or for those with BMI of 27 or greater and at plaques that may show vesicles and weeping acutely,
a't Ieast one obesity-associated comorbid condition who or dry scaling and lichenification at later stages.
Item 76 Answer: D
Ed u cationa I O bjective : Treat allergic contact Item 77 Answer: D
dermatitis. Educational Objective: Avoid aspirin for primary
prevention in patients on anticoagulation.
The most appropriate treatment is to add triamcinolone
cream (Option D). This patient developed allergic contact Aspirin is not recommended (Option D) fbr patients at
dermatitis to bacitracin ointment. Contact dermatitis is higher risk for bleeding, such as this patient taking rivarox
an eczematous eruption that occurs at the site of allergen aban. The U.S. Preventive Services Task Force recommends
contact with the skin and is characterized by pruritic, ery- low-dose aspirin for the primary pievention of atheroscle
thematous patches or plaques that may show vesicles and rotic cardiovascular disease (ASCVD) and colorectal can-
weeping acutely. or dry scaling and lichenification at later cer in willing adults aged 50 to 59 years with a 1O-year
stages. Although the patient discontinued using bacitracin ASCVD risk of 10'X, or higher, a life expectancy of at least
3 days before presenting, erythema and pruritus persist and 10 years, and no increased risk for bleeding. This patient has
would be best treated with a topical glucocorticoid, such as a 10 year ASCVD risk greater than 10'){, and may be consid
triamcinolone cream. Bacitracin, polymlxin B, and neomy ered for aspirin therapy lor primary prevention of ASCVD.
cin are common topical antibiotics used to treat or prevent Tempering the recommendation lor primary prevention of
wound infections and are fiequently associated with allergic ASCVD with aspirin are multiple trials, including ASCEND.
contact dermatitis. For clean wounds, such as a skin surgical ARRIVE, and ASPREE. and a 2019 meta-analysis showing
site, the application of plain petrolatum is recommended no net mortalify benefit with aspirin; reductions in nonfatal
over the use oftopical antibiotics. myocardial infarction and ischemic stroke were offset by
Candida infections are rare in clean surgical wounds, an increased risk for major bleeding, including intracranial
and treatment with ketoconazole cream (Option A) is not hemorrhage. Factors that increase risk for bleeding include
indicated for this patient. Candido infections occur more increasing age, male sex, concurrent anticoagulant or NSAID
commonly in intertriginous areas and are characterized by use, history of gastrointestinal bleeding, upper gastrointesti
erythema with satellite papules or pustules. This patient's nal pain, uncontrolled hypertension, chronic kidney disease,
skin flndings have no features of Candida infection. and thrombocytopenia. Because of these data, the American
Although mupirocin ointment (Option B) is less likely College of Cardiologr and the American Heart Association
than bacitracin to cause allergic contact dermatitis, there recommended that aspirin should be used infrequently in
is no indication of wound infection in this patient. Wound primary prevention of ASCVD and that low-dose (81 mg/d)
208
Answers and Critiques
aspirin (Option A) might be considered for primary preven- Recommending l pitl box (Option B) and selecting an
tion of ASCVD only in some individuals aged 40 to 70 years anticoagulant rvith the lowest copayment (Option D) are
who are at high risk for ASCVD but do not have an increased recognized strategies [o increase meclication adherence. Both
bleeding risk. methods are usually acceptable to patients. Selecting ir med
I The addition of a proton pump inhibitor, such as icrrtion with the lowcst copayment is challenging Llecause
omeprazole (Option B), is recommended by the American it requires physicians to maintain knowledge of various
College of Cardiologr for patients with established ASCVD rnd shifting copayments with a hrge uurnber oi'insurauce
disease who require treatment with two or more antithrom conrparries and is tin-re intensive. anci its inrpact is unkuort,n.
botic agents. These patients should also avoid NSAIDs. How- More important. it appears that the primary reason ft)r this
ever, this patient should not take aspirin and therefore does patient's nonadherence r,ras lack of trnderslanding ol the
not need to take a proton pump inhibitor. prolonged treatment duration tbr deep venous thrombo
Higher doses of aspirin, such as 325 mg (Option C), have sis. Patient education is more likely kl increase meclication
not been shown to be more effective in primary prevention adherence than either a pill bor or lowest copaymerlt. t
o
of ASCVD than low-dose aspirin and are associated with Scr-eening lbr klrv health literacy (Option C) may uncover
\ increased risk for bleeding. Ifaspirin were indicated as a pri health literacy pnrblems but u,ill not impror,e adherence to
ET
mary prevention measure in this patient, low dose aspirin tl.re required ar.rticoagulation plan. C.lirricians can adclress rJ
would be appropriate. low lrealth literacy by using techniques b improve patient
understanding, such as rcpeating inlirrnration and supplying
=,
XEY POITIIS l!
the patient with eclucational materirls writtel-r in plain lan UI
. Aspirin should be used infrequently in primary pre- guage. the teach back method, rn,herein the clinician asks
(l,
vention of atherosclerotic cardiovascular disease but patients kl describe their understzrncling of instructior.rs in U!
=
should not be used in patients at increased risk for their ou.n worcls, is especially helpfLrl in rlpidly identilying
bleeding. a potential health literacy problern beftrre a patient leaves
. Low-dose aspirin may be considered for primary pre- a mcdical setting. Ihe teach back nrethod coupled with
vention of atherosclerotic cardiovascular disease eclucation about the prtient's medical pniblem and need fbr
(ASCVD) in some individuals aged 40 to 70 years who prolonged treatmelrt is more likely to increase nredication
are at high risk for ASCVD and do not have an
aclhcrence than is screening for lort' health literacy alone.
tr Item 78 Answer: A
Educational Objective: Provide patient education to
promote medication adherence.
209
Answers and Critiques
tr
CONI.
highest risk patients. Pharmacologic thromboprophylaxis
fbr patients undergoir.rg nrajor surgery for cancer should
be continued for at least 7 to 10 days. Extended prophy'1axis
present \\-ith acute onset ol local symptonls (d)'suria. uri
nary tiequencl' and urgcncy, sr-rprapubic and or perineal
pain) and s!'stemic symptonls (i'evers. chills. nausea r,om
with LMWH for up to ,1 weeks postoperativell,' is reconr iting, rnalaise). Most cases of acute bacterial prostatitis are
mended for patients unclergoing n.rajor open or laparoscol.ric caused by' gram negative bacteria, including Escherichict
abdominal or pelvic surgery ltlr cancer rvho have high risk coli arrd Klebsiello, Enterobacter, Proteus, and Serrotio
Ieatures. such as restricted rnobilitl,. obesitl,l or history of species. Oral trimethoprinr suliamethoxazole has both
VTE. or have additional risk factors. This patient is excellent co\:erage of grarn negative bacteria and prostatic
"vho
undergoing high-risk surgery and has obesiS' as an addi tissue penetration. lt is a reasonable empiric antibiotic
tional risk fbr VTE, and extended prophylaxis is u'arranted. choice in patients able to tolL'rate oral intake. A fluoro-
'Ihe ASCO and other guidelincs assert that there is ntr quinolone antibiotic (ciprofloxacin. ler,ofloracin) is also
role fbr inferior vena cava (l\tC) filter insertion (Option A) an appropriate empiric antibiotic choice. Antibiotic ther
fbr VTE prophylaxis ou,ing to collcerns about its long-term apy should be initiated after a urine sample is obtained
UI harm. IVC filter placement may be considered in patients fbr ar.rall'sis and culture and should be continued for 2 to
.D with absolute contraindicatiorls to anticoagulant therapy in 4 r,r,eeks: some experts recornrnertd up to 6 r,reeks. Treatntent
=
ut the acute treatment setting olV'l'E if the thron-rbus burden is can be modified on the basis of results of the urine culture.
o, considered life-threatening. CT or MRI of the prostate (Option A) is not indicated fbr
EL Prophylaxis n,ith LMWH fbr 7 days (Option B) is patients with symptoms and pl.rysical examination findings
n unlikely to provide adeqrlate prophylaxis for this patient ol.acute bacterial prostatitis. CT can be obtained if there is
st with cancer undergoing rrajor surgery Registry data have diagnostic uncertainB' or if the patient does not improre
(D
documented that postoperative VTE is detected after hospital atter appropriate antibiotic theraplr because antibiotic f,ail
art discharge in 54')1, of cases. n,itl.r the highest risk correspor.rd ture raises the suspicion for prostate abscess.
ir.rg to colorectal and genitourinary cancer surgery Studies Although nitrofurantoin (Option B) is appropriate for
have demonstrated that administriition of LMWH for 30 clays uncornplicated cystitis, it has poor penetration into prostatic
after the day of surgery reduccs tl.re risk for VTE rvitl.rout tissue. lt is not indicated fbr a patient with acute bacterial
increasing the risk for bleeding cornplicatior.rs. prostatitis.
No additional prophylaxis is not appropriate therapy Although prostatic nlassage follorved by urine culture
(Option D). The 2019 ASCO guideline for VTE propl.rylaxis (Option C) has a role in patients rvith suspected chronic
in patients u,ith cancer recommends pharmacologic pro bacterial prostatitis. it should be avoided in patients u,ith
phylaxis fbr hospitalized patients u,ith active cancer in the acute bacterial prostatitis because it can potentialll,cause
absence olbleeding or other contraindications. bacteremia and sepsis.
Placement of a urinary catheler (Option E) is neces
TEY POITITS
sary in patients lr,'ith prostatitis experiencing acute urirrlry'
o Pharmacologic venous thromboembolismprophy rctention. Acute urinary retention is usually associated n,ith
laxis is recommended for hospitalized patients with inability to urinate and lolr,er arbdon.rir.ral and or suprapubic
active cancer in the absence of bleeding or other discomfbrt. Although this patient has obstructive urinar-r'
contraindications. svmptoms, he has no evidence olacute urinary retention.
o Extended venous thromboembolism (VTE) prophy f,EY POIXTS
laxis with lowmolecular-weight heparin for up to
4 weeks postoperatively is recommended for patients
. Bacterial prostatitis is associated with acute onset of
local symptoms (dysuria, urinary frequency and
undergoing major open or laparoscopic abdominal or
urgency, suprapubic and/or perineal pain) and systemic
pelvic surgery for cancer who have high-risk features,
symptoms (fevers, chills, nausea/vomiting, malaise).
such as restricted mobility, obesity, or history of VTE,
or who have additional risk factors. o Oral trimethoprim-sulfamethoxazole or an oral fluo-
roquinolone (ciprofloxacin, levofloxacin) is an appro
Bibliography priate empiric antibiotic for acute bacterial prostatitis.
Key NS, Khorana AA, Kuderer NM, ettl. Venous thromboembolism prophy
laxis and treatment in patients with c:]ncer: ASCO clinical practice Bibliography
guideline update. J CIin Oncol. 2020:38:496 520. IPMID: 31381464]
doi:10.1200/JCO.19.01461 Gill BC. Shoskes DA. Bacterial prostatitis. Curr Opin Infect Dis. 2016:29:
86 91. IPMID: 26555038] doi:lO.lO97 QCO.OOOOO0000o000222
80
tr Item Answer: D
Educational Objective: Treat acute bacterial prostatitis.
Item
Ed
81
ucation a
Answer: A
I O bj ective : Treat infl amed seborrheic
keratoses.
lhe most appropriate next step in management is en.rpiric
trirnethoprim-sulfamethoxazole (Option D). This patient The most appropriate treatment is cryotherapy (Option A).
has acute bacterial prostatitis. These patients typically This patient has seborrheic keratoses, the most common
210
Answers and Critiques
benign skin tumors in adults. Typically occurring after mid be tested for rubella immuni[2, and those who lack immunity
dle age, seborrheic keratoses present as warI/, "stuck on" should be ollered the MMR vaccine; women should avoid
papuies or plaques, most commoniy on the trunk; they conceiving for at least 4 weeks after vaccination. If a woman
involve only the epidermis. Seborrheic keratoses range in lacks immunity to rubella but is already pregnant, she should
size from a few millimeters to several centimeters, are usu- receive the MMR vaccine aller delivery but before leaving the
ally hyperyigmented, and range in color from tan to brown hospital or at the time of pregnancy termination. The MMR
or black. The sign of Leser Trdlat is the sudden eruption of vaccine is a live virus vaccine and should not be administered
numerous seborrheic keratoses and is associated with an to immunocompromised individuals or pregnant women.
internal malignancy. Diagnosis of seborrheic keratoses can Patients should be administered the human papilloma
usuallybe made clinically, but shave biopsy can be conflrma virus (HPV) vaccine series (Option A) at age 11 or 12 years
tory when there is concern for melanoma or squamous cell or between 13 and 26 years of age if not given previously. In
carcinoma. Reassurance is typically the appropriate manage unvaccinated patients aged 27 to 45 years, vaccination can
ment. but seborrheic keratoses are often treated if irritated be considered on the basis of risk using a shared decision tn
(l,
or pruritic, such as in this patient, or for cosmetic reasons. making process. If administered before 15 years of age, a 3
CT
Seborrheic keratosis usually become irritated and inflamed two dose series is recommended. whereas a three-dose
owing to chronic friction from clothing, but inflammation series is recommended in older individuals. Vaccination is U
of seborrheic keratoses may occur during chemotherapy; not recommended during pregnancy, although no harmful "!El
C
cytarabine and docetaxel are the most implicated drugs. effects have been noted when inadvertently given to preg- .E
UI
When indicated, the most common treatment is cryother- nant women and pregnancy testing is not necessary before
apy with liquid nitrogen, but rinly if the diagnosis is certain. vaccination. This patient received two doses of the HPV
o
3
ut
Otherwise, a shave removal and submission for histologic vaccine befbre 15 years ofage and does not need a third dose.
evaluation is most appropriate. Other destructive therapies All pregnant women should receive the tetanus toxoid,
include curettage, electrodessication, or laser therapy. reduced diphtheria toxoid, and acellular perhrssis (Tdap) vac
Mupirocin ointment (Option B) is effective in treat cine (Option C) during each pregnancy between 27 weeks'
ing superflcial skin infections typically caused by gram and 36 weeks'gestation. This patient is currently not preg
positive bacteria (staphylococci and streptococci). There is no nant and her Tdap vaccination is up to date. In patients who
indication of inf'ection in this patient, and mupirocin is not have previously received the Tdap vaccine, booster vaccina
appropriate treatment. tion with either tetanus and diphtheria toxoids (Td) or Tdap
Topical 5 fluorouracil (Option C) is an approved treat- booster is recommended every 10 years. Patients who have
ment for actinic keratoses. It can also be used to treat super not yet received the Tdap vaccine can receive it as one ofthe
ficial basal cell carcinomas and squamous cell carcinoma in scheduled booster vaccinations. This patient should wait until
situ. Topical 5 fluorauracil would not be appropriate man the appropriate time interval to receive the Tdap or Td vaccine.
agement for a seborrheic keratosis. Because the patient is planning pregnancy and does not
Topical glucocorticoids, such as triamcinolone aceton have immunity to rubella, not offering the MMR vaccine is
ide (Option D), have no role in the treatment of seborrheic not the best strategz (Option O).
keratoses because they are ineffective and over time may
XEY POITITS
thin the normal surrounding skin.
. Women who are planning pregnanry and do not have
I(EY POIl{T immunity to rubella should receive the measles, mumps,
. Seborrheic keratoses, the most common benign skin and rubella vaccine before conceiving and shottld'nait at
tumors in adults, present as "stuck on" papules or least 4 weeks after vaccination to conceive.
plaques; removal, most commonly with cryotherapy, . The measles, mumps, and rubella vaccine is a live
is considered if the keratoses are irritated or pruritic, virus vaccine and should not be administered to
or fbr cosmetic reasons. immunocompromised individuals or pregnant
women.
Bibliography
Ranasinghe GC, Friedman AJ. Managing seborrheic keratoses: evolving
strategies for optimizing patient outcomes. J Drugs Dermatol. 2017;
Bibliography
16:106,1 8. [pMtD: 29141054] Freedman MS, Bernstein H. Ault KA. Recommended adult immunization
schedule. United States. 2021. Ann lntern Med. 2021. IPMID, 33571011]
doi:10.7326lM20 8080
Item 82 Answer: B
Educational Objective: Provide measles, mumps, and
rubella vaccination in a woman planning pregnancy. Item 83 Answer: A
Educational Objective: Diagnose alopecia areata.
The measles, mumps, and rubella (MMR) vaccine (Option B) is
the most appropriate vaccine for this patient who iacks immu- This patient has alopecia areata (Option A), which results
nity to rubella. AII women in the preconception period shouid in well-circumscribed patches of nonscarring hair loss
211
Answers and Critiques
without scale. Alopecia areata aflects men and women TEY POIXIS
equally and most commonly presents before the age of . Alopecia areata is nonscarring hair loss that results in
30 years. It can progress to complete loss of scalp hair (alo
smooth, hairless patches of skin, most commonly
pecia totalis) and all ofthe body (alopecia universalis). Other
appearing on the scalp.
than the hair loss, alopecia areata is usually asymptomatic.
On examination, "exclamation point" hairs, which are small . "Exclamation point" hairs, which are small broken
broken hairs at the edges of the hair loss that are thicker hairs at the edges ofthe hair loss that are thicker at the
at the top and thinner near the scalp, can be observed; top and thinner near the scalp, are pathognomonic of
these are a pathognomonic indicator ofalopecia areata. The alopecia areata.
pathogenesis ofalopecia areata is not fully understood, but
an autoimmune process is postulated. Alopecia areata can Bibliography
be associated with atopic dermatitis, asthma, thyroid dis- Strazzulla LC. Wang EH, Avila L, et al. Alopecia areata: disease characteris
ease, and tlpe 1 diabetes mellitus. Patients with mild disease tics, clinical evaluation, and new perspectives on pathogenesis. J Am
Acad Dermatol. 2018;78:1 12. IPMID: 29241771) doi:10.1016 j.jaad.2ol7.
tt will often have spontaneous hair regrowth. More extensive 04.1141
E involvement at onset has a poorer prognosis. Treatment
.D
includes potent topical or intralesional glucocorticoids for
ut
o,
CL
limited disease.
Androgenic alopecia (Option B), or patterned bald
Item 84 Answer: B
Educational Objective: IdentiS high value care.
tr
r.l ness, is due to the postpubertal terminal hair replace- The American College of Physicians (ACP) High \hlue Care
lr ment, first with miniaturized follicles and eventually with initiative (Option B) ainrs to improve health. aroid harms.
atrophic follicles. It is seen in both men and women. The irnd elinrinate r,rasteful practices. This initiatire addresses
.D
.A onset begins after puberty and can be gradual. In men, it high r,alue care broadlyl olt'ering leirrning resources for clini
presents as bitemporal hairline recession, followed by ver- cians and medical educators. curricula. clinical guidelines.
tex thinning and then baldness (as shown). This patient's best practice advice, case studies. and patient resources on
circumscribed hair loss is not compatible with androgenic a wide variety of related topics. ACP believes that it is thc
alopecia. responsibility of the medical prof'ession to become cost
conscious and decrease unnecessary care that does not ben
cfit patients. ACP has developed r five step framer,','ork to assist
ph1'sicians in delir,ering high value care: (1) understand the
benefits. harms. and relative costs of the intenentions that
being considered: (2) decrease or eliminate the use of inter
ventior.rs that provide r.ro benefits ancl or may be harmtulr
(3) choose interventions and carc settings that manimize bene
fits. rninimize harms. and reduce costs (using conlparative
eff'ectiveness and cost eflectiveness data); (,1) customize lr
care plan nith the patient that incorporates their values
and addresses their concerns: rrnd (5) identify systenr level
opportunities to improve outcomes. rninimize harms, and
reduce health care \^,aste. Not perfbrming unnecessary test
ing, as shorvn in this case. is an erample ol high lalue care.
The practice of defensive medicine (Option A). thc
Discoid lupus erythematosus (DLE) (Option C) is ordering of excessire tests and procedures b1- ph1,-sicians to
a scarring alopecia that presents with erythematous avoid malpractice litigation. is the irr.rtithesis of high raluc
plaques with areas of scarring; it often occurs in the care. 'Ihe prevalence ol defbnsive medicine is a challenge
absence of systemic disease. The conchal bowls of the ears to the provision of high laluc care. Remedies that include
and other areas of the face are often involved. The absence ernphasizing patient saf'ety. pronroting a culture ol quality
of erythema and scarring in this patient is inconsistent improvement and coordinated care, and training physicians
with DLE. in best practices to ar,oid errors and reduce risk rvill pre
Tinea capitis (Option D) is most commonly caused vent harm and reduce the rvaste associated $,ith defensive
by Trichophyton and Microsporum species and typically medicine.
presents with pruritic single or multiple scaly patches. [-ean is a quality intproventent model (Option C) that
It is a common cause of hair loss in children but a less aims to maximize value and minimize lvaste by closely
common cause in adults. Severe cases may present with a examining a system's processes ancl eliminating non
kerion, an inflammatory plaque with pustules and drain- value added activities uithin the system. High ralue care
age. Cervical lymphadenopathy is common. The age of this may be an outcome of the Lean qualiry- improvement pro
patient and lack of pruritus or scaling make tinea capitis cess, but this physician's uppropriate decision to forgo
an unlikely diagnosis. unnecessary testing is more indicative of'high value care.
212
Answers and Critiques
tr
CONI.
i\,ledical rttioning (Option D), dcfinecl by the allocatiorr
of scarcc nteclical resources to ir sclcctecl group of'patients,
lelcls kr uncleruse of potentially lltltrrrpriirtc care. I Iigh valuc
Topical metronidazole vaginal gel (Option D), 5 g
daily fbr 5 days, is an eflective option for the treatment of
bacterial vaginosis. This form of metronidazole, however,
care uclv<lcates for rational clrc thirt is cl'uracterized by care is not curative and is inferior to a single 2 g dose of oral
that is clinically ellective, thus avoidilrg overuse or misuse ol' metronidazole or tinidazole for the treatment of tricho
carc that is inappropriate and wastcfirl. moniasis.
TIY POITTS t(EY POll{rS
o The aim of high value care is to improve health, avoid . Vaginal trichomoniasis is a sexually transmitted infec
harms, and eliminate wasteful practices. tion that can be asymptomatic or present with pale
. The practice of defensive medicine, the ordering of yellow, green, or gray frothy vaginal discharge with
excessive tests and procedures by physicians to avoid itching and burning; it is best treated with a single 2-g
malpractice litigation, is the antithesis of high value dose of oral metronidazole or tinidazole. Ut
q,
care and leads to waste and potential harm through . Women treated for trichomoniasis should be retested GT
unnecessary testing and treatment. in 3 months with a nucleic acid amplification test.
TJ
Bibliography Bibliography E
Carnrll AE. The high costs of unnecessary care. JAMA. 2017;318:1748 9 Workowski KA, Bolan GA; Centers tbr Disease Control and Prevention. |!
IPM I D: 29136432] doi:10.1001 ijama.2017.16193 Sexually transmitted diseases treatment guidelines, 2015. MMWR u!
Recomm Rep. 2015;64:1 137. IPMlD, 260.12815] o
UI
=
86 Answer:
Item 85 Answer: C
Educational Objective: Treat trichomoniasis.
Item D
Educational Objective: Avoid preoperative testing for
tr
low-risk surgery.
The most appropriate treatment is a single 2 g dose of oral
metronidazole (Option C). Vaginal trichomoniasis is a sexu No testing is indicated (Option D) ftrr this patient belirre
ally transmitted infection (STI). Patients can be asymptom ciltaract surgery. Preoperative testing can help define risks
atic or present with pale yellow, green, or gray frothy vaginal bcfirre undergoing a procedure and is intended to guide
discharge with itching and burning. Although trichomonia- lnanrgerrent. Hou,ever, there is considerable overuse of pre
sis can be diagnosed with microscopy when motile tricho operativc testing. particularl-v- in lorn, risk procedures. When
monads are noted on wet mount, nucleic acid amplification rletcrnrinir.rg appropriate preoperativc testir.rg ltrr electivc
testing (NAAT) is recommended because it can be dilflcult to procedures. the first branch point is detcrmining the risk
establish the diagnosis with a wet mount. Because trichomo ol surgeryi ln general, low risk surgeries (such as skin sur
niasis is sexually transmitted, patients should be screened for gcrics, hernia repair. cataract sLlrgery and arthroscopy) are
other STIs, including chlamydia, gonorrhea, HIV, and syph- o{ shorter duration. involve klcirI anestlretics. incur minimal
ilis. A single 2 g dose ofl oral metronidazole (or tinidazole) is blrxrd loss, and are associated with fl'w contplications (<1'X,
efl'ective treatment for established Trichomonas infection. It rnujor artverse cardiac evenls). Multiple studies have shown
is also important to treat the patient's partner to help prevent that preoperative testir.rg has little value ir.r low' risk proce
reinf'ection and spread of infection to other sexual contacts. clurcs. Even when patients have substantial comorbidities,
Patients should also be retested for Trichomonos in 3 months such as this patient, chest radiography, echocardiography,
with NAAT because of high reinf.ection rates. and UCG do not predict carclilc complications. Therefbre,
A single 1-g dose of oral azithromycin (Option A) is provided rnedical issues are stable, preoperative testiug is not
appropriate treatment for chlamydia but does not treat rec<lnrnrended tbr low risk pr<-rcedurcs.
trichomoniasis. Testing for chlamydia is appropriate in this Although this patient has cardiopulmonary disease,
setting because trichomoniasis is a STI, but treatment should inclucling COPD, her sylxptonls arc stable and chest radi
be limited to patients with established chlamydia infection. ography (Option A) results will n<-rt change perioperative
Fluconazole (Option B), 150 mg orally, is an appropriate nlilrlagement. Preoperative chest radiography in nontho
treatment for vulvovaginal candidiasis but is not appropriate racic surgery is indicated only in patients with new sigrrs
for the treatment of trichomoniasis. Vulvovaginal candidi- or sylxptoms of pulmonary illr.ress or unclerlying cardiac or
asis is typically characterized by vaginal itching, irritation, pulmonary disease with new or unstable symptoms.
and discharge. Examination reveals vulvar edema and exco Civen the low rates of cardiovascular complicatior.rs
riation, with thick, white, curdy vaginal discharge. Diagnos and minimal preciicti'n'e value they provide, screenitrg
I tic testing involves 10'7, potassium hydroxide wet mount of I')CC (Option B) is not helpful prior to lort' risk proce
the discharge showing yeast, hyphae, or pseudohyphae or a clures. Cardiac testing prior to ltx,r, risk surgery should
positive NAAT. Vulvovaginal candidiasis is not an STI; there be obtained only if the prrtient has nert, symptoms sug
is no need to treat partners of women diagnosed with vulvo gcstive ol cardiac disease (palpitations, chest pain, short
vaginal candidiasis. ncss olbreath).
213
Answers and Critiques
Echocardiography (Option C) should not be routinely Echocardiography (Option B) is not indicated as a com
Fl
lfl perlbrmed preoperatively and cerlainly not before lou, risk ponent of the preparticipation physical examination unless
t0*' ,r.g".y. Specific indications lbr cchocardiography befbre there is known or suspected cardiac disease. Although
surgery that is other than lou'risk include the presence ol hypertrophic cardiomyopathy may very rarely present as
dyspnea of unknown origin. heart tailure with worsening sudden death in the young athlete, the prevalence of the
clyspnca or change in clinical status. known left rentricular disease is so low that routine echocardiography is not an
dysfunction u,ithout echocarcliographic assessment in the ell'ective screening strates/. Findings that would prompt
lasl year. and know'n or suspected nloderate to severe val echocardiography include exertional syncopei family history
vular stenosis or regurgitation u'itl.tout echocardiographic ofsudden death; and physical findings suggestive ofcardiac
assessment in the past year or u'ith a change in clinical disease. including an unexplained murmur. This patient has
status. none of these flndings, and echocardiography is not nec-
essary. An ECG is clearly indicated if the patient has unex
I(EY POI ilI plained syncope; palpitations; or a family history of sudden
t,l
o In patients undergoing low risk surgery preoperative death that is suggestive ofa congenital abnormality, such as
E
(D
screening tests seldom predict perioperative compli- long QT syndrome. Although some guidelines suggest rou
aa cations and are not indicated even in patients with tine ECG, the Americat.r College of Cardiology and American
A, substantial comorbidities. Heart Association do not.
EL
n Bibliography
I(EY POIIITS
(jrhnSL. I)reoperative e\aluation firr nor]clrtliirc surgery: Ann lntern Med.
. Routine testing beyond the history and physical
lt 2016:165:lTC81 96. [PN,llD:')7919097)doi:10.7:]26 AITC201612060 examination is not necessary in the preparticipation
(D
la physical examination unless concerning symptoms
and physical findings are uncovered.
Item 87 Answer: D
o Mandatory components olthe preparticipation physi
Educational Objective: Evaluate patient for
a
cal examination include evaluating for exertional
preparticipation physical examination.
symptoms, family history of premature or sudden
There is no indication lor testing (Option D). Prepartici cardiac death, and presence of a heart murmur.
pation physical examination is often required for adoles
cents and young adults befbre participation in organized Bibliography
sports. The goal of a preparticipation physical evaluation Nlirlbellil\,1H. Devine MJ. Singh J. et xl. 'lhe preparticipation sports evtluil
is to determine an individual's safe participation in sports, tion. Am Fxm Physician.2015r92:il7l 6. It'MID,26371570l
although studies have not fbund that the preparticipation
screening examination prevents morbidity and mortal
ity associated with sports. Advocates claim that it may Item 88 Answer: D
detect conditions that predispose the athlete to injury
Educational Objective: Diagnose squamous cell
or illness, which then may guide advice or intervention.
carcinoma.
lhe examination should not be a barrier to competitive
sports participation, and unnecessary testing should be Squamous cell carcinoma (SCC) (Option D), the second most
avoided owing to its attendant costs and unintended con common skin cancer, is a malignant neoplasm of keratino
sequences, such as downstream testing fbr incidental flnd cytes. It usually presents as pink, scaly, indurated plaques,
ings. Mandatory components include evaluating fbr exer papules, or nodules that can ulcerate, bleed, or become crusty.
tional symptoms, family history ol premature or sudden SCCs tend to retain their surlace scale. and as an SCC accumu
cardiac death, and presence of a heart murmur. Routine lates a large amount of scale, it is called a cutaneous horn. Risk
testing beyond the history and physical examination is factors are exposure to UV light, ionizing radiation, or chem
not necessary unless concerning symptoms and physical ical carcinogens (coal tar, soot, and arsenic); viruses (human
findings are uncovered. This patient has exercise-induced papillomavirus); and immunosuppression (organ transplant,
bronchospasm but an unlimited exercise capacity and no hematologic malignancies, HIV infection). SCC on the central
other concerning symptoms or physical flndings. There is face, lips, or ear, as seen in this patient, is considered high
no indication for testing. risk and should be treated with Mohs micrographic surgery.
Chest radiography and spirometry (Options A, C) are Mohs micrographic surgery is a highly specialized surgical
not indicated for a preparticipation physical examination technique that combines pathologz and surgery for complete
in a patient with either well controlled asthma or exercise margin control and tissue conservation. It is appropriately
induced bronchospasm. as is the case fbr this patient. The used fbr cancers in the head and neck region, those that are
indications for these tests are the same as those in other large or recurrent, or those in areas where tissue-sparing is
patients, specifically a suspicion of pulmonary disease on critical for function.
the basis of symptoms or abnormal flndings on pulmo- Basal cell carcinoma (BCC) (Option A) is the most com
nary examination. mon skin cancer. It commonly presents as a pink, pearly
214
An s w9_11_11
{ !_1 11
i qyes
papule with rolled borders and arborizing telangiectasias. epiglottitis are less likely than chilclren to progress to air
Patients commonly report bleeding of BCCs with any manip way clbstruction, if acute epiglottitis is suspected and air
ulation, such as shaving or washing the face. BCC does not way obstruction is a consideration, airway management is a
present as hyperkeratotic lesions, as seen in this patient. priorily. Patients rarely require intubation or tracheostomy;
Similar to SCC, BCC is usually treated with surgical excision. however, hospital admission is recommended, often to an ICU
Mohs micrographic surgery is appropriate for BCC in the setting and with surgical cor.rsultation, to ensure close mon
head and neck region. itoring of the airway. Parenteral antibiotic therapy directed
SCC in situ is referred to as Bowen disease (Option B). toward common respiratory pathogens is additionally indi
ln this condition, malignant keratinocytes are conflned to -fypical
cirted. choices tbr empiric thcrapy include ceftriaxone
the epidermis. Bowen disease appears as larger scaly pink or tcfolaxime and rantomlr'irr.
and tan plaques with well defined borders and does not Although parenteral glucocorticclids. such methylpred
resemble this patient's hyperkeratotic nodule. Small lesions nisolone (Option B), are somctimes used to minimize lir
can be excised for cure; larger lesions may be more amenable way inflammation in acutc epiglottis, there is little evidence t
(l,
to destructive therapies, such as curettage and electrodesic supporti ng eflectiveness. Other measures, including airway
ET
cation or cryotherapy. management and initiation of parenteral antibiotics, arc ol
Seborrheic keratoses (Option C) are benign pigmented far greater urgency.
U
neoplasms of keratinoclte origin that are common in adults. FLtsobacterium necrophorum inf'ection can cause
They are "stuck-on" papules and plaques that occur any Lemierre syndrome. a rare suppllrative complication of
=,
.E
where on the body but are most common on the trunk and pharyngitis caused by local spread of inf'ection with resul t,l
spare the palms, soles, and mucous membranes. Usually tant septic thrombosis of the internal jugr-rlar vein. Clinicians
o
UI
they are brown, but they can range in color from tan to sl-roulcl suspect Lemierre synclronre in patients with severe =
black. When seborrheic keratoses are symptomatic, they pharlrngitis and neck pain that do not respond to :rpprcr
may be treated with cryotherapy or shave removal. This priate antibiotics. Diagnosis is made with neck CI' with
patient's hyperkeratotic nodule on the ear is most consistent contrast (Option C). In patients with suspected epiglottitis.
with SCC. radiographic imaging with ultrasonography or plain racliog
raphy may be helpful in gauging the severity of'neck tissue
KEY POI lIIS
inflammation and predicting likctihood of progression. ltrr
. Cutaneous squamous cell carcinoma presents as pink, this patient, CT of the ncck is nnwarrantecl and would delay
scaly, indurated plaques, papules, or nodules that can hcr transf'er to the ICU fbr carcful aint':ry monitoring ancl
ulcerate, bleed, or become crusty. inten ention if required.
. Squamous cell carcinoma on the central face, lips, or Although a rapid antigen detection test (RADT) (Option
ear is considered high risk and should be treated with D) is a reasonable test lor gronp A streptococcal pharyngitis,
Mohs micrographic surgery. this patier-rt has acute epiglottitis r,r,ith possible ainvay com
promise and there is no role lor IIAI)T.
Bibliography r(EY P0t1{rs
Kim JY, Kozlow JH, Mittal B, et al; W)rk Croup. Cuidelines of care for the
management of cutaneous squamolls cell ctrrcinoma. J Am Acad
. Patients with epiglottitis typically appear seriously i11,
Dermatol. 2018;78:560 78. Il,MIl): 293313861 doi.l0.l0161j.jaad. with excessive salivation and drooling, tachypnea,
2017.1 o.o07
stridor, and severe pharyngitis and dysphagia.
. If severe acute epiglottitis is suspected and airway
Item 89 Answer: A obstruction seems imminent, airway control should
precede diagnostic evaluation.
Educational Objective: Manage a patient with
suspected epiglottitis.
Bibliography
The most appropriate next stcp in management is admission Klein MR. Infections of the orophlrynx. limerg Med Clin North Am
'2019:37:69 80. IPMI D: 30454781 ] doi:10.1016/i.emc.2018.09.002
to the ICU (Option A). this prtient likely has acute epiglotti
tis. Epiglottitis is a severe and rare inflammatory response to
a respiratory infection that requires emergent identification
and treatment, r.tith close attention tcl airway management. Item 9O Answer: D
Although viral causes are common, epiglottitis may also be
Educational Objective: Manage cyclic breast pain.
caused by Streptococcus pneumotiioe and other bacterial
respiratory pathogens. Betbre universal vaccination. Hae The most appropriate management is patient education and
rrroplrilus influenzae type B accounted fbr most infections. reassurance (Option D). This patient has bilateral cyclic breast
Patients n,ith epiglottitis may present with flndings of sep pain. Most common in women aged 20 to 39 years, cyclic
sis, including fever. tachycarclia, severe sore throat, drooling breast pain is often diffuse and bilateral, occurs during the
that suggests difficulty managing normal secretions, striclor, premenstrual phase, and resolves with onset of menstruation;
and inability to open the mouth. Although adults with acutc it is usually associated with hormonal changes and is typically
215
Answers and Critiques
benign. If flndings on clinical breast examination (CBE) are patches involving most of the soles in a "moccasin" distribu
normal, additional screening for malignancy beyond age- tion. "Two feet one hand" tinea is a characteristic pattern of
appropriate screening is not required. Cyclic breast pain should involvement. Tinea pedis can have a more acute form, with
be managed conservatively with education, reassurance regard 1- to 2 mm vesicles, and can be extremely pruritic. The inter-
ing the absence of malignanry, and wearing a supportive and digital variant of tinea pedis shows flssures and maceration
well-fltting bra. Although some patients beneflt from topical in the folds (as shown).
NSAID therapy pain resolves spontaneously for most women.
There is a lack of evidence to support limiting caffeine intake or
using vitamin E as a means of mitigating the pain.
Breast ultrasonography (Option A) is not indicated for
this patient. Women with cyclic diffuse breast pain and a
normal CBE do not require additional imagrng. For women
younger than 30 years who have focal breast pain or an abnor
(,t mal CBE, breast ultrasonography is appropriate for further
evaluation. This patient is 33 years old; if indicated, mammog-
=
.D
!n raphy would be the appropriate diagnostic modality.
o, Danazol (Option B) is the only FDA-approved agent for
EL cyclic breast pain, but its use is limited because of side effects,
at including amenorrhea, hirsutism, and adverse changes in
st lipid proflle. It should be considered only in women with
severe pain in whom conservative management has failed. with serpiginous and arcuate scaly
This patient's plaques
(D
tl Ir4ammography (Option C) is not indicated. Given this borders in a moccasin distribution should raise suspicion for
patient's age, she does not qualify for routine screening dermatophy.te infection. Tinea should be treated to prevent
mammography, and her family history does not suggest a skin breakdor.r,n, which can be an entry port for bacterial
hereditary cancer syndrome that would quali$r for early infection. Dermatophytosis of non hair bearing skin with
screening. Mammography would be the most appropriate Iimited involvement can be treated topically with the use of
management in patients older than 30 years who present topical terbinaflne or imidazole creams, such as miconazole,
with noncyclic focal breast pain or have an abnormal CBE. clotrimazole, and ketoconazole, applied once to twice daily for
Noncyclic breast pain is often caused by medication use 2 to 4 weeks, ensuring that the application extends a few cen-
(contraceptive agents, hormone replacement therapy, psy- timeters beyond the advancing border. Combined antifungals
chiatric medications), underlying breast disease (flbrocys- and topical glucocorticoids should be avoided because they
tic disease, infection, trauma, cancer), or stretching of the can lead to increased recurrences and treatment failures.
Cooper Iigaments (connective tissue that maintains breast Adalimumab (Option A) and clobetasol (Option B) are
structural integrity). not helpful in the treatment of dermatophytosis but are effec
tive in the treatment of psoriasis, which can affect the soles
of the feet and nails. Psoriasis appears as discrete, sharply
. If a clinical breast examination is normal, manage- marginated red plaques with silvery lamellar scale. The arcu
ment of bilateral cyclic breast pain is conservative and ate and serpiginous scale and the dull brown red color of this
includes education, reassurance regarding the patient's skin flndings are not consistent with the presenta
absence of malignancy, wearing a supportive and tion ofpsoriasis.
well-fitting bra, and topical NSAID therapy. Topical nystatin cream (Option C) is beneflcial in the
. treatment of cutaneous candidiasis, but it is not effective in
Danazol is the only FDA approved agent for cyclic
the treatment of dermatophyte infections, such as tinea pedis.
breast pain but should be considered only in women
Although systemic antifungal agents, such as itracon
with severe pain in whom conservative management
azole (Option D), can be beneflcial in the treatment of ony-
has failed.
chomycosis, this patient reports she is not concerned with
treating her toenails. Furthermore, itraconazole is rarely
BibHography
used for the treatment of onychomycosis due to its side
Salzman B, Collins E, Hersh L. Common breast prcblems. Am Fam Physician.
2019;99:505-14. [PMlD, 30990294] effect proflle. Systemic terbinaflne or fluconazole are more
commonly utilized.
'fitrI'POlXttt., r :,' .
216
Answers and Critiques
tr Item 92 Answer: B
Educational Obiective: Manage postoperative ileus.
Bibliography
Vilz TO, Stoffels B, Strassburg C, et al. Ileus in adults. Dtsch Arztebl lnt. a
(l,
2017;114:508 18. IPMIDT 28818187] doi:10.3238/ailtebI.2017.0508
The nrost appropriate managentent is to decrease the
ET
patient's opioid use (Option B). This patient likely has a
postoperative ileus (POI). POI is common after both intra
abdominal and nonabdominal surgery. It is caused by a com
bination of intraperitoneal inflammation, neuroinhibition ol
Item 93 Answer: C
Educational Objective: IdentiS overdiagnosis as a carrse
tr 1.,
It
.u
ofharm due to screening. rrl
the sympathetic nenzous system, and medications (such as (,
opioids and anesthesia). There is no specific therapy for post Overdiagnosis (Option C) is most likely responsible for the
UI
operative ileus other than supportive. Supportive therapy small reduction in cancer related deaths and increase in =
consists of minimizing the use of opioids, hydration, bowel harm. Length-time bias occurs when screening detects more
rest, electrolyte repletion, postoperative ambulation, and use cases of disease with a prolonged asymptomatic phase than
of chewing gum. Minimally invasive surgical approaches, aggressive cases of disease with a shorl asymptomalic phase.
multimodal analgesia techniques, prophylactic bowel regi- Slowly progressive disease is more likely than aggressive dis
mens, and early ambulation may help to prevent POI. ease to be detected with screening, leading to an overesti-
Plain radiography is usually adequate to diagnose POl mation of survival benefit in patients with screen-detected
and to distinguish it fiom small bowel obstruction. CT of disease. Overdiagnosis, or finding and treating illness that
the abdomen (Option A) is indicated if any suspicion fbr otherwise would not have become clinically apparent or
small bowel obstruction remains. particularly if the ileus is caused harm in the patient's litbtime, is an extreme example
prolonged beyoncl 3 to 4 days. There is no indication for CT of length time bias. Overdiagnosis not uncommonly leads to
for this patient at this time. harm due to overtesting and unnecessary treatments.
In patients receiving long-standing opioid therapy and All-cause mortality (Option A) rather than cancer
who have severe constipation. a peripherally acting p-opioid specilic moftality has been proposed as a pref'erable end
receptor antagonist, such as methylnaltrexone, can be eflec point in randomized controlled trials on the basis of possible
tive in improving bowel motility. Althotigh opioids may be biases in assignment of cause of death. I\{isassignment can
contributing to this patient's POI, they are unlikely to be falsely elevate or decrease cancer related deaths. All-cause
the only cause and methylnaltrexone therapy (Option C) is mortality is a less ambiguous assignment and reduces bias
unlikely to be successful in reversing the ileus. and does not account for the results of this study.
Gastric decompression with a nasogastric tube (Option Lead time bias (Option B) occurs when early detection
D) is indicated for patients with significant vomiting, bowel arlificially results in an increase in measured survival. The
distention, or abdominal pain. These indications are not time between early detection and clinical diagnosis is mis-
present. takenly counted as survival time; howevel only the mea
Acute colonic pseudo-obstruction (Ogilvie syn- sured time with diagnosed disease, not survival time, has
drome) is characterized by acute dilatation of the colon in increased. Using moftality rates rather than survival time
the absence of a mechanical obstruction. This condition as the prin-rary outcome in studies of screening tests in ran-
i
is usually found in severely i1l hospitalized patients or domized controlled trials can help minimize lead time bias.
after surgery that is complicated by a metabolic distur This randomized controlled trial used a specific disease
bance or medications that inhibit peristalsis. Patients related end point as well as overall mortality. The risk for
have abdominal distention with pain, anorexia, nausea. lead time bias is low in this studv and cannot account fbr
and vomiting. Plain radiography docunrents a dilated the results.
colon. The initial management is supportive care, but Selection bias (Option D) occurs when patients who
treatment with neostigmine is often attempted in patients undergo screening tests are healthier and more interested
in r,r,hom conservative therapy has failed. On the basis of in their health than the general population. Intention-to
findings on this patient's radiograph, she does not have treat analyses, in which patients are analyzed according
acute colonic pseudo obstruction and does not require to their original group assignment in randomized clinical
neostigmine (Option E). trials regardless ofintervention received, reduce the risk for
217
Answers and Critiques
l[ selection bias. Participants in tiris study nere randonlized A single dose of intramuscular celtriarone and a 10 da-v-
E inle the lwo anns olthe studv and the results'utere analvzed course oloral levofloxacin (Option C) is appropriate empiric
-""' according to their original group assignment; risk fbr selec
CONT,
therapl, for patients with acute epididl'mitis ulto prac
tior, bias was thus reduced. and such bias cannot account tice insertive anal intercourse because it provides coverage
for the results. against coliform bacteria. N. gonorrhoerie, and C. tracho'
motis. This regimen is not indicated fbr this patient u'ho is
XEY POIIITS
sexualll' active onl!' with his wif'e.
. Overdiagnosis is finding and treating illness that oth-
erwise would not have become clinically apparent or
IEY POITTS
caused harm in the patient's lifetime and is often . Enteric bacteria are the most commonly implicated
associated with patient harm. pathogens causing epididyrnitis in men older than
. Lead-time bias occurs when early detection artifi- 35 years and at low risk for sexually transmitted
.€t Bibliography
(D
ur tr Item 94
Ed ucationa I Objective
Answer:
:
D
Manage acute epididymitis
McConaghy JR, Panchal B. Epididymitis: an oven-ie\-. Am Fam Ph]'sician.
2Q16:94:723 6. IPMID: 27929213)
in an older man.
The most appropriate management is oral levofloxacin Item 95 Answer: C
(Option D). This patient has acute epididymitis characterized
Educational Objective: Diagnose phototoxic reaction to
by unilateral scrotal pain of gradual onset that u,orsens o\,€r a
doxycycline.
period of days. In men older than 35 years and in those u,ho
are at lorar risk fbr sexually translxitted infections (STIs). the The most likely diagnosis is phototoxic drug reaction (Option
most likely pathogen is an enteric bacterium. such as Esche C). This patient presents with the characteristic erythema
richia coli. The most appropriate therapy is ar-r empiric 10 day and blistering ofa phototoxic drug reaction. Phototoxic drug
course of oral levofloxacin. Other important management reactions usually present as exaggerated sunburn due to direct
measures include rest. NSAIDs. and scrotal suppofl. At the cellular injury. Unlike photoallergic reactions, they are not
time of presentation, urinall,sis should be obtained: if results immunologically mediated and do not require prior sensi
are abnormal, urine culture should be performed. Hor,rerer. tization; reactions occur minutes to hours after sun expo-
antimicrobial therapy should not be delaled rvhile ar,raiting sure. Tetracyclines, including doxycycline, are one of the most
culture results. common causes of phototoxic drug reactions. In addition
Acute epididymitis is a clinical diagnosis. Doppler ultra- to exaggerated sunburn, tetracycline induced phototoxic-
sonography ol the scrotum (Option A) is not indicated in lty may present with photo onycholysis (separation of the
patients who present with both historical and eramination nail from the nail bed) and pseudoporphyria. Minocycline
flndings consistent with acute epididymitis (as is the case for may cause less photosensitization than other tetracyclines.
this patient) . Doppler ultrasonography is helpful r,t hen there Other common causes of phototoxic reactions include sul
is diagnostic uncertainty; Doppler findings suggestir,e of fonamides, hydrochlorothiazide, fluoroquinolones, antima-
acute epididymitis include increased epididymal blood flon' Iarials, amiodarone, and voriconazole. Discontinuation of the
and swelling. Lack of blood florv r,r,ould indicate a urologic photosensitizing drug and sun avoidance measures will lead
emergency, such as testicular torsion. r,r,hich is unlikely in to resolution of the reaction. Topical glucocorticoids can be
the presence ofan intact cremasteric reflex. used for symptomatic treatment.
A single dose of intramuscular ceftriarone and a 10 day Bullous pemphigoid (Option A) is a pruritic autoim-
course of oral doxycycline (Option B) is appropriate empiric mune blistering disorder that presents most commonly in
antimicrobial therapy in patients r,r,ith acute epididyntitis elderly persons. It does not present acutely after sun expo
who are 35 years of age or younger and in men are at high sure and is a very unlikely diagnosis in this young patient.
risk for STIs. In these cases, common causatir,e organisms A photoallergic drug reaction (Option B) is a type IV
include l{eisserio gonorrlrceae and Chlamydia tracho- hypersensitivity reaction that is less common than photo-
mofis; nucleic acid ampliflcation testing for these organ toxic drug reactions and occurs in patients with prior sen
isms should be obtained. This regimen is not indicated for sitization to the offending drug. This type of drug reaction
patients who are older than 35 years and in those at lor,v [zpically presents more than24 hours after exposure. It clin
risk for STls. ically presents as eczematous dermatitis and less commonly
218
Answers and Cr ues
as exaggerated sunburn. Although many drugs can cause Transvaginal ultrasonography (Option D) is not
both a phototoxic and photoallergic drug reaction, doxycy- required to diagnose or treat dysmenorrhea in the setting of
cline is not known to cause a photoallergic drug reaction. a normal physical examination. Even if there is a concern for
Porphyria cutanea tarda (pCT) (Option D) is a blistering a secondary cause ofdysmenorrhea, such as endometriosis,
disorder that presents most commonly in persons older empiric treatment is recommended before initiating diag
than 40 years, usually on the dorsal hands. It is associated nostic studies.
with hepatitis C virus infection, alcohol-related liver dis
ease, hemochromatosis, and HIV infection. Although pCT is
XEY POII{TS
a photosensitive blistering disorder, it is not the most likely . Management options for primary or secondary dys
diagnosis in this young woman taking doxycycline. menorrhea include empiric treatment with either
combined hormonal contraceptives or progesterone-
XEY POI]IT
only methods.
. Phototoxic reactions present as exaggerated sunburn
r UI
NSAIDs are an effective treatment for dysmenorrhea, (l,
within minutes to hours of sun exposure; tetracy-
acting by reducing the release of prostaglandins and
clines are a common inciting agent. ET
thereby helping to reduce cramping and menstrual
Bibliography
blood Ioss. L'
tt
Monteiro AF, Rato M, Martins C. Drug induced photosensitivity: photoal
lergic and phototoxic reactions. Clin Dermatol. 2016;34:571-81. [pMID: Bibliography ag
')76381351 doi:1 0.1016 lj.clindermarol.20l 6.05.006 tt
Kho KA. Shields JK. Diagnosis and management ol primary dysmenorrhea.
6,
JAMA. 20201323:268-9. [PMID: 31855238] doi:10.1001 ijama.2019.16921
UI
=
Item 96 Answer: B
Educational Objective: Treat dysmenorrhea. Item 97 Answer: C
Educational Objective: Screen for abdominal aortic
The most appropriate management is a combined hormonal
aneurysm.
oral contraceptive pill (Option B). Dysmenorrhea is charac
terized by pain and cramping during menstruation and can The most appropriate screening test is abdominal ultrasonog
be associated with low back pain, nausea, headache, and gas raphy (Option C). This patient has a 50 pack year smoking
trointestinal symptoms. Dysmenorrhea is classified as either history. According to the U.S. Preventive Services Task Force
primary when there is no identified cause, or secondary (USPSTF), one time abdominal ultrasonography is the pre
for which endometriosis is the most common cause. Many ferred screening modality for abdominal aortic aneurysm
women with dysmenorrhea have symptoms severe enough (AAA) in men aged 65 to 75 years who have smoked at least
to result in missed school, work, or other activities. Whether five packs of cigarettes in their lifetime (grade B). Screening
primary or secondary dysmenorrhea, empiric treatment with with ultrasonography is noninvasive, is simple to perform,
either combined hormonal contraceptives (pill, patch, or ring) has high sensitivity (94"/,, lOO"/,,) and speciflcity (98%-100')(,)
or progesterone only methods (pill, injection, implant, or for detecting AAA, and does not expose patients to radiation.
intrauterine device [lUD]) are appropriate options. Before Abdominal ultrasonography screening can also be performed
prescribing a combined hormonal contraceptive, it is impor selectively in men in the same age range who have not smoked
tant to assess for contraindications to estrogen, including a but are at increased risk for peripheral vascular disease or
history of venous thromboembolism or migraine with aura. who have a flrst-degree relative who had an AAA (grade C).
In this patient without contraindications and a need for more The USPSTF assessed the benefits of screening for AAA in
eflective contraceptive therapy, a combined hormonal oral women aged 65 to 75 years who have ever smoked as uncer
contraceptive pill is a reasonable option. tain and makes no recommendation regarding screening in
Acetaminophen (Option A) is often used for the treat this population (l statement). The USPSTF speciflcally recom-
ment of dysmenorrhea; it may be more effective than pla mends against routine screening in women who have never
cebo but is not as effective as NSAIDs. NSAIDs reduce the smoked and who have no family history of AAA (grade D).
release of prostaglandins and thereby help reduce cramps Smoking cessation and treatment of hypertension can prevent
and menstrual blood loss. Because of this patient's need for the development of aneurysmal disease.
more effective contraception, a combined hormonal oral Auscultation of the abdomen for bruits and palpation
contraceptive is a more attractive option than either acet (Option A) to detect pulsatile enlargement of the aorta have
aminophen or NSAIDS. low sensitivity (39'/. 68'1,,) and speciflcity (zs%) for AAA and
A copper IUD (Option C) would not improve the are not recommended for screening, especially in patients
patient's dysmenorrhea. Some women report heavier men with obesi[2, such as this patient with a BMI of 35.
strual bleeding and increased cramping with use of the Abdominal CT (Option B) is an accurate tool for identi
copper IUD. Levonorgestrel releasing IUDs, however, are an fying AAA; however, it is not recommended as a screening
effective treatment modality for women with dysmenorrhea method because of the expense and potential for harms
who are interested in long acting contraceptive options. from radiation exposure. Once an AAA is identifled, CT can
219
Answers and Critiques
provide precise information on its size and location, which RADT has comparable sensitivity and speciflcity to
is needed to plan surgery. throat culture (Option C). ln patients who are at high risk for
Although magnetic resonance angiography (MRA) complications (immunocompromised, high-risk contacts or
(Option D) of the abdomen would detect an AAA, it is environment), a negative RADT result should be confirmed
expensive and time consuming compared with ultrasonog by throat culfure; otherwise, it is unnecessary. Results of
raphy. Like CT, MRA is typically reserved for surgical plan throat culture generally take 24 to 48 hours. Empiric treat
ning, if indicated. ment with antibiotics while results are pending is not rec
ommended because short delays in therapy have not been
TEY POIIIIS
associated with higher complication rates.
. According to the U.S. Preventive Services Task Force, No testing or therapy is inappropriate (Option D).
one-time abdominal ultrasonography is the preferred Because this patient meets four Centor criteria, placing him
screening modality for abdominal aortic aneurysm in at risk for streptococcal pharyngitis, RADT is needed to
men aged 65 to 75 years who have smoked at least five guide therapy.
UI packs of cigarettes in their lifetime.
t(tY PorilIs
=
.D . The U.S. Preventive Services Task Force specifically o The Centor criteria (fever, tonsillar exudates, tender
UI recommends against routine screening for abdominal
o, anterior cervical lymphadenopathy, and absence of
aortic aneurysm (AAA) in women who have never
CL cough) can used to guide the need for testing (rapid
n smoked and who have no family history of AAA.
antigen detection test) for group A streptococcal
pharyngitis.
4t Bibliography
E
(D O\\,ens DK, Davidson KW. Krist AH, et ali US Preventive Services Task ljorce.
. Patients presenting with acute pharyngitis and two
UI Screening for abdominal aortic aneurysm: US Preventi\e Senices'l-ask or fewer Centor criteria do not need to be tested for
I.orce recommendation statement. JAI\,1A. 2019:322:2211 8. IPMID:
318214371 doi:10.1001ijama.2019.18928
group A Streptococcus.
Bibliography
Item 98 Answer: B Mustafa Z, Ghaffari M. Diagnostic methods, clinical guidelines, and antibi
otic treatment for group A streptococcal pharyngitis: a narrative review.
Educational Objective: Evaluate a patient with acute trront Cell Infect Microbiol. 2020:10:563627. [PMID: 33178623] doi:10.
pharyngitis. :ll]891fcimb.2020.563627
220
Answers and Critiques
tr
CONT,
nrortalit, lry 30'1, fbr each clay the dmg r,.,as absent before the
dcvclopmerrt ol blisters and erosions. 'lhis observation did not
holcl true frrr cl-ugs r,r,ith prokrnged hall lives.
Decisions regarding perioperative a ntiplatelet tl.rerapy
are inft)rmed in part by a ranclomized controlled study of
patients with previous coronary ar1ery stenting who subse-
'freatrnent fbr SJS, TEN orerlap s)'ndrome rernlins con quently lrad surgery. 'lhe data suggested that patients lvho
troversial. Intrirvenous glucocorticoicls (Option A) or intra continued aspirin throughout the perioperative period had
venous immunoglobulin (Option B) are likely the most the lowcst rate of a major cardiac aclverse event. Withholding
crirnmonlv used treatments. but neither is su1-rported Iry perioperative aspirin eitherwith oru,ithout ir P2Y1, inhibitor
strong eviclence. Regarclless, initiating therapy r,r,ith eithcr (Options C, D) is not recommencled. Withholding both aspi
intravenolls glucocorticoicls or intravenous imrnune globtr rin ancl clopidogrel may be reasonable if the risk for bleeding
lins is secondary in importance to discontinuing the poten is extreme or the consequences catastrophic, such as those
tial ofl'ending agent. associated with neurosurgeryr
Skin biopsy (Option D) may be indicated in the evalua Surgery is associated with proinflamnratory eflects that
tion of possible SJSI'I'EN overlap syndnrrne if the diugnosis is may increase the risk lbr thronrbosis, prrticularly coronary tt
not conclusivc hased on clinical flndings. When the possibil
o
artery thrombosis and stent thrombosis. Aspirin, clopido
ET
it1'oISJS TIIN is present. however, skin biopsy is secondary grel. and otl.rer P2Y,, inhibitors are antiplatelet agents. n hich
in impofiancc to immediate discontinuation of the possible can mitigate this risk. Balancing the risk fbr bleeding and t,
oflending agent. thronrbosis is key, and continuation of both agents may lead
=,
to unacceptable bleeding risk. In general, bleeding risl< with IE
rEY POTXT UI
aspirin is lor,rer than that r.l,ith clopidogrcl in the perioper
o The most important first step in preventing progres- (,
ative period, so colltinuing aspirin alone is recommended
sion of Stevens Johnson syndrome/toxic epidermal during this time fiame. vt
=
E
necrolysis is to immediately discontinue any potential
inciting medications.
XEY POIlII'
o In patients with coronary stents, guidelines recom-
Bibliography mend that dual antiplatelet therapy should be contin-
ll'lustafa SS, Ostrov D. Yerly D. Severe cutaneous adverse drug reactions: ued uninterrupted for 14 to 30 days after bare metal
presentation, risk factors, and management. Curr Allergl Asthma Rep.
stent placement and a minimum of 3 to 6 months
2018118:26. [PMID: 29574562] doi:10.1007rs11882 OIA 0778 6
after drug eluting stent placement.
. In patients with an urgent need for surgery discontin-
100 Answer: B
tr Item
Educational Objective: Manage perioperative dual
uation of a P2Y,, inhibitor can be considered after a
minimum of 3 months in patients with a drug-eluting
antiplatelet therapy for a patient undergoing urgent stent; aspirin should be continued ifat all possible.
noncardiac surgery.
'lhe most appropriate preoperative management of this Bibliography
BanerjeeS. Angiolillo DJ. Boden WE, et al. Use ofantiplatelet therxpy/DAPT
patient's dual antiplatelet therapy (DAP'f) is to continue aspi for post PCI patients undergoing noncardiac surgery. J Am Coll Cardiol.
rin and withhold clopiclogrel (Option B). In patients with 2017 ;69 :1 867 70. I PM I D : 2838531 5] doi : 1 0.1016/i.iacc .2077 .O2.O 12
221
Answers and Critiques
annually. All women receiving hormone therapy should also commonly unilateral than bilateral. Nodular variants of epi
receive individualized breast cancer risk assessments. This scleritis also occur. Although most causes of episcleritis are
patient does not have a flrst degree relative with breast can idiopathic, it can be associated with systemic conditions,
cer or other markers of elevated risk fbr breast cancer. but including rheumatoid arthritis, vasculitis, and inflammatory
she should be advised that hormone therapy is associated bowel disease. Women are more commonly afl'ected than
with an increased risk for breast cancer in a dose-dependent men, with most cases occurring in those who are young or
fashion; estrogen therapy alone is lower risk than estrogen middle aged. Use ofa topical vasoconstrictor, such as phenyl
plus progesterone (Option A). When hormonal therapy is ephrine, results in improvement in eye redness and can help
indicated, transdermal estrogen may be preferable to oral confirm the diagnosis.
estrogen because it is associated with less thromboembolic Although herpes simplex keratitis (Option B) is in the
risk. However, this is based on Iimited observational data, differential diagnosis of red eye, it typically causes signifl
and head to head comparisons are lacking. Guidelines are in cant eye pain. photophobia, and visual blurring, which are
general agreement in recommending transdermal estrogen not present in this patient. Herpes simplex keratitis can lead
UI in patients with moderate risk for coronary artery disease, to vision loss due to corneal damage and, when suspected,
increased risk fbr venous thromboembolism, hypertriglyceri should prompt immediate referral to an ophthalmologist.
=
.D
UI demia, or high or intermediate risk for breast cancer. Scleritis (Option C), inflammation of the sclera, is an
q,
Oral gabapentin (Option B) taken nightly can be an unlikely diagnosis for this patient because it causes intense.
CL effective nonhormonal option for the management of vaso boring eye pain and photophobia. The cardinal sign ofscleri
n motor symptoms of menopause. However, it is less effec tis is edema of the sclera often associated with a violaceous
tive than hormonal therapy, and given the severity of this discoloration ofthe underlying sclera and intense dilation of
a patient's symptoms and the absence of contraindications, the episcleral blood vessels accounting for the red eye.
o
vt she should be offered transdermal estrogen. Uveitis (Option D) is a potentially vision threatening
Transdermal or oral estrogen plus oral progesterone condition that typically causes severe ocular pain and photo
(Option D) is not appropriate treatment. A progestin is only phobia, both of which are absent in this patient. In addition.
needed for patients with a uterus to prevent estrogen related the eye redness observed with uveitis is most prominent at
endometrial proliferation and hyperplasia. This patient pre- the boundary between the iris and sclera, a pattern termed
viously underwent hysterectomy. When used, progestins can ciliary flush. In some patients, a collection of leukocytes
be dosed either continuously or cyclically. Women receiving (hypopyon) is seen pooling in the inf'erior aspect of the ante
progestins cyclically may experience withdrawal bleeding. rior chamber.
Viral conjunctivitis (Option E) is an unlikely diagnosis
I(EY POII{TS
because it causes diffuse eye redness, as opposed to the
. Hormone therapy is the most effective treatment for localized redness seen in this patient. Symptoms of viral
vasomotor symptoms associated with menopause. conjunctivitis classically start in one eye before spreading to
. Transdermal estrogen is recommended for patients the other eye. Eye redness is Szpically accompanied by watery
with moderate risk for coronary artery disease. discharge, and the tarsal conjunctiva may appear bumpy. Pre
increased risk for venous thromboembolism, hyper auricular lymphadenopathy is commonly present.
triglyceridemia, or high or intermediate risk for breast XEY POIlIIS
cancer. r The classic presentation ofepiscleritis is an abrupt
onset of unilateral eye redness due to dilated episcle-
Bibliography
ral blood vessels, irritation, and tearing, with no pain,
The NAMS 2017 Hormone Therap! Position Statement Advisory Panel. The
2017 hormonc therapy position statement ol the North American photophobia, or change in vision.
Menopause Society. Menopause. 2017;2.1:728 53. [PMlD, 28650869]
doi :l 0.1097/Cl\4 F].000000000ooo0921
. The cardinal sign of scleritis is edema of the sclera
often associated with an underlying violaceous dis
coloration of the sclera and intense dilation of the
Item 102 Answer: A episcleral blood vessels, accounting for the red eye.
Ed ucationa I Objective: Diagnose episcleritis.
Bibliography
lhe most likely diagnosis episcleritis (Option A), or inflam
is 'larff A. Behrens A. Ocular emergencies: red e1e. Med Clin North Am.
mation of the episclera, the highly vascularized connective 2017rl01:61.5 39. IPMID: 283727771 doi:10.1016/i.mcna.2016.12.013
tissue layer that lies between the conjunctiva and sclera. This
patient has a classic presentation of episcleritis, r.r'ith abrupt
onset of unilateral eye redness, irritation, and tearing. Pain, Item 103 Answer: D
photophobia, and vision changes are typically absent. Exam
Educational Objective: Diagnose erythrasma.
ination shows dilation of superflcial episcleral blood ves
sels. with normal sclera visualized between the blood vessels. Wood lamp evaluation (Option D) is the most appropriate
Redness can be localized or diffuse: involvement is more diagnostic test to perform nexl. This patient has ery,.thrasma,
222
Answers and cll"igues
a superficial skin infection secondary to Corynebacterium microscopic identification of the mites, eggs, or f'eces in skin
minutissimum infection. Symptoms, limited to pruritus, are scrapings prepared with mineral oil. The patient's presenta
minimal in most patients. Efihrasma manifests as thin, atro tion is not typical for dermatophytes, Candida, or scabies.
phic, flnely wrinkled pink brown plaques in the intertrigi-
t(EY POl]tTS
nous areas, such as the axillae or inguinal or gluteal fblds, or
less commonly in the interdigital or intramammary regions. . Erlthrasma, a superficial skin infection, manifests as
The skin is often described as having "cigarette paper" appear thin, atrophic, finely wrinkled pink brown plaques
ance. A Wood lamp evaluation reveals a soft coral red or in the intertriginous areas; Wood lamp evaluation
pink fluorescence, as shown in this patient with interdigital reveals a soft coral red or pink fluorescence.
erythrasma. . Symptoms of erythrasma are limited to mild pruritus,
and treatment for localized disease is topical erythro-
mycin.
vt
q,
Bibliography ET
Irorouzan P, Cohen PR. Erythmsma revisited: diagnosis, diflerential diagno
ses, ancl comprehensive review' ol treatment. Cureus. 2020;12:e10733.
IPMID: l]:ll45l38l doi:10.7759, cureus.l073ll
t,
E
t!
vt
L
o
104 Answer: D
Item
Educational Objective: Manage perioperative tr t
=
medications.
'I
he most appropriate perioperative nreclicatior-r rn:lnagement
is to colltinue all meclications (Option D). tligh quality evi
Treatment is with oral or topical erythromycin (preferred dence kr gr.ride perioperative mediclrtion lxanrgement is lack
fbr localized disease). Typical skin flndings plus fluorescence ing. Reconrnrendatiorrs are. largely derived fr<trn tlreoretical
with Wood lamp illumination conflrms the diagnosis; addi drug interirctions ancl expert consensus. Althor-rgh nrost nted
tional testing is not needed. ications are tolertrtecl thnrughout thc perioperxtile perioil.
Acanthosis nigricans can appear in the axillae as thick- there arc some importlnt exceptions. Fbr exirmple, the Amer
ened, hypertrophic velvety hyperpigmented plaques. In ican College ol Rheumatolog; reconrnrends that biologic
patients with acanthosis nigricans, it is important to obtain medications should be u,ithheld as closc. to one d<tsing cyclc
a hemoglobin A,. measurement (Option A) to evaluate for ls schedr.rling permits belbre elective hip ancl l<nee arthro
the presence of diabetes mellitus or glucose intolerance. plasty and restarted after evidence oI wor-rnd healing, typically
The plaques on this patient are atrophic rather than hyper- l4 days. N<lnbiologic discase.modifying :intirhcumatic drugs
trophic; a Wood lamp evaluation is the most appropriate (l )\lARt)s) (nrethotrexatc. sulfasalazine. hldrrxl,cl.rl,rroquinc.
initial test. lellunomirle) rnay be continued throughout the periopcrl
Examination of a potassium hydroxide preparation tive periocl. Meth<ltrcxatc is administcrcd or.rce weekly, mtist
of'skin scrapings (Option B) can show fungal elements in commonll, bv oral or subcutaneous routcs. Studies in patients
the case of dermatophytes or yeast, whereas mineral oil u'ith rheurnatoid arthritis have denronstrated thirt continuir.rg
preparation (Option C) can highlight ectoparasites, such as I)MARDs. includir-tg nrethotrexate. thnrugh the lterioperative
Sorcoptes scobiei or Demodex species. However, the charac period is safb without an increase in infbction ratc (Option
teristic clinical presentation oftinea corporis is annular scaly A), er,en with prosthetic irr.rplantation, and rc'duces the risk
patches with central clearing and varying degrees of inflam tor disease fllres.
mation; microscopic examination of a potassium hydrox- Accorcling to the 201.'l Americln College ol C:rrdiology
ide preparation will show branching hyphae in the scale. A mericir n He:rrt Associ :rtion pcrioperative carcl i ovasculirr
Intertrigo is an inflammatory skin disease that involves the guidelinc. p:rtients on B blocker therapy, such as meto
axillae, inframammary and inguinal folds. Condido is the prolol. sl.rould c<lntinue therapt' in the perioperative period
most common secondary inf'ection in intertrigo, with obe (Options B. C). Cessirtion ot thesc ilgents may cause rebound
sity being an important risk factor. It presents as erythema tachycardiu and pronrote dysrhythmias in thc periopcrri
tous patches with satellite pustules that are often macerated. tive perirxl because of the elevatcd adrenergic tone fronr
Diagnosis is fiequently made on clinical grounds alone, pain, strcss. :rnd volunre shifts. l)reoperative initiation ol
but a potassium hydroxide preparation can show spores B blockers is generalll' reserved tbr pirlients n'ith a Rer,isetl
and pseudohyphae. Scabies infestation is characterized by Cardiac Risk lndex score of 3 or mol'e without contraindi
intensely pruritic, crusted papules, nodules, and burrows cxtions (such :rs reslilrs bradycarclia or hypotcnsion) ar-rcl
that develop in the interdigital spaces, wrists, ankles, breasts, who havc enough tir-t're bclore sllrgcry to trial n'reclications
periumbilical area, and genitals. Diagnosis is performed by (>l r,r'eek) to ensure satety ancl tolerribility
223
Answers and Critiques
tr
CONT.
Sufficient evidence has demonstrated benelits ot sta-
tin agents, such as atorvastatin, during the periopcrative
period. Pleiotropic effects of these agents are belie\ed to
Reviewing the benefits of a healthy diet and exercise
(Options B, C) is necessary but not sufficient to help this
patient achieve healthy lifestyle goals. Behavioral change
be responsiblc Ibr numerous benefits in major surgeries. counseling that includes motivational interviewing and
presumably due to anti inflanlmabry properties. Statins introduction to behavioral change techniques as well as
have been consistently associated with lower risk Ibr car goal setting is more likely to achieve the desired end than
diovascular c<lmplications in both cardiac and noncardiac solely providing information about the benefits of diet
surgery In surgeries u'ith highest cardiovascular risk (major and exercise. The USPSTF notes that there is adequate
vascular. cardiothoracic). it is reasonable to start :l statin evidence that counseling interventions reduce overall
preoperatir,ely if the patient is not already taking one. T1-tis CVD events (myocardial infarction and stroke); improve
recommendation does not apply fbr patients ur.rdergoing blood pressure, lipid and fasting blood glucose levels.
noncardiac sllrger):. as is this case firr this patient: il a and body weight/adiposity; and improve healthy eating
D patient has bcen on long tcnn statin therapll it sl.rould be habits.
t^ continued. Although it is important that patients understand the
E long term complications of their diseases (Option D), pro
.D f,EY POIXTS
t viding a stronger means of supporting behavioral change.
o, . Nonbiologic disease-modifying antirheumatic such as a behavioral counseling program, is more likely to
CL drugs (DMARDs) can be continued throughout the make a signiflcant impact.
n perioperative period; biologic DMARDs should be
I(EYPO I XI
discontinued and restarted after evidence ofwound
tt o The U.S. Preventive Services Task Force recom
healing.
.D
UT
o mends behavioral counseling to promote a healthy
Patients on p blocker and statin medications should
diet and physical activity for adults aged 18 years or
continue therapy in the perioperative period.
older who are at increased risk for cardiovascular
disease.
Bibliography
Coodman SM. Springer B, Guyatt G, et al. 2017 American College of
Rheumatolos//American Association of Hip irnd Knee Surgeons guide Bibliography
line for the perioperative management of antirheumatic medication in Krist AH. Davidson KW. Mangione CM. et irl: US Preventive Sen'ices Task
patients with rheumatic diseases undergoing elective total hip or total Force. Behavioral counseling intenentions to promote a healthy diet and
knee arthroplasty. Arthritis Crre Res (Hoboken). 2017:69)111 2.1. [PMID: physical activity for cardiovascular disease pre\ention in adults u'ith
286209171 doi: I 0. 1002/acr.2327.1 cardiovascular risk factors: US Preventive Services Task Force recom
mendation statement. JAMA. 2020;32.1:2069-75. IPMID: 332316701
doi:l0.lo0l jrm3.2020.21ilq
Item 105 Answer: A
Educational Objective: Provide counseling for
behavioral change. Item 106 Answer: D
Educational Objective: Avoid prostate cancer
The most appropriate management is counseling for behav
screening in a patient who does not express a preference
ioral change (Option A). Behavioral counseling is an inte
for screening.
gral part of helping patients attain a healthy lifestyle. The
U.S. Preventive Services Task Force (USPSTF) recommends The most appropriate prostate cancer screening stratery
behavioral counseling to promote a healthy diet and phys for this patient is no screening (Option D). Screening for
ical activity for adults aged 18 years or older who are at prostate cancer in asymptomatic. average risk men has
increased risk for cardiovascular disease (CVD). Patients been controversial, and recommendations among profes
at increased risk for CVD are identified by the USPSTF as sional organizations continue to evolve. The U.S. Preven
patients with hypertension or elevated blood pressure; tive Services Task Force (USPSTF) recommends that for
dyslipidemia; or mixed or multiple risk factors, such as the men aged 55 to 69 years, the decision to undergo periodic
metabolic syndrome or an estimated 10 year atherosclerotic prostate speciflc antigen (PSA) based screening for
CVD risk of 7.5'1, or higher. The USPSTF provides suggestions prostate cancer should be an individual decision. Before
for behavioral change implementation, including a median deciding whether to be screened. men should have an
of 12 contacts. with an estimated 6 hours of contact over opportunity to discuss the potential benefits and harms
12 months; some one-on one time with an interventionist; of screening with their clinician and to incorporate their
and use of motivational interviewing and behavioral change values and preferences in the decision. Screening offers
techniques, such as goal setting, active use of self monitoring, a small potential benefit of reducing the chance of death
and addressing barriers related to diet, physical activity, or from prostate cancer in some men that is estimated to
weight change. The USPSTF suggests that counseling for be a reduction in one prostate cancer related death for
behavioral change can be done in person or referred to out every 1000 men screened for 10 years. However, many men
side counseling services, or patients can be informed about will experience potential harms of screening, including
media based interventions. false-positive results that require additional testing and
224
t
t
I
Answers and Critiques
t
possible prostate biopsy; overdiagnosis and overtreatment; including erythrotelangiectatic (as shown), papulopustu-
and treatment complications, such as incontinence and lar, phymatous, and ocular rosacea.
L
erectile dysfunction. In determining whether this service
I
is appropriate in individual cases, patients and clinicians
t should consider the balance ofbeneflts and harms on the
i
basis of family history race/ethnicity, comorbid medical
I conditions, patient values about the beneflts and harms
I
t of screening and treatment specific outcomes, and other
I
health needs. Clinicians should not screen men who do not
lr express a preference for screening.
i Conducting a digital rectal examination (Options A, B)
i as part of prostate cancer screening is not recommended by
i the USPSTF because there is no evidence that it adds beneflt U!
o
i
to the PSA assay.
I ET
Because the patient has not expressed a preference for
i
screening, obtaining a PSA measurement (Option C) is not (J
t
I
indicated. If the decision is made to proceed with screening, 'El
) the American Urological Association (AUA) recommends .g
choosing less frequent screening intervals (>2 years), which UI
The papulopustular variant of rosacea can be confused
may reduce overdiagnosis and the number of false,positive with acne, but rosacea does not present with comedonal
o
results while preserving most of the screening benefit. The rn
lesions. Telangiectasias are seen in the erythrotelangiec =
AUA also recommends that the interval for rescreening tatic subtype. Most patients also have frequent flushing in
may be based on the baseline PSA level. Screening is not response to triggers, such as stress, alcohol consumption,
recommended for men with less than a 10 to lS-year life heat, and excessive sunlight. Ihe erythema of the cheeks
expectancy. can mimic the malar rash of systemic lupus erS,thematosus.
IEY POITII Unlike lupus, however, the erythema of rosacea includes
the nasolabial folds. Ocular rosacea may present in isola-
. For men aged 55 to 69 years, the U.S. Preventive
tion or with associated skin flndings. In ocular rosacea, the
Services Task Force recommends that clinicians
conjunctiva appears injected, and patients often describe a
engagein a discussion ofthe potential benefits versus "gritty sensation." In phymatous rosacea, severe sebaceous
harms ofscreening for prostate cancer before ordering hyperplasia and chronic inflammation lead to flbrous over-
testing. growth of the skin, creating a nodular, tumor-like deforma-
. The U.S. Preventive Services Task Force recom tion of the facial structures. Rhinophyma is the form most
mends that clinicians should not screen men for frequently encountered, and it is almost exclusively found
prostate cancer unless they express a preference for in men. Treatment for rosacea is targeted toward the most
screening. prominent signs and symptoms in each patient. Topical
metronidazole is a reasonable flrst-line option for patients
Bibliography with predominant papulopustular variant of rosacea as seen
Crossman DC, Curry SJ, Owens DK, et al; US Preventive Services Task Force. in this patient. Other effective topical agents include sodium
Screening for prostate cancer: US Preventive Services Task Force recom sulfacetamide/sulfur, azelaic acid, topical calcineurin inhib-
mendation statement. JAMA. 2018;319:1901 13. [PMID: 29801017]
doi:10.1001/jama.2018.3710
itors (pimecrolimus, tacrolimus), and ivermectin. Oral anti-
biotics, such as low-dose doxycycline, have been shown to
help control inflammation in ocular and papulopustular
Item 107 Answer: C rosacea. Most patients will also beneflt from avoidance of
identifled triggers, proper use ofsun protection, and use of
Educational Objective: Treat rosacea.
gentle skin cleansers.
The most appropriate treatment is topical metronidazole Chloroquine (Option A) can be very helpful in the
cream (Option C). This patient's rash consists of red mac- treatment of cutaneous and systemic lupus erythematosus.
ules with pustules and telangiectasias within the rash, However, this patient has rosacea and chloroquine is not
which after exposure to certain triggers becomes more indicated.
prominent with stinging, burning, and itching. These Topical clobetasol cream (Option B) is a glucocorticoid
flndings are most consistent with rosacea. This condition and may exacerbate existing rosacea or cause rosacea de
is deflned by chronic inflammation of the pilosebaceous novo on the face. It is not a treatment for rosacea.
units with increased vascular reactivity; it is most com- Although topical tacrolimus cream (Option D) can
mon among patients of Irish and English descent with treat inflammatory variants of rosacea, such as perioriflcial
light complexions, usually in the third to sixth decades of dermatitis, topical metronidazole cream is a flrst-line agent
life. Ihere are a variety ofclinical presentations ofrosacea, for rosacea.
225
Answers and Critiques
226
Answers and Critiques
Bibliography
American College of Obstetricians and Cynecologists. ACOG committee
opinion no. 736: Optimizing Postpartum Care. Obstet Gynecol. 2018;
131 :e140 50. doi:10.1097iAOG.0000000000002633
227
Answers and Critiques
tr Item 1t 1 Answer: A
Ed u cationa I Obj ective : Prevent postoperative
operative pulmonary complications.
Bibliography
pulmonary complications.
Odor PM. Bampoe S, Gilhooly D, et al. Perioperati\e interventions for pre
Perioperative prophvlactic respirator)' ph1'siritl.rerapl' \€ntion of postoperative pulmonary complications: s),stematic review
and meta analysis. BMJ. 2020;368:m540. [PMID: 32161042] doi:10.1136
(Option A) that includes increased r.nobilit-r. sputunl bmj.m540
clearance, deep breathing erercise, and inspiratitn' mus
cle training is consistentl! sho$,n to reduce pulntonan,
D
complications ,fter surgery A 2020 s)'stematic revieu'and Item 112 Answer: A
meta-analysis cvaluated intenentions to reduce tl're inci Educational Objective: Treat acute otitis externa.
U!
€ dence of postoperati\e pulmonary complications in patients
o undergoing noncardiac surgery Postoperatire pulmonary The most appropriate additional treatment for this patient
UI with moderate severity acute otitis externa (AOE) (swimmer's
o, con.rplications u,ere defincd as a contposite of respiraton
ir.tfbction, respiratory failure, pleurirl eflusion. atclectasis, or ear) is ciprofloxacin dexamethasone otic drops (Option A).
CL
n pneumothorax. Perioperative (either bcfilre or afler surgery AOE is diffuse inflammation of the external ear canal and
may involve the pinna and tragus. In 987, of cases of AOE, the
or both) prophylactic respiratory. phl,siotherapl' shou'ed an
ll cause is bacterial infection, most commonly with Pseudomo-
overall benefit (risk ratio. 0.55: 95'll, CI. 0.32 0.911) in reduc'
.D ing the development o{ postoperative pulmonary compli nos or Stophulococcus oureus. Symptoms include otalgia,
(a
cations. Other intenentions :rssociated with a reduccd risk
pruritus, and ear fullness. The history should include assess
fbr postoperative puimonan' complications inclucled lung ment for risk factors, including swimming/water activities,
protecti\,e int raoperatire vcntilation. goal directecl hemod)' trauma/injury associated skin conditions (including contact
namic therapics. epidural irnalgesia (replacing opioid anal- dermatitis, eczema, and psoriasis), immunocompromising
gesia), prophylactic mucolytics. and postoperative continu- conditions (such as diabetes mellitus, HIV infection, or can-
ous positive pressure ventilation. All intcnentious sho\fing cer chemotherapy), history of radiation, and tympanostomy
benefit \tere associated rvith lou- qualit] er,idence. tubes or perforation of the tympanic membrane, which may
Although incentir,e spirornetry, (Option B) is olten alter choice of treatment. On physical examination, the classic
included in postoperativc management, moderatc qualit_v sign of AOE is tendemess with pushing on the tragus or pull-
evidence shons no benelit in pre\€nting postoperative pul
ing on the pinna. Topical treatments, including antibiotics,
glucocorticoids, antiseptics (acetic acid), and combination
n1onary complications (risk ratio. 1.06: 95'1,, CI. 0.85 1.31).
Otl.rer recent systematic revielvs concur u'ith this finding. therapies, are flrst line management for uncomplicated AOE.
Routine use of nasogastric tubes (Option C) is not rec In this patient with moderate severity disease characterized
ommended lor the prerentitnt of postoperatir,e pulmonary' by pain and occlusion olthe ear canal with debris (cerumen,
complications. Sl,stematic revierrs and meta anall'sis hare desquamated cells, purulent material), a topical antibiotic/
concluded that the routine use of'a nasogastric tube lo clecon.r glucocorticoid combination is reasonable after cleaning out
prcss the stornach followirrg abdominal surgery is associated the debris from the ear canal. Patients should be instructed
u,ith an increased risk lor pulmonary.complications. includ on proper technique for instillation of ear drops, including
ing pneumonia and atelcctasis. \asogtstric tubes do hare a cleaning excess debris before instilling the drops and then
role in patients rvitl.r postoperati\,e gastric distention or in the lying down with the affected ear facing upward, remaining in
treatment of nausea and v<tmiting. but nrutine use fitllou,ing this position for 3 to 5 minutes.
abdominal surgery is not recontmended. In most patients with AOE, systemic antibiotics.
Maxintizing oral h1'eiene u-ith preoperativc use o1' such as intravenous ceftriaxone (Option B), are ineffec
chlorhexidir.rc ntouth$'ash (Option D) reduces thc risk for tive and should be avoided. However, immunocompro-
postoperative pneumonia in patients undergoing cardiac mised patients require topical and systemic therapy and
surgeryi the ef licacy in patients undergoing noncardiac tho are at increased risk flor deeper infections, including bone
racic surgery or abdominal surgery, is less clear. involvement (malignant otitis externa), which if present
requires urgent referral to an otolaryngologist and intrave
I(EY POITTS nous antibiotics.
o Perioperative prophylactic respiratory physiotherapy Fungal infection of the external ear canal is uncom
that includes increased mobility, sputum clearance, mon and is usually suspected when appropriate treat-
deep breathing exercise, and inspiratory muscle train ment for AOE fails. The use of topical antibiotics can be a
ing reduces the incidence of postoperative pulmonary predisposing factor for fungal external otitis. Treatment
complications. includes meticulous cleaning of the external ear canal
(Continued) and application of a topical antifungal agent, such as
clotrimazole (Option C).
228
Answers and Critiques
Herpes zoster infection involving the ear may present less costly than transdermal formulations and has similar
with vesicles around the external ear canal and may be com- efflcacy and safety.
plicated by peripherai facial nerve paralysis and auditory/ Oral phosphodiesterase 5 (PDE 5) inhibitors, such
vestibular symptoms (Ramsay Hunt syndrome). These as sildenafll (Option E), are first-line medical therapy fbr
patients require antiviral therapy, such as valacyclovir erectile dysfunction and are safe and effective in most
(Option D), and referral to an otolaryngologist. Inspection of patients. Testosterone therapy may also be indicated in
this patient's ear and ear canal does not suggest the presence cases of confirmed androgen deflciency. When prescribed
of vesicles, and treatment for herpes zoster with valacyclovir fbr hypogonadal men with erectile dysfunction, testos
is not indicated. terone therapy will enhance the efficacy of PDE 5 inhib
I(EY POITTS
itors. However, neither PDE 5 inhibitors nor testosterone
therapy is indicated until the patient is fully evaluated fbr
o The classic sign of acute otitis externa is tendemess
hypogonadism.
with pushing on the tragus or pulling on the pinna. ta
r First-line treatment of acute otitis externa includes
rtY PortTS c,
229
Answers and Critiques
The USPSTF reported on two trials that studied the burning after application. Both drugs have been associated
effects of vitamin D on cardiovascular disease and found no with an increased risk for viral skin infections, such as her
effect on disease incidence or mortality. The USPSTF lound pes simplex and varicella zoster. These medications have a
only one trial that reported cardiovascular disease incidence black box warning fbr the rare development of'skin cancer
and mortality and all cause mortaliry with combined use and cutaneous lymphoma. Replacing clobetasol cream with
of vitamin D and calcium (Option C). and it fbund no ellect tacrolimus ointment is the most appropriate choice to con-
after 7 years of lbltow up. tinue to treat this patient's atopic dermatitis in the setting
The USPSTF found adequate evidence that supple- of glucocorticoid atrophy.
mentation with p carotene or vitamin E (Option D) in Calcipotriene cream (Option A) is a vitamin D analogue
healthy populations without known nutritional deflcien used fbr the treatment of psoriasis. Although adding calcipo-
cies does not reduce the risk for cardiovascular disease. triene cream would not worsen this patient's skin atrophy. it
The USPSTF recommends against the use of p-carotene or is not effective for treating atopic dermatitis. Regardless, this
D vitamin E supplements for the prevention ol cardiovascu patient should discontinue clobetasol cream, which is the
U! lar disease. cause ol her skin atroPhY.
Ketoconazole cream (Option B) is effective for treating
=
.D rEY PO!TIT
superficial fungal infections. l{owever, the annular plaques
t^
o, . No vitamin or supplement can be recommended to and scale of tinea are not seen in this patient. and keto
CL prevent cardiovascular disease due to lack of evidence conazole would not be efl'ective in treating this patient's skin
n for efficacy. atrophy, striae, and telangiectasias.
4t . The U.S. Preventive Services Task Force recom Triamcinolone cream (Option D) is a mid potency
E mends against the use of B-carotene or vitamin E topical glucocorticoid. Although it is effective in treating
.D
ta
supplements for the prevention of cardiovascular atopic dermatitis and is Iess potent than clobetasol cream,
disease because of evidence demonstrating lack of
it can potentially exacerbate the current atrophy, striae,
and telangiectasias seen in this patient's skin. Topical glu-
efficacy.
cocorticoids are frequently commercially combined with
topical antifungal agents (clotrimazole betamethasone).
Bibliography
These combinations should also be avoided. The use of a
N{oyer \A: U.S. Prcvcnti\,e Senices'lask Forcc. Vitirmin. mineral. and mul
tivitlnrin supplements Ibr the primary prevention ot cardiovirscular combination drug can worsen some tinea inf'ections and
diseasclnd cancer: U.S. Preventive Services'lhsk Forcr recommendation when used in the groin area has a high risk fbr causing
statement. Ann lntern Med. 201.1:160:558 6.1. IPMII): 21566,174] doi:10.
7326 l\,ll-1 0198 striae.
t(tY P0 t t{T
Item 115 Answer: C . Topical tacrolimus and pimecrolimus are immu
Educational Objective: Treat inflammatory skin nomodulators (calcineurin inhibitors) that can be
disorders with topical immunomodulators. effective in treating atopic dermatitis without the risk
The most appropriate treatment is to replace clobetasol for skin atrophy, striae, and telangiectasias that can
cream with tacrolimus ointment (Option C). This patient occur with topical glucocorticoid use.
has signs of skin atrophy, telangiectasias, and striae caused
by overuse of clobetasol cream, a high-potency topical glu- Bibliography
cocorticoid. Side effects are most fiequently seen when l)rugs lbr iltopic dermatitis. Med l.ett Drugs Ther 2020:62:89 96. IPMID:
325ssl22l
higher potencies are used lbr longer periods or when used
under occlusion, and they can appear as soon as 2 weeks
Answer:
after initiation of therapy. Use of glucocorticoids around
the eyes can exacerbate glaucoma and cause cataracts. Skin
atrophy is more likely to occur in intertriginous areas, the
Item 116
Educational Objective: Prevent venous
E
tr
thromboembolism in a hospitalized patient.
face, and often in the popliteal or antecubital fossa. Topical
glucocorticoids rnay also cause easy bruising. Although The most appropriate management lbr prever.rting venolls
this patient has signs ofskin atrophy. she continues to have thromboembolism (VTE) in this I.rospitalized patient is
flares ot her atopic dermatitis requiring treatment. Topical lou,molecular rveight l.reparin (LMWII) (Option E). Rates
tacrolimus and pimecrolimus are immunomodulators (cal of VTE (both pulmonary embolism and deep vcnous thron.r
cineurin inhibitors) that can be effective in treating atopic bosis) are elevated in acutely i1l medical patients. Acutely
dermatitis, having similar efflcacl' to low to medium ill medical patients are putients hospitalized fbr a n.redical
potency glucocorticoids. Topical calcineurin inhibitors can illness. Irr acutely ill medical patients, the 2018 American
be used on the face and intertriginous areas without the Society of Hematologr (AStl) guideline suggcsts parenteral
risk fbr skin atrophy, telangiectasias, and striae that can anticoagulation using LNIWH. unfractionated heparin (UFH).
occur with topical glucocorticoid use. The most common or fbndaparinux to prevent V'l'E. In critically ill patients. the
side effect of topical calcineurin inhibitors is stinging or ASI{ guicleline prefers LI\{WH to UFH.
230
Answers and Critiques
tr
CON],
In acutelv ill medical patients requiring V'l'l.l chemo
prophylaris. thc ASH and othcr guidelines recommend use
of parentcral anticoagulants over oral anticorrgullnts. Direct
preventable death in the United States. Individuals with
unhealthy alcohol use often interact with the health care
system but rarely receive appropriate treatment. Screen
oral ar.rticoagulants (Option A) are increasingly uscd as the ing instruments to identiflz harmful drinking include the
agent of choice for postsurgicrl VTE prophylaxis. and in the Alcohol Use Disorders Identiflcation Test (AUDIT), AUDIT
2019 ASll VTFI guideline, clirect oral anticoagulants are pre Consumption (AUDIT C), and Single Alcohol Screening
lerred over LMWH for postopcrrtive VTE prophylaxis in total Question (SASQ). The AUDIT is a validated 10 item screen
knee and hip arthroplastl. ing test that takes approximately 2 to 3 minutes to admin
ln ircutclv or criticalll, ill ntedical patients. the ASH ister; the AUDIT C is a briefer (three item) version of the
guideline suggests using pharntacologic VTE pr<tphylaxis AUDIT. With single item screening, the clinician asks, "llow
over mechanical VTE prophylaxis, such as intcrnlittent many times in the past year have you had five [four fbr
pneumatic compression (lPC) (Option B) or gradient women and persons aged >65 years] or more drinks in
stockir.rgs. IPC is indicatecl lirr a patient at incrcased risk 1 day?" Patients with a positive screening result should be tl
fbr VTF. arld at high risl< fttr n.rajor bleecling or expe-
o
evaluated for alcohol use disorder and medical complica
ET
riencing bleeding on prophylactic dose anticoagulants. tions ofexcessive alcohol intake.
It is also vcry unlikely tl.rat this patient u,ith exter.rsive The USPSTF recommends screening for diabetes ntel (.,
cellulitis r,r,ould tolerate the cliscomfbrt of IPC or gradient litus (Option A) in adults aged 40 to 70 years who are over
stockings. weight or obese. The American Diabetes Association (ADA)
=,
.E
In acutcly ill medicll patients. the ASf t guiclclinc panel recommends that screening be perfbrmed in patients of l^
q,
does uot suggest combined use of lPC devices ancl paren- any age who are overweight or obese and have one or
ta
teral anticoagulation prophl,laxis over parenterul anticoag- more risk factors fbr diabetes. The ADA also recomnlends =
ulation rlolle (Options C, D).-[his recommendation is based screening all adults beginning at age 45 years, regardless
on lor,r, quality evidence bccrusc there are no trials com of risk factors. This patient has no indication for diabetes
paring both strategies in this population. The AStl guide- screening.
line recommends combined mechanical and pharnracologic According to the USPSTF, evidence is insufficient to
prophylaris over pharmacologic prophylaxis alone in the assess the balance of benefits and harms of screening fbr
hospitalizecl surgical patient. obstructive sleep apnea (Option C) in asymptomatic adults
with the currently available tools.
t(EY P0tltIt The USPSTF concludes that evidence is insufficient to
. In acutely ill medical patients, parenteral anticoagula- recommend for or against screening fbr thyroid dystunc
tion using low-molecular-weight heparin, unfraction tion (Option D). The American Thyroid Association and the
ated heparin, or fondaparinux is suggested to prevent American Association of Clinical Endocrinologists, however,
venous thromboembolism. recommend measuring thyroid-stimulating hormone in
o In acutely ill medical patients, parenteral anticoagu- individuals at risk for hypothyroidism (e.g.,personal history
Iation to prevent venous thromboembolism is sug- of autoimmune disease, neck radiation, or thyroid surgery);
gested over oral anticoagulants and over mechanical they additionally suggest considering screening in adults
prophylaxis. aged 60 years or older. This patient has no indication fbr
thyroid function screening.
Bibliography f,EY POIlIIS
Schtinemann llJ, Cushman M. Burnett AE, et al. Americirn Society of
tlematolos, 2018 guidelines fbr mrnagement ol venous thromboembo
. The U.S. Preventive Services Task Force recommends
lism: prophylaxis for hospitalized and nonhospitalized nredical patients. screening for unhealthy alcohol use in primary care
Blood Atlv 2018;2:3198-225. IPMID: 30482763] doi:10.1182/bloodad
\ances.20l 8O2295,1
settings in adults aged 18 years or older, including
pregnant women, and providing persons engaged in
risky or hazardous drinking with brief behavioral
counseling interventions to reduce unhealthy alcohol
Item 117 Answer: B
use.
Educational Objective: Screen a patient for
hazardous drinking.
. Screening instruments to identify unhealthy alcohol
use include the Alcohol Use Disorders Identiflcation
The most appropriate screening is for hazardous alcohol Test (AUDIT), AUDlT-Consumption (AUDIT C), and
drinking (Option B). The U.S. Preventive Services Task Single Alcohol Screening Question (SASQ).
Force (USPSTF) recommends screening for unhealthy alco
hoi use in primary care settings in adults aged 18 years or Bibliography
older, including pregnant women, and providing persons (lurry SJ, Krist AH, Owens DK, et al; US Preventive Services Task I'irrce.
engaged in risky or hazardous drinking with brief behav- Screening and behavioral counseling interventions to reduce unhellthy
alcohol use in adolescents and adults: US I'r€'ventive Seruices lhsk lbrce
ioral counseling interventions to reduce unhealthy alcohol recommendation statement. JAMA. 2018:320:1 899 909. IPMID: 304221 99]
use. Unhealthy alcohol use is the third Ieading cause of doi :1 0.1001 ijama-2018. I 6789
231
Answers and Critiques
r(EY PO t 1{TS
. Poor communication between inpatient and outpatient
clinicians and between providers and patients is asso-
r Clinical findings do not sufficiently distinguish among
ciated with increased risk for hospital readmission.
the common causes olvaginitis, and laboratory testing
is necessary to establish a diagnosis.
. A multidisciplinary team approach to discharge plan
ning decreases the risk for readmission.
o Diagnostic testing for vaginitis includes assessment of
vaginal pH; amine (whiffl test; and microscopic evalu-
ation of the specimen to look for clue cells on the wet
Bibliography
Kamermayer AK, Leasure AR, Anderson L. 'l-he eflbctiveness of transitions
mount and yeast, hyphae, and pseudohyphae on the of care interventk)ns in reducing hospital readmissions and mortality:
potassium hydroxide preparation. , systematic review. Dimens Crit Care Nurs. 2017:36:311 6. IPMID:
'289764801 doi : 10. 1097i DCC.OO0000oooooo0266
232
Answers and Critiques
to help in the evaluation of bullous conditions because of she was initially vaccinated before the age of 65 years.
its lack of speciflcity. This patient's blistering disease is The ACIP recommends that adults aged 50 years or
conflned to her hands and is not consistent with bullous older receive the two-dose recombinant herpes zoster vac-
lupus erythematosus. cine (Option A) administered 2 to 6 months apart. Although
Anti tissue transglutaminase antibody testing (Option this patient received her doses 7 months apart, this is not
B) is the initial step in the diagnosis of dermatitis her- expected to decrease the efficacy ofthe vaccine series; a third
petiformis, an intensely pruritic bullous disease most dose is not indicated for this patient.
The tetanus and diphtheria toxoids (Td) vaccine (Option B)
commonly found on the elbows, knees, and buttocks that
is not indicated for this patient at this visit. The Td is indicated
is associated with celiac disease. This patient's clinical
flndings are not consistent with dermatitis herpetiformis, every 10 years, with at Ieast one dose replaced by the tetanus
and anti tissue transglutaminase antibody testing is not toxoid, reduced diphtheria toxoid, and acellular pertussis (Tdap)
vaccine. This patient received the Tdap vaccine 7 years ago and
needed.
The onset of bullous pemphigoid is usually in adults will not be due for her next Td vaccination until age 68 years.
aged 60 years or older and presents with urticarial and xtv Porilrs
eczematous lesions that progress to tense bullae on an o Adults aged 65 years or older who are not in high-risk
erythematous base. The most common sites of involve
groups (immunocompromised, presence of a cochlear
ment are the trunk, extremity flexures, and axillary and
implant or cerebrospinal fluid leak) should be engaged
inguinal folds. Oral involvement is present in approx-
in shared decision making regarding administration of
imately 20% of patients. Antibody tests are helping for
the 13-valent pneumococcal conjugate vaccine in addi-
monitoring disease activity in patients with bullous pem
phigoid. This patient's findings are not consistent with tion to the 23 valent pneumococcal polysaccharide
bullous pemphigoid, and antibody testing (Option C) is vaccine.
(Continued)
not appropriate.
233
Answers and Critiques
. Patients who want to receive both pneumococcal . Patients with chronic bacterial prostatitis require an
vaccines should receive the l3-valent pneumococcal extended course (at Ieast 4-6 weeks) of antimicrobial
conjugate vaccine first and the 23 valent pneumococ- therapy that has good prostatic tissue penetration and
cal polysaccharide vaccine at least 1 year later. covers the causative organism.
Bibliography Bibliography
F-reedman MS. Bernstein tl. Ault KA. Recommended adult immunization Cill BC. Shoskes DA. Bacterial prostatitis. Curr Opin lnfect Dis. 2016r29:
schedule, United States. 2021. Ann Intern Med. 2021. [PMID: 33571011] 86 91. IPMID: 26555038] Oo"rO.,697rQCO.OOOOOOOO0OOO0222
doi:10.7326 M20-8080
234
Answers and Critiques
23s
Answers and Critiques
disturbance, photophobia, and watery discharge, which are Because of the asymmetry and color variation of this
not present in this patient. patient's lesion, malignant melanoma (Option A) could be
Viral conjunctivitis (Option E), most commonly caused considered in the differential diagnosis. However, the rolled
by adenovirus, frequently occurs in the setting of upper borders ofthis patient's lesion and pearly translucent nature a
respiratory tract symptoms. Patients with viral conjunctivitis of the tumor are features of BCC and stand in contrast to the
often present with eye redness and clear, watery discharge berry like appearance of nodular melanoma (as shown). :
ftY ?otrTs
D
a^
o Acute bacterial conjunctivitis, most commonly caused
E
(D
by Staphylococcus aureus, typically presents with
l,I several days of unilateral diffuse conjunctival redness
o, and mucopurulent discharge.
EL
a'f
. Hyperacute bacterial conjunctivitis is a potentially
vision-threatening condition caused by Neisserio gon-
lt orrhea infection and is characterized by rapid onset of
.D profuse purulent discharge and diffirse bright red
a
conjunctival hemorrhage.
Bibliography
Varu DM, Rhee MK, Akpek EK, et al; American Academy of Ophthalmologr
Preferred Practice Pattern Cornea and External Disease Panel.
Conjunctivitis Preferred Practice Pattern'. Ophthalmolog. 20t9 ;t26 :P9 4
169. [PMID: 30366797) doi:10.1016/j.ophtha.2o18.10.020
236
lndex
Note: Page numbers followed by f and t denote figure and table, respectively. Bariatric surgery 44 46, 46t, 47t, Q9, Q47
Test questions are indicated by Q. Bartonella quintono, 7ll
Basal cell carcinoma, 123 124, 123f , l24f , Ql26
A Basal cell layer,82f
ABCDEs, of melanoma, 124 Beau lines, 1161
Abdominal aortic aneurysm (AAA), 4, Q97 Bed bugs, 1111, 112f
Acellular pertussis vaccine, 12, 16, Q40 Behavioral counseling, lT 18, Q105
Acne,83,96 98,97t 981, Q61 Behavioral therapy,44
Acne keloidalis nuchae, 114 U5, 115f Benign prcstatic hyperplasia (BPH), 29, 51-52, Q8
Acne vulgaris, 96-98,97f, 98t Benzoyl peroxide, 97-98
Acneiform eruptions, 96-100, 97t p Carotene, 19
Acral lentiginous melanoma, 125f Biologic therapy,4l
Acrochordons, l2O, l21f Biopsy
Actinic keratosis, 727 722, 722f endometrial,6l, Q45
Acute febrile neutrophilic dermatosis, 128-129 excisional, 841, 121, Q5
Acute generalized exanthematous pustulosis, 95, 95f punch,84l 84t
Acute kidney injury (AKI),40 shave,84l 84t
Acute otitis externa (AOE),77-78, Q112 Biotin, drug interactions with, l9
Acute otitis media(AOM),77 78 Black cohosh, 20t
Acute pelvic pain, Q57 Blepharitis, T0 77,77f
Acute sialadenitis, 80-81 Blue nails. u6t
Acyclovir,110 Body mass index (BMl), 6, 42t. See olso Obesity
Adenovirus, conjunctivitis and, 68 Bo rrelio recurrentis. ll7
Adrenal insufficienry, 39-40, 401, Q4 Bowen disease, 122 123,723f
Age-related macular degeneration (AMDI, 73-74, 74f BRCA]/2 gene mutations, 11
Alcohol use disorder Q117 Breast
Allergens, common,8Tt imaging, 55t
Alopecia, 113-115 masses, Q49
management of, 1131 symptoms,55-56, Q90
nonscarring, 113-114, 1141 Q83 Breast cancer, 8-9, 8t, 9t, Q123
scarring, 114-115, 115f Breast Imaging and Reporting Data System (BI RADS), 9, 55, 55t
a-Blockers,51 52 Bremelanotide, 65
Alprostadil,48 Breslow depth,125
Amoxicillin, drug eruptions and, 91 Bug bites, 83
Ampicillin, drug eruptions and, 91 Bullae, characteristics ol 83t
Analgesic agents, perioperative, 30t Bullous impetigo. 105-106. l06f
Androgen deficiency, 50, 501, Q113 Bullous pemphigoid, 102, 1021 103t
Androgenic alopecia, 113-114, 1131, 114f Burns, classification ol 112, 112t, Q65
Anemia, perioperative management ol 38-39 Burrows, characteristics of, 83t
Angle-closure glaucoma (ACG), 72,73f , Q37
Annual wellness visits, Medicare, 1 c
Anogenital warts, 119f Calcineurin inhibitors, 87
Antibiotics, topical, 85t, 86, 97 Calcium, 18, 18t
Anticoagulant therapy, 37-38, Q56, Q62, Q77, Q100 Call-outs,26t
Antifungals, topical, 85 86, 85t Callus. 120
Antihistamines, Q66 Canadian Task Force on the Periodic Health Examination, 1
237
Index
238
lndex
239
Index
240
lndex
241
lndex
i
l
l
I
l
\
242 i
i
i
l.
t
t
I
t.
t
I
I
t
t
t
t
t
t
I
t